You are on page 1of 440

NGOI NG 24H 1

WWW.NGOAINGU24H.VN

NGY 1: CC THNH PHN C BN CA CU

Ni dung bi hc
1. Cc thnh phn cu
2. Nm vng cu v cc cu trc b ngha
3. Nm vng cu m rng
4. Luyn tp phn tch cu

Bi tp luyn
Bi tp kim tra

ng k hc: 0962 60 8801 04 6260 3948 1


a ch: S 18 Trn i Ngha Q Hai B Trng H ni
NGOI NG 24H 2
WWW.NGOAINGU24H.VN

1. Cc thnh phn cu
Chc nng ca t loi
T loi Chc nng V d

Din t hnh ng ca ch ng.


ng t rt quan trng trong cu, The plant manager organized a day-long safety
ng t
do khi c cu, phi xc nh workshop. ng t
ng t trc tin.

Construction for the new plant will begin next


C vai tr l ch ng ( u cu) v
Danh t danh t
tn ng (sau ng t).
week.

Medical experts expressed concerns about the


C chc nng b ngha cho danh t
Tnh t increasing use of antibiotics.
v thng ng truc danh t.
tnh t danh t

C chc nng b ngha cho ng t We recently hired several sales personnel.


Trng t
hoc tnh t. trng t ng t

C chc nng lin kt cu vi cu Because Pierce was sick, he missed the


lin t ch ng ng t ch ng ng t
Lin t hoc lin kt cc ng t trong
(1) (2)
cng mt cu. meeting.

The concert has been canceled because of the


C chc nng lin kt danh t, gii t
Gii t
thng ng trc danh t. heavy rain.
(cm) danh t

2. Nm vng cu v cc cu trc b ngha


(1) Cc hnh thc cu trc b ngha
Mo t + trng t + tnh t + danh t : an increasingly popular hobby
Danh t phi ng sau mo t.
Tnh t ng trc danh t, b ngha cho danh t.
Trng t ng trc tnh t, b ngha cho tnh t.
Hnh thc b ngha sau danh t
Danh t + gii t + danh t : A newproduct ( under development) will be introduced next
year.
Cu trc b ngha gii t (under development) ng sau danh t (product) v b ngha cho danh
t.
ng k hc: 0962 60 8801 04 6260 3948 2
a ch: S 18 Trn i Ngha Q Hai B Trng H ni
NGOI NG 24H 3
WWW.NGOAINGU24H.VN

Trong cu trc cu ny, ch ng (a new product) v ng t (will be introduced) ng cch xa


nhau.
Danh t + mnh quan h (who / which / that) + ng t
The accounting manager ( who was transferred to London branch) was fired last week.

Mnh quan h (who was transferred to London branch) ng sau danh t (manager) v b
ngha cho danh t.
Trong cu trc cu ny, ch ng (the accounting manager) v ng t (was fired) ng cch xa
nhau.
Ghi ch
Phn tch cu trc cu theo th t di y:
1. Xc nh ng t.
2. Tm ch ng (thng l danh t).
3. Xc nh cu trc b ngha ri phn tch.
4. Xc nh s ha hp gia ch ng v ng t.

3. Nm vng cu m rng

(1) M rng cu bng cu trc b ngha

Khi thm cu trc b ngha vo cu c bn, cu s di v phc tp hn. Trc ht, ta phi xc nh
c cu trc ca cu c bn (ch ng + ng t + tn ng).

Cu c bn: The company fired employees


ch ng ng t tn ng

Thm trng t: The company recently fired employees.

Thm tnh t: The company recently fired several employees.

Thm (cm) gii t: The company on 5th Avenue recently fired several employees.

Thm mnh quan h: The company, which is located on 5th Avenue, recently fired several
employees.

(2) M rng cu bng cu trc lin kt

Lin kt danh t bng gii t


hiu cu nhanh chng, trc ht ta nn b qua cu trc lin kt bng gii t, ch tp trung vo cu
trc cu c bn trc.

The company, which is located in the business district, recently fired several employees
because of the economic slump.

ng k hc: 0962 60 8801 04 6260 3948 3


a ch: S 18 Trn i Ngha Q Hai B Trng H ni
NGOI NG 24H 4
WWW.NGOAINGU24H.VN

gii t cm danh t

Lin kt cu bng lin t

The company, which is located on 5th Avenue, recently fired several employees
because its sales have declined substantially.
lin t ch ng ng t

(= Because its sales have declined substantially, the company, which is located on 5th Avenue,
recently fired several employees.)
4. Luyn tp phn tch cu

1. Gch di ch ng, ng t v tn ng
2. Xc nh cu trc b ngha v cu trc lin kt
ex. The plant manager organized a day-long safety workshop.
ch ng ng t tn ng

Bi tp luyn: Tm ch ng, ng t v tn ng
1. The human resources department will take further stops.
2. All new employees must attend this week's session on company policies.
3. The company announced the opening of its new production line.
4. To meet customer's demands is our most important priority.
5. Retaining competent employees is important to succeed in business.
6. A new fax machine will be installed in the copy center.
7. Customers are requested to complete the survey.
8. This medical appliance is guaranteed for one year from the date of purchase.
9. All financial reports must be sent to the accounting department.
10. Special discounts will be given to all new subscribers.
11. The mediator in the merger talks was replaced.
12. The contents of this journal are carefully examined for errors.
13. Regional managers who exceeded sales goals will receive special bonuses.
14. The central library, which was built in 1970, is currently under renovation.
15. Ms. Leona, who is in charge of the accounting department, will be promoted.
16. The concert has been canceled because of the heavy rain.
17. According to a recent survey, interest rates are expected to rise slightly.
18. Because his flight was late, Mr. Lim missed the meeting.
19. The project has been a failure though we invested a lot of money in it
20. International Affairs, a monthly journal, has been published for more than 10 years.

ng k hc: 0962 60 8801 04 6260 3948 4


a ch: S 18 Trn i Ngha Q Hai B Trng H ni
NGOI NG 24H 5
WWW.NGOAINGU24H.VN

BI TP THC HNH
Bi tp 1:
Choose one correct option in the parentheses.
01 (Register, Registration) begins at 7 a.m.
02 The (cost, costly of housing rose dramatically last year.
03 (Write, To write) a novle is a goal of many creative writers.
04 (Eating, Eat) well leads to good health.
05 (Our (strategic, strategy) is to launch the product in Europe.
06 The (satisfaction, satisfactory) of our clients is our highest concern.

Choose the correct option for each of the following sentences.


07 The ______ of accidents should take priority in the workplace.
(A) prevent (C) prevention
(B) prevented (D) preventive
08 _____ should submit their evaluations of this new hairdryer before leaving.
(A) Participants (C) Participate
(B) Participated (D) Participatory
09 The _____ in the research budget caused a delay in the design process.

(A) reductive (C) reduction


(B)reduced (D) reduction
10 Because of this poor performance, the _____ dismissed the companys financial manager.

(A) direct (C) director


(B) directive (D) directed

Bi tp 2: Choose correct option in the parentheses.


01 These positions (require, requirement) extensive experience.
02 The company (to organize, should organize) a training program for staff.
03 Mr. Lee (purchased, purchasing) new office furniture.
04 Dialogue (helpful, can help) people to resolve problems.

Actual Question: Choose the correct option for each of the following sentences.
05 The CEO _____ a new management system in the board meeting.
(A) will suggest (C) suggesting
(B) suggestion (D) to suggest
06 The members of the HR department _____ every applicant's file.
(A) to review (C) reviewed
(B) reviewing (D) reviewer

ng k hc: 0962 60 8801 04 6260 3948 5


a ch: S 18 Trn i Ngha Q Hai B Trng H ni
NGOI NG 24H 6
WWW.NGOAINGU24H.VN

Bi tp 3:
Choose one correct option in the parentheses.
01 He reported the (problem, problematic) to the administrator.
02 We signed an (agree, agreement) with a foreign company.
03 Many young consumers prefer (to shop, shop) online.
04 The labor union demanded the (resign, resignation) of the president.
05 Mr. Kato considered (launching, launch) a campaign to promote a new PDA.
06 Flyaway Service guarantees (deliver, delivery) in three days.

Choose the correct option for each of the following sentences.


07 Our team can design the _____ according to customers' requirements.
(A) produced (C) productive
(B) product (D) productively
08 The proper _____ of equipment will prevent injuries.
(A) utilization (C) utilize
(B) utilizing (D) utilized
09 A broadband connection offers the _____ of immediate access to the Internet.
(A) advantaging (C) advantageous
(B) advantageously (D) advantage
10 Mr. Hornby did not notice any _____ between the two brands of camera.
(A) difference (C) different
(B) differ (D) differs
Bi tp 4:
Choose one correct option in the parentheses.
01 The new buildings are (spacious, spaciously) with high ceilings.
02 New LCD screens made the enterprise (popular, popularly).
ng k hc: 0962 60 8801 04 6260 3948 6
a ch: S 18 Trn i Ngha Q Hai B Trng H ni
NGOI NG 24H 7
WWW.NGOAINGU24H.VN

03 Airplane tickets are not (transferable, transfer) to other individuals.


04 We consider Ms. Elson a qualified (apply, applicant).
05 It is (advise, advisable) to obtain a second opinion if you plan to invest.
06 Jack is a (manager, manage) in the computer business.

Choose the correct option for each of the following sentences.


07 It was a great _____ that you were not hired for the position.
(A) disappoint (C) disappoints
(B) disappointedly (D) disappointment
08 Steven Williams was a market _____ when he worked for KMN, Inc.
(A) analyze (C) analyst
(B) analyzes (D) analyzing
09 Researchers found the recycling system _____ in reducing garbage.
(A) effect (C) effective
(B) effects (D) effectively
10 State governments and local citizens are _____ in developing and financing educational
programs.
(A) cooperative (C) cooperated
(B) cooperate (D) cooperatively

Bi tp 5:
Put brackets ([ ]) around the modifier(s) in the following sentences.
01 Despite the bad weather, the event is continuing.
02 Tenants cannot renovate their homes without the approval of the owner.
03 Because of a technical problem, we postponed the performance.
04 It was Mr. Downes who was involved in stealing funds.
05 Customers seeking refunds should present a receipt.
06 Economic experts met to analyze the impact of higher interest rates.

Choose the correct option for each of the following sentences.


Questions 7 and 8 refer to the following memorandum.

PRESS CONFERENCE ON BARKLEY-MILLER MERGER

The Barkley Software Company will hold a press conference to announce its merger with Miller
Associates. Barklev Software has developed an innovative _____ to restruct the two companies. It
07 (A) strategic (C) strategize
(B) strategy (D) strategically

will be ____ because it will increase product output and employee creativity.

08 (A) beneficial (C) benefited


(B) beneficially (D) benefit

ng k hc: 0962 60 8801 04 6260 3948 7


a ch: S 18 Trn i Ngha Q Hai B Trng H ni
NGOI NG 24H 8
WWW.NGOAINGU24H.VN

Bi tp 6:
Practice with TOEIC Actual Questions
Decide which of the choices - (A), (B), (C), or (D) best completes the sentence.
1. Mr. Smith decided to _____ for the job advertised in the newspaper.
(A) apply
(B) recommend
(C) do
(D) pay
2. The company is going to run a new commercial to _____ sales.
(A) return
(B) quit
(C) transfer
(D) promote
3. I have to _____ this final report by the end of the week.
(A) solve
(B) repair
(C) participate
(D) complete
4. The director strongly _____ Mr. Watson for the job of manager.
(A) contacted
(B) recommended
(C) quit
(D) transferred
5. The board of .directors will gather to _____ the problem next month.
(A) do
(B) participate
(C) solve
(D) apply
6. The manual _____ lots of information on how to use the machine.
(A) makes
(B) is
(C) has
(D) seems
7. Jacksons proposal about the new marketing policy looks _____
(A) interests
(B) interest
(C) interesting
(D) to interest
8. Jacksons proposal about the new marketing policy looks _____
(A) interests
(B) interest
(C) interesting
(D) to interest
ng k hc: 0962 60 8801 04 6260 3948 8
a ch: S 18 Trn i Ngha Q Hai B Trng H ni
NGOI NG 24H 9
WWW.NGOAINGU24H.VN

9. Many people _____ in line in the lobby.


(A) made
(B) recommended
(C) wrote
(D) waited
10. All of the applicants _____ the system very inconvenient.
(A) became
(B) found
(C) turned
(D) mentioned
11. He _____ the doctors office to make an appointment.
(A) called
(B) calling
(C) caller
(D) to call
12. The Madison Hotel _____ guests free coffee and drinks in the lobby.
(A) becomes
(B) finds
(C) offers
(D) keep
13. The _____ looked effective, and the board of directors liked it.
(A) propose
(B) proposal
(C) proposed
(D) proposes
14. The factory manager sent a memo _____ all of the employees.
(A) of
(B) for
(C) in
(D) to

Questions 15-16 refer to the following letter


Dear Lost and Found,

I am writing this letter to ask you about my missing wallet.

Last Friday, I ____ on your airline to go to Boston. I had my wallet when I boarded Flight No. 053

ng k hc: 0962 60 8801 04 6260 3948 9


a ch: S 18 Trn i Ngha Q Hai B Trng H ni
NGOI NG 24H 1
WWW.NGOAINGU24H.VN 0

15. (A) flew


(B) rode
(C) boring
(D) useful

in Vacouver. After arriving in Boston, I took a taxi. When I tried to ____ for the taxi, I found my
16. (A) fill
(B) pay
(C) enter
(D) make

wallet was missing. I guess that I dropped my wallet on the plane. The wallet is brown leather, and
there are several credit cards in my name in it. Could you kindly check if there is a missing wallet on
the plane?

Thank you in advance.

Regard,

Luke Danes

ng k hc: 0962 60 8801 04 6260 3948 10


a ch: S 18 Trn i Ngha Q Hai B Trng H ni
NGOI NG 24H 1
WWW.NGOAINGU24H.VN 1

BI TP KIM TRA
1. She has spent many .nights examining the questions.
A. sleepy
B. sleepless
C. sleeplessly
D. sleep
2. Many people like her because of her
A. polite
B. politeness
C. impolite
D. politely
3. You will have to work hard if you want to ..
A. succeed
B. successfully
C. success
D. successful
4. Read the book and you can find the information you need.
A. care
B. careful
C. carefulness
D. carefully
5. Lonan Imports will work .... with any distributor to fulfill a customer request.
A. directly
B. directs
C. directed
D. directness
6. Products made by Izmir Vitamins are designed to promote... and well-being.
A. health
B. healthy
C. healthful
D. healthfully
7. Jane Turngate was elected chairperson of the board of directors at Stellmann Corporation by a .....
margin.
A. narrow
B. narrows
C. narrowly
D. narrowness
8. Grove Canoes' prices may .... vary depending on changes in the cost of raw materials.
A. occasion
B. occasions
C. occasional
D. occasionally
9. A 20 percent increase in revenue makes this the .... year yet for the Sorvine HotelGroup.
A. more profitable
B. most profitable
C. profiting
D. profitably
10. McLellan Associates, the ..... of the two law firms, is presently advertising several job openings
for paralegals.
ng k hc: 0962 60 8801 04 6260 3948 11
a ch: S 18 Trn i Ngha Q Hai B Trng H ni
NGOI NG 24H 1
WWW.NGOAINGU24H.VN 2

A. largeness
B. larger
C. largely
D. large
11. The route to Sandy Shores Inn is .... marked from exit 262 on the coastal highway.
A. clearly
B. clear
C. clarity
D. clearance
12. A ... from Jensen-Colmes Corporation will be happy to meet with prospective job applicants at
the Westborough Job Fair.
A. represent
B. representing
C. representative
D. representation
13. The board of trustees will .... vote on the revised proposal at its meeting later this month.
A. define
B. definite
C. definitive
D. definitely
14. Payment is ... at the time you check out of the conference center.
A. expect
B. expecting
C. expected
D. expectation
15. Dr. Sato has indicated that there may soon be an ... for an apprentice technician in the chemical-
production laboratory.
A. open
B. opened
C. openly
D. opening
16. Although Dr. Obetz is qualified in several areas, nutritional health is one of her ....
A. special
B. specializes
C. specialties
D. specialists
17. The analyst's commentary in the progress report was sharply ..... of the leadership team.
A. critical
B. critic
C. critically
D. criticism
18. The ... of the Municipal Park Restoration Project was announced yesterday by the local
hospitality association.
A. completed
B. completes
C. completion
D. complete
19. The loan application process at Palau Bay Bank is very .....
A. efficient
ng k hc: 0962 60 8801 04 6260 3948 12
a ch: S 18 Trn i Ngha Q Hai B Trng H ni
NGOI NG 24H 1
WWW.NGOAINGU24H.VN 3

B. efficiency
C. efficiently
D. efficiencies
20. According to a new study, the United States will face ever increasing .. in the global economy
over the next decade.
A. competes
B. competition
C. competitively
D. competitive
21. Almost everyone who prepares to start a business finds that the financial projections and market
analysis are than they expected.
A. difficult
B. difficulty
C. more difficult
D. much difficulty
22. The hotel is .... located in Londons downtown within walking distance from banks, post office
and stores.
A. convenience
B. convenient
C. conveniences
D. conveniently
23. Over the years we have earned the .... trust and confidence of hundreds of students worldwide
through our unmatched performance in English education.
A. complete
B. completely
C. completed
D. completing
24. If you have .... received an incorrect order or your shipment has arrived damaged please see our
Customer Service section for our Return Policy.
A. mistook
B. mistake
C. mistaken
D. mistakenly
25. In order to use these materials for profit, you must gain formal .... from our company in writing.
A. approving
B. approval
C. approved
D. approves
26. A rapidly growing beverage company is seeking qualified and .... regional and district sales
managers to work with local distributors.
A. motivate
B. motivator
C. motivated
D. motivation
27. Our university has one of the most .... financial support programs in the US to ensure that
students can meet the cost of their college education, regardless of background.
A. extend
B. extent
C. extensive
ng k hc: 0962 60 8801 04 6260 3948 13
a ch: S 18 Trn i Ngha Q Hai B Trng H ni
NGOI NG 24H 1
WWW.NGOAINGU24H.VN 4

D. extending
28. Because it would generate a lot of money, the .... to win the contract will likely be aggressive and
fierce.
A. competitively
B. competitive
C. competes
D. competition
29. This book will teach you how to get .... out of debt including your mortgage.
A. total
B. totaling
C. totally
D. totaled
30. The software company .... sent out e-mails containing a virus to about 50,000 members on an e-
mail discussion list.
A. accident
B. accidental
C. accidents
D. accidentally
31. Most anti-virus programs are scheduled to perform a background check .... throughout the day on
a set schedule.
A. periodically
B. periodic
C. periodical
D. period
32. The interior of the high-speed trains is .... and able to carry more than 500 passengers.
A. space
B. spacious
C. spacing
D. spaciousness
33. In a recent survey, people appear to consider changing their job when they find their current
work too ....
A. repeat
B. repetitive
C. repeating
D. repetition
34. Today a lot of manufacturing firms have been too .... about investment, opting to amass cash
reserves instead.
A. caution
B. cautiousness
C. cautious
D. cautiously
35. There are many folk cultures on the island, which are .... distinct from the mainland.
A. clearer
B. clearly
C. clearest
D. cleared
36. During the war, a lot of companies in Japan were able to add .... to their wealth by selling deadly
weapons.
A. consider
ng k hc: 0962 60 8801 04 6260 3948 14
a ch: S 18 Trn i Ngha Q Hai B Trng H ni
NGOI NG 24H 1
WWW.NGOAINGU24H.VN 5

B. consideration
C. considerately
D. considerably
37. The .... of these ancient burial grounds has given us a lot of useful information about their
culture.
A. survive
B. survivor
C. survival
D. survey
38. A variety of pictures are scanned into a form of proper .... that computers can recognize.
A. informal
B. informative
C. information
D. informing
39. In order to .... the internal market, exports were controlled, and foreign exchange and issuance of
permissions were used as foreign trade policies.
A. regulate
B. regulating
C. regulation
D. regular
40. Visitors to the library are asked to speak .... and keep conversations brief when in the main
reading room.
A. quiet
B. quietly
C. quietest
D. quietness

ng k hc: 0962 60 8801 04 6260 3948 15


a ch: S 18 Trn i Ngha Q Hai B Trng H ni
NGOI NG 24H 1
WWW.NGOAINGU24H.VN 6

NGY 2: TH CA NG T

Ni dung bi hc
Ch 1: Trng t ch thi gian din t qu kh
Ch 2: Trng t ch thi gian din t tng lai
Ch 3: Trng hp phi dng th hin ti

Bi tp luyn
Bi tp kim tra

ng k hc: 0962 60 8801 04 6260 3948 16


a ch: S 18 Trn i Ngha Q Hai B Trng H ni
NGOI NG 24H 1
WWW.NGOAINGU24H.VN 7

Ch 1: TRNG T CH THI GIAN DIN T QU KH


Chn cu ng.
1. a. Kathy will send an e-mail last week.
b. Kathy sent an e-mail last week.
2. a. Kathy provided a free meal yesterday.
b. Kathy provides a free meal yesterday.
ng:
Kathy sent an e-mail last week.
Kathy provided a free meal yesterday.
Trng t ch thi gian qu kh
=> ng t c chia th qu kh
Ng php b sung
Cc trng t c dng th qu kh:
yesterday
last ~ : last week, last month, last year
~ ago: a week ago, a year ago, ten years ago
Pierce submitted the document yesterday / last week / a week ago

Bi tp 1:
1. Mr. Kim (will attend/ attended) the conference last year.
2. The company (conducted/ will conduct) the survey two weeks ago.
3. I (included/ include) the report in my e-mail yesterday.
4. J&J (held/ will hold) a press conference a month ago here.
5. Early last week, we (will inspect/ inspected) the quality of the products.

Bi tp 2:
1. Ms. Helen________transferred to the Chicago branch last week.
(A) is (B) will be
(C) has been (D) was
2. The sales manager________our company six years ago.
(A) will join (B) joined
(C) joins (D) is joining
3. The money________deposited in the bank last Friday.
(A) was (B) is
(C) has been (D) will be
4. The City Hall________constructed in 1945.
(A) were (B) was
(C) is (D) will be
5. He________elected as vice president of our company in the last weeks meeting.
(A) is (B) was
(C) will be (D) were

Ch 2: TRNG T CH THI GIAN DIN T TNG LAI


Chn cu ng:
1. a. Kathy will work next Saturday.
b. Kathy worked next Saturday.
2. a. Kathy delivered good news tomorrow.
b. Kathy will deliver good news tomorrow.
ng:
ng k hc: 0962 60 8801 04 6260 3948 17
a ch: S 18 Trn i Ngha Q Hai B Trng H ni
NGOI NG 24H 1
WWW.NGOAINGU24H.VN 8

Kathy will work next Saturday.


Kathy will deliver good news tomorrow

Trng t ch thi gian tng lai => ng t cng th tng lai


Ng php b sung
Cc trng t c dng th tng lai:
tomorrow
next ~ : next week: tun sau / next month :thng sau/ next year :nm sau
Pierce will submit the document tomorrow/ next week/ next month/ next year.

Bi tp 3:
1. I (submitted/ will submit) the proposal in person next week.
2. During next weeks seminar, the managers (will discuss/ discusses) the problem.
3. The company (constructed/ will construct) a new assembly plant next year.
4. There (was/ will be) an orientation for new employees tomorrow morning.
5. We will (implement/ implemented) a new vacation policy as of next month.

Bi tp 4:
1. Ms. Jin__________from her trip next week.
(A) returned (B) returns (C) to return (D) will return
2. The manager__________with the president tomorrow.
(A) will meet (B) met (C) meets (D) meeting
3. The bank __________ interest rates next month.
(A) raises (B) is raising (C) will raise (D) raised
4. The economy __________ from the recession next year.
(A) will recover (B) recovers (C) recovered (D) is recovering
5. Effective next week, Mr. Park __________ in charge of the sales department.
(A) is (B) will be (C) be (D) was

Ch 3: TRNG HP PHI DNG TH HIN TI


Vi th hin ti, ngoi vic nhn bit thng qua trng t ch thi gian, ta cn c th nhn bit qua
ngha ca cu
TRNG HP DNG TH HIN TI
Khi din t s tht ni chung
Khi din t mt s vic lp i lp li hay mt thi quen
Trng hp khc

1. Khi din t s tht ni chung


Ex: Managers spend considerable time training new employees
Manager: gim c, qun l
Spend time (in) + V-ing: dnh thi gian lm ~
Considerable: nhiu, ng k
Train: o to

2. Khi din t mt s vic lp i lp li hay mt thi quen


Ex: We usually spend much money on advertising
Nhng trng t din t hnh ng lp i lp li hoc thi quen
Always often usually
frequently every day every year
ng k hc: 0962 60 8801 04 6260 3948 18
a ch: S 18 Trn i Ngha Q Hai B Trng H ni
NGOI NG 24H 1
WWW.NGOAINGU24H.VN 9

3. Trng hp khc
S vic din ra trong thi gian ngn by gi, phi dng th hin ti tip din
Be + V-ing
Ex: They are planning to build a new plant now.
Plan to do: d nh lm
Build: xy dng
Plant: nh my

Bi tp 5:
1. It usually (will take/ takes) one hour to get to the City Hall.
2. I (read/ have read) a book every morning.
3. He usually (goes/will go) to work at 7:30
4. The company (recruits/ is recruiting) an experienced sales manager now.

Bi tp 6:
1. The public library usually no fees for local residents to borrow books.
(A) will change (B) charging
(C) to charge (D) charges
2. Every month, we a monthly business meeting.
(A) will have (B) have
(C) having (D) has
3. The department heads frequently with the overseas marketing team.
(A) to meet (B) will meet
(C) meet (D) meeting
4. A good supervisor encouragement to employees.
(A) gave (B) is giving
(C) given (D) gives
5. We are now our efforts to provide better services to our customers.
(A) increase (B) increasing
(C) increases (D) increased

BI TP LUYN TP:
Part V Chn t thch hp in vo ch trng di y.
1. The blueprint for the office building __________ finished by next week.
(A) will have (B) have
(C) are (D) will be
2. The president __________ the office early every Friday.
(A) will leave (B) left
(C) is leaving (D) leaves
3. The recruitment of sales personnel __________ last week.
(A) is ending (B) will end
(C) ends (D) ended
4. By next Friday, all budget reports __________ reviewed by the accounting
team.
(A) will be (B) are
(C) have (D) were
5. I knew Mr. Kims resignation when I __________ his office last Tuesday.
(A) will visit (B) visited
(C) visiting (D) visit
ng k hc: 0962 60 8801 04 6260 3948 19
a ch: S 18 Trn i Ngha Q Hai B Trng H ni
NGOI NG 24H 2
WWW.NGOAINGU24H.VN 0

Part VI in vo ch trng trong mu qung co sau.


Starting next week, the new City Library _________ a variety of services to meet the
needs of all
6. (A) will provide (B) provides
(C) provided (D) providing
citizens. In the past, it _________ impossible to provide those services due to the lack
of
7. (A) has (B) was
(C) is (D) will be
technology and equipment. We always _________ comments and suggestions that may
help us serve you better. If you have any suggestions on how we can improve, let us know.
8. (A) welcomed (B) will welcome
(C) welcome (D) welcomes

BI TP THC HNH
Bi tp 1:
Choose one correct option in the parentheses.
01 Mr. Chen (finished, finish) reviewing the summary yesterday.
02 We often (announce, will announce) the date for interviews in a local paper.
03 The board (convened, will convene) an executive session tomorrow.
04 Many workers (go, went) abroad in 2003.
05 Next Tuesday, the ship (will leave, left) the harbor at 12 p.m.
06 Ms. Cooper (met, meets) one of her European clients three days ago.

Choose the correct option for each of the following sentences.


07 The hotel's recreational facilities _____ a swimming pool and tennis courts.
(A) include (C) inclusion
(B) includes (D) to include
08 Some experts predict that urban areas _____ 98 percent of the world's population by 2050.
(A) hold (C) will hold
(B) held (D) holding
09 Some experts predict that urban areas _____ 98 percent of the world's population by 2050.
(A) hold (C) will hold
(B) held (D) holding
10 Several computer manufacturers _____ their new lines in the next several months
(A) introduce (C) introduction
(B) will introduce (D) be introduced

Bi tp 2: Choose one correct option in the parentheses.


01 Ms. Tracy (is preparing, was preparing) the annual report now.
02 Our CEO (will be attending, was attending) an important workshop at this time yesterday.
03 This time tomorrow, Chris (is attending, will be attending) an important conference.
04 She (worked, was working) on her report when the fire took place.
05 Mr. McCain (is returning, was returning) to Korea now.

ng k hc: 0962 60 8801 04 6260 3948 20


a ch: S 18 Trn i Ngha Q Hai B Trng H ni
NGOI NG 24H 2
WWW.NGOAINGU24H.VN 1

06 We (were making, will be making) arrangements for the Trade Conference tomorrow.

Choose the correct option for each of the following sentences.


07 Ms. Thomas _____ in the employee development session at 3 o'clock yesterday
(A) participates (C) is participating
(B) to participate (D) was participating
08 Wellton supermarket _____ its customers a 20 percent discount until Monday.
(A) offered (C) will be offering
(B) is offering (D) was offering
09 The manufacture _____ changes to its billing procedures now.
(A) implements (C) implementing
(B) implemented (D) is implementing
10 The memo informed us that the company _____ the renovation of the office next month.
(A) is carrying out
(B) has been carrying out
(C) will be carrying out
(D) is being carrying out

Bi tp 3:
Choose one correct option in the parentheses.
01 Mr. Heath (has appointed, will have appointed) Richard Barth as a financial officer by next May.
02 Before the results came in, she (had already gone, will have already gone).
03 The price of gas (increases, has increased) considerably since 2000
04 The revenue of the company (falls, has fallen) significantly for two years.

Choose the correct option for each of the following sentences


05 The cost of living In the country_____ by 17 percent over the last 15 years.
(A) will rise (C) rising
(B) has risen (D) rise
06 Mr. Jantick _____ in the military for six months by the end of the year.
(A) been served (C) will serve
(B) has served (D) will have served

Questions 7 and 8 refer to the following advertisement.

We are looking for a corporate trainer. Applicants should hold a master's degree in business or
communications studies. Also, five years of experience is required. The Business Training Program
_____ the most popular of all the programs we have offered over the past two years. The

01 (A) will be (C) had been


(B) was (D) has been

qualified candidate _____ training programs related to marketing strategies and management

ng k hc: 0962 60 8801 04 6260 3948 21


a ch: S 18 Trn i Ngha Q Hai B Trng H ni
NGOI NG 24H 2
WWW.NGOAINGU24H.VN 2

02 (A) conducts (C) will conduct


(B) conducted (D) has conducted

starting next month. If you have any questions, please contact Mr. Barry at 555-7800.

Bi tp 4: Choose one correct option in the parentheses.

01 Toxic chemicals should always be (practiced, handled, prevented) with care.


02 Effective advertisements (broaden, attract, convene) many new customers.
03 If you wish to attend the seminar, do not (expand, consider, hesitate) to contact me.
04 Computer programs are (labored, utilized, proceeded) in the shipment of orders.

Choose the correct option for each of the following sentences.


05 A deposit of $50 is required to _____ seats for the upcoming season.
(A) respond (C) connect
(B) appoint (D) reserve
06 The annual meeting _____ as planned last month without any interruption.
(A) reviewed (C) proceeded
(B) obtained (D) marched

Questions 7 and 8 refer to the following e-mail.

From: Julia Banks, CrackEnglish Publishing


To: Cathy Jones, Customer Service

Dear Ms. Jones,

Due to defects in the audio tapes we ordered from your company, we have received numerous
complaints from our customers. As we think that this problem could have been _____ if you had

07 (A) protected (C) reported


(B) solved (D) prevented

not sent us faulty merchandise, we believe that your company should seriously _____ how we can

08 (A) counsel (C) consider


(B) impact (D) notify

be compensated for the losses. We would like to hear from you as soon as possible

Bi tp 5:
ng k hc: 0962 60 8801 04 6260 3948 22
a ch: S 18 Trn i Ngha Q Hai B Trng H ni
NGOI NG 24H 2
WWW.NGOAINGU24H.VN 3

Decide which of the choices - (A), (B), (C), or (D) - best completes the sentence.
1. ICD Co. announced that it would _____ a new product next year.
(A) remain
(B) conduct
(C) launch
(D) renew
2. The secretary _____ a ticket to New York.
(A) booked
(B) complained
(C) increased
(D) met
3. The company decided to _____ the workforce in the Maintenance Department.
(A) afford
(B) reserve
(C) postpone
(D) reduce
4. Mr. Brown _____ the sales report last week.
(A) submitted
(B) submit
(C) submits
(D) submission
5. We _____ a new model to the China market in a month.
(A) introduce
(B) introduced
(C) will introduce
(D) were introducing
6. Last year, my team_____ a seminar every month.
(A) held
(B) holds
(C) holding
(D) will holding
7. There _____ increased concern about our competitors for the last year.
(A) have been
(B) has been
(C) was
(D) be
8. The committee _____ the proposal at the moment.
(A) was reviewing
(B) is reviewing
(C) reviews
(D) reviewed
9. By next month, Morgan _____ in the department for 3 years.

ng k hc: 0962 60 8801 04 6260 3948 23


a ch: S 18 Trn i Ngha Q Hai B Trng H ni
NGOI NG 24H 2
WWW.NGOAINGU24H.VN 4

(A) is
(B) was
(C) has been
(D) will have been
10. Ms. Sanderson _____ for a marketing firm before she came here.
(A) has worked
(B) works
(C) working
(D) had worked
11. Ms. Kimberly _____ out. of the country until next Tuesday.
(A) was
(B) is
(C) will be
(D) has been
12. The manager asked if there _____ any calls for her.
(A) is
(B) are
(C) had been
(D) has been
13. By 2020, the demand for digital cameras _____ twofold.
(A) will increase
(B) has increased
(C) will have increased
(D) increases
14. If managers _____ their employees to work overtime, they will have to pay them more
money.
(A) want
(B) will want
(C) wants
(D) have wanted

Questions 15-16 refer to the following letter.

Dear Mr. Lee,

I am so sorry that we _____ a mistake while processing your order last week.

15. (A) made


(B) makes
(C) making
(D) has made

I understand this caused a terrible inconvenience for your business. I _____ the product you
ng k hc: 0962 60 8801 04 6260 3948 24
a ch: S 18 Trn i Ngha Q Hai B Trng H ni
NGOI NG 24H 2
WWW.NGOAINGU24H.VN 5

16. (A) sent


(B) will send
(C) sending
(D) has sent

ordered as soon as possible as well as a free gift. Once again, 1 am sorry about the inconvenience,
and I hope to continue to do business with you for many years.

BI TP KIM TRA
1. My wife and I ______the company picnic last weekend.
(A) were attending
(B) attend
(C) attended
(D) attending
2. ______ coming into work today because I am sick.
(A) I am
(B) I will
(C) I'm not
(D) I didn't
3. No, I _____ think that an office assistant would be useful.
(A) don't
(B) does
(C) doesn't
(D) am doing
4. The company ______taking the stand that even being five minutes late is the same as missing a
day's work.
(A) is
(B) will
(C) am
(D) isn't
5. I usually eat my lunch in the breakroom, but today I _____my lunch at my desk.
(A) eat
(B) ate
(C) eaten
(D) have eaten
6. Planning the new office layout has been the hardest task I ______ in a long while.
(A) had have
(B) did have
(C) did had
(D) have had
7. Everyone_______ been wonderful to work with, and I will sorely miss working here.
(A) have
(B) did
(C) hasn't
(D) has

ng k hc: 0962 60 8801 04 6260 3948 25


a ch: S 18 Trn i Ngha Q Hai B Trng H ni
NGOI NG 24H 2
WWW.NGOAINGU24H.VN 6

8. The committee _______that we do not pursue legal action at this time, but wait for the result of
the tests.
(A) suggesting
(B) is recommending
(C) has
(D) is recommended
9. The worst that can happen _______ that the store will have to alter its operating hours until we can
hire replacement staff.
(A) are
(B) was
(C) is
(D) have
10. The boss was so angry at me that I was sure I _______ be fired.
(A) will
(B) would
(C) may
(D) might
11. She said she_______ rather not meet with you face to face.
(A) had
(B) could
(C) did
(D) would
12. My arm hurts. A bunch of us _______blood today at work.
(A) give
(B) to give
(C) gaved
(D) gave
13. She _______always on time.
(A) is
(B) does be
(C) are
(D) be
14. She _______spoken to me about your proposal.
(A) did
(B) have
(C) is
(D) has
15. There_______ no I in "Team."
(A) be
(B) isn't
(C) ain't
(D) is
16. Tomorrow, I_______ going on a week-long business trip to the Maldives.
(A) will
(B) to
(C) is
(D) am
17. We'll start when he _______ready.
(A) will
ng k hc: 0962 60 8801 04 6260 3948 26
a ch: S 18 Trn i Ngha Q Hai B Trng H ni
NGOI NG 24H 2
WWW.NGOAINGU24H.VN 7

(B) will is
(C) will be
(D) is
18. We _______at a party two months ago.
(A) meet
(B) met
(C) have met
(D) meeting
19. So far, there _______no word from them.
(A) is
(B) was
(C) has
(D) has been
20. When I called on her, Mary_______ her room.
(A) clean
(B) cleans
(C) cleaning
(D) was cleaning
21. All things _______, Professor Kim is the best instructor I've ever had.
(A) considered
(B) to consider
(C) considering
(D) considerable
22. I'm quite sure this business will_______.
(A) paying
(B) pay for
(C) pay off
(D) paid
23. I'm sort of _______with his flattery.
(A) disgust
(B) to disgust
(C) disgusting
(D) disgusted
24. These pills will surely _______your pain.
(A) ease
(B) easy
(C) easily
(D) easiness
25. I _______ like to see his daughter right now.
(A) will
(B) shall
(C) would
(D) could
26. Losing interest in her business, Kimberly has recently_______.
(A) retired
(B) be retired
(C) to be retired
(D) been retired
27. Almost every part of our lives _______computerized over the past to years.
ng k hc: 0962 60 8801 04 6260 3948 27
a ch: S 18 Trn i Ngha Q Hai B Trng H ni
NGOI NG 24H 2
WWW.NGOAINGU24H.VN 8

(A) have been


(B) has been
(C) was
(D) had done
28. I went to Los Angeles fifteen years _______ .
(A) before
(B) ago
(C) since
(D) next to
29. Kate usually _______ to class by bicycle, but today she went by bus because of the rain.
(A) go
(B) goes
(C) went
(D) gone
30. Frank_______ a TV show when Karen got home.
(A) watched
(B) watches
(C) watch
(D) was watching
31. We _______her for more than twenty years.
(A) know
(B) knows
(C) have known
(D) are known
32. His father _______a lawyer, but now he's a politician.
(A) is used to be
(B) was used to be
(C) use to be
(D) used to be
33. We were asked not to disturb the baby because he _______ .
(A) is sleeping
(B) slept
(C) was sleeping
(D) sleeping
34. He has studied English _______ since he was in elementary school.
(A) during
(B) while
(C) as long as
(D) Since
35. While Steve was washing his car, he _______ some dents in the doors.
(A) discovered
(B) is discovering
(C) was discovering
(D) has discovered
36. Ever since he arrived, he _______ quietly in the corner.
(A) sat
(B) has been sitting
(C) sits
(D) is sitting
ng k hc: 0962 60 8801 04 6260 3948 28
a ch: S 18 Trn i Ngha Q Hai B Trng H ni
NGOI NG 24H 2
WWW.NGOAINGU24H.VN 9

37. When he got home, his wife _______dinner.


(A) fix
(B) fixes
(C) fixing
(D) was fixing
38. On my trip to Italy, not only _______ a suitcase, but I broke my glasses.
(A) I lost
(B) I lose
(C) lost I
(D) did I lose

Questions 39 through 42 refer to the following letter.


Questions 39 through 42 refer to the following letter.

Mr. Albert Di Beni,


333 Spring Road
Penshurst
Kent

Dear Mr. Di Beni,

The Penshurst Medical Practice invites you to make an appointment for a medical _______. After
39. (A) exam
(B) examination
(C) quiz
(D) test

the age of forty, we recommend that you have a full physical every year. Our records show that you
recently _______ your fiftieth birthday. However, it is over seven years since were

40. (A) celebrated


(B) have celebrated
(C) had celebrated
(D) were celebrating
celebrating your last appointment with our clinic. If you contact the Penshurst Medical Practice
before September 15s, you will be able to take advantage _______ a free check up. In this

41. (A) from


(B) in
(C) of
(D) to

medical to we will check blood pressure, blood cholesterol, and blood sugar. For a small extra
charge. it is possible to have a more detailed examination. If you are ____ in this offer, please call
ng k hc: 0962 60 8801 04 6260 3948 29
a ch: S 18 Trn i Ngha Q Hai B Trng H ni
NGOI NG 24H 3
WWW.NGOAINGU24H.VN 0

42. (A) available


(B) wanting
(C) interested
(D) believe
the Medical Practice at your earliest convenience.

Sincerely,

Penshurst Medical Practice

Questions 43 through 46 refer to the following memo.


This semester, be prepared for your exams. Follow these handy hints from the Student Advice
Center.

Don't _______ until the night before an exam. The best students revise and learn early and regularly

43. (A) be wait


(B) waiting
(C) wait
(D) had waited

throughout the semester. Revise each night what you learned in class that day. Reread class notes,
assignments, handouts or previous tests and textbooks. _______ key words and points with a colored

44. (A) Highlight


(B) Notice
(C) Spotlight
(D) View

pen or pencil. It is very helpful to write summaries of the notes you take in class.

If that sounds like too much effort, then you start studying at least two weeks before _______ your

45. (A) had to


(B) should
(C) would
(D) shouldn't

exams. Make a study schedule and stick to it, but be realistic about your goals. Don't try to do too
much in one day. Resting is as _______ as studying.

ng k hc: 0962 60 8801 04 6260 3948 30


a ch: S 18 Trn i Ngha Q Hai B Trng H ni
NGOI NG 24H 3
WWW.NGOAINGU24H.VN 1

46. (A) important


(B) importance
(C) duty
(D) must

Questions 47 through 50 refer to the following information.


Choosing a college

_______ a college is one of the most important decisions you will have to make. The following

47. (A) To choice


(B) Chosen
(C) Choosing
(D) Having chosen

suggestions should help you _______ an informed decision. Visit the school you are considering.

48. (A) have


(B) suggest
(C) make
(D) perform

While you are at the school, take some time to look at the equipment and facilities. Talk to lecturers
and _______ students. Ask questions that will give you first-hand knowledge about the school:

49. (A) now


(B) current
(C) presently
(D) immediate

Do the instructors seem knowledgeable? What is the students' opinion of the instructors? What
_______ they like most and least about the school or program? Finally, look at several schools that

50. (A) do
(B) are
(C) had
(D) can

offer similar programs. Compare program length, schedule, cost, transferability of course credits,
financial aid availability, and any other factors that are important to you.

ng k hc: 0962 60 8801 04 6260 3948 31


a ch: S 18 Trn i Ngha Q Hai B Trng H ni
NGOI NG 24H 3
WWW.NGOAINGU24H.VN 2

NGY 3: NG T NGUYN MU

Ch 1
1. Lin t ca mnh trng ng ch thi gian
2. Lin t ca mnh trng ng ch iu kin
Ch 2
1. Trong mnh that theo sau nhng ng t ch yu cu, mnh lnh, ngh ta lun dng ng t
nguyn mu d hnh ng hin ti, qu kh hay tng lai.
2. Nhng tnh t ch s cn thit cng dng vi ng t nguyn mu.
Ch 3
1. Dng lm trng t ch mc ch
2. Dng lm ch ng tht v tn ng tht cho cu c i t it
Ch 4
1. Phn bit danh t v danh ng t
Ch 5
1. Cu trc i vi ng t nguyn mu c to
2. Cc ng t i vi ng t nguyn mu c to
3. Cc danh t i vi ng t nguyn mu c to
4. Cc cu trc i vi danh ng t
5. ng t i vi danh ng t

Bi tp luyn tp 1
VERB + V-ing/ To-V
1. Cc ng t + V-ing
2. Cc ng t + To V
3. Cc ng t + O + To V
Bi tp luyn tp 2
Bi tp kim tra

ng k hc: 0962 60 8801 04 6260 3948 32


a ch: S 18 Trn i Ngha Q Hai B Trng H ni
NGOI NG 24H 3
WWW.NGOAINGU24H.VN 3

Ch 1
1. Lin t ca mnh trng ng ch thi gian
when (khi)
before (trc khi)
after (sau khi)
as soon as (ngay khi)
while (trong khi)

Ex: Please dont forget to turn off the computer before you leave the office.
2. Lin t ca mnh trng ng ch iu kin
once (mt khi)
if (nu)
unless (tr phi)
Ex: If you return this survey, we will give you a 10% discount coupon.
Trong mnh trng ng ch thi gian v ch iu kin
Tng lai -> Hin ti n
Tng lai hon thnh -> Hin ti hon thnh

Quick quiz
1. If you ___________ any help to organize the meeting, please dont hesitate to ask me.
(A) needs (B) needed
(C) will need (D) need

Ch 2
1. Trong mnh that theo sau nhng ng t ch yu cu, mnh lnh, ngh ta lun dng
ng t nguyn mu d hnh ng hin ti, qu kh hay tng lai.
request, recommend, suggest, insist + that + ch ng + ng t nguyn mu
Ex: The accounting manager requested that all budget reports be submitted by Friday.

2. Nhng tnh t ch s cn thit cng dng vi ng t nguyn mu.


It is imperative, essential, important, necessary + that + ch ng + ng t nguyn mu
Ex: It is imperative that the facilities in the plant be checked regularly.

Quick quiz
2. The system engineer has requested that the existing computer system ________ as soon as
possible.
(A) upgrades (B) is upgraded
(C) be upgraded (D) upgraded

Ch 3
1. Dng lm trng t ch mc ch
Ex: To provide better service, we receive feedback from our customers.
ng t nguyn mu c to (to provide) c tc t l better service, c dng lm trng t ch mc
ch ( phc v khch hng tt hn).

2. Dng lm ch ng tht v tn ng tht cho cu c i t it

ng k hc: 0962 60 8801 04 6260 3948 33


a ch: S 18 Trn i Ngha Q Hai B Trng H ni
NGOI NG 24H 3
WWW.NGOAINGU24H.VN 4

a) Khi ch ng qu di, ngi ta thng dng ch ng gi it. Lc ny, ch ng tht l mnh danh
ng that hoc ng t nguyn mu c to
Ex: It is impossible for our team to meet the deadline.
Ch ng gi ch ng tht
b) Khi tc t qu di, ngi ta thng dng tn ng gi it. Lc ny, tn ng tht l ng t nguyn
mu c to.
Ex: This software program made it possible to reduce production time.
Tn ng gi tn ng tht
Quick quiz
3. In almost every business, it is really imperative __________ responsive to customers needs.
(A) to be (B) being
(C) is (D) that

Ch 4
1. Phn bit danh t v danh ng t
Danh ng t c chc nng l danh t trong cu v c th ng vai tr lm tn ng.
Danh ng t khc danh t cc im sau:
- Danh ng t khng c mo t nh danh t.
- Danh ng t c th c tc t theo sau (thng l danh t)
Ex:
Danh ng t
The management has decided to discontinue producing ladys hats.
Danh t
The management has decided to discontinue production of lady's hats.

Quick quiz
4. It is expected that we will begin ________ our new line of portable printers next month.
(A) production (B) productive
(C) producing (D) produce

Ch 5
1. Cu trc i vi ng t nguyn mu c to
be going to do be eligible to do
be able to do be unable to do
be sure to do in order to do
so as to do feel free to do
be liable to do be pleased to do
be delighted to do be likely to do
be supposed to do be designed to do
be reluctant to do be willing to do
would like to do be ready to do
hesitate to do

2. Cc ng t i vi ng t nguyn mu c to
propose/ intend/ plan/ decide/ fail to do/ want/ wish/ hope to do

3. Cc danh t i vi ng t nguyn mu c to
ability/ right/ way/ need to do

ng k hc: 0962 60 8801 04 6260 3948 34


a ch: S 18 Trn i Ngha Q Hai B Trng H ni
NGOI NG 24H 3
WWW.NGOAINGU24H.VN 5

4. Cc cu trc i vi danh ng t
be capable of ~ing succeed in ~ing
have difficulty ~ing be busy ~ing
upon ~ing be devoted to + Noun
be committed to + Noun/~ing look forward to +Noun/~ing
be entitled to + Noun
y, to l gii t nn sau to l danh t hoc danh ng t.

5. ng t i vi danh ng t
consider/ discontinue/ avoid/ finish/ suggest/ include/ keep ~ing

BI TP LUYN TP
Bi tp 1:
Part V: Chn t thch hp in vo ch trng.
1. Early last week, the city council formally ________ the downtown renewal project.
(A) approved (B) will approve
(C) approves (D) has approved

2. After considerable effort, the company has succeeded in ________ the new environment-
friendly home appliances.
(A) development (B) develops
(C) developed (D) developing

3. If commercial banks ________ mortgage rates, many households will have difficulty in
repaying interest.
(A) Will raise (B) raise
(C) to raise (D) raising

4. It is important for security guards ________ the security guidelines while they are on duty.
(A) following (B) to follow
(C) followed (D) follow

5. Some technical support workers ________ at the headquarters to learn how to operate the
new network access system over the past two months.
(A) trained (B) have trained
(C) have been trained (D) are trained

6. The newly installed security system _______ requires that every user the four-digit access
number.
(A) have (B) to have
(C) having (D) has

7. Construction workers are responsible for _______ their own equipment and tools.
(A) provides (B) providing
(C) provided (D) provision
Part VI: Chn p n ng cho cu 8 ~ 10 trong mu qung co sau y.
Want to sell your property?
Why only sell to local buyers?
Intemational-Properties.com offers the first international private property sales service.
ng k hc: 0962 60 8801 04 6260 3948 35
a ch: S 18 Trn i Ngha Q Hai B Trng H ni
NGOI NG 24H 3
WWW.NGOAINGU24H.VN 6

Our bilingual sales team enables the Intemational-Properties.com network to sell your property
directly to our database of international clients. If you ____________ to sell your property
internationally, we will be the best choice.
8. (A) will plan
(B) plans
(C) plan
(D) planned
We ___________ working as a specialist for international property over the past 20 years.
9. (A) were
(B) have been
(C) are
(D) will be
By ________ your property description to Intemational-Properties.com, your ad will be
10. (A) provide
(B) provision
(C) providing
(D) provided
promoted on our network until the sale of your property.

VERB + V-ing/ To-V


1. Cc ng t + V-ing
avoid (trnh) recollect (nh ra)
admit (tha nhn) recommend (nhc nh)
advise (khuyn nh) resent (bc tc)
appreciate (nh gi) resist (khng c)
complete (hon thnh) risk (ri ro)
consider (xem xt) suggest ( ngh)
delay (tr hon) tolerate (tha th)
deny (t chi) understand (hiu)
discuss (tho lun) cant help (ko th trnh/ nhn c)
dislike (khng thch) cant stand (ko th chu ng c)
enjoy (thch) cant bear (ko th chu ng c)
finish (hon thnh) It is no use/ It is no good (v ch)
keep (tip tc) would you mind (c phin...ko)
mention ( cp) to be used to (quen vi)
mind (phin, ngi) to be/ get accustomed to (dn quen
miss (nh, b l) vi)
postpone (tr hon) to be busy (bn rn)
practice (luyn tp) to be worth (xng ng)
quit (ngh, thi) to look forward to (trng mong)
recall (nhc nh, nh)
2. Cc ng t + To V
agree (ng ) consent (bng lng)
appear (xut hin) decide (quyt nh)
arrange (sp xp) demand (yu cu)
ask (hi, yu cu) deserve (xng ng)
beg (ni n, van xin) expect (mong i)
care (chm sc) fail (tht bi)
claim (i hi, yu cu) hesitate (do d)
ng k hc: 0962 60 8801 04 6260 3948 36
a ch: S 18 Trn i Ngha Q Hai B Trng H ni
NGOI NG 24H 3
WWW.NGOAINGU24H.VN 7

hope (hi vng) refuse (t chi)


learn (hc) wish (mong)
manage (sp xp) seem (dng nh)
mean ( nh) struggle (u tranh)
need (cn) swear (xin th)
offer ( ngh) threaten (e do)
plan (ln k hoch) volunteer (tnh nguyn)
prepare (chun b) wait (i)
pretend (gi v) want (mun)
promise (ha) afford ( kh nng)
3. Cc ng t + O + To V
advise (khuyn)
allow (cho php)
ask (yu cu)
beg (van xin)
cause (gy ra)
challenge (thch thc)
convince (thuyt phc)
dare (dm)
encourage (khuyn khch)
expect (mong i)
forbid (cm)
force (buc)
hire (thu)
instruct (hng dn)
invite (mi)
need (cn)
order (ra lnh)
permit (cho php)
persuade (thuyt phc)
remind (nhc nh)
require (i hi)
teach (dy)
tell (bo)
urge (thc gic)
want (mun)
warn (bo trc)

ng k hc: 0962 60 8801 04 6260 3948 37


a ch: S 18 Trn i Ngha Q Hai B Trng H ni
NGOI NG 24H 38
WWW.NGOAINGU24H.VN

FORGET, REMEMBER
+ Ving: Nh (qun) chuyn lm
I remember meeting you somewhere last year
+ To inf:
Nh (qun) lm chuyn g
Don't forget to buy me a book.

TRY
+ Ving: th lm g
I try eating the cake he makes
+ To inf: c gng ...
I try to avoid meeting him

REGRET
+ Ving: hi hn chuyn lm
I regret lending him the book
+ To inf: ly lm tic ......
I regret to tell you that ... ( ti ly lm tic ni vi bn rng...) - cha ni - by gi mi ni

STOP
Stop + to inf: Dng lm g
After Id been working for 3 hours, I stopped to eat lunch.
Stop + Ving: Dng vic g li
We stop eating meat 5 years ago.

MEAN
Mean + to inf: D nh
I mean to go out
Mean + Ving: mang ngha
Failure on the exam means having to learn one more year.

NEED
1.1 Need dng nh mt ng t thng:
a) ng t i sau need ch dng nguyn th khi ch ng l mt vt th sng:
Ex:
- My friend needs to learn Spanish.
He will need to drive alone tonight.
John needs to paint his house.
b) ng t i sau need phi dng verb-ing hoc dng b ng nu ch ng khng phi l vt th
sng.
Ex: The grass needs cutting
OR The grass needs to be cut.
The telivision needs repairing
OR The TV needs to be repaired.
Your thesis needs rewriting
OR Your thesis needs to be rewritten.
1.2 Need dng nh mt tr ng t

ng k hc: 0962 60 8801 04 6260 3948 38


a ch: S 18 Trn i Ngha Q Hai B Trng H ni
NGOI NG 24H 39
WWW.NGOAINGU24H.VN

Ch dng th nghi vn hoc ph nh thi hin ti. Ngi th ba s t khng c "s" tn cng. Khng
dng vi tr ng t to do. Sau need (tr ng t) l mt ng t b to:
Ex: We needn't reserve seats - there will be plenty of rooms. Need I fill out the form?

Bi tp 2:
1. Even though assigned tasks are somewhat complex, we will have to complete them before
we_____ for the day.
a. leave b. leaving
c. to leave d. have left
2. Due to_____ fuel prices, the company has decided to add a surcharge to all
deliveries.
a. rise b. arisen
c. rose d. rising
3. Speakers should be prepared_____ their finding to the audience and be able to answer questions
regarding them.
a. has presented b. presenting
c. present d. to present
4. Abigail Sanchez is going to attend a conference in Hamshire tomorrow.
She remembers_____ there last year.
a. to come b. came
c. to coming d. coming
5. Please remember_____ your identification and sufficient documents
with you when you come here to register.
a. to bring b. to bringing
c. bringing d. being brought
6. Mr. Michael needs_____ all the CVs of applicants before contacting
them for interviewing.
a. review b. to review
c. reviewing d. to be reviewed
7. The first solar-energy motorbike of Apo Motors needs_____ carefully before introducing to the
public next month.
a. test b. to test
c. testing d. to be testing
8. One of the most effective methods to protect precious animals is that people should stop_____
them for leather and feather.
a. hunt b. to hunt
c. hunting d. be hunted
9. According to the schedule, after leaving the National Park, the visitors
will stop_____ the view beside the Sachihiro River.
a. to enjoy b. to be enjoyed
c. enjoying d. be enjoying
10. Auto-car manufactory tried_____ the fixed cost and variable cost
to increase the profit.
a. to cut b. to be cut
c. be cutting d. cutting
11. In order to ensure that the medication does not have side effects, pharmacists have tried_____
it on a variety of animals.
a. to experiment b. experimenting
c. to be experimented d. experimented
ng k hc: 0962 60 8801 04 6260 3948 39
a ch: S 18 Trn i Ngha Q Hai B Trng H ni
NGOI NG 24H 40
WWW.NGOAINGU24H.VN

12. In financial terms, IPO( initial public offering) means_____ a companys shares to the public for
the first time and then the stock is said to be listed on the stock exchange.
a. sell b. to sell
c. selling d. be sold
13. The full version of Salems newest song that meant_____ was leaked.
a. to launch b. to be launched
c. launching d. be launched
14. Ms. Saffron regrets_____ protective gloves when using this cleaning
liquid. Serious burns have resulted from direct contact with her skin.
a. to wear b. wearing
c. not to wear d. not wearing
15. Mrs. Happy, the founder of Happys Pie Hut, regrets_____ that she will retire and leave her
business next year
a. to inform b. informing
c. to be informed d. being informed

ng k hc: 0962 60 8801 04 6260 3948 40


a ch: S 18 Trn i Ngha Q Hai B Trng H ni
NGOI NG 24H 41
WWW.NGOAINGU24H.VN

BI TP THC HNH
Bi tp 1:
Choose one correct option in the parentheses.
01 We hope (gain, to gain) a greater market share
02 Our goal is (to attract, attraction) more customers.
03 Simons mistake was (to forget, forget) the date of the conference.
04 I am writing (inquire, to inquire) whether my order was shipped.
05 She is beginning (prepare, to prepare) the company s anniversary celebration
06 We will need (to enhance, enhancement) the security of our website.

Choose the correct option for each of the following sentences.


07 The board members unanimously agreed _____ Mr. Webers suggestion.
(A) accept (C) accepted
(B) to accept (D) acceptance
08 Some residents wish _____ their phone numbers after they move.
(A) to keep (C) kept
(B) keep (D) keeper
09 One way of boosting profits is _____ the cost of production.
(A) reduced (C) to reduce
(B) reduce (D) reduction
10 The accounting department may request receipts _____ your expenses.
(A) verifying (C) be verified
(B) verify (D) to verify

Bi tp 2:
Choose one correct option in the parentheses.
01 Many people prefer (invest, to invest) in the IT industry.
02 (To update, Update) our website is my main responsibility.
03 Our policy is (donation, to donate) money to charities every year
04 He has a presentation (to make, make) on sales strategies.
05 (To protect, Protection) your skin, you should use UltraCare sunblock.
06 She called (to reschedule, reschedule) her appointment with Mr. Lee.

Choose the correct option for each of the following sentences.


07 _____ the facilities before the upcoming event, the firm required several staff members to work
overtime.
(A) To inspect (C) Inspection
08 The meeting's purpose is _____ improvements in employee benefits.
(A) discuss (C) discussion
(B) will discuss (D) to discuss
09 Every company has an obligation _____ its workers with a safe and healthy work atmosphere.
(A) provide (C) provision
(B) provides (D) to provide
10 Mr. McCarran checked the advertisements in several newspapers _____ for a used digital camera.

ng k hc: 0962 60 8801 04 6260 3948 41


a ch: S 18 Trn i Ngha Q Hai B Trng H ni
NGOI NG 24H 42
WWW.NGOAINGU24H.VN

(A) of looking (C) look


(B) to look (D) looks

Bi tp 3:
Choose one correct option in the parentheses.
01 The presenter forced attendees (to set, setting) yearly goals.
02 Bigtown, Inc. permits employees (moving, to move) to other branches.
03 The corporation wishes (relocating, to relocate) outside New York.
04 The manager promised (to lower, lowering) the sales quotas tor new dealers.

Choose the correct option for each of the following sentences.


05 We would like to remind customers _____ this device cautiously.
(A) handling (C) handle
(B) handled (D) to handle
06 The team failed ____ the project, so they did not receive bonuses.
(A) complete (C) completes
(B) to complete (D) completed
Questions 7 and 8 refer to the following notice.

Date: August 10
Subject: Employee Awards Night

We have scheduled the ceremony _____ awards to the company's employees of the year for

01 (A) present (C) to present


(B) presents (D) presented

August 18. There will be a rehearsal on August 16, so please ask Jane Williams and Ryan Monk to
report to the second-floor conference hall at 5:00 p.m. My secretary plans _____ out the invitations to

02 (A) to mail (C) is mailing


(B) mailed (D) mail

the employees and their families soon.

Bi tp 4:
Decide which of the choices - (A), (B), (C), or (D) - best completes the sentence
1. We call the mall ____ out whether the produce were sold out out.
(A) to finding
(B) for finding
(C) to find
(D) find
2. Mr. Taylor has the ability____ people.
ng k hc: 0962 60 8801 04 6260 3948 42
a ch: S 18 Trn i Ngha Q Hai B Trng H ni
NGOI NG 24H 43
WWW.NGOAINGU24H.VN

(A) persuading
(B) persuade
(C) to persuade
(D) persuasion
3. All the employees were asked ____ the meeting on Sunday.
(A) attend
(B) attended
(C) attending
(D) to attend
4. Ms. Watson tried hard to help the customers only ____ her.
(A) upset
(B) have upset
(C) upsetting
(D) to upset
5. _____ sales, we decided to advertise our product in the local newspaper.
(A) Increase
(B) Increasing
(C) Increased
(D) To increase
6. They finished the research but failed ____ the results.
(A) reporting
(B) to report
(C) report
(D) for reporting
7. The company made every effort _____ customer satisfaction.
(A) boost
(B) boosting
(C) to boost
(D) for boosting
8. I am very pleased ___ you of our decision.
(A) inform
(B) to inform
(C) to be informed
(D) information
9. The purpose of this meeting is____ an agreement on our marketing strategy.
(A) to reach
(B) reach
(C) reached
(D) to reaching
10. We believe that Jason is ready _____ with others.
(A) cooperate
(B) cooperating
(C) to cooperate
ng k hc: 0962 60 8801 04 6260 3948 43
a ch: S 18 Trn i Ngha Q Hai B Trng H ni
NGOI NG 24H 44
WWW.NGOAINGU24H.VN

(D) of cooperating
11. You have the right ___ for a refund.
(A) asking
(B) to ask
(C) of asking
(D) ask
12. They tried to ____ the disease from spreading.
(A) deal
(B) comment
(C) prevent
(D) apologize
13. Many companies took ___ of the recent rising oil prices to raise the prices of their products.
(A) point
(B) exposure
(C) advantage
(D) means
14. We have to take ____ to deal with the problems.
(A) foots
(B) steps
(C) stairs
(D) feet

Questions 15-16 refer to the following letter.

Mr. David Hornsby


190 Broadway
Vancouver, BC

Dear Mr. Hornsby,

I am writing this letter _____ that you will start working for Big Office Supplies on April 12. Your

15. (A) confirm


(B) to confirm
(C) confirming
(D) for confirming

job duties will be explained during the training session.

As we discussed before, you will be able ____ twenty days of vacation and three sick days. If you

16. (A) have

ng k hc: 0962 60 8801 04 6260 3948 44


a ch: S 18 Trn i Ngha Q Hai B Trng H ni
NGOI NG 24H 45
WWW.NGOAINGU24H.VN

(B)having
(C) to having
(D) to have

have any questions, please let us know.

Bi tp 5:
Decide which of the choices - (A), (B), (C), or (D) best completes the sentence.
1. ____ the restaurant will attract more customers.
(A) Renovation
(B) Renovating
(C) Renovated
(D) To renovating
2. The newly hired secretary is good at _____
(A) organizing
(B) to organize
(C) organize
(D) organization
3. The president suggested ____ a new staff lounge to better serve the employees.
(A) to build
(B) build
(C) built
(D) building
4. We would like to thank you for ____ in our annual conference.
(A) participate
(B) participating
(C) to participate
(D) participated
5. Mr. Carter has had difficulty ____ experienced research specialists since last month.
(A) find
(B) finding
(C) to find
(D) found
6. Because of the bad economic situation, we could not help ___ our New York branch.
(A) close
(B) closed
(C) closing
(D) to close
7. You can simply book your ticket by ___ our website or dropping by one of our stores.
(A) visits
(B) visiting
(C) visit

ng k hc: 0962 60 8801 04 6260 3948 45


a ch: S 18 Trn i Ngha Q Hai B Trng H ni
NGOI NG 24H 46
WWW.NGOAINGU24H.VN

(D) to visit
8. We regret ___ you that your application has been rejected.
(A) to tell .
(B) told
(C) telling
(D) tells
9. They stopped ____ the old model in order to promote a newly released one.
(A) to sell
(B) selling
(C) sold
(D) sell
10. ____ the number of security guards will hopefully, reduce the chance of thefts.
(A) Increase
(B) Increasing
(C) Increased
(D) To increasing
11. Atlantis Co. strictly prohibits its employees from ____ an office phone for personal reasons.
(A) use
(B) to use
(C) used
(D) using
12. The company has considered ____ its branch to Boston for the past few weeks.
(A) relocation
(B) to relocate
(C) relocating
(D) relocated
13. The ____ a simple survey to find out about their customers needs
(A) talked
(B) conducted
(C) filled
(D) notified
14. It usually takes a lot of time and effort to ___ to a new environment.
(A) accustom
(B) refer
(C) postpone
(D) adjust

Questions 15-16 refer to the following letter.

Dear Mr. Myer,

According to our records, your membership with California Fitness will expire in a few weeks. We

ng k hc: 0962 60 8801 04 6260 3948 46


a ch: S 18 Trn i Ngha Q Hai B Trng H ni
NGOI NG 24H 47
WWW.NGOAINGU24H.VN

would like to ____ you with a special offer. With your renewal of your membership, you can

15. (A) provide


(B) schedule
(C) cancel
(D) approve
continue ____ our facilities and services at a discounted price.

16. (A) use


(B) used
(C) using
(D) uses

Sincerely,
Jane Miller

BI TP KIM TRA
1. The Trattoria Restaurant requests that patrons _______ reservations for a table at least two weeks
ahead of time.
(A) makes
(B) making
(C) made
(D) make
2. The manufacturer insists that the defective air bags_______ replaced in spite of the high cost of
recalling thousands of affected cars.
(A) are
(B) be
(C) have
(D) has
3. Due to the urgency of this matter, it is imperative that Mr. Lambert _______one of our customer
representatives by 5 p.m.
(A) is contacting
(B) will contact
(C) contact
(D) contacted
4. While Harman's used to _______imported furniture, it is now sticking to local products to avoid
the hassles of customs.
(A) have sold
(B) sold
(C) sell
(D) selling
5. It is essential that no unauthorized persons _______ into the building once it has been locked by
the security personnel.
(A) admitted
(B) admit
(C) be admitted

ng k hc: 0962 60 8801 04 6260 3948 47


a ch: S 18 Trn i Ngha Q Hai B Trng H ni
NGOI NG 24H 48
WWW.NGOAINGU24H.VN

(D) admittance
6. Financial advisers suggest that setting aside cash for emergencies _______it easier-for the first-
time investor to recover from losses.
(A) makes
(B) be made
(C) make
(D) made
7. The notice on the community bulletin board requested that the purse_______ to its owner and
stated that a small reward would be given.
(A) returns
(B) returning
(C) be returned
(D) to return
8. At the meeting, we will _______ about the problem concerning the misuse of company telephones
and fax machines.
(A) object
(B) oppose
(C) talk
(D) discuss
9. Once all the proposals have been submitted. we will _______ which company is best suited for the
new construction project.
(A) determination
(B) determine
(C) determined
(D) determines
10. The canned chicken soup manufacturer insists that all the cans _______, regardless of losses to
the company.
(A) recall
(B) recalling
(C) be recalled
(D) are recalled
12. A test was given to the applicants to eliminate those who did not _______ the basic knowledge
requirements for the job.
(A) meet
(B) meeting
(C) be met
(D) to meet
13. The final report should _______ on the more recent findings rather than on the data collected a
few months ago.
(A) base
(B) based
(C) be based
(D) to be based
14. Only after one of the employees expressed concern did they _______ investigating harassment
claims against the supervisor.
(A) started
(B) starting
(C) starts
(D) start
ng k hc: 0962 60 8801 04 6260 3948 48
a ch: S 18 Trn i Ngha Q Hai B Trng H ni
NGOI NG 24H 49
WWW.NGOAINGU24H.VN

15. Even though assigned tasks are somewhat complex, we will have to complete them before we
_______ for the day.
(A) leave
(B) leaving
(C) to leave
(D) have left
16.. Many researchers are doing research on how to reduce the time it takes _______ and forget an
accident.
(A) will overcome
(B) be overcome
(C) to overcome
(D) has overcome
17. Speakers should be prepared _______ their findings to the audience and be able to answer
questions regarding them.
(A) has presented
(B) presenting
(C) present
(D) to present
18. Please _______ any information, no matter how insufficient it may seem, to your client in the
very near future.
(A) forwards
(B) forward
(C) forwarding
(D) to forward
19.. So as to _______ that your medical treatment will be paid for, you must present a note from
your doctor.
(A) ensure
(B) ensuring
(C) ensured
(D) be ensured
20. The employees in the planning department spend most of their time _______ proposals for a
better management structure.
(A) write
(B) writing
(C) written
(D) for writing
21. He expects _______ soon.
(A) arrive
(B) arrival
(C) to arrive
(D) arriving
22. All I want is _______ to return safe.
(A) he
(B) him
(C) of him
(D) for him
23. I'm sorry about _______ their feelings.
(A) hurt
(B) to hurt
ng k hc: 0962 60 8801 04 6260 3948 49
a ch: S 18 Trn i Ngha Q Hai B Trng H ni
NGOI NG 24H 50
WWW.NGOAINGU24H.VN

(C) hurting
(D) I hurt
24. Christine promised to meet _______ at the theater.
(A) our
(B) us
(C) we
(D) ours
25. He is making every possible effort to _______ his opponent.
(A) best
(B) well
(C) good
(D) better
26. When _______ to resign his position, the manager reacted badly.
(A) ask
(B) to ask
(C) asking
(D) asked
27. When we arrived, the film was about _______.
(A) start
(B) starting
(C) to start
(D) started
28. The prince was married to the Duchess of Kent and _______ by his brother.
(A) to succeed
(B) succeeded
(C) succeeding
(D) successfully
29. Who is responsible for _______ the dishes tonight?
(A) do
(B) to do
(C) doing
(D) will do
30. She forced him _______ the work.
(A) does
(B) do
(C) did
(D) to do
31. We _______ to inform you that the position has been filled.
(A) sorry
(B) apology
(C) apologize
(D) regret
32. Would you mind _______ the window?
(A) to open
(B) opening
(C) to opening
(D) if opening
33. Does she remember _______ the report to the secretary last week?
(A) give
ng k hc: 0962 60 8801 04 6260 3948 50
a ch: S 18 Trn i Ngha Q Hai B Trng H ni
NGOI NG 24H 51
WWW.NGOAINGU24H.VN

(B) to give
(C) gives
(D) giving
34. They needed to practice _______ the ball.
(A) catch
(B) catching
(C) catches
(D) caught
35. Frank has been really busy _______ the new products ready for the exhibition.
(A) getting
(B) get
(C) to get
(D) to getting
36. She asked which chapter _______.
(A) read
(B) reads
(C) to read
(D) reading
37. I enjoyed _______ in the park with Jane this afternoon.
(A) walk
(B) walking
(C) to walk
(D) to walking
38. I'm looking forward _______ her tomorrow.
(A) to seeing
(B) to see
(C) seeing
(D) see
39. It was necessary that she _______ her father the truth.
(A) tell
(B) to tell
(C) tells
(D) does tell

Question 40 through 41 refer to the following advertisement.


A smoke detector is an easy way to protect your family. It is very easy to install one. Most people
expect _______ if there is a fire, but poisonous fumes from smoke kill
40. (A) to wake up
(B) woke up
(C) wake up
(D) waking up
hundreds of people every year. _______ a smoke detector will give your family a chance to escape in
the case of a fire. Ask at your local fire station for advice.
41. (A) Putting
(B) Fixing
(C) Installing
(D) Hanging
Question 42 through 45 refer to the following letter.
Hi Mom,
ng k hc: 0962 60 8801 04 6260 3948 51
a ch: S 18 Trn i Ngha Q Hai B Trng H ni
NGOI NG 24H 52
WWW.NGOAINGU24H.VN

You were _______ when I left, so I'm leaving this note for you to read. I am going to
42. (A) off
(B) out
(C) over
(D) in
spend the night at Anna's, so don't wait up for me. I have also taken the spare keys from the hook in
the kitchen, so don't be _______ to find them missing. I can't find my
43. (A) surprising
(B) surprised
(C) surprise
(D) to surprise
keys - don't worry, because I know they are in the house somewhere. I used them to let myself in
earlier today. My room is ______ mess that I can't find them in there. I'll be sure
44. (A) such a
(B) such
(C) so
(D) too

to tidy up when I get back tomorrow evening. I know that you hate my room being so messy.
Anyway. I'll see you tomorrow. Hope you managed _______the coat you wanted.
45. (A) buy
(B) buying
(C) bought
(D) to buy
Love,
Melanie

Questions 46 through 49 refer to the following notice.


Notice to all Movie-rama movie house managers.
Now that the summer movie season is approaching, we at Movie-rama would like to remind all of
our managers that food and drink sales should _______ by 200%.
46. (A) increase
(B) decrease
(C) stay the same
(D) be better
Especially in the case of cola and flavored water, sales should increase by 300%. To achieve the
intended sales goals, we recommend putting _______ salt on the popcorn,
47. (A) salty
(B) less
(C) more
(D) no
which will make customers more thirsty, which, in turn, will increase drink sales. Also, be reminded
that Movie-rama movie theatres are now selling Sugar Cone ice cream bars. The suggested selling
price is two dollars, but each theatre can set their own_______.
48. (A) cost
(B) price
(C) ice cream
(D) movies
Thank you all, and have a _______ summer.
ng k hc: 0962 60 8801 04 6260 3948 52
a ch: S 18 Trn i Ngha Q Hai B Trng H ni
NGOI NG 24H 53
WWW.NGOAINGU24H.VN

49. (A) hot


(B) terrible
(C) wonderful
(D) big

The Management

NGY 4: TH B NG

Ch 1: Lm quen vi th b ng
Ch 2: Hnh thc ch ng v b ng cc th
Ch 3: Ng php nng cao

Bi tp luyn tp
Bi tp kim tra
Ch 1
1.1: Lm quen vi th b ng
He delivered the document in person.
S V O

The document was delivered by him in person.


So be + PII by + ch th hnh ng (S)
ch ng ca th ch ng (S) i thnh by + ch th hnh ng

1.2: Exercise
1. You must [sign/ be signed] the employment contract.
2. The employment contract must [sign/ be signed] by you.
3. The sales department will [hold/ be held] a marketing seminar.
4. The marketing seminar will [hold/ be held] by the sales department.
Ghi ch
1. Ch ng trong cu b ng l tn ng ca ngoi ng t trong cu ch ng.
You must sign the employment contract.
2. Ni ng t khng i hi c tn ng nn khng bao gi c trng hp cu b ng vi ni ng
t.
The price increases rapidly.
Ch ng => tr ng t + ng t nguyn mu
B ng => tr ng t + be + qu kh phn t

BI TP LUYN TP
Bi tp 1:
1. The manager will _________ the new safety guidelines.
(A) distributing (B) distribute
(C) distributed (D) distribution

ng k hc: 0962 60 8801 04 6260 3948 53


a ch: S 18 Trn i Ngha Q Hai B Trng H ni
NGOI NG 24H 54
WWW.NGOAINGU24H.VN

2. The parts could not _________ to the factory by next week.


(A) be shipped (B) shipping
(C) ship (D) shipment
3. Attendance records should _________ to the personnel department.
(A) submission (B) submit
(C) submitting (D) be submitted
4. The contents of this proposal can _________ by the manager.
(A) be revised (B) revise
(C) revision (D) revised
5. The company will _________ its staff by 20 percent.
(A) be reduced (B) reducing
(C) reduce (D) reduction

Ch 2
Hnh thc ch ng v b ng cc th

Th Th ch ng Th b ng V d
Hin ti S + V (s/es) + O S + am/is/are + PII Ex: The manager sends a memo to all
n + distributors.
by + O -> A memo is sent to all distributors
by the manager.

Hin ti S + am/is/are + Ving S + am/is/are + Ex: The manager is sending a memo


tip din +O being + PII + by + to all distributors at the moment.
O -> A memo is being sent to all
distributors by the manager at the
moment.

Hin ti S + have/has + Ex: The manager has sent a memo to


hon thnh S + have/has been + PII + by + all distributors.
+ PII + O O -> A memo has been sent to all
distributors by the manager.

Qu kh S + Ved/ P-I + O S + was/were + PII Ex: The manager sent a memo to all
n + by + O distributors yesterday.
-> A memo was sent to all
distributors by the manager
yesterday.

Qu kh S + was/were S + was/were + Ex: The manager was sending a


tip din + Ving + O being memo to all distributors this time
+ PII + by + O yesterday.
-> A memo was being sent to all
distributors by the manager this time
yesterday.

ng k hc: 0962 60 8801 04 6260 3948 54


a ch: S 18 Trn i Ngha Q Hai B Trng H ni
NGOI NG 24H 55
WWW.NGOAINGU24H.VN

Qu kh S + had + PII + O S + had + been Ex: The manager had sent a memo to
hon thnh + PII + by + O all distributors before the electricity
went off.
-> A memo had been sent to all
distributors by the manager before the
electricity went off.

Tng lai S + will/ shall S + will/ shall + be Ex: The manager will send a memo to
n +V+O + PII + by + O all distributors tomorrow.
-> A memo will be sent to all
distributors by the manager
tomorrow.

Tng lai S + will/ shall + S + will/ shall + Ex: The manager will have sent a
hon thnh have + PII + O have + been + PII memo to all distributors by next
+ by + O Monday.
-> A memo will have been sent to all
distributors by the manager by next
Monday.

Tng lai S + is/am/are + S + is/am/are + Ex: The manager is going to send a


gn going to + V + O going to + be + PII memo to all distributors this holiday.
+ by + O -> A memo is going to be sent to all
distributors by the manager this
holiday.

ng t S + modal verbs S + modal verbs + Ex: The manager must send a memo
khuyt +V+O be to all distributors.
thiu + PII + by + O -> A memo must be sent to all
distributors by the manager.

Bi tp 2:
1. The company has [reduced/ been reduced] its production cost.
2. Its production cost has [reduced/ been reduced] by the company.
3. The secretary has [distributed/ been distributed] this pamphlet.
4. This pamphlet has [distributed/ been distributed] by the secretary.

Bi tp 3:
1. The proposal has _________ by my secretary.
(A) revised (B) revising
(C) revise (D) been revised

2. The accounting manager has _________ all budget reports.


(A) reviewing (B) reviewed
(C) been reviewed (D) reviews

ng k hc: 0962 60 8801 04 6260 3948 55


a ch: S 18 Trn i Ngha Q Hai B Trng H ni
NGOI NG 24H 56
WWW.NGOAINGU24H.VN

3. The companys travel budget has _________ substantially.


(A) reduced (B) reduce
(C) been reduced (D) reducing

4. The researchers have _________ a new vaccine.


(A) been developed (B) developing (C) developed (D) develops

5. They have consistently _________ quality services and products.


(A) provides (B) been provided (C) providing (D) provided
CH 3
Ng php nng cao
Xc nh nhanh th ch ng/ b ng thng qua tn ng
1. Hnh thc cu ch ng
ch ng + ng t + tn ng
Ex:
The manager sent a memo to all distributors.
-> A memo was sent to all distributors by the manager.
Tn ng ca cu ch ng tr thnh ch ng trong cu b ng
cu b ng khng c tn ng.

Do , c th on nhanh cu ch ng v cu b ng thng qua s tn ti ca tn ng.

2. be + ... + (cm) danh t: c tn ng sau nn l cu ch ng.


t in vo ch trng l ngoi ng t dng hin ti phn t.

Ex: He is inviting all his friends to the party.

be + ... + (cm) trng t: khng c tn ng (danh t) sau nn l cu b ng


=> t in vo ch trng l ngoi ng t dng qu kh phn t.
Ex: All his friends are being invited to the party
Quick Quiz
1. Please remember that all application materials should ________ by next Friday.
(A) send (B) be sending (C) be sent (D) sending

2. Money refunds will ______ into your account within 7 days of your claim.
(A) deposited (B) to deposit (C) depositing (D) be deposited

Bi tp 4:
Part V: Chn t thch hp in vo ch trng di y.
1. The new employee handbook was ________ to all employees.
(A) distributed (B) distributing (C) distributes (D) distribute

2. You should ________ the rent before the deadline.


(A) paid (B) be paid (C) pay (D) paying

3. The research and development budget has ________ substantially.


(A) reduced (B) reduces (C) been reduced (D) be reduced

4. The president has ________ a new strategy to increase sales.


(A) proposing (B) proposed (C) been proposed (D) propose
ng k hc: 0962 60 8801 04 6260 3948 56
a ch: S 18 Trn i Ngha Q Hai B Trng H ni
NGOI NG 24H 57
WWW.NGOAINGU24H.VN

5. The terms of the contract must ________ carefully.


(A) be reviewing (B) review (C) reviewing (D) be reviewed

Part VI in vo ch trng trong mu qung co sau.


Teen Ice Cream Party & Game Night
Thursday August 31 @ 6:30 P.M.
All teens and their friends are __________ for an End-of-the-Summer Ice Cream Party and
6. (A) invite
(B) invited
(C) inviting
(D) invites
Game Night. All the food for the party will __________, along with cards and various board games.
7. (A) provide
(B) provides
(C) provided
(D) be provided
Registration is __________ to join the event.
8. (A) requiring
(B) requires
(C) require
(D) required
Bi tp 5:
Part V: Chn t thch hp in vo ch trng.
1. Though a great deal of money had ___________, the project was a failure.
(A) invested (B) be invested (C) been invested (D) invest

2. Public buildings throughout the country will soon __________ a no-smoking policy.
(A) implement (B) implementation (C) be implemented (D) implementing

3. It is a great pleasure to inform you that your company has __________ as our new supplier.
(A) selecting (B) selected (C) be selected (D) been selected

4. If you require additional information, __________ one of our sales representatives.


(A) contacting (B) contact (C) contacts (D) contacted

5. Because of a mechanical problem, the replacement parts could not __________ by tomorrow.
(A) shipped (B) be shipped (C) be shipping (D) ship

6. A special luncheon will be held in honor of the sales department, which has __________
monthly sales goals.
(A) reached (B) been reached (C) reaching (D) reaches

7. According to a new policy, overnight camping in all national parks is no longer __________.
(A) permits (B) permitting (C) permitted (D) permission

8. Sales personnel ____extra office supplies should get permission from their supervisor.
(A) purchasing (B) to purchase (C) purchased (D) have purchased

9. Beginning next month, a technician will ____new security cameras on every floor of the building.
(A) install (B) be installed (C) installing (D) installed

ng k hc: 0962 60 8801 04 6260 3948 57


a ch: S 18 Trn i Ngha Q Hai B Trng H ni
NGOI NG 24H 58
WWW.NGOAINGU24H.VN

10. A number of large corporations have recently been _____ considerable downsizing.
(A) undergo (B) undergone (C) underwent (D) undergoing

11. Tickets to Bob Mollen's charity concert will ____online starting next Tuesday.
(A) have sold (B) selling (C) be sold (D) have been selling

12. Applications for bank mortgages must ____by the end of this month.
(A) to receive (B) be received (C) received (D) receiving

13. Overall company profits at Kaplin Tech, Inc. ____ steadily for the last five years.
(A) risen (B) were risen (C) have risen (D) have been risen

14. The revisions ____ by the senior accountant should be included in the following budget report.
(A) were made (B) was made (C) to make (D) made

Part VI: Chn p n ng cho cu 15 ~ 17 trong thng bo sau y.


Enrollment policies:
Requests to withdraw from a course must ________ in writing via email before your allotted
15. (A) receive
(B) be receiving
(C) receiving
(D) be received
time has expired.
___________ an email to registration@utuniversity.edu requesting a withdrawal.
16. (A) Sending
(B) Send
(C) To send
(D) Sends
You will need to include your name, the course title and the course section number. Following the
proper withdrawal procedure will ensure an appropriate grade assignment.
No extensions will __________ for this course.
17. (A) grant
(B) have granted
(C) be granted
(D) have been granting

ng k hc: 0962 60 8801 04 6260 3948 58


a ch: S 18 Trn i Ngha Q Hai B Trng H ni
NGOI NG 24H 59
WWW.NGOAINGU24H.VN

BI TP THC HNH
Bi tp 1:
Choose one correct option in the parentheses.
01 Our appliances will (be serviced, service) for free.
02 We (convey, are conveyed) gratitude to loyal customers.
03 The CEO (was paid, paid) a bonus to the employees at the end of the quarter.
04 Scientists (argue, are argued) that greenhouse gas emissions are a major cause of global warming.
05 Management (required, were required) to develop a good relationship with the new partners.
06 The process of filling prescriptions should (change, be changed) to prevent mistakes.

Choose the correct option for each of the following sentences.


07 The current tenants must ___ the offices by the end of June.
(A) vacant (C) vacate

(B) vacancy (D) be vacated


08 The company's future sales may ____ by the growth of its competitors.
(A) affecting (C) be affecting
(B) be affected (D) affect
09 The airline industry by a government body before 1979.

(A) regulates (C) was regulated


(B) regulation (D) was regulating
10 The finance manager ____ that the company spent too much money on office equipment.
(A) is concluded (C) to conclude
(B) concluded (D) conclusion

Bi tp 2:
Choose one correct option in the parentheses.
01 The Direct-Help Organization is dedicated (to, at) helping people.
02 The firm is engaged (in, for) buying and selling real estate.
03 New car models are equipped (by, with) anti-theft locks and air bags.
04 The development team was (pleased, pleasing) with the performance of the new diesel engine.

Choose the correct option for each of the following sentences.


05 People living near manufacturing plants are exposed ____ various pollutants.
(A) to (C) for
(B) by (D) at
06 The report said that many health problems are ___ to a lack of exercise.
(A) relate (C) related
(B) relating (D) relation

Questions 7 and 8 refer to the following article.

ng k hc: 0962 60 8801 04 6260 3948 59


a ch: S 18 Trn i Ngha Q Hai B Trng H ni
NGOI NG 24H 60
WWW.NGOAINGU24H.VN

The Internet is now available on some of the largest planes of Bobkin Airlines. The service ____

07 (A) is allowed (C) allowing


(B) to allow (D) allows

passengers to connect to the Internet and utilize various web services, including e-mail. Sean Monk,
the president of Bobkin, said in a press conference that passengers, especially those on business,
____ with its new service. He said it marks a first in the travel industry

08 (A) are satisfied (C) satisfied


(B) are satisfying (D) satisfying

Bi tp 3:
Decide which of the choices - (A), (B), (C), or (D) -best completes the sentence.

1. A lot of errors ____ in the final report.


(A) were found
(B) found
(C) find
(D) be found
2. The company is faced ___ a financial crisis.
(A) in
(B) from
(C) with
(D) on
3. The annual conference ___ in the Schicago Convention Center
(A) holing
(B) holds
(C) is held
(D) to hold
4. A new assistant can ___ within
(A) a month.
(B) be found
(C) is found
(D) finds
5. This marketing position ___ at least two years of experience in a related field.
(A) require
(B) requiring
(C) requires
(D) is required
6. The training is going to ___ place in the seminar room at 1 p.m.
(A) took

ng k hc: 0962 60 8801 04 6260 3948 60


a ch: S 18 Trn i Ngha Q Hai B Trng H ni
NGOI NG 24H 61
WWW.NGOAINGU24H.VN

(B) is taken
(C) taking
(D) take
7. The lawyer is dedicated ____ helping small businesses in trouble.
(A) to
(B) with
(C) of
(D) in
8. Many changes ___ to the construction plan because the president didnt like it.
(A) have made
(B) have been made
(C) made
(D) are made
9. The president ____ after he had worked for the company for 30 years.
(A) retired
(B) retiring
(C) was retired
(D) retirement
10. The air conditioner ___ to your office no later than tomorrow.
(A) delivered
(B) will deliver
(C) delivering
(D) will be delivered
11. The team has been devoted ___ upgrading the system.
(A) in
(B) with
(C) to
(D) at
12. The company technicians ____ to fix the broken computers.
(A) asks
(B) is asking
(C) was asking
(D) were asked
13. The R&D team is going to ____ the new product.
(A) examine
(B) relocate
(C) solve
(D) refrain
14. We need to discuss it in detail before we ___ any kind of conclusion.
(A) replace
(B) quit
(C) reach
(D) fill
ng k hc: 0962 60 8801 04 6260 3948 61
a ch: S 18 Trn i Ngha Q Hai B Trng H ni
NGOI NG 24H 62
WWW.NGOAINGU24H.VN

Questions 15-16 refer to the following letter.

Best Buy, Inc.


3452 Seaside Street
Seattle

Dear Mr. Denis Lee,

We thank you for your order from Best Buy. We are glad to meet your order for ten printers, three
copy machines, and two fax machines. Since this is your first order, we ____ a special 20%

15. (A) evaluated


(B) dedicated
(C) canceled
(D) enclosed

discount coupon for you. You can use this on your next purchase. We are certain you will be ____

16. (A) satisfy


(B) satisfying
(C) satisfied
(D) satisfaction
with the quality of both our products and services.

We hope to do more business with you soon.

Sincerely,

John Hopkins
Sales Manager

BI TP KIM TRA
1. The mistake has already been_______ by him.
(A) correct
(B) correcting
(C) correction
(D) corrected
2. The family has _______ from their vacation already.
(A) return
(B) returned
(C) been return
(D) been returned
3. They were seen _______ kites in the park last weekend.
(A) fly
ng k hc: 0962 60 8801 04 6260 3948 62
a ch: S 18 Trn i Ngha Q Hai B Trng H ni
NGOI NG 24H 63
WWW.NGOAINGU24H.VN

(B) flown
(C) flying
(D) to be flown
4. The manager was preoccupied _______ checking the reports.
(A) in
(B) at
(C) with
(D) from
5. Losing interest in her business, Kimberly has recently _______.
(A) retired
(B) be retired
(C) to be retired
(D) been retired
6. Almost every part of our lives _______ computerized over the past 10 years.
(A) have been
(B) has been
(C) was
(D) had done
7. We _______ her for more than twenty years.
(A) know
(B) knows
(C) have known
(D) are known
8. I never see him without being _______ of his grandfather.
(A) remember
(B) memory
(C) remained
(D) reminded
9. During the experiment, people _______ to use calculators if necessary.
(A) were let
(B) were allowed
(C) were allowing
(D) let
10. I'm quite sure you will soon grow_______ to the new work environment.
(A) accustom
(B) accustoming
(C) accustomed
(D) to be accustomed
11. The United Nations _______ to begin fund raising for the earthquake victims.
(A) are expect
(B) is expect
(C) are expected
(D) is expected
12. The instructions _______ precisely.
(A) must follow
(B) must followed
(C) must to be followed
(D) must be followed
13. You had _______ have your teeth checked at least once a year.
ng k hc: 0962 60 8801 04 6260 3948 63
a ch: S 18 Trn i Ngha Q Hai B Trng H ni
NGOI NG 24H 64
WWW.NGOAINGU24H.VN

(A) should
(B) good
(C) better
(D) worse
14. _______snow, the mountain looks fantastic.
(A) Covering
(B) To cover
(C) Covered with
(D) To have covered
15. Everybody _______ with the results of the meeting.
(A) was satisfy
(B) satisfied
(C) was to satisfy
(D) was satisfied
16. Left alone, the baby _______ into tears.
(A) cried
(B) burst
(C) poured
(D) sobbed
17. Why was the soccer game _______ yesterday?
(A) to cancel
(B) canceling
(C) cancel
(D) cancelled
18. Last week, John an award for his community service.
(A) gave
(B) was gave
(C) was given
(D) had been given
19. A second attempt was made to collect _______ from the space probe.
(A) informations
(B) knowledges
(C) data
(D) fact
20. Once the files are ready, please have them_______ me.
(A) send
(B) send to
(C) sent to
(D) sent into
21. The pictures, _______ by a professional photographer, are going to be on display.
(A) taken
(B) which taken
(C) were taken
(D) was taken
22. I can't _______ it anymore. That website seems to have suddenly vanished.
(A) reaching
(B) log out
(C) access
(D) pass
ng k hc: 0962 60 8801 04 6260 3948 64
a ch: S 18 Trn i Ngha Q Hai B Trng H ni
NGOI NG 24H 65
WWW.NGOAINGU24H.VN

23. _______ at a distance, it looks like a miniature city.


(A) Seen
(B) Seeing
(C) Having seen
(D) Having been seen
24. This math problem _______ solved in ten minutes.
(A) will able to be
(B) be able to is
(C) is ably
(D) can be
25. Brian was seen _______ his car.
(A) washing
(B) wash
(C) have washed
(D) washed
26. My umbrella _______ by that terrible wind this morning
(A) broke
(B) was broken
(C) break
(D) had broken
27. With only one more week, he _______ better prepared for the concert last Saturday.
(A) could have been
(B) could be
(C) could well have
(D) could well be
28. The photocopier needs _______ .
(A) to fix
(B) to be fix
(C) fixing
(D) to be fixing
29. It is reported that the boat _______ about 6o miles off the coast of South Africa. (A) disappeared
(B) was disappeared
(C) was being disappeared
(D) has been disappeared
30. I have no money. I'm _______ .
(A) broke
(B) broken
(C) breaking
(D) break
31. Let's have this letter _______ by express mail.
(A) sends
(C) sent
(B) send
(D) being sent
32. Ms. Parker was very _______ with the answers the applicant gave during the job interview.
(A) impress
(B) impressionable
(C) impression
(D) impressed
ng k hc: 0962 60 8801 04 6260 3948 65
a ch: S 18 Trn i Ngha Q Hai B Trng H ni
NGOI NG 24H 66
WWW.NGOAINGU24H.VN

33. The benefits program_______ in the next few months.


(A) had changed
(C) changed
(B) were changed
(D) will be changed
34. Mr. Honda is a terrific worker. He _______ two promotions this year.
(A) has been giving
(B) gave
(C) was given
(D) giving
35. I'll be home for dinner unless, the boss _______ me to work overtime.
(A) will ask
(C) asks
(B) is asking
(D) asked
36. Make sure you get these contracts _______ before you meet with the lawyer.
(A) signed
(B) to sign
(C) signing
(D) sign
37. This group of doctors _______ in important research.
(A) is involved
(B) are involved
(C) be involving
(D) been involving
38. The trade newsletter where we advertise _______ widely distributed.
(A) has
(B) have
(C) is
(D) are

Questions 39-40 refer to the following report.


A survey of mobile phone use was conducted _______ the National
39. (A) to
(B) of
(C) by
(D) from
Telecommunications Institute. They discovered that only five percent of people have downloaded a
mobile phone game. The study found that many were confused about whether their handset could
play games or how to download them. 2,500 phone users were _______ across the U.S. and several
European countries.
40. (A) interviewed
(B) interview
(C) interviews
(D) interviewing

Questions 41-44 refer to the following advertisement.


Are you looking for an economical car to rent?
Look no more.
ng k hc: 0962 60 8801 04 6260 3948 66
a ch: S 18 Trn i Ngha Q Hai B Trng H ni
NGOI NG 24H 67
WWW.NGOAINGU24H.VN

Come to
Mr. Miser's Car Rental Agency
Why pay more than you have to?
At Mr. Miser's, we have _______ prices in town.
41. (A) low
(B) lower
(C) lowers
(D) the lowest
Why travel farther than you have to?
Mr. Miser's has three convenient _______.
42. (A) cars
(B) prices
(C) locations
(D) schedules
We have offices at the airport, at the train station, and downtown on Main Street.
At Mr. Miser's we _______ to serve you with a friendly smile.
43. (A) always ready are
(B) always are ready
(C) are always ready
(D) are ready always
Next time, _______ your car from Mr. Miser's.
44. (A) rent
(B) rents
(C) renting
(D) will rent

Questions 45-47 refer to the following letter.


Dear customers of Equips Shipping,
A recent _______ in fuel prices has made Equips Shipping adjust our shipping rates.
45. (A) rise
(B) proposal
(C) shortage
(D) benefit
Prices for deliveries within the NYC area will remain unchanged; however, rates for out-of-city and
our-of-state shipments will incense by five percent. The revised price list is _______.
46. (A) enclose
(B) enclosed
(C) enclosure
(D) enclosing
All the changes will be effective as of July 1. We sincerely _______ any
47. (A) doubt
(B) bother
(C) concern
(D) regret
inconvenience this will cause you.
We appreciate your continuous support of Equips Shipping.
Sincerely,
Robert Matson
Marketing Manager, Equips Shipping
ng k hc: 0962 60 8801 04 6260 3948 67
a ch: S 18 Trn i Ngha Q Hai B Trng H ni
NGOI NG 24H 68
WWW.NGOAINGU24H.VN

Questions 48-50 refer to the following e-mail.


I am writing to let you know that the IT department has completed its network overhaul and we will
begin the changeover to the new system - HAL - immediately. Al employees will now be able to
access electronic files and account information at the touch of a button. HAL _______ in response to
the network problems we have
48. (A) will be created
(B) has created
(C) was created
(D) is creating
experienced since last year.
Before running HAL on your desktop computers, it is important that you all download and view the
guidelines on using HAL and _______ related software you require.
49. (A) whatever
(B) which
(C) whom
(D) elsewhere
All content available through HAL is safe from viruses, and I am sure it will prove to be very
convenient to use. HAL will be available through the HAL site, www.grblx.org/HAL.html.
Passwords will be e-mailed to all employees within the next 5 days. Do not share your _______
information with anyone, including
50. (A) bank
(B) access
(C) income
(0) contact
supervisors.
Recovering lost passwords will result in lost time and productivity for IT.

ng k hc: 0962 60 8801 04 6260 3948 68


a ch: S 18 Trn i Ngha Q Hai B Trng H ni
NGOI NG 24H 69
WWW.NGOAINGU24H.VN

NGY 5: DANH T

Cc ui danh t ph bin

CH 1: Khng c ch ng th khng c danh t


Ng php b sung
1. Danh t
2. i t

CH 2: Danh t cng ng vai tr lm tn ng ca ng t


Ng php b sung

CH 3 Mo t v t s hu gn vi danh t

KIN THC M RNG


Tm danh t gc (danh t c i t thay th)

Bi tp luyn tp
Bi tp kim tra

ng k hc: 0962 60 8801 04 6260 3948 69


a ch: S 18 Trn i Ngha Q Hai B Trng H ni
NGOI NG 24H 70
WWW.NGOAINGU24H.VN

Cc ui danh t ph bin

-tion application completion -ment enrollment investment


-sion extension compassion -ance attendance maintenance
-ure departure exposure -sis basis crisis
-y inquiry security -al removal denial
-ant attendant applicant -ness kindness happiness

CH 1: Khng c ch ng th khng c danh t


Ch ng c v tr u cu nn ng vai tr lm ch th ca hnh ng. V l ch th ca hnh ng
nn v tr ca ch ng thng do danh t m nhim. Nu danh t nm u cu l ch ng th s
rt d tr li cu hi. Hy xem xt k hn v vn t loi no s lm ch ng.
1-1: Chn cu ng.
1. Is great. (X)
Paid vacation is great. (O)
Ng php b sung
C 2 loi t v tr ch ng
1. Danh t
Overtime work is not allowed.
Danh t work uc dng v tr ch ng.
* Khng ch c danh t m i t (pronoun) hoc cm danh t (danh ng t, ng t nguyn mu
c to) cng c th t v tr ch ng.
Working overtime is not allowed. (danh ng t)
To work overtime is not allowed. (ng t nguyn mu c to)
2. i t
Nhng i t ch nh ch ngi hay s vt cng c chc nng ch ng.
You are invited to attend the seminar.
i t you c dng v tr ch ng.
S t S nhiu
it they
he she they
I you we / you

Bi tp 1:
1. (Compete /Competition) in the global market has increased.
2. Customer (satisfy /satisfaction) is our top priority.
3. (Encouragement /Encourage) will be given to each trainee.
4. (To produce /Products) can be ordered through Internet.
5. (Construction /Construct) on the road is on schedule.
ng k hc: 0962 60 8801 04 6260 3948 70
a ch: S 18 Trn i Ngha Q Hai B Trng H ni
NGOI NG 24H 71
WWW.NGOAINGU24H.VN

+ [cm t b ngha] + ng t => ch trng l v tr ca ch ng nn cn phi in danh t

NOTE:

Danh t ng t
competition compete
satisfaction satisfy
encouragement encourage
product produce
construction construct

Bi tp 2:
1. ___________ will be completed by next month.
(A) Constructive (B) Construct (C) Construction (D) Constructed

2. Every ___________ will receive a written response.


(A) apply (B) applicant (C) applied (D) applicable

3. Closer ___________ is required to meet the safety standards.


(A) supervision (B) supervise (C) supervised (D) supervisory

4. Certain ___________ must be met to apply for a loan.


(A) conditioning (B) conditioned (C) to condition (D) conditions

5. The research and development ___________ has gradually increased.


(A) spend (B) spending (C) spent (D) spends

ng k hc: 0962 60 8801 04 6260 3948 71


a ch: S 18 Trn i Ngha Q Hai B Trng H ni
NGOI NG 24H 72
WWW.NGOAINGU24H.VN

CH 2: Danh t cng ng vai tr lm tn ng ca ng t

Danh t ng vai tr nh ngi gim h ca ng t, ngoi vai tr lm ch th ca ng t, n cng


c vai tr lm tn ng ca ng t.
2-1. Chn cu ng.
1.We hired for the construction. (X)
We hired supervisors for the construction. (O)

Nh trnh by trn, nu khng c tn ng th khng th bit i tng tuyn dng v khng


to thnh cu. Nn nh rng sau ngoi ng t chc chn phi c tn ng v danh t cng c th
ng vai tr tn ng.
Ng php b sung
Tn ng l g?
Tn ng (O) l i tng cho hnh ng ca ng t. V d:
We hired supervisors for the construction.
S V O

(1) ng t hired din t hnh ng tuyn dng ca ch ng we.


(2) Danh t supervisors tr thnh tn ng v l i tng ca hnh ng tuyn dng.

Bi tp 3: Chn t ng trong cc cu sau:


1. We do not accept (responsible / responsibility) for lost items.
2. The company has begun (produce / production)of new digital cameras.
3. Mr. Ram provides (translate / translation) of official documents.
4. The storm has caused (damage /damaged) to the building.
Ngoi ng t (V. trans) + ... + gii t (prep) => in danh t ng vai tr lm tn ng vo ch
trng
Danh t

Bi tp 4:
1. An important part of a managers job is offering _________ to new employees.
(A) encourage (B) encouragement (C) encouraging (D) encourages
2. The new product will give_________ to our customers.
(A) satisfaction (B) satisfy (C) satisfied (D) satisfying
3. You should receive official_________ for taking a day off.
(A) approve (B) approved (C) approving (D) approval
4. The new airline has increased_________ in the airline industry.
(A) compete (B) competition (C) competitive (D) competitively
5. You should seek_________ from your supervisor.
(A) advised (B) advisable (C) advice (D) advise

ng k hc: 0962 60 8801 04 6260 3948 72


a ch: S 18 Trn i Ngha Q Hai B Trng H ni
NGOI NG 24H 73
WWW.NGOAINGU24H.VN

CH 3 mo t v t s hu gn vi danh t

Sau mo t (a/an/the) v t s hu (my/his/her/our/their/its/your) chc chn phi l danh t. Mo t


l t dung lm r ngha ca danh t theo sau. Do sau mo t lun phi c danh t km theo.
Hn na, nu khng c danh t th cc t s hu chc chn phi l danh t.

Ng php b sung
1. Mo t
(1) Mo t bt nh a/an: ch i tng khng r rang v khng th xc nh c.
a company
Mo t bt nh c ngha l mt, danh t c mo t bt nh c th hin s t.
(2) Mo t xc nh the: ch i tng c th c xc nh.
The company
The performance was very exciting.
Mo t xc nh ch i tng c th, ch yu dung khi bit i tng.
2. T s hu
(1) T s hu ng trc danh t mang ngha l ca ai.
His performance was very exciting.
Bui trnh din l ca anh y ch khng phi ca ai khc.

T s hu
My Our
His Her
its Their
Your
Bi tp 5:
Chn t ng trong cc cu sau.
1. They made many changes to the[proposal/ propose].
2. Our bank will review your [apply/ application]
3. The company reserves the [rightful/ right] to revise the contents.
4. Please turn off your cellular phone during the [perform/ performance].
5. We have to maximize our [produce/ productivity].

(mo t/ t s hu+ => phi in danh t vo ch trng)

Cu trc mo t/ t s hu+danh t
Mo t/t s hu Danh t ng t
proposal Propose
The Application Apply
Your
Right Right
our
Performance Perform
Productivity produce

Bi tp 6. Xc nh danh t ng sau mo t hoc t s hu


1. You have to refuse the _____ of packages from unknow senders.
a. Deliverable b. delivered c. delivery d. deliver
2. You need to get your supervisors _____ for the seminar.
a. Approval b. approve c. approved d. approving

ng k hc: 0962 60 8801 04 6260 3948 73


a ch: S 18 Trn i Ngha Q Hai B Trng H ni
NGOI NG 24H 74
WWW.NGOAINGU24H.VN

3. I learned this program through the _____ in todays newspaper.


a. Advertise b. advertisement c. advertised d. advertising
4. Companies should evaluate employees _____.
a. Performance b. performed c. performing d. perform
5. All members are looking forward to a fruitful _____.
a. Conclude b. conclusive c. concluded d. conclusion

KIN THC M RNG


I. Tm danh t gc (danh t c i t thay th)
1. Danh t gc v i t
i t c dng thay th cho danh t, nhm trnh s lp li danh t . Vic xc nh xem i
t trong cu thay th cho danh t no l k nng rt quan trng m bi thi TOEIC nh gi.

Danh t gc i t
S nhiu (c ngi v vt) they
S t Vt it
Ngi (nam) he
Ngi (n) she

PIA Investment Inc. announced that it will invest more in emerging markets.
danh t gc s t, ch vt i t
Cng ty PIA Investment thng bo l h s tng cng u t vo cc th trng mi ni.

BI TP LUYN TP
PART 5: Incomplete Sentence

Questions 1-10 Choose the word that best completes each sentence.

l. The_____for the building renovation is now available on the company bulletin board
(A) scheduled (B) scheduled (C) schedules (D) schedulers

2. Faster Shipping offers exceptional _____in customer satisfaction.


(A) performer (B) performing (C) performed (D) performance

3. The bank will celebrate the _____of the senior accountant at the upcoming monthly meeting.
(A) retire (B) retirement (C) retiring (D) retired

4. Thanks to his _____to the project, we could complete it on time.


(A) commit (B) committed (C) commitment (D) committing

5. The newly installed system requires that every _____ have a savings account.
(A) user (B) useful (C) used (D) using

6. We introduced an innovative compensation plan to increase staff_____.


(A) produce (B) product (C) productive (D) productivity

7. It took the division almost a week to receive a _____from the manufacturer.


ng k hc: 0962 60 8801 04 6260 3948 74
a ch: S 18 Trn i Ngha Q Hai B Trng H ni
NGOI NG 24H 75
WWW.NGOAINGU24H.VN

(A) response (B) respond (C) responding (D) responsive

8. When you receive your _____ to the grand opening, please respond to us ASAP.
(A) honor (B) invitation (C) expression (D) ovation

9. Executives agree that there is a ____for better equipment and financial resources.
(A) need (B) look (C) control (D) center

10. Employees can find out about the _____of the recent survey on the Web.
(A) events (B) chances (C) matters (D) results

PART 6: Text Completion

Questions 1-3 refer to the following letter.

Dear Prudent investment customer,

We are writing to inform you that an error might have occurred on your bill due to a recent ____to
our online system.
l.(A) update
(B) updated
(C) updating
(D) to update

Please contact our customer service representative regarding any suspected discrepancies within 30
days of receiving your bill.

The Prudent Investment Group feels____ about any inconvenience this might cause,
2.(A) regret
(B) regretted
(C) regrettable
(D) regrettably

and we will try our best to resolve any problems as ____as possible.
3. (A) temporarily
(B) currently
(C) effortlessly
(D) promptly

We sincerely appreciate your support and will continuously provide you with the best quality service
you will ever experience.

Best regards,
Francesca Gonzalez

Customer Relations
ng k hc: 0962 60 8801 04 6260 3948 75
a ch: S 18 Trn i Ngha Q Hai B Trng H ni
NGOI NG 24H 76
WWW.NGOAINGU24H.VN

The Prudent Investment Group


BI TP THC HNH
Bi tp 1:
Choose one correct option in the parentheses.
01 Our company strengthened (security, secure) by hiring more guards.
02 The (grow, growth) of the IT market was larger than expected.
03 Oil companies earned (profits, profitable) during the recent quarter.
04 It is an (advantage, advantageous) to have experience in a related field.
05 The supervisor will examine her (apply, application).
06 The movie drew widespread (criticism, criticize).

Choose the correct option for each of the following sentences.


07 This is a ___ of recent comments from our product users.
(A) summarize (C) summarizes
(B) summary (D) summarized
08 Interviewees normally have a fear of ____
(A) rejection (C) rejected
(B) rejecting (D) to reject
09 The book was released after it got final ___ from the author.
(A) approved (C) approve
(B) approving (D) approval
10 Once the __ of the parking lot is completed, visitors will have no problem parking their cars.
(A) construction (C) constructive
(B) construct (D) constructing

Bi tp 2:
Choose one correct option in the parentheses.
01 Diana obtained (access, accesses) to classified files.
02 The supplier has many different models of (furnitures, furniture).
03 Mr, Anderson has received some (advice, advices) from his co-workers.
04 Jennifer will stay in Hong Kong for (a month, month) to hire more engineers.
05 He complained to the airline officials about his lost (luggages, luggage).
06 The partners failed to reach (an agreement, agreement).

Choose the correct option for each of the following sentences.


07 This article will discuss the ___ of the new commercial law.

(A) implicates (C) implicating


(B) implicated (D) implications
08 If you have questions regarding your pur

chase. please provide our staff with the relevant ____


(A) informed (C)inform
(B) information (D)informations
09 The director has asked us lo send monthly ___ to Ms. Shriver in the finance department.

ng k hc: 0962 60 8801 04 6260 3948 76


a ch: S 18 Trn i Ngha Q Hai B Trng H ni
NGOI NG 24H 77
WWW.NGOAINGU24H.VN

(A) statements (C) states


(B) statement (D) state
10 ____ of labor union met with management to discuss the contract for the next year.

(A) Represent (C) Representative


(B) Representing (D) Representatives

Bi tp 3:
Choose one correct option in the parentheses.
01 The committee interviewed one of the (candidate, candidates) this morning.
02 (Several, Every) calculators were found to be defective.
03 (Another, Some) staff members will attend the seminar tomorrow.
04 The policy change caused (much, many) debate.

Choose the correct option for each of the following sentences.


05 Recent studies have found that there are a number of ___ why customers prefer placing orders
online.

(A) reasons (C) reasoning


(B) reason (D) reasoned
06 ___ franchises comply with strict rules on structure and operations.
(A) Another (C) Most
(B) Much (D) Little
Questions 7 and 8 refer to the following memorandum.
From: Tom Klein, Training Manager
To: All employees

I would like to remind you about the training program you are scheduled to take. You will be asked
to assess the trainer's ____ at the end of the program and submit a report to the department.

07 (A) performance (C) performed


(B) perform (D) performable

Also, ____ employees who have good attendance records will receive certificates for completing the

08 (A) another (C) every


(B) all (D) each

program.

Bi tp 4:
Choose one correct option in the parentheses.
01 A well-organized work environment will increase staff (interest, productivity, compliance)
02 The baggage (weight, security, allowance) on international flights is 20 kilograms.
03 Advances in computer (technology, concern, relation) are giving companies a lot of advantages.

ng k hc: 0962 60 8801 04 6260 3948 77


a ch: S 18 Trn i Ngha Q Hai B Trng H ni
NGOI NG 24H 78
WWW.NGOAINGU24H.VN

04 The results of recent customer (concerns, allowances, surveys) show a high level of satisfaction
with our services.

Choose the correct option for each of the following sentences.


05 Due to time ___, the executives were not able to interview all the applicants.

(A) obstacles (C) allowances


(B) records (D) constraints
06 Employees ____ records are included in the performance evaluation.

(A) attendance (C) interest


(B) maximum (D) retirement
Questions 7 and 8 refer to the following e-mail.
From: Margie Weston, Marketing Manager
To: Sarah Hall, Human Resources Manager

Dear Ms. Hall,

It is my understanding that Ms, Lee has applied for the ___ of Marketing Manager with your

07 (A) position (C) question


(B) request (D) reason

company. Ms. Lee has been working for our company's marketing ___ for two years and has shown a

08 (A) campaign (C) department


(B) management (D) sales

keen interest in sales. I have complete confidence in Ms. Lee's ability to attract potential customers to
GlobalNetwork.

Bi tp 5:
Choose one correct option in the parentheses.
01 There is no (inconvenience, collection, exception) to this rule.
02 Ms. Lewis received positive (feedback, complaint, objection) after the conference.
03 Clients make a frequent (discount, thought, complaint) that the products arrive late.
04 The company only allows (representatives, subjects, opponents) to attend its meetings.

Choose the correct option for each of the following sentences.


05 High Airlines is offering conference attendees a 40 percent ___ on economy class tickets.
(A) discount (C) retail
(B) feedback (D) improvement
06 Hotel employees do their best to create a friendly ____
(A) atmosphere (C) apology
(B) temperature (D) responsible

ng k hc: 0962 60 8801 04 6260 3948 78


a ch: S 18 Trn i Ngha Q Hai B Trng H ni
NGOI NG 24H 79
WWW.NGOAINGU24H.VN

Questions 7 and 8 refer to the following letter.

Dear Mr. Jones,


We are sorry to inform you that the large teddy bears at AllToys.com will be temporarily unavailable.
The manufacturer that makes these toys has moved its ____ and the items are not in production at
this

07 (A) qualities (C) facilities


(B) responsibilities (D) categories

time. We apologize for any ___ this may cause you. We will let you know when they are available.

08 (A) income (C) exception


(B) inconvenience (D) incentive

Bi tp 6:
Choose one correct option in the parentheses.
01 Respondents will receive the (results, chances, proportions) of the survey before July.
02 The company spent a large (proportion, size, incentive) of Its budget on research.
03 Credit cards and checks are the preferred (methods, efforts, concerns) of payment.
04 Cities offer (obligations, incentives, complaints) to companies that hire disabled workers.
Choose the correct option for each of the following sentences.
05 The regional office is always available and will promptly respond to any___ that may arise.
(A) agreement (C) importance
(B) incentives (D) concerns
06 Consumers have no ____ to pay for goods that they did not order.
(A) obligation (C) promise
(B) effort (D) exception
Questions 7 and 8 refer to the following memorandum.
From: Warren Becker, PR Department
To: All staff

Managers have agreed to set aside 5 percent of the companys funds for competency-based training.
The training department has redesigned its program in an ___ to improve employee performance.

07 (A) objection (C) effort


(B) opinion (D) influence

This will be a good opportunity for staff members to upgrade their capabilities. Since we believe a
truly successful training ultimately depends on the ____ of all employees, we will require everyone

08 (A) participation (C) obligation


(B) proportion (D) conclusion

to attend the training sessions.

ng k hc: 0962 60 8801 04 6260 3948 79


a ch: S 18 Trn i Ngha Q Hai B Trng H ni
NGOI NG 24H 80
WWW.NGOAINGU24H.VN

Bi tp 7:
Choose one correct option in the parentheses.
01 The company needs a consultant with {inquiry, expertise, defects) in real estate.
02 The CEO should ensure the (stability, issue, request) of the company.
03 Stock price is an (indicator, invitation, expertise) of expected future profits.
04 The medication reduces the (duration, profits, purpose) of symptoms associated with the flu.

Choose the correct option for each of the following sentences.


05 Mr. Robert recently received an ___ to participate in the annual conference.
(A) expression (C) indicator
(B) invitation (D) honor
06 ___ for additional copies should be addressed to the Service Center.
(A) Requests (C) Retirements
(B) Developments (D) Facilities
Questions 7 and 8 refer to the following letter.

Dear Ms. Pena,

We ordered car accessories from your company and were very impressed with the speedy delivery.
However, some of the accessories were damaged. Because of the ___ , we were not able to meet an

07 (A) alterations (C) charges


(B) shortages (D) defects

important deadline, and the actual ____ from the sale of these cars was much less than we estimated.

08 (A) incentive (C) price


(B) profit (D) concern

Since the problem was caused by your company, we feel that our losses should be compensated.

Bi tp 8:
Choose one correct option in the parentheses.
01 Guests will have the (complaint, effort, opportunity) to try new wines.
02 The following is a (total, point, summary) of activities scheduled for this year.
03 The hotel is gaining (participation, discount, popularity) because of its facilities.
04 Mr. Dyen will secure his (schedule, position, order) as a manager for another year

Choose the correct option for each of the following sentences


05 Every customer will have the ___ of canceling the order at any time.
(A) summary (C) exception
(B) option (D) expertise
ng k hc: 0962 60 8801 04 6260 3948 80
a ch: S 18 Trn i Ngha Q Hai B Trng H ni
NGOI NG 24H 81
WWW.NGOAINGU24H.VN

06 Changing the locks frequently is a highly recommended ___.


(A) plot (C) spot
(B) point (D) practice
Questions 7 and 8 refer to the following memorandum.

I am pleased to announce that Mary Murphy was recently promoted to manager of the real estate
development department. She will start her new position on May 15. Most of you know that the
company's sales department has been under the _____ of Ms. Murphy for two years. During that

07 (A) attendance (C) provision


(B) supervision (D) sight

period, a comprehensive____ of the property development industry has allowed her to increase the

08 (A) knowledge (C) ability


(B) opinion (D) obligation

company's real estate sales by 150 percent.

Bi tp 9:
Choose the correct noun in brackets for each sentence. Then, name its function.
1. The (meeting / meet) will be delayed __________
2. The event was a huge (success / succeed) _____
3. What we need now is your (cooperation / cooperate)____
4. The (manager / manage) is not fulfilling his job____
5. We need to arrange a (gathering / gather) ______
6. My boss is satisfied with his (employ / employees)_____

Choose the correct option to complete each sentence.


1. Unfortunately, the subject is out of order.
(A) copier (B) copy (C) copying (D) to copy
2. The R&D Department at Info Tech is looking for object .
(A) special (B) specialize (C) specializing (D)specialists

Bi tp 10:
Choose the correct noun from the adjective or verb given.
1. distribute (distributor / distributence)
2. conduct (conductor / conductant)
3. train (trainee / trainment)
4. apply (applicant / applier)
5. supervise (superviser / supervisor)
Choose the correct option to complete each sentence.
6. ____ can get a brochure on the second floor.
(A) To participate (B) Participate (C) Participants (D) Participating
7. ____ in computer courses has increased for the past few years.
(A) To enroll (B) Enrollment (C) Enrolls (D) Enrolled

ng k hc: 0962 60 8801 04 6260 3948 81


a ch: S 18 Trn i Ngha Q Hai B Trng H ni
NGOI NG 24H 82
WWW.NGOAINGU24H.VN

Bi tp 11:
Look at the following sentences and place the nouns in brackets in the correct position - (A), (B),
(C), or (D).
1. (applicant) The (A) is not (B) qualified (C) for (D) the job.
2. (supervisor) (A) Your (B) will inform (C) you (D) next week.
3. (campaign) An (A) aggressive (B) will (C) be effective (D).
4. (renovations) The (A) shop (B) is closed (B) for (D).
5. (accountant) An (A) experienced (B) will (C) be hired (D) soon.
6. (satisfaction) We will try (A) to solve your (B) problem (C) to your (D).

Look at the hints printed in bold and choose the correct option to complete each sentence.
7. An early ____ is required, especially in this kind of situation.
(A) decide (B) decision (C) to decide (D) decided
8. We are looking for someone with two years of ____.
(A) experiencing (B) to experience (C) experience (D)experienced

Bi tp 12:
Choose the correct noun in brackets for each sentence.
1. The book provides general (information / informations) on health.
2. Your country should sign (agreement / an agreement).
3. My boss is always late for (meeting / meetings).
4. You cannot carry a lot of (luggages / luggage).

Choose the correct option to complete each sentence.


5. You need to wear protective ____ at the construction site.
(A) cloth (B)clothe (C) clothings (D) clothing
6. The research shows that this ____ can cause big noise problems.
(A) some machineries (B)machineries (C) machinery (D) a machinery

Bi tp 13:
Fill each gap with some or any.
1. ____ companies make a lot of profits in this economic situation
2. Do you have _____ concerns about your new job?
3. Our office needs _____ equipment before we start a new project.
4. You should not touch ____ furniture in this lobby.
5. Mr. Norris found ____ mistakes in the final report.
6. The management doesnt want to fire _____ employees next year.
Look at the hints printed in bold and choose the correct option to complete each sentence.
7. The trainees didnt have ___ questions about the new system.
(A) theirs (B) any (C) some (D) a
8. The engineers reported that ____ machinery has critical problems.
(A) they (B) some (C) any (D) a

Bi tp 14:
Practice with TOEIC Actual Questions
Decide which of the choices - (A), (B), (C), or (D) - best completes the sentence.
1. Make sure you bring ____ ID to open an account.
(A) you

ng k hc: 0962 60 8801 04 6260 3948 82


a ch: S 18 Trn i Ngha Q Hai B Trng H ni
NGOI NG 24H 83
WWW.NGOAINGU24H.VN

(B) your
(C) yours
(D) yourself
2. _____ for the job must pass a very difficult test.
(A) Applying
(B) Apply
(C) To apply
(D) Applicants
3. I need to find ____ investors for this business.
(A) any
(B) some
(C) yours
(D) yourself
4. Your -___ in this matter will be appreciated.
(A) cooperation
(B) cooperating
(C) cooperate
(D) to cooperate
5. The recent ____ conducted by Todays Report showed some surprising results.
(A) researching
(B) research
(C) to research
(D) researched.
6. The ____ will inform you of any changes happening here.
(A) managing
(B) manager
(C) managed
(D) manage
7. It costs a lot of money to conduct a(n) ____ to find out customerss need.
(A) office
(B) ourselves
(C) survey
(D) equipment
8. The ____ was established in 1977 to help poor children in the world.
(A) consultant
(B) agreement
(C) transportation
(D) organization
9. It is always better to use public ____ during rush hour.
(A) transportation
(B) convenience
(C) appointment
(D) agreement
ng k hc: 0962 60 8801 04 6260 3948 83
a ch: S 18 Trn i Ngha Q Hai B Trng H ni
NGOI NG 24H 84
WWW.NGOAINGU24H.VN

10. The company is going to hire a __ to help us with our management problems.
(A) consultant
(B) trainee
(C) candidate
(D) contractor
11. You can make a call or send an e-mail to make a(n) ____ with Dr. Stewart.
(A) development
(B) possibility
(C) renovation
(D) appointment

Questions 12-13 refer to the following advertisement.

Do you want to get some new home apliances?


Sears can help you. Enjoy our special offer!

This weekend, we are offering a special ______ on home appliances such as refrigerators, washing
12. (A) discounts
(B) discount
(C) discounting
(D) to discount

machines, and dishwashers.

The offer lasts only for three days. ____ should hurry up. This offer ends on March 3. Visit Sears
13. (A) You
(B) Your
(C) Yours
(D) Yourself

today and get the home applicances you want.

BI TP KIM TRA
1. Did you buy _______ I asked you for?
(A) a toothpaste
(B) a toothpastes
(C) the toothpastes
(D) the toothpaste
2. Have you seen _______ stapler?
(A) I'm
(B) my
(C) mine
(D) me
3. Do you have _______ rooms available?

ng k hc: 0962 60 8801 04 6260 3948 84


a ch: S 18 Trn i Ngha Q Hai B Trng H ni
NGOI NG 24H 85
WWW.NGOAINGU24H.VN

(A) some
(B) sometime
(C) sometimes
(D) any
4. I cut ten _______.
(A) slice of bread
(B) slices of bread
(C) slices of breads
(D) slice of breads
5. I met a lot of _______ at the party.
(A) peoples
(B) people
(C) person
(D) persons
6. For more _______, contact us at 721-3431.
(A) informations
(B) informed
(C) further information
(D) information
7. Let's take another _______ at the sales figure.
(A) view
(B) scene
(C) glance
(D) vision
8. My favorite color is _______.
(A) grapes
(B) apple
(C) orange
(D) pear
9. The managing director asked his _______ to contact the shareholders regarding the crisis
management meeting.
(A) aid
(B) aide
(C) aided
(D) addition
10. _______ all of the newly-hired employees were unhappy with the organization of the orientation
schedule.
(A) Most
(B) Mostly
(C) Almost
(D) Every
11. Fortunately, changing the members of the committee halfway through the project has had no
noticeable _______ on the price of shares.
(A) point
(B) affectation
(C) affect
(D) effect
12. I will be announcing to the media today that all _______ from sales of this CD will go to charity.
(A) proceeds
ng k hc: 0962 60 8801 04 6260 3948 85
a ch: S 18 Trn i Ngha Q Hai B Trng H ni
NGOI NG 24H 86
WWW.NGOAINGU24H.VN

(B) precedes
(C) precedent
(D) results
13. There has been a leak of confidential data to one of our biggest _______ It would seem that we
are the victims of corporate espionage.
(A) enemies
(B) oppositions
(C) rivals
(D) opposites
14. The company will spend more money on the _______ of environment-friendly vehicles.
(A) develops
(B) developed
(C) develop
(D) development
15. Mr. Kim, the plant manager, expressed his support for _______ a new parking lot.
(A) constructive
(B) construction
(C) constructively
(D) constructed
16. The accounting director decided to conduct an _______ on the cash flow problem.
(A) introduction
(B) investigation
(C) satisfaction
(D) exhibition
17. Mr. June has outstanding _______ for the position of sales manager.
(A) qualifying
(B) qualification
(C) qualified
(D) qualify
18. I have attached a copy of our new catalog to my e-mail for your _______.
(A) performance
(B) supervision
(C) encouragement
(D) convenience
19. The company reserves the _______ to change the prices without any prior notice.
(A) goal
(B) seat
(C) occupation
(D) right
20. You need to present one form of _______ to pick up a package.
(A) identity
(B) identification
(C) identifying
(D) identifiable
21. The results show that there were a significant number of customer _______ about our delivery
services.
(A) complaint
(B) complaining
(C) complains
ng k hc: 0962 60 8801 04 6260 3948 86
a ch: S 18 Trn i Ngha Q Hai B Trng H ni
NGOI NG 24H 87
WWW.NGOAINGU24H.VN

(D) complaints
22. We couldn't take _______ measure to deal with the problem.
(A) some
(B) any
(C) their
(D) the other
23. _______ of the applicants from around the country were qualified for each position.
(A) Either
(B) Every
(C) Each
(D) Most
24. We make it a rule to double-check_______ report with important contents.
(A) all
(B) some
(C) every
(D) other
25. The main _______ of the novel gives us a keen insight into the loss of humanity.
(A) ideas
(B) ideal
(C) idea
(D) idealess
26. Customers can access this website for _______ information about items on sale.
(A) almost
(B) more
(C) each
(D) another
27. Most of _______ living in this country fear another economic crisis after they went through the
currency crisis in the late 90s.
(A) peoples
(B) many people
(C) many peoples
(D) the people
28. Tommy Books will offer _______ of up to 20 percent off on the first book of the series
throughout this month.
(A) discount
(B) discounting
(C) discounts
(D) discounters
29. In order to brand your business, you'd better include your _______ in every letter you send to
customers, which helps remind them of whom they do business with.
(A) signing
(B) signed
(C) signature
(D) to sign
30. _______ for reconstruction contracts has been getting stiffer over the years since it can generate
enormously lucrative earnings.
(A) Competitively
(B) Competition
(C) Competitive
ng k hc: 0962 60 8801 04 6260 3948 87
a ch: S 18 Trn i Ngha Q Hai B Trng H ni
NGOI NG 24H 88
WWW.NGOAINGU24H.VN

(D) Competes
31. When the_______ for this new safety policy was under consideration by the managers, a number
of experts were asked for their comments.
(A) proposing
(B) propose
(C) proposes
(D) proposal
32. A questionnaire asked _______ to evaluate the appearance and the overall brightness of the new
merchandise.
(A) shopping
(B) shoppers
(C) to shop
(D) shopper
33. Please read the terms and conditions carefully to determine whether or not you can request a
refund or _______ for the products you ordered.
(A) replacement
(B) complaint
(C) receipt
(D) promotion
34. Superb _______ has been paid to even the smallest detail, as the project was expected to cost
billions of dollars.
(A) attends
(B) attention
(C) attended
(D) attendant
35. All _______ to our fabulous nation park will be given a complimentary lunch at the top of the
hill.
(A) visit
(B) visitations
(C) visitors
(D) visiting
36. We are proud to announce their upcoming _______ in an event whose purpose is to educate
attendees on e-mail management tools.
(A) participation
(B) participated
(C) participating
(D) participate
37. Employees must reserve a time to copy by writing their names on the _______ on the lid of the
photocopiers.
(A) schedule
(B) scheduled
(C) schedules
(D) schedulers
38. The two companies signed an _______ on the acquisition in spite of a few differing opinions.
(A) agreement
(B) agrees
(C) agreements
(D) agreed

ng k hc: 0962 60 8801 04 6260 3948 88


a ch: S 18 Trn i Ngha Q Hai B Trng H ni
NGOI NG 24H 89
WWW.NGOAINGU24H.VN

Ouestion 39 through 41 refer to the following email.


To: Everyone
From: cc021@exxnet.co
Subject: Party
Date: October 20th
Hi Everyone,
As you know, the office halloween party is coming up. As usual, we will have a costume
competition. It is a _______ for you all to have some fun and win some great
39. (A) chance
(B) change
(C) concept
(D) challenge
prizes. The party starts at 6 p.m. on Friday, October 30. We will be closing the office early, so you
will have one hour to get _______. I am looking forward to some
40. (A) rest
(B) real
(C) ready
(D) realize
great costumes. This year, the first prize is free membership at the company sports club for one year.
Second prize is dinner _______ the Magic Chef restaurant for four people.
41. (A) in
(B) at
(C) with
(D) for
See you on the 30th.
Miranda Kim

Questions 42 - 44 refer to the following notice.


Music Festival
To celebrate the 25th anniversary of ABC Times, we invite our _______ readers to
42. (A) value
(B) valuably
(C) valuation
(D) valuable
the Music Festival.
Our current _______, will receive two tickets for free.
43. (A) subscribe
(B) subscription
(C) subscribed
(D) subscribers
New readers will receive one free ticket for the concert.
The festival will_______ by the National Magazine Association.
44. (A) sponsor
(B) sponsoring
(C) be sponsored
(D) sponsored

Questions 45- 47 refer to the following advertisement.


Meetings & Conventions
ng k hc: 0962 60 8801 04 6260 3948 89
a ch: S 18 Trn i Ngha Q Hai B Trng H ni
NGOI NG 24H 90
WWW.NGOAINGU24H.VN

Taking_______ of discounted fares and benefits is easy with Concord Airlines'


45. (A) address
(B) advantage
(C) admission
(D) advice
Meetings & Conventions Travel Team. _______ your company is planning a business
46. (A) During
(B) Often
(C) Because of
(D) If
meeting or your association is planning a convention, let Concord Airlines be the official airline. Our
service representatives are ready to satisfy your air travel requirements.
You can call_______ to our Meetings & Conventions Travel Team to get any
47. (A) direct
(B) direction
(C) director
(D) directly
information about our discounted fares and group benefits.

Questions 48 - 50 refer to the following notice.


Please help us improve our website by answering a few questions_______ your
48. (A) because
(B) about
(C) if
(D) while
experience today. This _______ survey will take less than a minute and your answers
49. (A) shortly
(B) shorten
(C) shortage
(D) short
will remain confidential. Your _______ will be used solely to improve the quality and
50. (A) respond
(B) responded
(C) responsive
(D) responses
content of this site.

ng k hc: 0962 60 8801 04 6260 3948 90


a ch: S 18 Trn i Ngha Q Hai B Trng H ni
NGOI NG 24H 91
WWW.NGOAINGU24H.VN

NGY 6:
S HA HP GIA CH NG V NG T

CH 1:
Chia ng t ph hp vi ngi v s ca ch ng
Kin thc b sung

CH 2: Xc nh s ha hp v s ca ch ng v ng t, b qua cm t b ngha cho danh t


lm ch ng
Kin thc b sung

BI TP LUYN
BI TP KIM TRA

ng k hc: 0962 60 8801 04 6260 3948 91


a ch: S 18 Trn i Ngha Q Hai B Trng H ni
NGOI NG 24H 92
WWW.NGOAINGU24H.VN

CH 1: Chia ng t ph hp vi ngi v s ca ch ng
(1) S ca danh t: Danh t nm v tr ch ng c hnh thc s t v s nhiu.
1. Danh t c mo t ch s t: a book, a chair, an employee
2. Danh t gn vi (e)s" th hin s nhiu: books, chairs, employees
(2) S ca ng t:
1. ng t c gn (e)s th hin s t: expands, needs, requests
2. ng t dng bare inf th hin s nhiu: expand, need, request

Bi tp 1: Chn cu ng trong hai cu sau:


1. The company hires new employees.
The company hire new employees.
2. Many bank hires MBA graduates.
Many banks hire MBA graduates.

ng t s t hay s nhiu c quyt nh ty theo s ca danh t lm ch ng. Ngha l nu danh


t lm ch ng l s t th ng t cng l s t, danh t lm ch ng l s nhiu th ng t cng l
s nhiu. y chnh l nguyn tc v s ha hp gia ch ng v ng t.

Kin thc b sung

1. S khc bit gia s t / s nhiu ca ng t ch quan trng khi ng t th hin ti, cn th


qu kh th s ca danh t lm ch ng khng nh hng n vic chia ng t s t hay s nhiu
(ngoi tr ng t be).
He delivers products to customers.
Ch ng ngi th ba s t th hin ti nn ng t c ui l -s.
They deliver products to their customers.
Ch ng l ngi th ba s nhiu nn ng t cng l s nhiu.
He delivered products to the customer. vs They delivered products to their customers.
2. Tr ng t (auxiliary - aux) khng b nh hng bi s ca ch ng v lun c vit vi cng
mt hnh thc ging nhau cc ngi, s hay th ca ch ng.
He will / can / should deliver products to customers.
They will / can / should deliver products to customers.

Bi tp 2: Chn t ng trong cc cu sau.


1. Dr. Brown (has/ have) requested the report.
2. A notice (is/ are) posted on the bulletin board.
3. The documents (was/ were) sent by e-mail.

ng k hc: 0962 60 8801 04 6260 3948 92


a ch: S 18 Trn i Ngha Q Hai B Trng H ni
NGOI NG 24H 93
WWW.NGOAINGU24H.VN

4. They usually (charge/charges) high fees.


5. The company (need/ needed) a partner to expand its market.

Lu :
Ch ng ngi th 3 s t, th hin ti, ng t thung => ng t nguyn mu + (e)s
Ch ng ngi th 3 s t, th hin ti, ng t be => is
Ch ng ngi th 3 s t, th qu kh, ng t be => was

Tm tt:
Trng hp ca ng t be, khi th qu kh cng c s t v s nhiu.
The product was delivered to the customer.
Ch ng ngi th 3 s t nn chia l was.
The products were delivered to the customer.
Ch ng ngi th 3 s nhiu nn chia l were.
3 dng ca ng t be:
am / is was been
are were been
Bi tp 3:
1. Many companies________criticized the governments bill.

(A) have (B) has (C) having (D) to have

2. The committee________the proposed nonsmoking policy.

(A) reject (B) rejecting (C) to reject (D) rejected

3. The furniture________scheduled to be delivered on September 14.

(A) be (B) were (C) are (D) is

4. The post office________one form of identification.

(A) require (B) requires (C) requirement (D) requiring

5. All department managers________to attend the monthly business meeting.

(A) needs (B) to need (C) needing (D) need

CH 2: Xc nh s ha hp v s ca ch ng v ng t, b qua cm t b ngha cho


danh t lm ch ng

Chn cu ng
The plans for the project is prepared by Mr. Kim.
The plans for the project are prepared by Mr. Kim.
ng k hc: 0962 60 8801 04 6260 3948 93
a ch: S 18 Trn i Ngha Q Hai B Trng H ni
NGOI NG 24H 94
WWW.NGOAINGU24H.VN

p n: The plans for the project are prepared by Mr. Kim.

cu trn ta c th hiu nhm danh t project gn ng t are l ch ng, nn d chia thnh dng
s t is. xc nh ch ng chnh xc, ta nn (1) b cm t b ngha gii t (prep) + danh t [for
the project] vo du ngoc v (2) tm cu trc ch ng - ng t (plans - are) thng nht v s
ca ch ng v ng t.

Kin thc b sung


Phng php xc nh s ha hp v s ca ch ng v ng t bng cch tch cm t b ngha.
1. Xc nh ng t trc.
2. Tm danh t xc nh ch ng.
1. Tm danh t u cu xc nh ch ng.
2. Nu c danh t nm sau gii t th b gii t + danh t (cm t b ngha) vo du
ngoc.
3. Cm t b ngha chnh l by!
3. Nu nm vng c cu trc ch ng - ng t v b qua cm t b ngha th s tr li c
cu hi v s ca ch ng v ng t.

Bi tp 4:
1. The conditions for a promotion (are/ is) clearly explained in this handbook.
2. The material for the workshops (are/ is) going to be distributed.
3. The use of online games (have/ has) caused problems.
4. The committee for the 10th anniversary party (need/ needs) ideas.
5. All items in stock at this store (are/ is) discounted.

ng k hc: 0962 60 8801 04 6260 3948 94


a ch: S 18 Trn i Ngha Q Hai B Trng H ni
NGOI NG 24H 95
WWW.NGOAINGU24H.VN

Tm tt:

The material
Ch ng s t The use ng t s t
The committee
Cm t b ngha
All items
Ch ng s nhiu ng t s nhiu
The conditions

Bi tp 5:
1. Centers for public health_________become popular.
(A) have (B) has (C) having (D) to have
2. The prices in the new product catalog_________discounted by 10 percent.
(A) be (B) are (C) is (D) was
3. The newspaper company_________lowering subscription rates.
(A) are (B) were (C) has (D) is
4. Many prizes_________awarded to the marketing department.
(A) is (B) were (C) was (D) be
5. The suggestions in the report_________to be reviewed.
(A) need (B) needy (C) needs (D) needing

ng k hc: 0962 60 8801 04 6260 3948 95


a ch: S 18 Trn i Ngha Q Hai B Trng H ni
NGOI NG 24H 96
WWW.NGOAINGU24H.VN

BI TP LUYN
Part V Chn t thch hp in vo ch trng di y.
Bi tp 1:
1. The new museum________ to attract many tourists.
(A) continue (B) continues (C) continuing (D) continuity
2. The opening date of the new public libraries ________ been postponed.
(A) have (B) having (C) to have (D) has
3. The shipment of your orders ________ delayed due to some defective products.
(A) have (B) were (C) are (D) was
4. The new employee guidebook ________ finally distributed to the new employees.
(A) was (B) are (C) were (D) has
5. The contents of this report ________ thoroughly checked for errors.
(A) is (B) are (C) be (D) was

Part VI in vo ch trng trong mu qung co sau.

U.S. UNIVERSITY FAIR


ASIA FALL 2008
All interested students _______ invited to attend the U.S. University Fair.
6. (A) was
(B) are
(C) be
(D) is
Admission to the fair _______ free.
7. (A) were
(B) is
(C) are
(D) have
If you _______ to join the fair, please register now.
8. (A) wish
(B) wishing
(C) to wish
(D) wishes

ng k hc: 0962 60 8801 04 6260 3948 96


a ch: S 18 Trn i Ngha Q Hai B Trng H ni
NGOI NG 24H 97
WWW.NGOAINGU24H.VN

Bi tp 2:
Choose one correct options in the parentheses.
1. Holding the conference in our hotel (are, is) advantageous to your company.
2. The requested document (has, have) not yet arrived.
3. Access to this file (are, is) limited to authorized personnel.
4. Each report (contains, contain) information on key economic indicators.
5. Thar Mr. Smith was absent from work (were, was) suprising.
6. The editor (want, wants) to change the layout of the newspaper.
Choose one correct options for each of the following sentences.
1. The firm ___ profit increases with the new marketing strategy.
(A) anticipate (C) are anticipated
(B)anticipates (D) anticipating
2. This chemical ____ only approved for use in animal foods.
(A) was (C) has
(B) were (D) have
3. Obtaining a good job ____ difficult due to the economic situation.
(A) have remained (C) remain
(B) are remaining (D) remains
4. Every area manager ____ required to visit all the offices under his management once a week.
(A) is (C) were
(B) are (D)have

Bi tp 3:
Choose one correct option in the parentheses.
1. The applicants (have, has) plenty of experience in foreign trade.
2. Many offices (instruct, instructs) employees about what to do in case of lire.
3. The survey forms (was, were) not filled out completely.
4. The headquarters and the branch office (plan, plans) to reorganize some divisions
5. A few reports (provides, provide) accurate data on the current market.
6. Our facilities (are, is) available for family and company events every weekend.
Choose the correct option for each of the following sentences.
7. The company and the striking workers _____ to a settlement.
(A) has agreed (C) have
(B) agrees (D) agreeing
8. The cellphone charges for local calls ____ discounted for the first month.
(A) has (C) is
(B) was (D) are

Bi tp 4:
Choose one correct option in the parentheses.
1. All of the old magazines (is, are) available at a discounted price.
2. Some of the executives (refuse, refuses) to reduce expenditures on advertising.
3. The competition between low-cost airlines (has, have) increased.
ng k hc: 0962 60 8801 04 6260 3948 97
a ch: S 18 Trn i Ngha Q Hai B Trng H ni
NGOI NG 24H 98
WWW.NGOAINGU24H.VN

4. Senior employees in the workplace (know, knows) less about computers.


Choose the correct option for each of the following sentences.
5. The rest of the guests for the Best Employee Award ____ to arrive after 7:00 p.m.
(A) expects (C) are expected
(B) expecting (D) is expected
6. Some of the pens in the shipment ______ faulty parts and cannot be used.
(A) has (C) was
(B) is (D)have
Questions 7 and 8 refer to the following letter.

Dear Mr. Woods,

Half of the emplyees in the system maintenance department ____ recently. We think that

7. (A) have quit (C) quitting


(B) was quit (D) quits

dissatisfaction with their jobs ____ the cause. Therefore, we would like to involve employees in

8. (A) are (C) were


(B) is (D) have

decision-making by having monthly meetings with management. Also, they will be granted stock
options beginning next month. We believe these steps can help the company retain its workers.

Bi tp 5:
Practice with TOEIC Actual Questions
Decide which of the choices - (A), (B), (C), or (D) - best completes the sentence.
1. The article ____ that the population of the world is increasing.
(A) saying
(B) says
(C) say
(D) is said
2. The number of online shopping malls ______ increasing.
(A) is
(B) are
(C) to be
(D) being
3. Both of the applicants _____ scheduled for an interview today.
(A) to be
(B) was
(C) is
(D) are
4. Some of the luggage ______ stolen at the airport.
(A) has
ng k hc: 0962 60 8801 04 6260 3948 98
a ch: S 18 Trn i Ngha Q Hai B Trng H ni
NGOI NG 24H 99
WWW.NGOAINGU24H.VN

(B) are
(C) was
(D) were
5. The president and the vice president ______ in the conference room.
(A) are
(B) being
(C) is
(D) was
6. The good news ____ that I will get a pay raise.
(A) have
(B) were
(C) are
(D) is
7. Every country _____ interested in the global trade issue.
(A) have
(B) are
(C) is
(D) has
8. Most of the interviewees ____ very punctual.
(A) is
(B) are
(C) to be
(D) being
9. The sales meeting ____ place in the auditorium every two months.
(A) is taking
(B) is taken
(C) take
(D) takes
10. Tim Scott, the founder and CEO of the company, ____ in 2000.
(A) died
(B) die
(C) is dying
(D) is died
11. Fortunately, neither the driver nor the passengers ______ injured.
(A) had
(B) has
(C) was
(D) were
12. The prime minister was _____ in a bribe scandal.
(A) improved
(B) involved
(C) equipped
(D) concerned
13. This cellular phone is superior ______ the competitors in quality.
ng k hc: 0962 60 8801 04 6260 3948 99
a ch: S 18 Trn i Ngha Q Hai B Trng H ni
NGOI NG 24H 10
WWW.NGOAINGU24H.VN 0

(A) for
(B) to
(C) on
(D) than
14. Prices may be ____ to change without notice.
(A) subject
(B) engaged
(C) payable
(D) conscious

Questions 15-16 refer to the following letter.

Dear Mr. Rosenberg,

A number of the workers in the factory ______ off lately. That will have a bad effect on the rest of
15. (A) has been laid
(B) have been laid
(C) are laying
(D) is laid

the workers in our company. Some of them _____ already quit their jobs.
16. (A) have
(B) had
(C) has
(D) having

Therefore, we suggest that you hold a meeting to explain the financial situation of the company. This
will help the employees want to keep working here.

BI TP KIM TRA
1. Physics _______ one of my favorite subjects.
(A) be
(B) to be
(C) is
(D) are
2. There are some books on the table, which_______ about fish.
(A) does
(B) do
(C) is
(D) are
3. A third of the students_______ passed the test.
(A) are
(B) have
(C) has
(D) is
4. The bus comes here_______ 30 minutes.
(A) each

ng k hc: 0962 60 8801 04 6260 3948 100


a ch: S 18 Trn i Ngha Q Hai B Trng H ni
NGOI NG 24H 10
WWW.NGOAINGU24H.VN 1

(B) another
(C) every
(D) all
5. Teaching children_______ not easy.
(A) are
(B) is
(C) were
(D) am
6. Either Bill or we _______supposed to contact Sylvia about the meeting.
(A) is
(B) could
(C) was
(D) were
7. Kim Jones, together with her roommate, _______ to write a letter to the campus newspaper.
(A) will
(B) are
(C) is going
(D) wills
8. Most of the fish I caught _______ too small to bring home.
(A) will
(B) am
(C) were
(D) was
9. Two-quarters of the land _______ sold to investors.
(A) will
(B) have
(C) have been
(D) has been
10. Neither Professor Johnson nor his students _______ going to join the project.
(A) will
(B) is
(C) are
(D) either
11. Fifty cents_______ how much I owe you.
(A) does
(B) are
(C) do
(D) is
12. Nowadays, most _______ have four wheels.
(A) cars
(B) of the cars
(C) the cars
(D) among the cars
13. There was_______ left in the safe.
(A) two-hundred dollar bills
(B) two-hundreds dollar bills
(C) two-hundred dollars
(D) two-hundreds dollars
14. There are no_______ between the brothers.
ng k hc: 0962 60 8801 04 6260 3948 101
a ch: S 18 Trn i Ngha Q Hai B Trng H ni
NGOI NG 24H 10
WWW.NGOAINGU24H.VN 2

(A) same
(B) difference
(C) similarities
(D) variation
15._______ people are not happy with the class.
(A) Most of
(B) Every
(C) Many a
(D) A number of
16. The students, as well as the teacher, _______ happy about the test results.
(A) was
(B) are
(C) is
(D) has
17. All of the food_______ prepared by seven.
(A) were
(B) is
(C) have
(D) was
18. All of the people there_______ from Japan.
(A) were
(B) is
(C) be
(D) was
19. _______ remain complicated economic problems to be resolved, while the economic conditions
have been good.
(A) They
(B) It
(C) There
(D) That
20. Applicants for the managerial position_______ to possess high levels of motivation along with
basic computer skills.
(A) are required
(B) require
(C) requires
(D) has required
21. Please_______ any information, no matter how insufficient it may seem, to your client in the
very near future.
(A) forwards
(B) forward
(C) forwarding
(D) to forward
22. The revised version of an unpublished manuscript_______ due to arrive this morning at 10
o'clock, but unexpected problems delayed the shipping.
(A) was
(B) were
(C) is
(D) are

ng k hc: 0962 60 8801 04 6260 3948 102


a ch: S 18 Trn i Ngha Q Hai B Trng H ni
NGOI NG 24H 10
WWW.NGOAINGU24H.VN 3

23. When I closed the window, all of the files and folders on my desktop computer_______ except
for the icon.
(A) are disappearing
(B) will have disappeared
(C) had disappeared (
D) disappear
24. Due to recent changes, Nordon Express_______ delivery of equipment effective the first day of
the coming year.
(A) to discontinue
(B) will discontinue
(C) discontinued
(D) have discontinue
25. As portable audio items_______ currently unavailable in the store, you'd better check the Internet
for more information.
(A) is
(B) been
(C) are
(D) being
26. Employment application forms completed by applicants must be_______ to Mr. Keith by Friday
at noon.
(A) submitting
(B) submitted
(C) submit
(D) submission
27. Most stores _______special discounts, coupons on gifts and personalized gifts to retain their
existing customers.
(A) offering
(B) be offered
(C) to offer
(D) are offering.
28. Even though assigned tasks are somewhat complex, we will have to complete them before
we_______ for the day.
(A) leave
(B) leaving
(C) to leave
(D) have left
29. All the storage rooms are scheduled to undergo renovations and_______ closed for several days
next week.
(A) will be
(B) was
(C) are
(D) has been
30. According to many_______ of the industry, the majority of small-sized firms are hoping to enter
into lucrative businesses.
(A) survey
(B) surveys
(C) surveying
(D) surveyed

ng k hc: 0962 60 8801 04 6260 3948 103


a ch: S 18 Trn i Ngha Q Hai B Trng H ni
NGOI NG 24H 10
WWW.NGOAINGU24H.VN 4

31. Ms. Watanabe will be unable to finish filing the data by Friday unless she_______ one of the
office interns to help her.
(A) got
(B) has gotten
(C) gets
(D) will get
32. The factory manager reports that currently all the machinery_______ at full capacity.
(A) will have been operated
(B) was operating
(C) had operated
(D) is operating
33. _______ you require assistance with the installation of the product, contact a customer sales
associate at the number below.
(A) Would
(B) Have
(C) Should
(D) Had
34. Alterations to the work schedule cannot be made until they_______ by the shift supervisor.
(A) will be cleared
(B) were cleared
(C) are cleared
(D) will be clearing
35. Mr. Walter's business partners, who_______ a venture capital firm in Santa Barbara, will be at
the investors' meeting tomorrow.
(A) manage
(B) manages
(C) management
(D) managing
Questions 36 through 37 refer to the following advertisement.

World famous folk singer, Daniel Matthews, _______with the Smallville Choir, will
36. (A) next
(B) along
(C) by
(D) who
give a small concert on August 27th. Mr. Matthews will perform songs from his new CD Voices of
Heaven. Tickets are limited so_______ fans who missed his last concert
37. (A) every
(B) another
(C) all
(D) each
should hurry! Tickets go on sale on June 306. Call the Smallville Arts Hall on 021-333-0999 for
more information.

ng k hc: 0962 60 8801 04 6260 3948 104


a ch: S 18 Trn i Ngha Q Hai B Trng H ni
NGOI NG 24H 10
WWW.NGOAINGU24H.VN 5

Questions 38 through 40 refer to the following letter.

Woodworks
23 Culver Lane
Westford
12034

Alice Burns
934 Garden Highway
Fremont
Dear Ms. Burns,
Dank you for your recent enquiries about "Woodworks: Woodworks_______ a small
38. (A) are
(B) be
(C) is
(D) has
family-owned company. We are in our sixth year of business. We specialize in hand-carved pieces.
We_______ local wood and make sure three trees are planted to replace
39. (A) uses
(B) use
(C) used to
(D) using
every tree we use. Woodworks has its own forest, and all of our products are made from trees that
grow there. As you say, we are a little expensive. However, we make a unique, high-quality product.
Every piece we make is designed individually. It takes about six months to make______- of our
products. Of course, this costs a lot of money.
40. (A) every
(B) everyone
(C) each
(D) anyone
Please enjoy the enclosed catalog.
Sincerely,

ng k hc: 0962 60 8801 04 6260 3948 105


a ch: S 18 Trn i Ngha Q Hai B Trng H ni
NGOI NG 24H 10
WWW.NGOAINGU24H.VN 6

Questions 41 through 43 refer to the following note.

Dear Sally,

I am going to_______ grandmother's house this afternoon. I will be out when you get
41. (A) the
(B) our
(C) your
(D) yours

home from school, so you will need to get your own dinner. Help yourself_______ anything in the
42. (A) with
(B) to
(C) by
(D) over

refrigerator. There is some cheese and plenty of vegetables. There is bread, too. You can
pick_______ apples from the tree in the garden if you want. As a
43. (A) any
(B) some
(C) all
(D) almost

special treat, I have bought some chocolate fudge ice cream. Don't eat too much - I want to try some,
too! Do your homework, OK? be home about eight o'clock. See you later.

Mom

ng k hc: 0962 60 8801 04 6260 3948 106


a ch: S 18 Trn i Ngha Q Hai B Trng H ni
NGOI NG 24H 10
WWW.NGOAINGU24H.VN 7

Questions 44 through 46 refer to the following memo.

Memo
To: Sales Staff
From: Marketing
Re: Christmas sales target
Date: October 15th

It is time to start thinking_______ our Christmas sales. Last year, we saw very low profits, so we
44. (A) on
(B) about
(C) for
(D) off

really need to improve things this year. The store wants to double its profits, so we have a lot of
work to do. We will have a meeting next Tuesday at three o'clock to discuss this. Please think of
some good ideas to increase sales. If we reach the new target, _______ will get a bonus.
45. (A) everyone
(B) no one
(C) anyone
(D) each

If we are not successful, there will be no bonuses. The store manager will come to the meeting. He is
going to_______ us his ideas and listen to our opinions.
46. (A) say
(B) talk
(C) tell
(D) speak
This is a very important meeting. Don't miss it.

ng k hc: 0962 60 8801 04 6260 3948 107


a ch: S 18 Trn i Ngha Q Hai B Trng H ni
NGOI NG 24H 10
WWW.NGOAINGU24H.VN 8

Questions 47 through 49 refer to the following advertisement.

Getaway Tours

We now have the details of our Summer Specials. As always we have some real treats. Just take a
look at a sample of_______ we have on offer this year six
47. (A) that
(B) what
(C) which
(D) why

nights in Guam, staying in a luxury hotel with breakfast included - starts at just $50 per person per
night. Five nights In Hong Kong, staying in a 5 star hotel, including a guided harbor boat cruise -
starts at just $75 per person per night. _______ of our
48. (A) Almost
(B) All
(C) Nearly
(D) At all

prices include travel insurance and free transportation from the airport _______to your hotel. Give
49. (A) straightly
(B) directly
(C) firstly
(D) immediate

us a call at 234-0009 for reservations or further information.

ng k hc: 0962 60 8801 04 6260 3948 108


a ch: S 18 Trn i Ngha Q Hai B Trng H ni
NGOI NG 24H 10
WWW.NGOAINGU24H.VN 9

NGY 7:
TNH T V TRNG T

A- TNH T
CH 1: Tnh t ng trc danh t v b ngha cho danh t
Ng php b sung
BI TP LUYN

B- TRNG T
CH 2: Trng t b ngha cho tnh t
Ng php b sung
CH 3: Trng t b ngha cho ng t
Ng php b sung
CH 4: Trng t thng chen vo gia
Ng php b sung
TM TT
CH 5: Trng t b ngha cho ng t
1. Trng t ng cui mt cu hon chnh, b ngha cho ng t
2. Trng t ng sau ni ng t, b ngha cho ni ng t

Mt s cm ni ng t - trng t ph bin

BI TP LUYN
BI TP KIM TRA

ng k hc: 0962 60 8801 04 6260 3948 109


a ch: S 18 Trn i Ngha Q Hai B Trng H ni
NGOI NG 24H 11
WWW.NGOAINGU24H.VN 0

A- TNH T
CH 1: Tnh t ng trc danh t v b ngha cho danh t

Bi tp 1: Chn cu ng.
1. the attractive building 2. Kathys create idea
the attraction building Kathys creative idea

Tnh t:
(1) gia mo t v danh t, b ngha cho danh t
(2) gia s hu cch v danh t, b ngha cho danh t theo sau
Hn na:
(3) Tnh t trc danh t l tn ng ca ngoi ng t, b ngha cho danh t.

Ng php b sung
V tr ca tnh t b ngha cho danh t:
1. Mo t [a/an/the] + tnh t + danh t: tnh t nm gia mo t v danh t, b ngha cho danh t.
an impressive test result the popular online store

mo t adj n mo t adj n
2. T s hu [my / your / his / our /their / its / her] / s hu cch + tnh t + danh t: tnh t nm
gia t s hu v danh t, b ngha cho danh t.
its high profit CEO's important decision
t s hu adj n s hu cch adj n

Bi tp 2: Chn t ng trong cc cu sau.


1. He received the award for his (creative/ create) idea.
2. The special team will conduct an (extend/ extensive) inspection.
3. The country will experience an (annual/ annually) growth rate of 10 percent.
4. We will conduct a (thoroughly/ thorough) inspection of the facility.
5. We are not able to meet the (presently/ present) production schedule.

Bi tp 3: Nm cu trc: mo t / s hu cch + ... + danh t


1. We are looking for an__________ person for our sales department.
(A) experienced (B) experience (C) experiences (D) to experience
2. The president had a meeting with employees during his__________ visit to the plant.
(A) recently (B) recentness (C) recency (D) recent
3. The__________ attractions of the city continue to attract visitors.
(A) diversification (B) diversifying (C) diverse (D) diversity
4. We recommended a__________ review of all of the facilities in the hospital.
(A) comprehensive (B) comprehend (C) comprehensively (D) comprehensiveness

ng k hc: 0962 60 8801 04 6260 3948 110


a ch: S 18 Trn i Ngha Q Hai B Trng H ni
NGOI NG 24H 11
WWW.NGOAINGU24H.VN 1

5. Dr. Marriot recently completed a__________ study on economic trends.


(A) detail (B) details (C) in detail (D) detailed

BI TP LUYN
Bi tp 1:
Part V Chn t thch hp in vo ch trng di y.
1. Our company recorded _________ profits this year.
(A) impress (B) to impress (C) impresses (D) impressive
2. The university is seeking _________ individuals to participate in a survey.
(A) interest (B) interests (C) interested (D) interestingly
3. Managers will discuss _________ methods for the promotion of our products.
(A) addition (B) additional (C) additionally (D) additions
4. Due to the recent _________ conditions, the company made a lot of money.
(A) economy (B) economic (C) economist (D) economically
5. All _________ information should be submitted by next week.
(A) necessarily (B) necessity (C) necessaries (D) necessary

Part VI in vo ch trng trong mu qung co sau.

Business Cards

BizCard offers great products and services with ________ prices.


6. (A) excellence
(B) excellently
(C) excellent
(D) excel
We sometimes have sales and promotions for our ________ products.
7. (A) reliable
(B) reliably
(C) reliability
(D) rely
Our ________ designers guarantee the most impressive business cards for your business.
8. (A) experience
(B) experiential
(C) experientially
(D) experienced
On our website, you can find some of BizCards most amazing offers.

ng k hc: 0962 60 8801 04 6260 3948 111


a ch: S 18 Trn i Ngha Q Hai B Trng H ni
NGOI NG 24H 11
WWW.NGOAINGU24H.VN 2

B- TRNG T
CH 2: Trng t b ngha cho tnh t
Trng t l mt trong nhng loi t c t cu hi nhiu nht trong bi thi TOEIC. V trng t c
vai tr rt a dng: b ngha cho ton b cu, b ngha cho tnh t, ng t, trng t trong cu nn c
th gy kh khn nht nh cho hc vin. Tuy nhin, nu nm c nguyn tc th vic gii quyt
cc cu hi v trng t cng kh d dng.

Ng php b sung
1. Hnh thi ca trng t
=> Thng c cu to adj + -ly.
Tnh t + - ly = Trng t
increasing increasingly
current currently
close closely

2. Chc nng ca trng t


=> Trng t ng trc tnh t v b ngha cho tnh t.
a popular hobby => an increasingly popular hobby
mo t adj n mo t adv adj n
a competitive market => an extremely competitive market
mo t adj n mo t adv adj n

It is_______easy to get information through the Internet.


=> L v tri cn mt loi t b ngha cho tnh t easy nn phi l trng t v trng t increasingly
c ngha ph hp trong cu.

Bi tp 4: Chn t ng trong cc cu sau.


1. The (new/ newly) opened plant will hire more workers.
2. Reading is becoming an (increasing/ increasingly) popular hobby.
3. You can buy this product at (significantly/ significant) discounted prices.
4. It is (extremely/ extreme) important to finish this report by noon.
5. We need to develop (clear/ clearly) planned marketing strategies.

[Mo t + ... + tnh t + danh t => phi in trng t b ngha cho tnh t

Bi tp 5:
1. Our products are doing well in the _________ competitive market
(A) extreme (B) extremely (C) extremity (D) extremeness
2. Most companies provide _________ low wages to interns.
(A) signified (B) significantly (C) significant (D) signify
3. Patients private health records will be kept _________ confidential.
(A) completely (B) complete (C) completion (D) completed
ng k hc: 0962 60 8801 04 6260 3948 112
a ch: S 18 Trn i Ngha Q Hai B Trng H ni
NGOI NG 24H 11
WWW.NGOAINGU24H.VN 3

4. _________ discounted tickets have many restrictions.


(A) Heavy (B) Heavily (C) Heaviness (D) Heavier
5. They worked together on a _________ profitable development plan.
(A) highly (B) high (C) highness (D) higher

CH 3: Trng t b ngha cho ng t


Trng t b ngha cho ng t c v tr ng trc ng t b sung ngha ca hnh ng (ng
t) Trng hp khi trng t ch s thng xuyn din t thi quen, trng t c t trc ng t
thng v sau ng t be.

Ng php b sung
1. Trng t b ngha cho ng t
=> Trng t b ngha cho ng t din t thi gian, ni chn, mc v tnh thng xuyn xy ra
hnh ng (ng t) ca ch ng.
We frequently meet with regional suppliers.
Trng t frequently b ngha cho ng t din t mc thng xuyn xy ra hnh ng ca ch
ng. Trng t ny c v tr ng trc ng t meet.

2.V tr ca trng t b ngha cho ng t


=> Trng t ng trc ng t - gia ch ng v ng t - c chc nng b ngha cho ng t.
We recently employed several programmers.
Trng t recently c v tr ng gia ch ng v ng t, c chc nng b ngha cho ng t
employed.
=> Trng t ch tn sut c v tr ng trc ng t thung v sau ng t be.
Kathy always feels tired afternoon.
trng t ch tn sut ng t thng
Kathy is always late for the meeting.
ng t be trng t ch tn sut

Bi tp 6: Chn t ng trong cc cu sau.


1. He (original/ originally) suggested the ideas for the advertising campaign.
2. The manager (direction/ directly) reports to the president.
3. The company (unfairly/ unfair) raised the subscription rates.
4. He (thorough (thoroughly) reviews the monthly reports.
5. Every employee (secure/ securely) locks the door after work.

Bi tp 7: Tm v tr ca trng t b ngha cho ng t trong cu.


1. We recommend that you ________ purchase your home.
(A) quickly (B) quick (C) quicken (D) quickness
2. Our production team ________ inspects the quality of our Products.
(A) through (B) thoroughness (C) thoroughly (D) thorough
3. The stock analyst ________ predicted the increase in stock prices.
(A) correct (B) correctly (C) corrective (D) correctness
4. Most interviewers ________ examine applicants educational backgrounds.
(A) closely (B) closer (C) close (D) closest
5. Small-business owners ________ use local newspaper advertisements.
(A) frequent (B)frequency (C) frequentness (D) frequently

ng k hc: 0962 60 8801 04 6260 3948 113


a ch: S 18 Trn i Ngha Q Hai B Trng H ni
NGOI NG 24H 11
WWW.NGOAINGU24H.VN 4

CH 4: Trng t thng chen vo gia


phn trc, chng ta bit v tr ca trng t l ng trc v b ngha cho ng t v tnh t.
ngoi ra n cng b ngha cho ng t nhng mt s v tr khc.

Ng php b sung
1. Trng t:
- Trng t nm gia cu trc tng hp cu ng t v b ngha cho ng t. Ngoi trng t ra,
khng c loi t no c vai tr nh vy.
2. Cc trng hp ng gia ca trng t:
a. Tr ng t + trng t + ng t nguyn mu
Eg: The landlord will probably raise the rent.
( trng t probably ng gia tr ng t will v dng t nguyn mu raise, b ngha cho
ng t)
b. ng t have + trng t + qu kh phn t
Eg: The applicant has clearly shown his interest in the position.
( Trng t clearly ng gia tr ng t hon thnh have v qu kh phn t shown, b ngha
cho ng t.
c. ng t be + trng t + phn t (V-ing/p.p)
Eg: the directions to the meeting room are easily displayed.
( trng t easily ng gia ng t be v qu kh phn t displayed, b ngha cho ng t)

Bi tp 8: Chn t ng cho cc cu sau:


1. Interest rates will (probably/ probable) rise by 0.5 percent.
2. Every office door should be (secure/ securely) closed.
3. The company has (consistent/ consistently) donated money.
4. You can (easily/ easy) find our products in retail stores.
5. The local museum is (current/ currently) closed for renovations.

( [tr ng t + + ng t nguyn mu ] / [be + + phn t]


=> in trng t b ngha cho ng t vo trng)

TM TT

Tr ng t/ ng t be + trng t (probably/ securely/ consistently/ easily/ currently) + ng t


nguyn mu/ phn t(p.p/V-ing)

ng k hc: 0962 60 8801 04 6260 3948 114


a ch: S 18 Trn i Ngha Q Hai B Trng H ni
NGOI NG 24H 11
WWW.NGOAINGU24H.VN 5

Bi tp 9: Ch v tr ng gia ca trng t.
1. We are _____ seeking new university graduates to join our sales team.
A. Current
B. Currency
C. Currently
D. Currentness
2. Identification cards must be _____ displayed.
A. Clear
B. Clearly
C. Clearance
D. Clarity
3. The result was _____ predicted before this quarter.
A. original
B. origin
C. originally
D. originality
4. The hotel is _____ located within easy walking distance of the beach.
A. Perfect
B. Perfectly
C. Perfection
D. Perfecting
5. The company is _____ implementing new vacation policy.
A. Active
B. Activity
C. Actively
D. Activate

ng k hc: 0962 60 8801 04 6260 3948 115


a ch: S 18 Trn i Ngha Q Hai B Trng H ni
NGOI NG 24H 11
WWW.NGOAINGU24H.VN 6

CH 5: Trng t b ngha cho ng t


1. Trng t ng cui mt cu hon chnh, b ngha cho ng t

Before you install this software, please read the directions carefully.
carefully b ngha cho read
Trc khi ci t phn mm ny, xin vui lng c k hng dn.

2. Trng t ng sau ni ng t, b ngha cho ni ng t


Nhng ng t i km vi gii t m khng c tn ng c gi l ni ng t. Trong trng hp
ny, trng t ng gia ni ng t v gii t.

You need to respond promptly to questions from customers.


promptly b ngha cho respond
Anh/ch cn tr li cc cu hi ca khch hng ngay tc khc.

Mt s cm ni ng t - trng t ph bin
travel regularly grow rapidly
increase dramatically decrease slightly
work collaboratively meet frequently with ~
react calmly to delays rely heavily on tourism
speak clearly to the audience communicate easily with ~

Bi tp 10:
1. Due to the financial constraints, the company had to reduce its travel budget_________ .
(A) substantial (B) substance (C) substantiate (D) substantially
2. The population of this city has decreased_________ for the last few months.
(A) slight (B) slightly (C) slightness (D) slighted

BI TP LUYN

PART V: Incomplete Sentence


Bi tp 2:
1. The teleconference is scheduled to start ____at 10 A.M. next Tuesday.
(A) precisely (B) preciseness (C) precision (D) precise

2. Replacing the old building with a new one was a ____ impossible task.
(A) near (B) nearby (C) nearly (D) nearing

3. The construction site makes a visual impact that is ____ .


(A) impress (B) impression (C) impressed (D) impressive

4. The sales figures have been revised ____ due to the miscalculation.
(A) significant (B) significantly (C) significance (D) signification

ng k hc: 0962 60 8801 04 6260 3948 116


a ch: S 18 Trn i Ngha Q Hai B Trng H ni
NGOI NG 24H 11
WWW.NGOAINGU24H.VN 7

5. The sales staff was especially ____ and willing to help the customers.
(A) friend (B) friends (C) friendly (D) friendship

6. The team members would reach an agreement under more ____ circumstances.
(A) favor (B) favorite (C) favorable (D) favorably

7. The insurance company does not seem to be _____ sound.


(A) finance (B) finances (C) financial (D) financially

8. Best Computers is ____to increase its customer base in European countries.


(A) frequent (B) common (C) large (D) eager

9. New York state residents are ____to apply for a NYS library card.
(A) capable (B) valuable (C) eligible (D) flexible

10. Students who have not received their IDs should visit the Registrars Office ____.
(A) recently (B) especially (C) unexpectedly (D) immediately

PART VI: Text Completion

Questions 11-13 refer to the following letter.

Dear Ms. Angela,

Please be informed that we have recently obtained your request to renew your subscription to
Economy Weekly. Your new 2-year subscription will start on May 8,
2008, and the payment of $48.00 is due on the first day of each month ____the end of
11. (A) because of
(B) beyond
(C) regarding
(D) throughout

your contract term. In the ____ that you need to contact us regarding this subscription,
12. (A) affair
(B) event
(C) happening
(D) opening

please include your full subscription number (EW-200805223) on any correspondence with us.

This ____a timely response and efficient processing of your request or problem.
13. (A) ensure
(B) has ensured
(C) will ensure
(D) ensured

Respectfully,

Serena Glory
ng k hc: 0962 60 8801 04 6260 3948 117
a ch: S 18 Trn i Ngha Q Hai B Trng H ni
NGOI NG 24H 11
WWW.NGOAINGU24H.VN 8

Economy Weekly

Bi tp 3:
Choose one correct option in the parentheses.
1. Consumers offer (construct, constructive) advice on our new product lines.
2. (Success, Successful) candidates require excellent communication skills.
3. The service at the automobile repair center is (quick, quickly).
4. The applicants were provided with (specific, specify) guidelines on what to submit.
5. The Pinesville Resort is the (ideal, ideally) place to spend a holiday.
6. Susan is not (surely, sure) if she will accept the position.
Choose the correct option for each of the following sentences.
7. Ms. Bell is an especially ____ employee at Ewing, Inc.
(A) value (C) valuably
(B) values (D) valuable
8. Depositors should bring the _____ identification to complete a transaction.
(A) necessity (C) necessitate
(B) necessarily (D) necessary
9. All the visitors to the power plant must wear _____ clothing provided when entering the
laboratory.
(A) protect (C) protective
(B) protection (D) protecting
10. The new R&D head was highly ______ of our procedures for collecting data.
(A) critical (C) criticize
(B) critically (D) criticizes

Bi tp 4:
Choose one correct option in the parentheses.
1. Ms. Byrd (usually, usual) makes coffee when she arrives at the office.
2. The president (final, finally) decided to close down the old branch.
3. The parcel was sent (mistake, mistakenly) to the wrong address.
4. The unemployment rate has been decreasing (relative, relatively) slowly.
5. Please make sure that the invitation is addressed (correctly, correct) before mailing it.
6. The executive director travels (regularly, regular) for business.

Choose the correct option for each of the following sentences.


7. Mr. Gifford said the network system will be ____ operational by January.
(A) completion (C) completing
(B) complete (D) completely
8. All supervisors in the company agreed that Mr. Yoshio works _______.
(A) conscientiously
(B) conscientious
(C) conscientiousness
(D) conscience
ng k hc: 0962 60 8801 04 6260 3948 118
a ch: S 18 Trn i Ngha Q Hai B Trng H ni
NGOI NG 24H 11
WWW.NGOAINGU24H.VN 9

9. Consumer opinion is becoming an ______ essential factor in project planning.


(A) increase (C) increasing
(B) increases (D) increasingly
10. We ____ inform our subscribers that Kotins Press will stop publishing this month.
(A) regrettable
(B) regretfully
(C) regret
(D) regrets

Bi tp 5:
Choose one correct option in the parentheses.
1. The election was (close, closely) observed by the media.
2. The travel agency offers (reliant, reliable) service.
3. (Near, Nearly) 70% of companies say that their business operations are profitable.
4. The pilot is (responsible, responsive) for the safety of the passengers.
Choose the correct option for each of the following sentences
5. The attached file is a list of professors who will speak at the seminar.
(A) distinguish (C) distinguishably
(B) distinguished (D) distinguishable
Another branch could have been established under more _____ conditions.
(A) favorable (C) favorably
(B) favorite (D) favorableness
Questions 7 and 8 refer to the following advertisement.

If you are not sure of how to manage your savings, contact Investment Associates. Investment
Associates is a small group of _______ advisors that assist people in making the right decisions about
7. (A) financially (C) financing
(B) financial (D) finances

their money. We can give you information on what investment strategies can most help you. The best
time to begin setting investment goals Is while you are in your twenties. It is _____ recommended
8. (A) high (C) highly
(B) height (D) higher

that you sit down with one of our advisors to determine which plans are suitable for your age group.

Bi tp 6:
Choose one correct option in the parentheses.
1. Our staff members had (temporary, differing, qualified) opinions on the matter.
2. Future economic prospects look less (common, unprecedented, promising) than expected
3. Investors feel (incapable, unanimous, insecure) about the current market.
4. It is the obligation of the company to recall (untrue, defective, differing) products.

ng k hc: 0962 60 8801 04 6260 3948 119


a ch: S 18 Trn i Ngha Q Hai B Trng H ni
NGOI NG 24H 12
WWW.NGOAINGU24H.VN 0

Choose the correct option for each of the following sentences.


5. Retail prices increased after an ______ in oil prices.
(A) abundant (C) informed
(B) accomplished (D) unprecedented
6. Mr. Gunn served as a ______ replacement for an employee who is on leave.
(A) minor (C) temporary
(B) demanding (D) lengthy
Questions 7 and 8 refer to the following memorandum.
From: Ralph F. McKinney, CEO
To: All ChemField employees

It is fairly ______ for new businesses to take some time to become profitable. However, it took
7. (A) insecure (C) even
(B) common (D) level

ChemField only two months to establish a strong customer base in the food industry. This resulted
from our _____ improvements in serving our customers needs. To celebrate our rapid growth, we
8. (A) mature (C) continuous
(B) straight (D) promising

will be giving all employees a bonus of 20 percent of their current salaries on the last day of this
month. We look forward to your ongoing loyalty and dedication to ChemField.

Bi tp 7:
Choose one correct option in the parentheses.
1. Individuals should provide (annual, qualified, accurate) contact information on packages.
2. Companies use new methods to improve (mounting, maintaining, existing) technology.
3. The food industry is currently expanding at a(n) (active, powerful, rapid) rate.
4. Workers should not feel (sincere, apprehensive, surprising) about voicing their opinions.
Choose the correct option for each of the following sentences.
5. Conducting online transactions is less _____ for the elderly due to their lack of computer skills.
(A) sophisticated (C) convenient
(B) accurate (D) protective
6. The CEO is required to attend a(n) _____ conference to determine ways to achieve yearly goals
(A) recent (C) existing
(B) annual (D) typical
Questions 7 and 8 refer to the following notice.
We are sorry to announce that as a result of system crash in our main server, some of the music data
on our website was lost. For the moment, the data we have will be _____ at www.tempmusic.com.
7. (A) available (C) defective
(B) effective (D) potential

ng k hc: 0962 60 8801 04 6260 3948 120


a ch: S 18 Trn i Ngha Q Hai B Trng H ni
NGOI NG 24H 12
WWW.NGOAINGU24H.VN 1

Please accept our apologies for any inconvenience this unexpected problem may have caused you.
8. (A) original (B) completed
(C) sincere (D) estimated

Bi tp 8:
Choose one correct option in the parentheses.
1. The team made (considerable, confidential, rapid) efforts to collect the data.
2. Everyone congratulated Mr. Park on his (modern, late, recent) promotion.
3. Any (improper, complete, appropriate) behavior will not be tolerated in the office.
4. A (recent, pleasant, broad) atmosphere improves work performance.
Choose the correct option for each of the following sentences.
5. Management expects applicants to have a _____ knowledge of economics.
(A) high (C) round
(B) whole (D) broad
6. When the installation process is _____ the "Done tab will appear on the monitor.
(A) entire (C) total
(B) accurate (D) complete
Questions 7 and 8 refer to the following memorandum.
From: Olivia Johnson
To: All staff

This memo is to refresh everyone's memory on the importance of keeping records private. Please
remember that all _____ records must be kept in locked file cabinets when not in use. The company's
7. (A) confidential (C) limited
(B) former (D) unprecedented

reputation rests upon your ability to handle _____ client information properly. Employees who fail to
8. (A) apprehensive
(B) competitive
(C) sincere
(D) sensitive

do so will be warned, and if it happens repeatedly, access to the files will be restricted.
Bi tp 9:
Choose one correct option in the parentheses.
1. Oil prices increased (simultaneously, importantly, markedly) two years ago.
2. Companies work (initially, cooperatively, fairly) to find solutions to problems.
3. Clients have (repeatedly, markedly, briefly) requested information on our new products.
4. The employee who (seriously, voluntarily, widely) took on a difficult task was promoted.
Choose the correct option for each of the following sentences.
5. The New Chinese Financial Newspaper will be released _____ in Singapore and Taiwan.
(A) typically (C) conveniently

ng k hc: 0962 60 8801 04 6260 3948 121


a ch: S 18 Trn i Ngha Q Hai B Trng H ni
NGOI NG 24H 12
WWW.NGOAINGU24H.VN 2

(B) stringently (D) simultaneously


6. The country suffered a(n) _____ high unemployment rate for years.
(A) wrongly (C) unusually
(B) improperly (D) hugely
Questions 7 and 8 refer to the following advertisement.
Welcome to the Grand Opening of the Lake Park Plaza!
Please join us as we celebrate our Grand Opening on Friday, March 16 at 1:30 p.m. The Lake Park
Plaza shopping center will _____ open for business with a ribbon cutting. The president of the
7. (A) normally (C) presently
(B) officially (D) recently

Lake Park Plaza will then give a speech about her long-term vision for the shopping center. The
speech will be followed by a party which will begin _____ at 2:30 p.m.
8. (A) randomly (C) promptly
(B) relatively (D) ultimately

Bi tp 10:
Choose one correct option in the parentheses.
1. The item is (shortly, temporarily, efficiently) out of stock.
2. The product was (conveniently, widely, directly) advertised throughout the region.
3. Passwords should be changed (lately, clearly, frequently) to prevent unauthorized access.
4. For technical reasons, the website is (currently, precisely, highly) not accessible.
Choose the correct option for each of the following sentences.
5. Shareholders should not rely _____ on the fund's investment adviser for assistance.
(A) surely (B) recently
(C) nearly (D) solely
6. The session will help participants learn how to _____ conduct a survey.
(A) accidentally (C) effectively
(B) apparently (D) informally
Question 7 and 8 refer to the following advetisment
Alpha Auto Body has served Tokyo motorists for over 20 years. Since our opening, we have _____
7. (A) broadly (C) fairly
(B) formerly (D) consistently

provided our customers with superior customers service. We are located ___ in the citys center near
8. (A) gradually (C) conveniently
(B) quickly (D) marginally

department stores and subway stations. Our facility is equipped with the latest high-tech machines
and computerized systems. We ensure you an excellent repair job at a fair price.

Bi tp 11:
ng k hc: 0962 60 8801 04 6260 3948 122
a ch: S 18 Trn i Ngha Q Hai B Trng H ni
NGOI NG 24H 12
WWW.NGOAINGU24H.VN 3

Choose one correct option in the parentheses.


1. This year's budget has been reduced (late, widely, significantly).
2. Most stores stay open (well, much, late) on weekends to accommodate shoppers.
3. The number of clients has grown (politely, dramatically, carefully) in the last few years.
4. The food at the French restaurant was (initially, reasonably, solely) priced.
Choose the correct option for each of the following sentences.
5. Our goal is to maintain a _____ beneficial relationship with our customers and users operate the
computer more community.
(A) conveniently (C) regularly
(B) mutually (D) precisely
6. The manual provides information that helps users operate the computer more _____.
(A) frequently (C) cooperatively
(B) efficiently (D) simultaneously
Questions 7 and 8 refer to the following memorandum.
From: Erica Turner, Office Manager
To: All staff

I know that not everyone in the office is comfortable using computers, but all important office
announcements will now be sent by e-mail. So, it is very important that you _____ check your e-mail

7. (A) entirely (C) regularly


(B) flexibly (D) strongly

inbox for messages. If you are having trouble using the company's e-mail system, please _____ read
8. (A) carefully (C) mildly
(B) easily (D) softly

the directions in the employee manual.

Bi tp 12:
Practice with TOEIC Actual Questions
Decide which of the choices - (A), (B), (C), or (D) best completes the sentence.
1. The _____ report shows that Chilean wines market share increased this year.
(A) various
(B) legal
(C) official
(D) productive
2. Your company should take _____ action to solve the problem.
(A) proper
(B) permanent
(C) competitive
(D) complimentary

ng k hc: 0962 60 8801 04 6260 3948 123


a ch: S 18 Trn i Ngha Q Hai B Trng H ni
NGOI NG 24H 12
WWW.NGOAINGU24H.VN 4

3. The president said that it was the _____ meeting he has ever had.
(A) more productive
(B) productive
(C) most productive
(D) productivity
4. The team made a presentation in a very _____ way.
(A) impress
(B) impressive
(C) impressiveness
(D) impressively
5. We need to finish the report by next Friday at the _____.
(A)later
(B) late
(C) latest
(D) lately
6. Sales directors should go _____ to Room 100 for another meeting.
(A) direct
(B) directly
(C) direction
(D) directive
7. We found the manual absolutely _____ in dealing with customer complaints.
(A) importantly
(B) importance
(C) important
(D) importing
8. The more motivated the workers are, the _____ the productivity becomes.
(A) high
(B) higher
(C) highly
(D) more highly
9. Please read the instructions as _____ as possible.
(A) careful
(B) care
(C) carefully
(D) more carefully
10. The budget cut made our project _____ more difficult.
(A) many
(B) very
(C) so
(D) even
11. Make sure you have all the _____ data with you.
(A) need
(B) necessary
ng k hc: 0962 60 8801 04 6260 3948 124
a ch: S 18 Trn i Ngha Q Hai B Trng H ni
NGOI NG 24H 12
WWW.NGOAINGU24H.VN 5

(C) necessarily
(D) needs
12. They _____ renovated the stores in order to attract more customers.
(A) late
(B) recently
(C) likely
(D) hard
13. We will discuss who is _____ for the position.
(A) afraid
(B) aware
(C) eligible
(D) subject
14. The solar panels were, not so_____ as we had anticipated.
(A) efficient
(B) efficiently
(C) more efficient
(D) most efficient
Questions 15-16 refer to the following notice.

We would like to inform you that the employee cafeteria on the second floor will be closed for
renovations. These renovations will enable us to provide all of our employees with _____ service and
15. (A) good
(B) best
(C) better
(D) well

food than now.

The renovations will take _____ three days. The snack bar on the third floor would like to inform

16. (A) approximate


(B) approximations
(C) approximately
(D) more approximately

employee cafeteria renovation enable provide cooperation will be open for drinks and snacks.

Thank you for your cooperation.

BI TP KIM TRA
1. The conference's keynote speaker addressed_______ impacts of digital technology on the current
music industry.
(A) variety
ng k hc: 0962 60 8801 04 6260 3948 125
a ch: S 18 Trn i Ngha Q Hai B Trng H ni
NGOI NG 24H 12
WWW.NGOAINGU24H.VN 6

(B) variably
(C) variation
(D) various
2. We had to spend the whole month searching for better solutions, and_______ customers were
impressed by our efforts.
(A) fortunate
(B) fortunes
(C) fortune
(D) fortunately
3. The more we spent with the sales team, the more_______ we were with their innovative marketing
skills.
(A) impression
(B) impress
(C) impresses
(D) impressed
4. The manager said that it is_______ to test emergency equipment frequently to avoid any
malfunction.
(A) appropriate
(B) appropriateness
(C) appropriately
(D) most appropriately
5. The unexpected operating complexity was more serious than_______ anticipated. (A) origin
(B) original
(C) originally
(D) originated
6. As the company's strongest competitor released a new line of fall clothing, NTR Inc.
moved_______ to introduce its new leather jackets.
(A) regularly
(B) quickly
(C) softly
(D) tiredly
7. The expansion of the natural history museum is most_______ the cause of significant revenue
increases.
(A) probabilities
(B) probability
(C) probable
(D) probably
8. Many science majors enter graduate school_______ after completing a bachelor of science degree.
(A) directly
(B) direct
(C) directed
(D) direction
9. Customers who purchase more than 500 dollars worth of items can request an HDTV cable box at
no_______ charge.
(A) additional
(B) addition
(C) adding
(D) additionally

ng k hc: 0962 60 8801 04 6260 3948 126


a ch: S 18 Trn i Ngha Q Hai B Trng H ni
NGOI NG 24H 12
WWW.NGOAINGU24H.VN 7

10. Mr. Garry Noxon in accounting was_______ warned about having too many numerical errors in
the budget report.
(A) repeat
(B) repeatedly
(C) repeater
(D) repetition
11. During the one-week promotional period, we offer all the visitors and customers 10 dollars worth
of kitchen appliances_______ free of charge.
(A) extremely
(B) exclusively
(C) continually
(D) completely
12. Despite_______ efforts and partial success for the last few years, the unemployment rate is still
on the rise.
(A) proficient
(B) considerable
(C) secondary
(D) present
13. Employees will be given bonuses and incentives_______ depending on their performance
achievements, not based on their relationship with supervisors.
(A) exclusive
(B) excluding
(C) exclusively
(D) excluded
14. Over the past three months, employees have been working_______ with support staff to
accomplish their goals, working an average of eight hours per day.
(A) initially
(B) originally
(C) primarily
(D) numerically
15. While smokers are highly_______ of the ban on smoking in workplaces, most non-smokers
support the ban.
(A) critic
(B) critically
(C) critical
(D) criticism
16. If you send the_______ information, I'll drop by your area to look for suitable housing.
(A) enthusiastic
(B) serious
(C) pertinent
(D) appreciative
17. We are_______ considering a wide variety of potential applicants, so please fill out your
background information accurately.
(A) ordinarily
(B) currently
(C) commonly
(D) lately
18. _______ parts stored in the warehouse may be procured at special sale prices, which are
negotiable.
ng k hc: 0962 60 8801 04 6260 3948 127
a ch: S 18 Trn i Ngha Q Hai B Trng H ni
NGOI NG 24H 12
WWW.NGOAINGU24H.VN 8

(A) Lengthy
(B) Opportune
(C) Surplus
(D) Brief
19. The renovated French restaurant next to the post office attracted more customers as a result of
its_______ priced, various food selections.
(A) reasonably
(B) thoroughly
(C) gratefully
(D) virtually
20. All employees should be informed explicitly of their responsibilities in relation to_______ client
information.
(A) sensitive
(B) competitive
(C) affordable
(D) courteous
21. Since the corporate system has a_______ structured daily schedule, it would be nearly
impossible to impair the work efficiency.
(A) hopefully
(B) highly
(C) probably
(D) rarely
22. The local government is planning to construct_______ performing arts facilities in order to meet
the cultural needs of the population.
(A) infrequent
(B) additional
(C) ongoing
(D) incidental
23. Our experienced and skilled technicians can be of much help in the event that you_______ delete
something essential from the program.
(A) accident
(B) accidental
(C) accidents
(D) accidentally
24. Any employees who are_______ late or absent from work may be subject to disciplinary actions,
regardless of their position.
(A) consistently
(B) steadily
(C) sensibly
(D) exactly
25. If you need to hire_______ help for your growing business, our experts can assist with the hiring
process from start to finish.
(A) additions
(B) additionally
(C) additional
(D) addition
26. After the introduction of collaboration tool programs, the company, which was in temporary
financial trouble, was_______nable to increase its market share.
(A) lastly
ng k hc: 0962 60 8801 04 6260 3948 128
a ch: S 18 Trn i Ngha Q Hai B Trng H ni
NGOI NG 24H 12
WWW.NGOAINGU24H.VN 9

(B) at first
(C) meanwhile
(D) finally
27. Many people call fairy tales too_______ and obvious, but that's why they are needed.
(A) predicting
(B) predicted
(C) predictable
(D) predictably
28. Our GTA phone roaming service is suited for individuals who travel_______ anywhere in the
world for business.
(A) regular
(B) regularity
(C) regularly
(D) regulate
29. The president of Kasara Inc. _______ announced that Lopez Davis, who has served the company
for more than 30 years, would retire at the end of next month.
(A) regretting
(B) regretted
(C) regretfully
(D) regretful
30. It is_______ through broadcast media such as television and radio that companies place an
advertisement to attract prospective customers.
(A) predominant
(B) predominantly
(C) predominating
(D) predominated
31. Workshop participants were required to be_______ not to choose any reserved front-row seats.
(A) caution
(B) cautious
(C) cautiously
(D) cautiousness
32. Road and street signs must be_______ visible even in the distance in order for drivers to know
where they are heading.
(A) clearness
(B) clear
(C) clearly
(D) clearest
33. This e-mail is to let you know that the book you ordered is_______ on backorder. (A) quickly
(B) precisely
(C) currently
(D) temperately
34. We have worked_______ hard on health insurance plans to enhance employee satisfaction.
(A) exception
(B) exceptionally
(C) exceptional
(D) except
35. It appears to be a_______ impossible plan to charge fees to cars visiting the city park on the
weekend.
(A) nearing
ng k hc: 0962 60 8801 04 6260 3948 129
a ch: S 18 Trn i Ngha Q Hai B Trng H ni
NGOI NG 24H 13
WWW.NGOAINGU24H.VN 0

(B) nears
(C) nearly
(D) neared
36. The recent findings indicate that the effects of weight loss associated with calorie restriction are
quite_______.
(A) beneficial
(B) benefits
(C) beneficiary
(D) benefit
37. Please make sure that all the volts in overhead compartments are tightened_______.
(A) secure
(B) security
(C) securely
(D) more secure

Questions 38-40 refer to the following event personnel advertisement.

LaDon Talent has good opportunities for part-time personnel. Positions available are part-time,
mostly weekends, and typically 5-7 hours a day. Earn $8 for a 5-hour event plus bonuses. You will
be_______ for promoting brands by distributing brochures and samples or demonstrating products
38. (A) aware
(B) eligible
(C) responsible
(D) qualified

to prospective customers, _______ creating brand awareness of the product features.


39. (A) if
(B) given
(C) within
(D) besides

If you apply for this position and it's not available at the time, please be assured that LaDon Talent
will retain your application in our files for 60 days. During this time your application will remain
available for_______ until opportunities become open.
40. (A) consider
(B) considering
(C) considered
(D) consideration

ng k hc: 0962 60 8801 04 6260 3948 130


a ch: S 18 Trn i Ngha Q Hai B Trng H ni
NGOI NG 24H 13
WWW.NGOAINGU24H.VN 1

Questions 41-43 refer to the following letter.

Dear donors,
We are extremely grateful for your generous_______ to UNICEF.
41. (A) acquisitions
(B) benefits
(C) excursion
(D) contributions

The money you donated will help UNICEF provide both immediate relief and long-term
rehabilitation of essential services for children in need.

Our review committee recently found that in 55 out of 80 villages, water systems have been severely
damaged. Therefore, there's an_______ need for expanding the distribution of bottled water and
42. (A) urgent
(B) current
(C) prior
(D) neutral

sanitation systems.
Another major challenge includes the threat of landmines. Many children are presently receiving
treatment in the hospitals for injuries resulting from remnants of bombs. So as to alleviate the
problem, UNICEF is involved in a campaign to_______ people to the dangers of
43. (A) disregard
(B) alert
(C) volunteer
(D) promote
unexploded bombs.

We are appealing to you, companies, and foundations for continuous support.


Thanks again for your generosity.

ng k hc: 0962 60 8801 04 6260 3948 131


a ch: S 18 Trn i Ngha Q Hai B Trng H ni
NGOI NG 24H 13
WWW.NGOAINGU24H.VN 2

Questions 44-46 refer to the following message.

To: Employees <nkosCassoc.wc.edu>


From: Dax Shepard cDaxetpl.zs.us>.
Date: Tue.12. June 2007 14:58:23

This message is being posted to all employees. First of all. I'm pleased to announce that Dale
Spencer was appointed as the district manager. As you know, It's fairly common for our employees
to receive positive feedback from coworkers, but Dale's case is_______
44. (A) unique
(B) predictable
(C) common
(D) desirable

For the past few weeks, we have received many letters from customers expressing their_______.
45. (A) disappointment
(B) importance
(C) appreciation
(D) concern

They often said that Dale is exceptionally patient, courteous and quick to respond to special requests
from his customers. This sort of commitment indicates that Dale puts customer_______ before
46. (A) satisfaction
(B) satisfied
(C) satisfying
(D) satisfyingly

anything else.

ng k hc: 0962 60 8801 04 6260 3948 132


a ch: S 18 Trn i Ngha Q Hai B Trng H ni
NGOI NG 24H 13
WWW.NGOAINGU24H.VN 3

Questions 47 to 49 refer to the following email.

From: Kelly Cho, Maintenance Manager cho@const.com>


To: Emilio Francaise, Chief Operations Officer <francaise@const.corroi
Subject: The elevator

Dear Mr. Francaise,

We completed our annual office safety inspection today as scheduled. Most_______ of the structure
47. (A) types
(B) charts
(C) blueprints
(D) aspects

comply with standard building codes and requirements. However, while Inspecting the elevator this
morning, my crew encountered a_______ mechanical defect. The damage is extensive, and the repair
48. (A) major
(B) trivial
(C) necessary
(D) dynamic

work is likely to be difficult.

It is essential that these repairs be made as soon as possible. Unfortunately, there is no one on our
team with enough technical expertise in this area, so I will need to_______ the work to someone
else. 49. (A) donate
(B) subcontract
(C) undertake
(D) owe
Thanks,
Kelly Cho

ng k hc: 0962 60 8801 04 6260 3948 133


a ch: S 18 Trn i Ngha Q Hai B Trng H ni
NGOI NG 24H 13
WWW.NGOAINGU24H.VN 4

NGY 8:
TNH T CH S LNG/ SO SNH/ PHN T
CH 1: Tnh t ch s lng mt" i cng vi danh t s t
1. a /an mt

A new security system will be installed next week.


mo t + danh t s t
bt nh
H thng an ninh mi s c lp t vo tun ti.

2. another mt ci khc

We need to find another distributor who is willing to sell our products.


another + danh t s t
Chng ta cn mt nh phn phi khc sn sng bn sn phm ca chng ta.

3. each mi, every mi, mi

Every employee should enter the access code to enter the building.
every + danh t s t
Mi nhn vin phi nhp m s mi c th vo ta nh.

Quick Quiz
1. Interview schedules were sent to each job ___________ by email.
(A) applicants (B) apply (C) applying (D) applicant

CH 2: Tnh t ch s lng i cng vi danh t s nhiu: many, those, these, several


1. many (= a number of) nhiu

You have to consider many factors before you start your own business.
many + danh t s nhiu
Bn phi cn nhc nhiu yu t trc khi m mt doanh nghip ca chnh mnh.

2. these nhng ci ny, those nhng ci

We need to complete those reports by Friday.


those + danh t s nhiu
Chng ta cn phi lm xong cc bn bo co trc th su.

3. several mt vi

Several inspectors were sent to the factory to examine the facility.


Several + danh t s nhiu
Vi thanh tra vin s c c n cng ty xem xt cc my mc thit b.
ng k hc: 0962 60 8801 04 6260 3948 134
a ch: S 18 Trn i Ngha Q Hai B Trng H ni
NGOI NG 24H 13
WWW.NGOAINGU24H.VN 5

* few ngha l t v a few ngha l mt vi cng ng trc danh t s nhiu


There are few mistakes in Kathy's report.
There are a few mistakes in Kathy's report.

Quick Quiz
2. We have received many __________ for our new products since last week's advertisement
(A) request (B) requesting (C) requests (D) requested

CH 3: So snh bng v so snh tng i


1. So snh bng: as + tnh t + as (... nh ~)
Jason usually makes as many mistakes as his colleagues.
as + tnh t + as
Jason thng mc nhiu li cng nh nhng ng nghip ca anh y.

2. So snh tng i: more + tnh t + than / tnh t + -er + than (... hn ~)


Taking the subway is faster than any other available method.
tnh t + -er + than
i xe in ngm th nhanh hn bt k phng tin sn c no khc.

CH 4: So snh tuyt i
1. So snh tuyt i: the most + tnh t / the + tnh t + -est (... nht)
The advertising team has demonstrated the highest productivity in recent years.
the + adj + -est
Nhm ph trch qung co th hin nng sut cao nht trong nhng nm gn y.
We offer the most comprehensive medical service in this area.
the most + adj
Chng ti cung cp dch v y t ton din nht trong khu vc ny.

Quick Quiz
3. We have experienced more production problems _________ we originally predicted.
(A) as (B) whether (C) than (D) or
4. Our company has a reputation of offering our customers the _______ selection of briefcases in the
marketplace.
(A) wider (B) widest (C) more widely (D) most widely

CH 5: Danh t v tnh t ch s lng


T loi danh t ( s t, s nhiu, m c, khng m c) ta dng cc tnh t ch s lng khc
nhau.
The beautiful painting The beautiful paintings
Loi danh t Tnh t ch s lng
S Many (=a number of): nhiu ; several: mt vi
m c nhiu These/those: nhng ci ny/ nhng ci kia;
few/ a few: t, vi/ mt vi.
S t a/an: mt another: mt ci khc
each: mi every: mi, mi
Khng m c Much: nhiu little/ a little: t/ mt t
-This/ that + DT s t/ DT khng m c
-Other + DT s nhiu/DT khng m c

ng k hc: 0962 60 8801 04 6260 3948 135


a ch: S 18 Trn i Ngha Q Hai B Trng H ni
NGOI NG 24H 13
WWW.NGOAINGU24H.VN 6

C hai -A lot of/lots of/plenty of + DT s nhiu/DT khng m c


-some/most/all + DT s nhiu/DT khng m c.
CH 6: Tnh t ch s lng c bit
1. Tnh t ch s lng other ng trc danh t m c s nhiu v danh t khng m
c.
Maps and other resources for tourists are available from the tourist information center.
other + danh t m c s nhiu
Bn v cc ngun ti liu cho du khch c sn trung tm thng tin v du lch.
Workers are responsible for keeping and cleaning their tools and other construction equipment.
other + danh t khng m c
Cng nhn c nhim v gi gn v lau chi cc dng c ca mnh v cc thit b xy dng khc.

2. all (tt c), most (hu ht), some (mt vi /mt s) cng ng trc danh t m c s
nhiu v danh t khng m c.
Most participants expressed their satisfaction about this seminar.
most + danh t m c s nhiu
a s ngi tham d by t s hi lng ca h v bui hi tho.

Please submit all information that you have as soon as possible.


all + danh t khng m c
Xin np tt c cc thng tin m anh/ch c cng sm cng tt.

Quick Quiz
5. Financial Times is offering all new ________ 20% discounts to celebrate the launch of its new
business magazine.
(A) subscriber (B) subscribed (C) subscribe (D) subscribers

CH 7: Trng t nhn mnh cu trc so snh


1. Trng t nhn mnh tnh t bnh thng: very/quite

It is very expensive to buy new office furniture.


trng t + tnh t
Mua c vn phng mi rt tn km.

2. Trng t nhn mnh cu trc so snh


Trng t nhn mnh cu trc so snh tng i much, even, still, far, a lot, by far
Trng t nhn mnh cu trc so snh tuyt i by far, quite

This new machine is much faster than the old one.


trng t nhn mnh + so snh hn + than
Ci my mi ny chy nhanh hn nhiu so vi my c.

Quick Quiz
6. PDAs have become _____________ more versatile devices than before.
(A) very (B) really (C) so (D) much

CH 8: Phn t
Phn t l mt hnh thc bin i ca ng t, c chc nng nh tnh t. C 2 loi phn t: hin ti
phn t (present participle) v qu kh phn t (past participle). Hin ti phn t (c cu trc -ing)
ng k hc: 0962 60 8801 04 6260 3948 136
a ch: S 18 Trn i Ngha Q Hai B Trng H ni
NGOI NG 24H 13
WWW.NGOAINGU24H.VN 7

mang ngha ch ng cn qu kh phn t (c cu trc -ed/ng t bt quy tc ct 3) mang ngha


b ng.
1. Cu hi
in dng ng ca ng t cho sn vo ch trng.
(1) the ______ companies (participate tham gia )
(2) the ______ schedule (attach nh km)

2. Cch phn tch cu hi


T cn in vo ch trng phi l tnh t. Nhng y, t cho sn li l ng t, nn t cn in vo
ch trng phi l phn t. xc nh t cn in vo l hin ti phn t hay qu kh phn t, ta
xem danh t nh l ch ng ca ng t cho sn. Nu mang ngha ch ng, t cn in vo ch
trng phi l hin ti phn t. Nu mang ngha b ng, t cn in vo ch trng phi l qu kh
phn t.

(1) t companies lm ch ng, participate lm ng t


ngha ch ng (cng ty tham gia) hp l hn
t cn in vo ch trng l hin ti phn t participating
the participating companies ngha l cc cng ty tham gia
(2) t schedule lm ch ng, attach lm ng t
ngha b ng (thi kha biu c nh km) hp l hn
t cn in vo ch trng l qu kh phn t attached
the attached schedule ngha l thi kha biu c nh km
Quick Quiz
7. Because of financial concerns, our company decided to discontinue the ________ expansion
project.
(A) proposing (B) proposed (C) propose (D) proposal

CH 9: Phn t ch cm xc
1. Hai loi phn t ch cm xc
Phn t chi cm xc kt thc bng -ing (nh boring, exciting,...) c dng ch bn cht ca mt
ngi hay mt s vt no . Ngc li, phn t ch cm xc kt thc bng ~ed (nh bored, excited,
...) c dng ch mt trng thi tm thi, thng l ca mt ngi no do chu s tc ng t
bn ngoi.
The test result was very disappointing.
disappointing (ng tht vng) ch bn cht ca result (kt qu thi)
Kt qu thi rt ng tht vng.
He was very disappointed at the test result.
disappointed (tht vng) ch trng thi tm thi ca he (anh y) do kt qu thi gy nn.
Anh y rt tht vng v kt qu thi.

ng k hc: 0962 60 8801 04 6260 3948 137


a ch: S 18 Trn i Ngha Q Hai B Trng H ni
NGOI NG 24H 13
WWW.NGOAINGU24H.VN 8

2. Cc phn t ch cm xc thng gp

Dng ~ing Dng ~ed


disturbing phin toi disturbed cm thy phin toi, b lm phin
troubling lm phin troubled b lm phin
frustrating gy bc mnh frustrated bc mnh
interesting hay, th v interested thch, quan tm
exciting th v excited hng th
disappointing ng tht vng disappointed tht vng
encouraging lm phn khi encouraged phn khi
satisfactory tt satisfied tha mn, tha lng
fascinating th v fascinated cm thy th v

Ch bn cht (thng ch s vt) Ch trng thi cm xc tm thi (thng ch ngi)

Quick Quiz
8. We finally decided to terminate the partnership because some of the terms of the contract were too
_________ to accommodate.
(A) frustrating (B) frustration (C) frustrate (D) frustrated

Bi tp 1:
Choose the word or phrase that best completes the sentence.
1. Theres ____ news
A. Excite
B. Excitement
C. Exciting
D. Excited
2. She was _____
A. Surprise
B. To surprise
C. Surprisingly
D. Surprised
3. _____ tired, he decided to leave early.
A. Feels
B. To feel
C. Feeling
D. Felt
4. _____ from a distance, the painting appeared quite realistic.
A. Seen
B. Saw
C. Seeing
D. See it

ng k hc: 0962 60 8801 04 6260 3948 138


a ch: S 18 Trn i Ngha Q Hai B Trng H ni
NGOI NG 24H 13
WWW.NGOAINGU24H.VN 9

Choose the word or phrase that best completes the sentence.


Located in the heart of the city, the Plaza Ville Hotel is the most _____ luxury hotel in town. Each of
5. A. Center
B. Located
C. Central
D. Best
the 200 rooms has a wonderful view of either the river or the mountains. As part of our weekend
package we offer a free massage and use of the sauna to all guests. If you feel ____ after a busy
6. A. Tiring
B. Tired
C. tire
D. To tire
week of work, why not check into the Plaza Ville for some rest and relaxation?

Fill the blank with the appropriate word


7. The_____ for imports this year has already been filled.
A. Quota
B. Quote
C. Quarter
D. Court
8. For a_____ of reasons, he wouldnt accept our offer.
A. Variation
B. Variant
C. Variety
D. Varying
9. Wholesale and _____ sales registered negative growth last month.
A. Part
B. Retail
C. Division
D. Gross
10. In the 1930s, there was a world-wide economic _____ and mass unemployment.
A. Policy
B. Prosperity
C. Activity
D. Depression

BI TP LUYN:
Bi tp 1:
Part V: Choose the word or phrase that best completes the sentence.

1. For your convenience, our claims division will now open __________ weekday from 9 A.M. to 7
P.M.
(A) every (B) few (C) a lot (D) many
2. Many job seekers consider internship experiences to be at least as valuable qualification
________ a degree in business.
(A) than (B) more (C) as (D) most
3. Please contact your supervisor if you are __________ in the career development seminar.
(A) interest (B) interested (C) interesting (D) interests

ng k hc: 0962 60 8801 04 6260 3948 139


a ch: S 18 Trn i Ngha Q Hai B Trng H ni
NGOI NG 24H 14
WWW.NGOAINGU24H.VN 0

4. The admission fee for the new art museum is __________ more expensive than we expected.
(A) far (B) well (C) quite (D) pretty
5. The architect had to revise the blueprint because it contained too much unusable __________.
(A) space (B) spaced (C) spacious (D) spaces
6. The consultant believes that the __________ revisions to the ordering process will make it more
cost-effective.
(A) suggest (B) suggested (C) suggesting (D) suggestion
7. Please note that Del Electronics is not responsible for any damage which is caused by misuse,
improper management, or __________ negligence.
(A) the other (B) other (C) others (D) another

Part VI: Questions through refer to the following advertisement.

The Business Travel Channel !!

Helping you find air travel, hotel accommodations, travel agencies and more!

Booking business travel is a complex process. Business owners often look to the Web for convenient
ways to compare options and book quickly at the _________ rates.
8. (A) good
(B) better
(C) well
(D) best

But there are __________ categories to consider, including air travel, ground transportation,
9. (A) every
(B) many
(C) much
(D) that
limousines, shuttles, rail travel and others, so youll want solutions geared to business in general and
your business in particular.

Some of the business travel solutions youll find here include _____________ stay and corporate
10. (A) extend
(B) extending
(C) extended
(D) extensions
housing, rail travel and charter buses for your business.

Need travel arrangements fast on a budget? The Business Travel Channel can help you.

ng k hc: 0962 60 8801 04 6260 3948 140


a ch: S 18 Trn i Ngha Q Hai B Trng H ni
NGOI NG 24H 14
WWW.NGOAINGU24H.VN 1

Bi tp 2:
Choose the correct option in brackets for each sentence.
1. The chairman hasnt signed the contract (already / yet).
2. Their office supplies are good. (However / Furthermore), they are cheap.
3. They (seldom / not) have problems with the product.
4. We were short of money. (Nevertheless / Moreover), we decided to develop a new product.

Bi tp 3:
Choose the correct option given in brackets to complete each sentence.
1. Please refer to the (attached / attaching) price list.
2. They should put (warning / warned) labels on the products.
3. After the seminar was over, we were all (impressed / impressing).
4. Please refer to the prices (listing / listed) in the catalogue.
5. Customers found our products (fascinated / fascinating).
6. An (increasing / increased) number of people use online shopping.

Look at the hints given and choose the correct option to complete each sentence.
7. We are looking for Modifying a noun an workers.
(A) experience (B) experiences (C) experiencing (D) experienced
object complement
8. The president ordered the meeting .
(A) cancel (B) canceled (C) canceling (D) being canceled

Bi tp 4:
A. Fill the gaps with the correct forms of participial adjectives from the verbs given in brackets.
1. (disappoint) Stevens proposal was ____________________
2. (excite) The audience was____________________ about the performance.
3. (please) They were ____________________ with our products.
4. (disappoint) The chief executive was ____________________ with the survey results.
5. (confuse) The new road system was ____________________ to the citizens.
6. (interest) The representatives were ____________________ in importing our electronic goods.

Look at the hints given and choose the correct option to complete each sentence.
7. The customers are so _____ by the new ordering system.
People

(A) confusing (B) confuse (C) confused (D) confusion


8. The market share was so _____ that we decided to take stronger action.
A thing

(A) depress (B) depressing (C) depressed (D) depression

Bi tp 5:
Read the following sentences. Mark next to correct sentences and correct the mistakes in incorrect
sentences.
1. Entered the building, you should present a proper form of identification. __________
2. When sending the form, please put it in the enclosed envelope. __________
3. Locating in a busy area, this restaurant is visited by a lot of people. __________
4. When promoted to vice president, he got a pay raise. __________
ng k hc: 0962 60 8801 04 6260 3948 141
a ch: S 18 Trn i Ngha Q Hai B Trng H ni
NGOI NG 24H 14
WWW.NGOAINGU24H.VN 2

5. As explaining in the manual, this product is not compatible with color printers. __________
6. Being giving a speech, you should speak slowly and clearly. __________

Choose the correct option to complete each sentence.


7. _____ from the USA, some books were seriously damaged.
(A) Delivery (B) Deliver (C) Delivered (D) Being delivering
8. Before _____ on vacation, you should report it to the manager, go on vacation report.
(A) going (B) go (C) gone (D) went

Bi tp 6:
A. Change the underlined parts into participial phrases.
1. Although he was promoted, he didnt get a raise. _____________________
2. If you take a right turn, you will see Coit Tower. _____________________
3. When you make a decision, you should think twice. _____________________
4. As he looked around us, Mr. Spencer started the presentation. _____________________
5. Because it is updated regularly, the website provides lots of recent information. _______________

Choose the correct option to complete each sentence.


6. When _____ the payment, you should present your order number.
(A) sent (B) send (C) sending (D) sends
7. _____ in Spanish, the report is hard to understand.
(A) Write (B) Writing (C) Written (D) To write

Bi tp 7:
Practice with TOEIC Actual Questions
Decide which of the choices - (A), (B), (C), or (D) - best completes the sentence
1. Customers _____ our online store complained about the slow shipping.
(A) using
(B) used
(C) use
(D) to use
2. The building is in _____ condition.
(A) damaging
(B) damaged
(C) damages
(D) damage
3. When _____ out the application form, please make sure it does not have any typos.
(A) fills
(B) filled
(C) to fill
(D) filling
fill out application form typo
4. Anyone _____ in working abroad should talk to a supervisor.
(A) interest
(B) interests
(C) interesting

ng k hc: 0962 60 8801 04 6260 3948 142


a ch: S 18 Trn i Ngha Q Hai B Trng H ni
NGOI NG 24H 14
WWW.NGOAINGU24H.VN 3

(D) interested
5. Sales this year have been _____ to both the management and the staff.
(A) disappoints
(B) disappoint
(C) disappointing
(D) disappointed
6. They saw a group of people _____ the mayor's office.
(A) entered
(B) entering
(C) to enter
(D) entrance
7. _____ big bonuses last week, all the employees seemed pleased.
(A) Given
(B) Giving
(C) Give
(D) To give
8. Mr. Kims suggestion was _____, and some concepts were difficult to understand.
(A) confuses
(B) confused
(C) confuse
(D) confusing
9. Info Tech, is one of the _____ multinational companies in the nation.
(A) leading
(B) led
(C) to lead
(D) lead
10. Please post the _____ information on the bulletin board.
(A) request
(B) requesting
(C) to request
(D) requested
11. You can visit our website to find more _____ information on our products.
(A) detail
(B) detailed
(C) details
(D) detailing
12. They decided to renovate the _____ facilities in the factory.
(A) existing
(B) leading
(C) built
(D) extended
13. The participants at the book fair seemed quite _____.
(A) satisfaction
ng k hc: 0962 60 8801 04 6260 3948 143
a ch: S 18 Trn i Ngha Q Hai B Trng H ni
NGOI NG 24H 14
WWW.NGOAINGU24H.VN 4

(B) satisfying
(C) satisfied
(D) satisfy
14. H&P Co. is one of the _____ companies in the field of fashion.
(A) warning
(B) operating
(C) promising
(D) missing
Questions 15-16 refer to the following notice.
To All Tenants in Madison Apartments,

As _____ on March 1 of this year, all Madison apartment rents will increase by 10 percent on April
1.
15. (A) announce
(B) announced
(C) announcing
(D) announces

this increase is inevitable as we have recently made improvements in services and utilities. And we
have also renovated the parking structure since some of the tenants complained about the _____
16. (A) limited
(B) operated
(C) attached
(D) complicated

number of spaces.

We are sorry for this increase, but we will do our best to provide you with better services and
facilities.

Thank you.

Resident Manager

Bi tp 8:
Questions 1-10 Choose the word that best completes each sentence.
1. Enclosed is the complete listing of the _____ banks in the neighboring area.
(A) distinguish (B) distinguishing (C) distinguished (D) distinguishability
2. The recent oil spill along the Belgian coast resulted in _____ consequences for the industry.
(A) worried (B) worrying (C) worries (D) worrier
3. Please refer to the _____ bank statement for detailed information.
(A) enclose (B) enclosing (C) enclosure (D) enclosed
4. The keynote speech at the conference was both informative and _____.
(A) interest (B) interested (C) interesting (D) to interest
ng k hc: 0962 60 8801 04 6260 3948 144
a ch: S 18 Trn i Ngha Q Hai B Trng H ni
NGOI NG 24H 14
WWW.NGOAINGU24H.VN 5

5. The renovation project has been led by a highly _____ group of designers.
(A) motivate (B) motivation (C) motivating (D) motivated
6. We were ______ by Dr. Albert's outstanding contributions to the development of the new vaccine.
(A) impression (B) impressive (C) impressed (D) impresses
7. Employees who have not submitted a _____ report should discuss it with their manager.
(A) revised (B) revising (C) revision (D) revise
8. We expect museum patrons to make continuous donations for the _____ season.
(A) prior (B) first (C) upcoming (D) current
9. Extra specialists were employed due to an _____ need to secure the mobile devices.
(A) accidental (B) increasing (C) outgoing (D) effective
10. We regret to inform you that repairs for _____ applications will not be available any longer.
(A) conditional (B) temporary (C) partial (D) discontinued
Text Completion
Questions 11-13 refer to the following memorandum.
To: All Employees
From: John Parrison, Personnel Manager
Date: August 16th
Re: Workshop

Please be informed that the System Management Workshop has been scheduled weekly in July and
August. The workshop will be held in Room 310 of the main building from 4 P.M. to 5:30 P.M.
every Thursday during the two-month period.

The _____ of this workshop is to familiarize our employees with our upgraded in company
11. (A) study
(B) purpose
(C) influence
(D) decision

communication system. All employees are _____ to attend one of the sessions. Employees _____ to
12. (A) required 13. (A) advised
(B) imposing (B) have advised
(C) determined (C) are advised
(D) equal (D) were advised

schedule their participation with Jamie Song, a human resources coordinator, at least one week in
advance.

BI TP KIM TRA
1. There's_______ news.
(A) excite
(B) excitement
(C) exciting
(D) excited
2. She was_______.
(A) surprise
(B) to surprise
(C) surprisingly
(D) surprised
ng k hc: 0962 60 8801 04 6260 3948 145
a ch: S 18 Trn i Ngha Q Hai B Trng H ni
NGOI NG 24H 14
WWW.NGOAINGU24H.VN 6

3. _______ tired, he decided to leave early.


(A) Feels
(B) To feel
(C) Feeling
(D) Felt
4. _______ from a distance, the painting appeared quite realistic.
(A) Seen
(B) Saw
(C) Seeing
(D) See it
5. Professor Miller requires that all papers be_______ in ink.
(A) formatted
(B) copied
(C) written
(D) made
6. The baby was about to fall asleep because it was_______.
(A) funny
(B) small
(C) bored
(D) excited
7. The boss was_______ to fire his secretary.
(A) determine
(B) determining
(C) determined
(D) determination
8. The movie was very_______.
(A) move
(B) moving
(C) moved
(D) movement
9. He has his shoes_______ every day.
(A) put on
(B) shined
(C) made
(D) bought
10. This noise is very_______.
(A) annoys
(B) annoying
(C) annoyed
(D) annoy
11. It is not very easy to make her_______ once she gets upset.
(A) please
(B) pleasing
(C) pleased
(D) pleasure
12. Two teenagers were among the fourteen people_______
(A) arrests
(B) arrest
(C) arresting
ng k hc: 0962 60 8801 04 6260 3948 146
a ch: S 18 Trn i Ngha Q Hai B Trng H ni
NGOI NG 24H 14
WWW.NGOAINGU24H.VN 7

(D) arrested
13. Extremely_______ from the hard work, he couldn't walk an inch.
(A) busy
(B) excited
(C) happy
(D) exhausted
14. Who is that man_______ the red jacket?
(A) wearing
(B) having
(C) doing
(D) inside of
15. She said that the report was a bit_______.
(A) confused
(B) confuses
(C) confusing
(D) confuse
16. _______ water is not safe for drinking.
(A) Polluted
(B) Polluting
(C) Pollute
(D) Pollutes
17. He could easily make himself_______ if he tried.
(A) understand
(B) understanding
(C) understood
(D) be understood
18. Don't leave your bags_______ as they may be removed without notice.
(A) attended
(B) attending
(C) unattended
(D) unattending
19. Some customers were very_______ about the return policy.
(A) confusing
(B) confused
(C) confuse
(D) confuses
20. _______ at a distance, it looks like a miniature city.
(A) Seen
(B) Seeing
(C) Having seen
(D) Having been seen
21. The photocopier needs_______.
(A) to fix
(B) to be fix
(C) fixing
(D) to be fixing
22. 1 am considering_______.
(A) taking a new job
(B) to take a new job
ng k hc: 0962 60 8801 04 6260 3948 147
a ch: S 18 Trn i Ngha Q Hai B Trng H ni
NGOI NG 24H 14
WWW.NGOAINGU24H.VN 8

(C) take a new job


(D) do take a new job
23. _______ in a lake is very different from swimming in an ocean.
(A) To swim
(B) Swimming
(C) To be swimming
(D) Swim
24. _______ is fun.
(A) Swimming
(B) A swimming
(C) Swim
(D) Be swimming
25. We enjoy_______ in the mountains.
(A) hike
(B) to hike
(C) hiking
(D) to hiking
26. That movie was very_______.
(A) interest
(B) interested
(C) interesting
(D) to interest
27. The business consultant suggested a_______ sales plan.
(A) modify
(B) modifies
(C) modified
(D) modifying
28. The contractor is_______ for his quality workmanship.
(A) respected
(B) respects
(C) respect
(D) respecting
29. The news was so_______ that the company invested even more money in the project.
(A) encourage
(B) encouraged
(C) encouraging
(D) encourages
30. This construction project is as important as any other project_______ this year. (A) undertaking
(B) undertook
(C) undertaken
(D) undertake
31. My cousin's_______ advice about selling our stock saved us thousands.
(A) amazingly
(B) amazed
(C) amazing
(D) amazement
32. Mr. Hansen's_______ from his position as chief comptroller has been a shock to all of us.
(A) resigning
(B) resigned
ng k hc: 0962 60 8801 04 6260 3948 148
a ch: S 18 Trn i Ngha Q Hai B Trng H ni
NGOI NG 24H 14
WWW.NGOAINGU24H.VN 9

(C) resigns
(D) resign
33. This new trade agreement has created all kinds of_______ possibilities for both our countries.
(A) excited
(B) exciting
(C) excitement
(D) excitable
34. Remember that feeling confident and_______ is an important part of giving a presentation.
(A) relaxing
(B) relaxes
(C) relaxed
(D) relax
35. While some people enjoy receiving calls from telemarketers, other people find such
calls_______.
(A) annoys
(B) annoying
(C) annoyingly
(D) annoyed
36. Mr. Chang will serve as_______ director until a permanent director can be found. (A) act
(B) acting
(C) actor
(D) acted
37. Stop_______ your money.
(A) to waste
(B) wasted
(C) wasting
(D) waste
Questions 38-39 refer to the following advertisement.
Located in the heart of the city, the Plaza Ville Hotel is the most_______ luxury hotel in town. Each
38. (A) center
(B) located
(C) central
(D) best

of the 200 rooms has a wonderful view of either the river or the mountains. As part of our weekend
package we offer a free massage and use of the sauna to all guests. If you feel_______ after a busy 2.
39. (A) tiring
(B) tired
(C) tire
(D) to tire

week of work, why not check into the Plaza Ville for some rest and relaxation?

Questions 40-43 refer to the following notice.


Grow your own vegetables! Its_______ you think.
40. (A) easier
(B) easier than
(C) more easy
(D) the easiest
ng k hc: 0962 60 8801 04 6260 3948 149
a ch: S 18 Trn i Ngha Q Hai B Trng H ni
NGOI NG 24H 15
WWW.NGOAINGU24H.VN 0

The Bradley Community Allotment Society has to new allotments available for local residents. We
will distribute them on a first come, first served basis, so if you are_______, don't delay.
41. (A) interesting
(B) interested in
(C) interested
(D) interest

Annual rent for each allotment is 950 dollars. You will be able to grow a lot of the vegetables you
normally buy, so eventually you will be saving money. The Allotment Society will be_______
42. (A) making
(B) showing
(C) giving
(D) taking

lessons on how to grow vegetables successfully at the community center this coming weekend. If
you are interested in_______ one of the allotments, why not come along to the lectures and get some

43. (A) rent


(B) rented
(C) renting
(D) to rent

useful tips.

Questions 44 through 47 refer to the following notice


Volunteers Wanted

The Psychology Department of Queenstown University is_______ volunteers to take part in an


44. (A) searching
(B) looking for
(C) necessary
(D) investigating

experiment. We need 150 volunteers: 75 men and 75 women. Volunteers should be aged_______ 20
45. (A) under
(B) by
(C) between
(D) from

and 26 and should not be taking any kind of medication. Volunteers must not be enrolled in any of
the university's current psychology courses.

The experiment will take between thirty to forty minutes to complete. Participants will be asked to
answer a series of questions after_______ a short film. A small fee will be paid to participants.
46. (A) have watched
(B) watch
(C) watching
ng k hc: 0962 60 8801 04 6260 3948 150
a ch: S 18 Trn i Ngha Q Hai B Trng H ni
NGOI NG 24H 15
WWW.NGOAINGU24H.VN 1

(D) to watch

Coffee and tea will be also served during the film.

If you are_______, please contact the Psychology Department secretary on 990-8887 before
47. (A) interested
(B) interesting
(C) have an interest in
(D) interest

January 20th.

Questions 48-50 refer to the following memorandum.


To: Debra Fullerton
From : Susan Drake
Re : New Recycling Project

This past summer we implemented a new recycling project. New bins were placed at a significant
number of locations throughout the city. Residents had opportunities to provide feedback on the
style, positioning and location of the new recycling bins. Advertisements on the bins provided
residents_______ information about a telephone survey where they would have their voices_______.
48. (A) to 49. (A) hear
(B) for (B) to hear
(C) on (C) heard
(D) with (D) hearing

In addition to this survey feedback mechanism. we hired some people to conduct a street-level
survey. Public_______ to the survey were noticeable.
50. (A) respond
(B) responding
(C) response
(D) responses

In total, approximately 4,500 surveys were completed. City officials designed the survey tools and
collected all of the data.

If you want to go over these survey results, contact me at any time.

ng k hc: 0962 60 8801 04 6260 3948 151


a ch: S 18 Trn i Ngha Q Hai B Trng H ni
NGOI NG 24H 15
WWW.NGOAINGU24H.VN 2
NGY 9: LIN T V GII T
LIN T KT HP, LIN T KP

CH 1: Lin t c chc nng lin kt cu


Chng ta khng ch i thoi bng cc cu ngn trong cuc sng hng ngy m cn thng xuyn s
dng cc cu di. to thnh cc cu ghp v phc nh vy t nhng cu n ngn, ta cn dng
lin t.
1-1. Chn cu ng.
Kathy met her old friend she was visiting the London office.
Kathy met her old friend while she was visiting the London office.
Kathy met her old friend but she was visiting the London office.
SAI

Kathy met her old friend x she was visiting the London office.
s v s v : khng c lin t ni 2 cu

Kathy met her old friend but she was visiting the London office.
s v conj s v : c lin t nhng khng thch hp
NG
Kathy met her old friend while she was visiting the London office.
s v conj s v :lin t while lin kt 2 cu vi nhau
y l dng cu ghp vi cu trc cu n l S + V + O nn ch c lin t mi ghp uc 2 cu
n vi nhau. Do , khi lm bi thi TOEIC thc t, khi c 2 ng t trong cng mt cu th nht
thit phi xc nh xem c lin t hay khng.
Ng php b sung
1. Trng hp phi in lin t.
(1) S + V + O S+V+O
(2) S + V + O, S + V + O
2. Lin t c nhiu ngha.
(1) Din t thi gian: while, when, since, before, after
(2) Din t l do: because, since
(3) Din t s nhng b: although [= though / even though]
(4) Din t iu kin : if, unless (= if ~ not), once

Bi tp 1:
Chn t ng trong cc cu sau.
1. (If /Often) you want to see this report, please let me know.
2. (When /At) you develop products, it is important to conduct a market research.
3. Mr. Terry couldnt attend the meeting (because/ due to) he was on a business trip.
4. (Despite /Although) she is young, Ms. Rowell has much experience in the field.
5. You should turn off your cellular phone (during/ while) the seminar is in progress.
S + V + O ... S + V + O => in lin t vo ch trng
ng k hc: 0962 60 8801 04 6260 3948 152
a ch: S 18 Trn i Ngha Q Hai B Trng H ni
NGOI NG 24H 15
WWW.NGOAINGU24H.VN 3

S + V + O, S + V + O

NOTE:

Lin t ngha Gii t

if iu kin nu often (adv): thng

when thi gian khi at + thi gian: lc

because l do v due to

although nhng b mc d despite

while thi gian trong lc during

Bi tp 2:
1. You need to contact your supervisor ________ you are late to work.
(A) despite (B) often (C) if (D) according to
2. ________ he joined our company, Mr. Park has been working in the sales team.
(A) Since (B) During (C) Within (D) Due to
3. ________ the internship period has ended, you will get a chance to be hired as a full-time
employee.
(A) Beyond (B) During (C) Between (D) After
4. You are not allowed to renew the books ________ other patrons are waiting for them.
(A) due to (B) because of (C) because (D) from
5. ________ we receive your orders, we will check our inventory to fill your orders.
(A) During (B) Sometimes (C) Once (D) Often

CH 2: Gii t lin kt danh t


Nu lin t dng lin kt cu th gii t c dng lin kt danh t trong cu.
Ng php b sung
1. Trng hp phi in gii t.
(1) Khi lin kt danh t u cu
According to the survey, many college students want to study abroad.
Prep n

(2) Khi lin kt danh t sau cu hon chinh


He was not able to finish the report because of the problem with his computer.
cu hon chnh prep n
Khi lin kt danh t sau cu hon chnh th dng gii t, khi lin kt cu th dng lin t.

2. Gii t cng c nhiu ngha.


(1) Thi gian : during, since, for, before (= prior to), after, within, by
(2) L do : because of (= due to)
(3) Nhng b : despite (= in spite of)
ng k hc: 0962 60 8801 04 6260 3948 153
a ch: S 18 Trn i Ngha Q Hai B Trng H ni
NGOI NG 24H 15
WWW.NGOAINGU24H.VN 4

(4) ngha khc: according to, about (= as to)


Bi tp 3: Chn t ng trong cc cu sau.
1. We have to accomplish our financial goals (for/ while) this year.
2. (Despite/ Although) the recent increase in sales, the overall profits have declined.
3. The network access will be suspended (due to/ because) the maintenance work.
4. Mr. Perez gave a speech (while/ during) the welcome reception.
5. Mr. Raymond succeeded in his business (because of/ if) his diligent efforts.
Danh t + ... + danh t => in gii t vo ch trng

Bi tp 4: Gii t lin kt danh t trong cu


1. _________ the recent high interest rates, there are many people to apply for a loan.
(A) Despite (B) Unless (C) Although (D) While
2. Refunds will be given only _________ the period of 30 days from the purchase.
(A) because (B) although (C) for (D) while
3. The City Museum will be open _________ the scheduled renovation.
(A) once (B) because (C) while (D) after
4. The commuter train service has been suspended _________ a technical problem.
(A) because (B) while (C) unless (D) because of
5. The new office can accommodate 1,000 people _________ the advertisement.
(A) according to (B) because (C) while (D) before

KIN THC M RNG


I. Lin t trong mnh trng ng khng nh hng n cu trc cu
1. Mt s lin t trong mnh trng ng khng nh hng n cu trc cu
|-- mnh trng ng --| |-----------------------------mnh chnh --------------------------------|
While I am away, you can contact my secretary for any urgent business.
Khi ti i vng, anh c th lin lc vi th k ca ti nu c bt k vic g khn cp.

2. Cc lin t ph bin trong mnh trng ng


Lin t Ngha
Thi gian since t khi
when (= as) khi
after sau khi
before trc khi
as soon as ngay sau khi
while trong khi
iu kin if / as long as nu / min l
unless tr phi
once mt khi
Nguyn nhn because / since / as bi v
for bi v
now that bi v
Nhng b although mc d
even though d cho
though
even if
ng k hc: 0962 60 8801 04 6260 3948 154
a ch: S 18 Trn i Ngha Q Hai B Trng H ni
NGOI NG 24H 15
WWW.NGOAINGU24H.VN 5

Mc ch so / so that m

Quick Quiz
1. We will begin processing the loan application ________ we receive the supporting documents.
(A) which (B) once (C) despite (D) according to

II. Lin t trong mnh danh ng c chc nng ch ng hoc tc t trong cu


1. Mnh danh ng c chc nng nh danh t (lm ch ng v tc t)
S + V+ -------------- + S +V + N. Lin t ca mnh danh ng c chc nng nh tc t
-------------- + S + V + N + V + N. Lin t ca mnh danh ng c chc nng nh ch ng
2. Cc lin t ca mnh danh ng
Cu hi trong TOEIC thng i hi th sinh phi phn bit c that v what
(1) Lin t that phi ng truc mt mnh hon chnh
Every visitor should remember that proper identification should be presented.
ng t (1) ng t (2)
Khch tham d phi nh xut trnh giy chng minh hp l.
Cn c lin t lin kt cc ng t (should remember/should be presented).
ng t remember c tc t l mnh danh ng that proper identification should be presented.
Sau lin t that l mt cu trc ch - v hon chnh proper identification should be presented.
(2) Lin t what ng trc mt cu trc khng hon chnh
To improve sales, you should know what customers want.
ng t (1) ng t (2)
tng doanh s, bn phi bit khch hng mun g.
Cn c lin t lin kt cc ng t (should know/ want).
ng t know c tc t l mnh danh ng what customers want.
Sau lin t what l mt cu trc khng hon chnh: customers want thiu tc t.

Quick Quiz
2. The financial consultant has suggested ________ the management consider hiring an outside
auditor.
(A) if (B) who (C) in (D) that

NOTE: lin t.
Bi tp 5:
Xc nh loi lin t trong cc cu sau y.
1. the company has received many orders since it put an ad in the local newspaper.
2. A new study shows that more people are interested in studying abroad.

Chn lin t thch hp cho cc cu sau y:


3. (that/if) the concert sold out so fast is a proof of the singers popularity.
4. (although/which) he is relatively inexperienced, Rick has a lot of potential.

ng k hc: 0962 60 8801 04 6260 3948 155


a ch: S 18 Trn i Ngha Q Hai B Trng H ni
NGOI NG 24H 15
WWW.NGOAINGU24H.VN 6

BI TP LUYN TP
Bi tp 1:
Part V Chn t thch hp in vo ch trng di y.
1. _________ your order was placed after the deadline, no discount will be given.
(A) However (B) During (C) Because (D) Despite
2. _________ the recent increase in sales, we decided to hire more sales personnel.
(A) Due to (B) In fact (C) While (D) Because
3. The feedback from our customers are great _________ a few bad comments.
(A) although (B) because (C) once (D) despite
4. _________ the government takes effective measures, the unemployment rate will continue to
rise.
(A) Unless (B) Also (C) Except (D) However
5. Overall profits of our company increased by more than 10 percent _________ Ms. Serenawas
serving as sales manager.
(A) during (B) because of (C) despite (D) while

Part VI in vo ch trng trong mu qung co sau.

Return Policy

We provide refunds only ________ the item is defective or damaged during delivery.
6. (A) if
(B) due to
(C) during
(D) despite
To receive a full refund, you need to return the item ________ 30 days of your purchase.
7. (A) because
(B) while
(C) within
(D) if
If you want to exchange the item with a new one, the new item will be sent within 7 business
days ________ we receive your returned item.
8. (A) due to
(B) prior to
(C) according to
(D) after

ng k hc: 0962 60 8801 04 6260 3948 156


a ch: S 18 Trn i Ngha Q Hai B Trng H ni
NGOI NG 24H 15
WWW.NGOAINGU24H.VN 7

Bi tp 2:
Part V: Chn t thch hp in vo ch trng.

1. The restaurant still tries to improve its quality and service ________ they usually receive
excellent feedback from customers.
(A) such as (B) in addition to (C) by means of (D) even though
2. The new convention center will not open until next January ________ the yesterdays press
release.
(A) as (B) so that (C) instead (D) according to
3. ________ the six month's internship period is completed, all participants will be eligible to get a
raise.
(A) Once (B) Soon (C) Then (D) Later
4. Dr. Kim's new study indicates ________ people are willing to spend money on career
development.
(A) while (B) that (C) on (D) however
5. The travel company introduced a new travel package _________ includes round-trip airfare and
accommodation.
(A) due to (B) what (C) since (D) that
6. You need to let one of your colleagues take care of your job _________ you are on vacation.
(A) in fact (B) during (C) unless (D) while
7. _________ the director wants to see the budget report, give him a copy of the report.
(A) That (B) Despite (C) If (D) For

ng k hc: 0962 60 8801 04 6260 3948 157


a ch: S 18 Trn i Ngha Q Hai B Trng H ni
NGOI NG 24H 15
WWW.NGOAINGU24H.VN 8

Part VI: Chn p n ng cho cu 8 ~ 10 trong bi bo sau y.

Career Development
Many career experts agree ____________ the best time to look for a new job is while comfortably
8. (A) which
(B) because
(C) with
(D) that
working in your old one.

____________ youre starting to feel unchallenged in your present position, you may be ready for a
9. (A) Then
(B) Often
(C) If
(D) For
promotion to the next level.

If there arent many career advancement opportunities where you work, the best next step may be to
look for a new job elsewhere.

Nowadays, its up to you to take control of your professional future and make sure you are progressing
wisely down the right career path.

Here are some useful tips ____________ can help you make better decisions.
10. (A) which
(B) because
(C) they
(D) it

Lin t kt hp v lin t kp

CH 1: Lin t lin kt cc cm t song song


1. T hoc cm t ng trc v ng sau and, or, but phi c cng hnh thc v t loi vi
nhau.
The restaurant serves fresh and organic vegetables.
tnh t and tnh t
Nh hng ny phc v rau ti v sch.

You can use my laptop or the computers in the lounge.


danh t or danh t
Anh c th dng my tnh xch tay ca ti hoc my tnh bn trong phng khch.

Email is a simple but powerful way to promote your products.


tnh t but tnh t
Th in t l mt phng tin n gin nhng hiu qu qung b sn phm ca bn.
and din t s kt hp (fresh v organic cng b ngha cho danh t vegetables)
ng k hc: 0962 60 8801 04 6260 3948 158
a ch: S 18 Trn i Ngha Q Hai B Trng H ni
NGOI NG 24H 15
WWW.NGOAINGU24H.VN 9

or din t s la chn (laptop hoc computers)


but din t s tng phn (simple tng phn vi powerful)
Quick Quiz
3. Mr. Simmons presented his creative and __________ design for the contest.
(A) innovatively (B) innovative (C) innovate (D) innovation

CH 2: Lin t kp
Lin t kp l lin t c hai thnh phn, chng hn both ... and either... or v.v. Nhng t (hoc cm
t) ng sau tng thnh phn ny phi cng t loi vi nhau.
1. both A and B c A v B
We are currently looking for both new and experienced sales personnel.
both tnh t and tnh t
Hin nay chng ti ang tm kim nhn vin bn hng va tr va c kinh nghim.

2. either A or B hoc A hoc B


Either Mr. Kim or Ms. Nori will be promoted
either danh t or danh t
Hoc ng Kim hoc c Nori s c bt.

3. neither A nor B khng A cng khng B


Neither the sales director nor the marketing director will be able to attend the meeting.
neither danh t nor danh t
C gim c kinh doanh ln gim c tip th u s khng th tham d cuc hp.

4. not only A but also B khng nhng A m cn B


We offer employees not only satisfactory salaries but also comprehensive benefits.
not only danh t but also danh t
Chng ti khng ch cho nhn vin mc lng tha ng m cn c phc li y .

Quick Quiz
4. The furniture you ordered yesterday is expected to arrive ___________ today or tomorrow.
(A) neither (B) both (C) either (D) not only

ng k hc: 0962 60 8801 04 6260 3948 159


a ch: S 18 Trn i Ngha Q Hai B Trng H ni
NGOI NG 24H 16
WWW.NGOAINGU24H.VN 0

KIN THC M RNG


I. Cu hi tm lin t thch hp
1. and v
Our products are available at major department stores and retail stores.
A and B
Sn phm ca chng ti c bn ti cc ca hng bch ha ln v cc ca hng bn l.

2. but (= yet) nhng


You already signed a contract but you also need to sign the official insurance documents.
A but B
Anh k hp ng nhng anh cng cn k cc giy t bo him chnh thc.

3. or hoc
Rents should be paid at or before the end of each month.
A or B
Tin thu phi c tr vo cui mi thng hoc trc .

Quick Quiz
5. Security specialists from private companies ______ government agencies will meet to discuss the
problem.
(A) in (B) both (C) by (D) and

II. Cu hi tm t thch hp trc v sau lin t


1. Lin t kt hp cc t cng t loi
The company plans to expand its operations and market share.
A and B
Cng ty c k hoch m rng hot ng v th phn ca mnh.
and lin kt danh t (operations) vi danh t (market share)

2. Lin t kt hp cc t cng hnh thc


The company plans to expand its operation and hire an experienced sales manager.
A and B
Cng ty c k hoch m rng hot ng v tuyn dng trng phng kinh doanh c kinh nghim.
and lin kt ng t nguyn mu (to expand) vi ng t nguyn mu ((to) hire)

Quick Quiz
5. Our primary goal is to educate children and ____________ them from violence and poverty
(A) protected (B) protecting (C) protection (D) protect

Bi tp 3:
Part V: Chn t thch hp in vo ch trng.

1. Please complete this ________ and easy survey to give your opinions about our service.
(A) quickly (B) quicken (C) quick (D) quicker

ng k hc: 0962 60 8801 04 6260 3948 160


a ch: S 18 Trn i Ngha Q Hai B Trng H ni
NGOI NG 24H 16
WWW.NGOAINGU24H.VN 1

2. The new promotion policy is applied to both new ________ existing employees.
(A) or (B) but (C) nor (D) and
3. Companies usually ask applicants to submit a resume which summarizes their ________ and
skills.
(A) education (B) educational (C) educate (D) educator
4. Students have to consider not only the tuition ________ the cost of meals and housing
accommodation.
(A) also (B) and (C) or (D) but also
5. Visitors who want to renew their license can either complete the application by hand ________
use the computers in the lobby.
(A) or (B) but (C) also (D) and
6. The two applicants have experience in accounting, ________ only one is familiar with our
accounting software.
(A) nor (B) but (C) by (D) or
7. Neither our company ________ the rival company will attend the trade show.
(A) for (B) and (C) nor (D) or

Part VI: Chn p n ng cho cu 8 ~ 10 trong mu qung co sau y.

Online Marketing
Active Online Marketing Inc. is a __________ and experienced industry leader in search engine
8. (A) professionally
(B) professionalism
(C) professional
(D) profession
marketing.

We offer customized marketing programs that meet the needs of your company and __________ a
9. (A) to provide
(B) provide
(C) provides
(D) providing
platform that will help you to launch your business onto the forefront of the information super highway.

We are one of the few search engine marketing firms that is concerned not only about the quantity of
traffic ___________ the quality.
10. (A) but also
(B) also
(C) or
(D) and

BI TP THC HNH
Bi tp 1:

ng k hc: 0962 60 8801 04 6260 3948 161


a ch: S 18 Trn i Ngha Q Hai B Trng H ni
NGOI NG 24H 16
WWW.NGOAINGU24H.VN 2

Choose one correct option in the parentheses.


1. The editor is under (press, pressure) to meet the deadline
2. All accountants must acquire the licenses in accordance with (regulations, regulates).
3. At his (suggestion, suggestive), we enter into the partnership with GH, Inc.
4. The traffic accident resulted from her (careless, carelessness).
5. Ms. Anderson went to New York last month for (pleasant, pleasure).
6. The automatic teller machine in the lobby was out of (ordered, order).
Choose the correct option for each of the following sentences.
7. The employees who recently joined the staff were referred to you for ____
(A) guide (C) guidance
(B) guided (D) guidable
8. The proposals are currently under ___ .
(A) consider (C) considerable
(B) considerably (D) consideration
9. Many parts of Africa are currently under ____ .
(A) development (C) develop
(B) developed (D) develops
10. Without ___, all Gainsborough Corporation workers must attend the summer training seminar.
(A) except (C) excepts
(B) exception (D) exceptional

Bi tp 2:
Choose one correct option in the parentheses.
1. The construction of the monument is scheduled to start (on, in) February 5.
2. The coupons for a free drink are valid (during, for) a month.
3. Employees must sign up for the workshop (by. until) tomorrow.
4. We will have an orientation for newcomers (at, in) noon.
5. The website will officially be launched (until, on) New Year's Day.
6. The National Museum will be closed for renovation (in, on) summer.
Choose the correct option for each of the following sentences.
7. The meeting about new marketing strategies will be held _____ 11 a.m.
(A) to (C) at
(B) on (D) for
8. All worker must submit their records to the supervisor _____ the end of the month.
(A) for (C) in
(B) by (D) to
9. All requests for vacation time must be presented in writing _____ June 15.
(A) at (C) in
(B) on (D) for
10. _____ her stay in Paris, Ms. Lernet supervised the new branch office.
(A) For (C) About
(B) Upon (D) During

Bi tp 3:
Choose one correct option in the parentheses.

ng k hc: 0962 60 8801 04 6260 3948 162


a ch: S 18 Trn i Ngha Q Hai B Trng H ni
NGOI NG 24H 16
WWW.NGOAINGU24H.VN 3

1. People are not allowed to sit (on, at) the grass.


2. Two vehicles are waiting (at, in) the traffic light.
3. The official ceremony will be held (on, in) City Hall.
4. Russia is the largest country (in, on) the world.
5. The new department store is located (at, in) the comer of Potrero Avenue and Main Street.
6. He walked (among, between) the two buildings and came upon a large square.
Choose the correct option for each of the following sentences.
7. The company's barbecue party will take place __ Hubbard Park.
(A) in (C) of
(B) on (D) as
8. The post office is conveniently located _____ the office and the subway entrance.
(A) among (C) into
(B) with (D) between
Choose one correct option in the parentheses.
1. Numerous parks can be found (along, into) the roads of Ohio.
2. Ms Jordan took some documents (into, out of) her bag.
3. His secretary came running (out of. across) the street
4. Water runs (against, through) this pipe.
Choose the correct option for each of the following sentences.
5. Mr. Huntington will offer advice _____ managers on how to increase sales.
(A) to (C) on
(B) with (D) upon
6. Visitors who want to enjoy beautiful scenery may drive _____ the coast.
(A) into (C) along
(B) under (D) among
Questions 7 and 8 refer to the following memorandum.
This is to inform you that a number of changes in the Ohio branch of Akron have been made in the
past week. The branch relocated to a new building _____ April 18.
7. (A) for (C) in
(B) at (D) on

Ms. Miller will give you the address and directions as well as the new telephone numbers v for the
branch. Their financial department is also under new _____ Tom Holt, who headed the department
8. (A) manage (B) management
(B) manageable (D) managed

for twenty years, retired on April 20. The new head is Sarah Douglas.

Bi tp 4:
Choose one correct option in the parentheses.
1. You may mail (so, or) fax your resume to Mr. Blunt.
2. The building has (both, either) an outdoor parking lot and an underground garage.
3. He made a request for help (but, and) did not receive any response.
4. The logo must be not only distinctive (and. but) also professional-looking.
5. Participants were told to arrive by noon, (yet, so) most of them were late.
6. The consultant advised neither expanding the factory (or, nor) hiring more workers.
Choose the correct option for each of the following sentences.
7. Ms. Douglas will help customers create an investment _____ savings plan.
ng k hc: 0962 60 8801 04 6260 3948 163
a ch: S 18 Trn i Ngha Q Hai B Trng H ni
NGOI NG 24H 16
WWW.NGOAINGU24H.VN 4

(A) so (C) yet


(B) and (D) but
8. Applicants are required to present _____ a reference letter and a photocopy of their social security
card.
(A) all (C) either
(B) both (D) neither
9. The website is most accessible before 8 a.m. _____ after 5 p.m.
(A) or (C) so
(B) nor (D) yet
10. Neither chatting online _____ checking personal e-mail is permitted during working hours.
(A) or (C) nor
(B) and (D) only

Bi tp 5:
Choose one correct option in the parentheses.
1. We postponed the construction (because, which) there was not enough funding.
2. (If, Although) the government tried to control fuel costs, they increased significantly.
3. The team conducted a survey (which, whether) assessed the market potential for diet food.
4. (Whether, Because) Ms. Fletcher will give the presentation is not certain.
Choose the correct option for each of the following sentences.
5. _____ promotional experience is helpful, it is not a requirement.
(A) And (C) Which
(B) Who (D) Although
6. Profits rose not _____ because of an increase in sales, but also a reduction in expenses.
(A) alone (C) over
(B) only (O) less
Questions 7 and 8 refer to the following letter.
I was just going over the records of the maintenance workers hired last year. They are eligible for
various tax benefits _____ they have held the jobs over six months. Please inform Lisa. One other
7. (A) whether (C) because
(B) but ` (D)

thing I would like to mention is that Mike Evans of purchasing said he would submit the equipment
specifications yesterday, _____ I do not see them anywhere. Let him know that l am waiting for
8. (A) so (B) yet
(C) for (D) both
them.

Bi tp 6:
Practice with TOEIC Actual Questions
Decide which of the choices - (A), (B), (C), or (D) - best completes the sentence.

ng k hc: 0962 60 8801 04 6260 3948 164


a ch: S 18 Trn i Ngha Q Hai B Trng H ni
NGOI NG 24H 16
WWW.NGOAINGU24H.VN 5

1. The executive manager neither sent me an e-mail ___ call me


(A) and
(B) or
(C) nor
(D) also
2. He did his best, ____ his presentation failed to satisfy the buyers.
(A) so
(B) and
(C) because
(D) but
3. ____ the vice president asked us a question, we became really nervous.
(A) Although
(B) Unless
(C) When
(D) So
4. ____ business improves, the company will go bankrupt.
(A) Since
(B) Although
(C) Unless
(D) If
5. The company is sure ____ the new product will boot its profits.
(A) because
(B) whether
(C) that
(D) if
6. Ms. Taylor is ____ intelligent that everybody respects her.
(A) such
(B) so
(C) very
(D) too
(D) intelligent
7. ____ the manager was absent, we decided to put off the meeting.
(A) Although
(B) Providing
(C) Whereas
(D) Because
8. He was hired ____ he didnt have the necessary qualifications.
(A) because
(B) although
(C) now that
(D) as
9. ___ the president will arrive on Friday is not certain.
(A) Whether
ng k hc: 0962 60 8801 04 6260 3948 165
a ch: S 18 Trn i Ngha Q Hai B Trng H ni
NGOI NG 24H 16
WWW.NGOAINGU24H.VN 6

(B) Since
(C) Because
(D) What
10. The items you requested last week will be delivered either on Monday ___ Tuesday.
(A) nor
(B) or
(C) and
(D) so
11. This project began almost five months ago _____ is expected to be finished in December.
(A) or
(B) also
(C) therefore
(D) and
12. Because Im too busy at work, I ____ ever have free time.
(A) hardly
(B) carefully
(C) readily
(D) usually
13. Mr. Kim is the person who is in charge of purchasing office ____ at the company.
(A) records
(B) satisfaction
(C) effort
(D) supplies
14. You are ____ invited to visit the branch office in Seattle.
(A) necessarily
(B) cordially
(C) highly
(D) impressively
Questions 15-16 refer to the following notice:
Accounting Employees:

We will have performance reviews over the next week, _____ I would like to tell you the review
15. (A) so
(B) but
(C) since
(D) because

process.

First of all, we will interview each of you in person. Second, you will have to fill out the evaluation
form for yourself ____ return it to the Personnel Department.
16. (A) so that

ng k hc: 0962 60 8801 04 6260 3948 166


a ch: S 18 Trn i Ngha Q Hai B Trng H ni
NGOI NG 24H 16
WWW.NGOAINGU24H.VN 7

(B) but
(C) and
(D) now that

Lastly, we will write a report that summarizes your performance.

We all hope that this is a great chance to improve your job performance.

Bi tp 7:
Practice with TOEIC Actual Questions
Decide which of the choices - (A), (B), (C), or (D) - best completes the sentence.
1. _____ this construction is over, the building will look different.
(A) For
(B) By
(C) By the time
(D) During
2. The post office and the police station are _____ Main Street.
(A) on
(B) in
(C) at
(D) of
3. If you have any problems _____ the service, just let me know.
(A) regardless
(B) regard
(C) regarding
(D) regarded
4. Thank you _____ your inquiry about our products.
(A) to
(B) as
(C) of
(D) for
5. The president was standing _____ the two bodyguards.
(A) among
(B) between
(C) on
(D) as
6. I have been working at the bank _____ three years.
(A) for
(B) during
(C) while
(D) with
7. _____ he has a lot of experience, he was able to get a job easily.
(A) Even though

ng k hc: 0962 60 8801 04 6260 3948 167


a ch: S 18 Trn i Ngha Q Hai B Trng H ni
NGOI NG 24H 16
WWW.NGOAINGU24H.VN 8

(B) Because
(C) Due to
(D) In spite of
8. The team members had a discussion _____ the advertisement.
(A) to
(B) with
(C) about
(D) from
9. You can get a full refund _____ 30 days of purchase.
(A) to
(B) within
(C) for
(D) as
10. The applicants have to wait _____ next week for the results.
(A) until
(B) by
(C) despite
(D) at
11. The editing must be completed by _____ next Monday the latest.
(A) on
(B) of
(C) in
(D) at
12. Please explain the current situation _____ detail
(A) in
(B) for
(C) on
(D) of
13. _____ the machine is extremely expensive, our company cant afford it.
(A) Because of
(B) Since
(C) Owing to
(D) Also
14. You should prepare the contract _____ duplicate.
(A) with
(B) on
(C) in
(D) for
Questions 15-16 refer to the following memorandum.
This memo is to inform you that our branch office will be renovated. The renovations will begin
____
15. (A) as

ng k hc: 0962 60 8801 04 6260 3948 168


a ch: S 18 Trn i Ngha Q Hai B Trng H ni
NGOI NG 24H 16
WWW.NGOAINGU24H.VN 9

(B) on
(C) in
(D) at

October 21 and are expected to be finished by December.

Therefore, all employees should use the small building on Broadway from Monday. If you have any
questions _____ the location of the building, contact Ms. Endley in the Personnel Department.
16. (A) concerning
(B) concerns
(C) concern
(D) concerned

Bi tp 8:
Correct the underlined prepositions in the following sentences.
1. He has worked in the factory since 11 years.
2. You must submit the applications within Monday.
3. We have lived in New York for last year.
4. Please send me the document at least five days prior the audit.

BI TP KIM TRA
1. He left home early_______ failed to catch the train.
(A) and
(B) therefore
(C) but
(D) or
2. _______ he was cooking, his wife was working in the yard.
(A) But
(B) And
(C) Yet
(D) While
3. He works to support his family, _______ he is in his seventies.
(A) during
(B) despite
(C) although
(D) because
4. I haven't seen him_______ ages.
(A) with
(B) for
(C) during
(D) until
5. He didn't fail the test; _______ he got the highest score.

ng k hc: 0962 60 8801 04 6260 3948 169


a ch: S 18 Trn i Ngha Q Hai B Trng H ni
NGOI NG 24H 17
WWW.NGOAINGU24H.VN 0

(A) likewise
(B) else
(C) otherwise
(D) on the contrary
6. Not only does Steve sing at a bar, _______ he also serves the patrons.
(A) and
(B) or
(C) but
(D) still
7._______ I was waiting in line for a visa interview, I ate my lunch.
(A) Although
(B) While
(C) Since
(D) Because
8. He wouldn't say anything_______ directly asked a question; he was a man of few words.
(A) since
(B) otherwise
(C) or else
(D) unless
9. _______ I'm retired, I can take a long journey throughout Europe.
(A) However
(B) Now that
(C) Thanks to
(D) Due to
10. I drive to school, _______ my brother always cycles.
(A) when
(B) even
(C) whereas
(D) if
11. I have to go to the meeting_______ I want to or not.
(A) because
(B) whether
(C) as soon as
(D) while
12. You'd better take an umbrella_______ it begins to rain.
(A) as soon as
(B) despite
(C) in case
(D) although
13. We're.planning to go camping_______ the summer vacation.
(A) during
(B) while
(C) when
ng k hc: 0962 60 8801 04 6260 3948 170
a ch: S 18 Trn i Ngha Q Hai B Trng H ni
NGOI NG 24H 17
WWW.NGOAINGU24H.VN 1

(D) since
14. She was standing by the window_______ her eyes closed, listening to music.
(A) while
(B) with
(C) about
(D) except
15. I have no objection, if everyone else agrees, _______ leading the meeting.
(A) to your
(B) for your
(C) to you're
(D) on your
16. Just_______ you and me, I prefer the restaurant where we ate last Friday.
(A) among
(B) around
(C) between
(D) from
17. I bought an umbrella_______ it was raining.
(A) so
(B) because
(C) also
(D) why
18._______ you were a member, you could come too.
(A) That
(B) If
(C) Than
(D) So
19._______ I was clean, I took a bath.
(A) What
(B) As long as
(C) Such
(D) Even though
20. I wanted to_______ down because I was quite tired.
(A) lie
(B) bring
(C) lay
(D) take
21._______ he was boarding the bus, he dropped his ticket in the gutter.
(A) Whether
(B) How
(C) While
(D) What
22. You can either rent_______ buy these skateboards.
(A) or
ng k hc: 0962 60 8801 04 6260 3948 171
a ch: S 18 Trn i Ngha Q Hai B Trng H ni
NGOI NG 24H 17
WWW.NGOAINGU24H.VN 2

(B) and
(C) nor
(D) either
23. I neither registered for_______ attended the class.
(A) or
(B) nor
(C) either
(D) neither
24. I have been very tired_______.
(A) late
(B) lately
(C) sometime
(D) yet
25. _______ you hear the bell, you need to leave the classroom.
(A) By the time
(B) As
(C) Whether
(D) As soon as
26. She wore sunscreen_______ her skin would not burn.
(A) because
(B) so
(C) neither
(D) what
27. My father has been working in the company_______ twenty years.
(A) to
(B) over
(C) at
(D) in
28. The Joneses must have left at least one week_______ .
(A) since
(B) ago
(C) advanced
(D) during
29. When my friend opened the door, I walked_______ the house.
(A) over
(B) into
(C) for
(D) with
30. The Joneses must have left at least one week_______.
(A) since
(B) ago
(C) advanced
(D) during
ng k hc: 0962 60 8801 04 6260 3948 172
a ch: S 18 Trn i Ngha Q Hai B Trng H ni
NGOI NG 24H 17
WWW.NGOAINGU24H.VN 3

31. He wouldn't leave the platform_______ her train went out of sight.
(A) so that
(B) until
(C) so
(D) therefore
32. _______ the research uncovered some problems, the company decided to stop the project.
(A) Since
(B) Wherever
(C) Despite
(D) Even though
33. What will become_______ the child now that his parents are gone?
(A) with
(B) at
(C) of
(D) to
34. A theme park will be built in this area_______ the construction plan is approved. (A) on
condition that
(B) so that
(C) whereas
(0) although
35. We have to keep the subsidiaries from bankruptcy_______ the holding company will not be
endangered.
(A) because
(B) in order that
(C) since
(D) as soon as
36. The cabin was so far away from town_______ I decided not to buy it.
(A) but
(B) or
(C) that
(D) if
37. The senior researcher will be promoted to head of the research center_______ the most
experienced researcher here.
(A) as
(B) unless
(C) though
(D) but
38. Though he made a terrible mistake, he acted_______ nothing had happened.
(A) as if
(B) so that
(C) unless
(D) whereas

ng k hc: 0962 60 8801 04 6260 3948 173


a ch: S 18 Trn i Ngha Q Hai B Trng H ni
NGOI NG 24H 17
WWW.NGOAINGU24H.VN 4

Questions 39 to 40 refer to the following article.

Experts say it is very important for children over the age of six to develop a hobby or take pan in a
club. Children can learn important social skills_______ doing something they enjoy. Even though
39.(A) during
(B) despite
(C) while
(D) provided

your child is playing, she will learn skills like turn taking, losing well, and following rules. Hobbies
and clubs are vital for the_______ of your child.

40. (A) be developed


(B) develop
(C) developed
(D) development

Questions 41 to 44 refer to the following email.


Dear Tracey,

Your mother and I were very happy to hear from you at last. It had been almost 4 months_______
41. (A) because
(B) although
(C) since
(D) during

your last email. I really do wish that you would get a telephone. I don't know why you_______ being
42. (A) insist to
(B) insist on
(C) insist by
(D) insist

so difficult to contact. I guess we will just have to respect your privacy. While you almost never
contact us, your sister telephones every day. As you know, I am willing to pay for a phone line
provided you promise to call at least once a month. We don't make many demands of you, Tracey.
_______ we want to see to you more, we don't visit you unexpectedly, do we? Well, I hope that your
43. (A) Since
(B) Even though
(C) Seeing that
(D) Whereas

job is going well. Remember, you know where to find us if you feel like visiting some time. I know

ng k hc: 0962 60 8801 04 6260 3948 174


a ch: S 18 Trn i Ngha Q Hai B Trng H ni
NGOI NG 24H 17
WWW.NGOAINGU24H.VN 5

that would_______ your mother very happy. Look forward to hearing from you.
44. (A) let
(B) cause
(C) make
(D) force

Lots of love, Dad


Questions 45 through 48 refer to the following letter.
Huntley School for Girls
Huntley,
H78 F.88

Dear Mrs. Miles,

It is with great_______ that I am writing this letter. As you know, your daughter Miranda has been
45. (A) pleasure
(B) joy
(C) regret
(D) upset

having problems with some of the other girls. Unfortunately, on Monday there was an unforgivable
incident_______ a math lesson. One girl needed to be hospitalized as a result. We cannot tolerate
46. (A) while
(B) despite
(C) during
(D) enduring

this kind of behavior. Miranda has not listened to any of our warnings and I need to talk to
you_______. We are considering expelling Miranda, but_______ to talk to you first. On the other
47. (A) in person 48. (A) would like
(B) with people (B) had liked
(C) in people (C) will like
(D) as person (D) have liked

hand, her sister Melinda is a delight to teach. We hope that she will continue her studies at Huntley.

Sincerely,
Marjorie Bartle Principal

Questions 49 through 51 refer to the following introduction.


Unpaid Endeavors by Paul Kang, president of Vine Corp., gives invaluable advice on _______ to
49. (A) what
ng k hc: 0962 60 8801 04 6260 3948 175
a ch: S 18 Trn i Ngha Q Hai B Trng H ni
NGOI NG 24H 17
WWW.NGOAINGU24H.VN 6

(B) which
(C) where
(D) how

survive in the competitive business field as a start-up company.

Based on his personal experience, this book tells us how he_______ his business again after a big
50. (A) ran
(B) failed
(C) got over
(D) followed

failure again after a big failure _______ made it a leading company in the software industry.
51. (A) but
(B) and
(C) yet
(D) for

Throughout the book, Kang stresses the importance of teamwork and leadership.

ng k hc: 0962 60 8801 04 6260 3948 176


a ch: S 18 Trn i Ngha Q Hai B Trng H ni
NGOI NG 24H 17
WWW.NGOAINGU24H.VN 7

NGY 10: I T QUAN H

Ch 1. i t quan h c chc nng ca c i t v lin t

1. i t quan h c khi l ch ng, c khi l tc t


Mc ch ca i t quan h l nhm trnh lp li nhiu ln mt t hay mt cm t no khi vit cu.
We will introduce a new product.
It is under development.
We will introduce a new product and it is under development.
(1) i t quan h gip lin kt hai cu v (2) ng vai tr l ch ng ca ng t is.
We will introduce a new product which is under development.
which va l lin t va l ch ng
2. Cc loi i t quan h
Tin t Ch ng S hu Tc t
Ngi who whose whom
S vt which whose / of which which
Ngi v s vt that that

The museum will open a new exhibition which features ancient civilization.
tin t (s vt) i t quan h (lm ch ng)
Bo tng s m mt cuc trin lm mi c ch l nn vn minh c i.
Cu trn cn c lin t lin kt hai ng t will open v features.
Ch ng ca features chnh l which.
V tin t ch s vt (exhibition ngha l cuc trin lm) nn ta phi dng i t quan h which.
Quick Quiz
1. The human resources department has announced a new incentive program ________ will begin next
month.
(A) that (B) such (C) when (D) until

Ng php nng cao

Ch 2. Nu tin t ch ngi, ta dng i t quan h who

ng k hc: 0962 60 8801 04 6260 3948 177


a ch: S 18 Trn i Ngha Q Hai B Trng H ni
NGOI NG 24H 17
WWW.NGOAINGU24H.VN 8

Students who want to apply for a scholarship should submit their transcripts.
tin t i t ng t (1) ng t
quan h (ch ng)
Nhng sinh vin no mun xin hc bng th cn phi np bn sao hc b ca mnh.
who gip lin kt hai ng t want v should submit trong cu.
who l i t quan h c chc nng ch ng (i vi ng t want).
V students l tin t ch ngi nn ta dng who.
Ch 3. Nu tin t ch vt, ta dng i t quan h which
You should first finish the budget report which is due this Friday.
ng t tin t (ch vt) i t ng t (1)
quan h (ch ng)
Trc tin anh nn hon thnh bn bo co ngn sch ht hn vo th su ny.
which gip lin kt hai ng t should first finish v is trong cu.
which l i t quan h c chc nng ch ng (i vi ng t is).
V the budget report l tin t ch s vt nn ta dng which.
Quick Quiz
2. Visitors ___________ need directions should ask at the information desk.
(A) which (B) they (C) who (D) since
3. The city council will sponsor the job fair _______ is scheduled to be held in the Maria Hotel.
(A) because (B) who (C) when (D) which
Ch 4. i t quan h that c dng cho c ngi ln (s) vt
i t quan h that c th thay th cho c who v which. Tuy nhin, i t quan h that khng ng ngay
sau du phy (,) v gii t.
The prices that are listed in the brochure are effective until further notice
tin t i t ng t
quan h (ch ng)
Gi c c lit k trong sch qung co c gi tr cho n khi c thng bo mi.
that gip lin kt hai ng t are listed v are trong cu.
that l i t quan h c chc nng ch ng (i vi ng t are listed),
that c dng thay cho which v tin t l s vt (price ngha l gi c).
Ghi ch
We welcome donations from employees who are interested in the charity event.

Chng ti hoan nghnh cc ng gp ca cc nhn vin quan tm n s kin t thin ny.
ng k hc: 0962 60 8801 04 6260 3948 178
a ch: S 18 Trn i Ngha Q Hai B Trng H ni
NGOI NG 24H 17
WWW.NGOAINGU24H.VN 9

Xc nh loi lin t cn dng


Nu ch trng c chc nng va l ch ng va l lin t th ch trng phi l i t quan h.
La chn i t quan h thch hp
Nu tin t l ngi th dng who, l (s) vt th dng which. That c dng cho c hai trng hp.
Bo m s ha hp gia ch ng (l i t quan h) v ng t trong mnh quan h
Trong v d trn, tin t l s nhiu (employees) nn ta dng ng t s nhiu (are).
Quick Quiz
4. Mr. Kewell developed the product __________ won him the design award.
(A) he (B) that (C) it (D) so that
Ch 5. Mnh quan h rt gn
(1) When the relative pronouns are in the objective case
- He is the only candidate whom we selected.
= He is the only candidate we selected.

- This is the book that I bought yesterday.


= This is the book I bought yesterday.

(2) When the relative pronouns are used with the verb to be
- The man who is talking with Janet is my brother.
= The man talking with Janet is my brother.

- The report which was written by Mike is accurate.


= The report written by Mike is accurate.

- The proposal was revised by Phil was outstanding. (X)


> The proposal that was revised/revised by Phil was outstanding. (O)

Bi tp 1: Chn t thch hp trong ngoc n.


1. The employee (who / whom) makes the most sales will get a bonus.
2. He is the person (who / which) is responsible for online purchase.
3. The people (who / whose) sales exceed $10,000 will get a bonus
4. We are offering a salary (that / that it) is highly competitive.
5. The candidate (whom / whose) they selected has extensive work experience.
6. Ms. Chao gave a presentation (that / whom) we fully appreciated.
7. The budget report (was submitted / submitted) by the Planning Department had some flaws.
8. She lived in France for several years, (which / that) helped her do businesses with Europeans.
9. Job applicants (whom / whose) rsums were received by last week will be contacted soon.
ng k hc: 0962 60 8801 04 6260 3948 179
a ch: S 18 Trn i Ngha Q Hai B Trng H ni
NGOI NG 24H 18
WWW.NGOAINGU24H.VN 0

10. We are looking for a location (why / where) we can bund our next factory.

Bi tp 2: Chn t thch hp in vo ch trng


l. Passengers ____wish to have vegetarian meals will have to notify flight attendants before takeoff.
(A) who (B) whose (C) when (D) what
2. Please refer to the attached agenda of yesterday's seminar ____ you asked for.
(A) then (B) that (C) what (D) when
3. We would like to welcome Dr. Zhang, ____ made such an excellent contribution by opening a branch
office in Africa.
(A) that (B) which (C) when (D) who
4. Our overseas staff is required to speak Spanish, ____is essential to doing businesses in Latin America.
(A) which (B) whose (C) who (D) that
5. In order to stay competitive in the global market, we should know ____our customers need.
(A) that (B) when (C) which (D) what
6. The training session is for programmers____ jobs involve entering computer codes and data.
(A) who (B) whom (C) whose (D) that
7. ____is shown in commercials does not always affect the purchasing decisions of smart buyers.
(A) Who (B) What (C) Which (D) That
8. In accordance with federal law, applicants must be treated equally ____age and gender.
(A) whatever (B) based on (C) whether (D) regardless of
9. ____the manual, all software programs should be shut down before installation,
(A) In common with (B) In case of (C) According to (D) On behalf of
l0. According to the report, items stay on the ____for periods of three to six weeks before they are sold.
(A) market (B) place (C) sale (D) advertisement
Bi tp 3: Chn p n ng cho cu 1 - 3 trong bc th sau y.

Dear Mr. Kurtz,

My name is Daniel Goddard, and I am a journalist from Entrepreneur Weekly. We are preparing our next
____, which will feature the growth of advertising companies.
1.(A) show
(B) investment
(C) issue
(D) episode

We listed some of the most ____ business leaders in the field, and you were selected

ng k hc: 0962 60 8801 04 6260 3948 180


a ch: S 18 Trn i Ngha Q Hai B Trng H ni
NGOI NG 24H 18
WWW.NGOAINGU24H.VN 1

2.(A) successful
(B) accurate
(C) consistent
(D) dramatic

as one of them.

We ____ to meet with you for an interview if you are available and willing to do so
3.(A) would like
(B) will be liked
(C) have liked
(D) liked

Please contact me at 217-447-2314.

We are looking forward to hearing from you soon.

Best regards,

Daniel Goddard
ENTREPRENEUR WEEKLY

BI TP LUYN
Bi tp 1:
Part V: Chn t thch hp in vo ch trng.

1. Mr. Kim knows the location of the office _________ issues parking permits.
(A) who (B) it (C) themselves (D) which

2. We are currently seeking an experienced sales manager _________ has at least five years of experience.
(A) he (B) if (C) who (D) because

3. The number of foreign students _________ want to learn Korean is gradually increasing.
(A) which (B) that (C) for (D) while

ng k hc: 0962 60 8801 04 6260 3948 181


a ch: S 18 Trn i Ngha Q Hai B Trng H ni
NGOI NG 24H 18
WWW.NGOAINGU24H.VN 2

4. A new security door, _________ can be operated by remote control, will be installed next month.
(A) that (B) it (C) them (D) which

5. Many people _________ were surveyed indicated that they are planning to buy a new car.
(A) they (B) who (C) which (D) those

6. The hotel _________ is under renovation is expected to reopen next month.


(A) that (B) such (C) when (D) until

7. The airport limousine service, _________ has been running since January, has been favorably received
by travelers.
(A) who (B) what (C) when (D) which

Part VI: Chn p n ng cho cu 8 ~ 10 trong thng bo sau y.

MEMORANDUM

To : All Employees
From : Personnel Office
Date: May 15
Subject : Leave Request

This is a reminder for those employees __________ are considering making a leave request.
8. (A) when
(B) who
(C) which
(D) until
Before submitting the request, you should first get approval from your supervisor. And you also need to
complete the leave request form _________ can be obtained from the personnel office.
9. (A) what
(B) who
(C) for
(D) that
Please submit the form with your supervisors approval to the personnel office, _________
10.(A) which
(B) it
(C) who
(D) they
is located on the 5th floor next to the employee lounge.
Your leave request cannot be granted if your supervisor denies your request for any reason. If you have
any questions, please contact the personnel office.
ng k hc: 0962 60 8801 04 6260 3948 182
a ch: S 18 Trn i Ngha Q Hai B Trng H ni
NGOI NG 24H 18
WWW.NGOAINGU24H.VN 3

Jerry McCarthy
Jerry McCarthy
Personnel Office

Bi tp 2:
Choose one correct option in the parentheses.
1. Randy Hayes book (which, it) studies international banking is a bestseller.
2. Warehouse workers stocked the item (there, that) the at are popular in stores.
3. The consultant (who, he) gave us advice will visit the office tomorrow.
4. Our department manager will produce reports (that, they) evaluate the employees performance.
5. The head office must hire someone (this, who) can upgrade its accounting software.
6. The report includes the sales figures (those, that) we corrected today.
Choose the correct option for each of the following sentences.
7. Shoppers ___ present a coupon cant get a 20 percent discount.
(A) who (C) there
(B) they (D) when
8. If the lawyer had had time, she would have verified the content of the document ____ you gave her
last week.
(A) who (C) whose
(B) those (D) that
9. The concert _____ was held in the theater yesterday is the best I have ever seen.
(A) who (C) whose
(B) which (D) they
10. The company will move into a new office building, ___ Atlanta Builders constructed.
(A) they (C) which
(B) it those

Bi tp 3:
Chose one correct option in the parentheses.
1. The company is producing a new tire (who, which) is extremely durable.
2. This organization is hiring a PR man (whom, whose) responsibilities will include fundraising.
3. The second edition (who, which) will be available soon has an additional chapter.
4. Writers (whose, who) work is admired internationally usually write about universal subjects.
Choose the correct option for each of the following sentences.
5. He recommends this website for anyone _____ is thinking about buying a car.
(A) when (C) which
(B) what (D) who
6. The secretary printed some invitations _____ he will send tomorrow.
(A) which (C) whose
(B)who (D) what
Questions 7 and 8 refer to the following article.

ng k hc: 0962 60 8801 04 6260 3948 183


a ch: S 18 Trn i Ngha Q Hai B Trng H ni
NGOI NG 24H 18
WWW.NGOAINGU24H.VN 4

Lunsford Publishers announced yesterday that it has released a new book called Lost Wind. It is a
novel about a family _____ lived in Massachusetts in the late nineteenth century. The novel _____
7. (A) who (C) which 8. (A) whatever (C) what
(B) these (D) they (B) this (D) which

gives an accurate picture of life in the 1890s was found in the attic of Mary Woods, a Virginia
resident. Her grandmother is its author.

Bi tp 4:
Practice with TOEIC Actual Questions
Decide which of the choices - (A), (B), (C), or (D) - best completes the sentence.
1. Anyone ____ wants to take a vacation this month needs to talk to me today.
(A) which
(B) whom
(C) who
(D) whose
2. This manual is for people _____ native language is not English.
(A) whose
(B) whom
(C) that
(D) what
3. The decision _____ the company made will cause a lot of problems.
(A) whose
(B) who
(C) what
(D) that
4. I met the representative _____ Mr. Reeves talked about.
(A) that
(B) which
(C) what
(D) how
5. Mr. Delaney met the person _____ she recommended for the managerial position.
(A) what
(B) which
(C) whom
(D) why
6. The specialist _____ we just met will be the manager of our team.
(A) what
(B) whose
(C) who
(D) when
7. This website is good for people _____ are thinking about renting a house.

ng k hc: 0962 60 8801 04 6260 3948 184


a ch: S 18 Trn i Ngha Q Hai B Trng H ni
NGOI NG 24H 18
WWW.NGOAINGU24H.VN 5

(A) they
(B) who
(C) whom
(D) them
8. This is the place _____ we have our annual meeting.
(A) when
(B) where
(C) why
(D) how
9. JW, Inc. is one of the major companies _____ manufactures furniture.
(A) where
(B) whom
(C) whose
(D) which
10. ____ I said to the interviewer was not very clear.
(A) What
(B) When
(C) Which
(D) Whose
11. I can't find the files _____ I saved all the important information.
(A) which
(B) why
(C) when
(D) where
12. The secretary booked the flight ticket _____ advance.
(A) with
(B) on
(C) in
(D) for
13. The second thing _____ the agenda is our poor sales.
(A) on
(B) in
(C) about
(D) at
14. Any car parked here will be towed _____ the owner's expense.
(A) to
(B) on
(C) at
(D) for
Questions 15-16 refer to the following article.
The Pelican Publishing Company announced that it will publish a new book entitled True Story in
March. This is a nonfiction book about the crew, of a spaceship _____ explored the universe.

ng k hc: 0962 60 8801 04 6260 3948 185


a ch: S 18 Trn i Ngha Q Hai B Trng H ni
NGOI NG 24H 18
WWW.NGOAINGU24H.VN 6

15. (A) where


(B) whom
(C) that
(D) when

the book will show readers ____ the crew member handled difficulties on the international spaceship.
16. (A) what
(B) how
(C) and
(D) also

BI TP KIM TRA
1. Soccer is the sport_______ I like the most.
(A) who
(B) that
(C) whom
(D) whose
2. The hotel_______ we stayed was built 100 years ago.
(A) when
(B) which
(C) that
(D) where
3. The man_______ Linda spoke was her English teacher.
(A) who
(B) whose
(C) to whom
(D) that
4. Could you give me_______ pen, please?
(A) whose
(B) other
(C) whom
(D) another
5. I didn't_______ whose car it was.
(A) saw
(B) see
(C) look
(D) speak
6. Jane is the girl_______ mother wrote a famous novel.
(A) whose
(B) who
(C) that
(D) which
7. This is the course in_______ we learned the history of England.
(A) that
(B) which
(C) where

ng k hc: 0962 60 8801 04 6260 3948 186


a ch: S 18 Trn i Ngha Q Hai B Trng H ni
NGOI NG 24H 18
WWW.NGOAINGU24H.VN 7

(D) whose
8. The greatest writer_______ I have ever met was Neil Smith.
(A) whose
(B) that
(C) which
(D) where
9. December 31st is the date_______ we meet every year.
(A) in which
(B) by which
(C) on which
(D) where
10. She is the teacher_______ helped me.
(A) who
(B) when
(C) whose
(D) what
11. _______ he says deserves recording.
(A) Which
(B) Who
(C) That
(D) What
12. Yesterday, I met a woman_______ grandfather was Swedish.
(A) who
(B) who's
(C) whose
(D) which
13. Toni, _______ has three sisters, has no interest in getting married.
(A) who
(B) whom
(C) that
(D) which
14. She put her prize in a place_______ everyone would notice it.
(A) where
(B) which
(C) in what
(D) to that
15. The instructor told us_______ to find the book.
(A) whom
(B) which
(C) where
(D) what
16. There are few mothers_______ don't love their own children.
(A) who
(B) which
(C) but
(D) except
17. Who is that man_______ black?
(A) wears
(B) is wearing
ng k hc: 0962 60 8801 04 6260 3948 187
a ch: S 18 Trn i Ngha Q Hai B Trng H ni
NGOI NG 24H 18
WWW.NGOAINGU24H.VN 8

(C) wear
(D) wearing
18. The Victoria Hotel,_______ we stayed during our summer vacation, will be torn down for
extensive renovations.
(A) when
(B) where
(C) in that
(D) in it
19. Emma Jean, _______ was nominated for an Employee of the Month Award for her exceptional
work performance, will deliver a speech this evening.
(A) who
(B) anyone
(C) whose
(D) whichever
20. Shopping on the Internet is for those consumers for_______ going to malls has become a
nightmare.
(A) who
(B) whom
(C) which
(D) that
21. The person to_______ you submitted your request is no longer in charge of this section.
(A) whom
(B) which
(C) who
(D) where
22. Buses in the metropolitan area are rarely on schedule_______ traffic is busy and hectic.
(A) which
(B) in case
(C) when
(D) in order that
23. The names of upper management to_______ you must report will be provided in order to get
budgets approved for projects.
(A) whom
(B) whoever
(C) what
(D) where
24. Customers_______ products are provided and maintained through an existing arrangement
should contact us immediately.
(A) who
(B) which
(C) whose
(D) that
25. The tourist was unable to find the local information center_______ was marked on his city map.
(A) who
(B) what
(C) which
(D) where
26. The personnel manager has decided to hire the candidate with the degree in engineering_______
he interviewed last week.
ng k hc: 0962 60 8801 04 6260 3948 188
a ch: S 18 Trn i Ngha Q Hai B Trng H ni
NGOI NG 24H 18
WWW.NGOAINGU24H.VN 9

(A) what
(B) which
(C) whom
(D) whose
27. Stanley Super Store is offering all Stanley VIP members free shopping coupons_______ are
valid until the end of the month.
(A) which
(B) what
(C) whatever
(D) who
28. The presenter showed a series of slides_______ detailed the results of her year-long study.
(A) which
(B) whose
(C) whom
(D) who
29. The writers of the contract must revise all of the clauses with_______ the clients disagreed.
(A) whom
(B) which
(C) that
(D) whose
30. This safety helmet has been especially designed for construction workers_______ work outdoors.
(A) in
(B) when
(C) if
(D) who
31. A fine of 5200 will be imposed upon any drivers_______ park illegally downtown during the
holiday parade.
(A) which
(B) whose
(C) whom
(D) who
32. Saturday's clearance sale will reduce old stock and make room for next season's products,
_______ will arrive very soon.
(A) when
(B) what
(C) where
(D) which
33. The Yunof brand of teas, _______ entered the market in Norway only three years ago, is already
among the top five best-selling brands.
(A) when
(B) who
(C) where
(D) which
34. New employees_______ wish to receive benefits should complete the necessary forms before the
10:00 A.M. orientation session.
(A) whose
(B) whom
(C) who
(D) which
ng k hc: 0962 60 8801 04 6260 3948 189
a ch: S 18 Trn i Ngha Q Hai B Trng H ni
NGOI NG 24H 19
WWW.NGOAINGU24H.VN 0

35. Ms. Reston and Mr. Parnthong were two of the senior partners_______ visited the clients last
week.
(A) who
(B) when
(C) what
(D) whose
36. People_______ are interested in career opportunities are invited immediately to submit their
rsums and cover letters to us.
(A) whose
(B) whom
(C) whomever
(D) who

Questions 37 through 39 refer to the following notice.

Notice

From next week, all cars _____ are parked in front of the building will be towed away. Any

37. (A) who


(B) whose
(C) that
(D) what

employees who want to park near the building must apply for a special permit. All cars _____ a
38. (A) by
(B) for
(C) in
(D) with

permit sticker will be allowed to park in the car park surrounding the building. Drivers who want a
permit must visit the personnel office and fill in a form. You will have to pay $5 to receive your
sticker when you fill in the form. Please call 843-9876 for _____ information.
39. (A) more
(B) add
(C) plus
(D) over

Questions 40 through 42 refer to the following letter.


Rirakku Designs, Ikaruga City 221900
Tel: 81-(0)73-998-743

Dear Ms. Hives,

I am _____ you the designs that you asked for. I hope that you like them. If you are not happy with
ng k hc: 0962 60 8801 04 6260 3948 190
a ch: S 18 Trn i Ngha Q Hai B Trng H ni
NGOI NG 24H 19
WWW.NGOAINGU24H.VN 1

40. (A) send


(B) sent
(C) sending
(D) to send

the work that I have done, please call me. My telephone number is _____ the top of this page. I tried
41. (A) at
(B) by
(C) with
(D) in

to include all the colon that you wanted. I have been In hospital. That is the reason _____ I have sent
42. (A) for
(B) which
(C) why
(D) when

this a little later than you expected. I am sorry for the inconvenience. Please let me know the exact
date that you need the finished product.

Thank you for using Rirakku Designs.

Sincerely,
Ken Miyoshi

Questions 43 through 45 refer to the following email.


To: Nina Tendo <cnintcn@cosmo.net>
From: Jane Sims <cisints@united_foods.ccia>
Subject: Customer Complaint 100234
Date: May 29th

Dear Ms. Tondo, Thank you for your email last week. I am very sorry to hear that you had a problem
with a United Foods product. You _____ us that you found ants in a packet of Jolly Cowboy
cookies.
43. (A) identified
(B) informed
(C) information
(D) identify

We went to the store where you bought the cookies. We found ants in several packets of Jolly
cowboy cookies. We found that ants had entered the store room. We treated the problem, and there
are _____ ants in the store room. We are very sorry that this happened, and hope you will continue to
44. (A) not more
ng k hc: 0962 60 8801 04 6260 3948 191
a ch: S 18 Trn i Ngha Q Hai B Trng H ni
NGOI NG 24H 19
WWW.NGOAINGU24H.VN 2

(B) more
(C) no more
(D) any more
buy our food products. I am sending you a $20 coupon, _____ you can print out and use in any store
45. (A) which
(B) what
(C) where
(D) who

to buy United Foods products.

Sincerely,
Jane Sims Customer Service Manager

Questions 46-48 refer to the following memorandum.


Yesterday, November 10, at approximately 3 p.m. we had a temporary of services _____. At the
time,
46. (A) affordability
(B) interruption
(C) status
(D) affair

an electrician was setting up some electric outlets in the machine room, _____ various network
47. (A) which
(B) nearby
(C) where
(D) however

servers are housed.

When he was finishing up, he found a loose wire which he connected the other day, so he had to
reconnect it. However, it appears that when reconnecting it, he made some mistakes. _____ the
48. (A) Among
(B) Amid
(C) So
(D) Until

machines affected were the new NS3, one of the primary name servers, and Happer, which controls
the new operating system.

Thanks,
Bill

ng k hc: 0962 60 8801 04 6260 3948 192


a ch: S 18 Trn i Ngha Q Hai B Trng H ni
NGOI NG 24H 19
WWW.NGOAINGU24H.VN 3

ng k hc: 0962 60 8801 04 6260 3948 193


a ch: S 18 Trn i Ngha Q Hai B Trng H ni
NGOI NG 24H 19
WWW.NGOAINGU24H.VN 4

NGY 11
DNG CU HI C MT NGI HOC
NHIU NGI TRONG HNH
I. DNG CU HI C MT NGI TRONG HNH.
Trng tm ca bi hc
Trong cu hi v hnh nh Part 1, cn lu con ngi l trng tm. Cn quan st k ng tc ca
ngi . y l dng cu hi t con ngui lm trng tm nn hc vin cn luyn tp cch miu t
nhng c im v ngoi hnh, ng tc ca nhn vt da theo cu trc be + V-ing, ng thi cn
lm quen vi nhng t vng miu t vt hay cnh vt xung quanh con ngi.
Phn tch hnh 001.mp3

Vocabulary
hold cm, nm operate vn hnh
look down at nhn xung photocopier my sao chp li liu
machine my mc along dc theo
make a copy sao chp ti liu closed b ng li

ng k hc: 0962 60 8801 04 6260 3948 194


a ch: S 18 Trn i Ngha Q Hai B Trng H ni
NGOI NG 24H 19
WWW.NGOAINGU24H.VN 5

Bi tp 1: 002.mp3

Nghe nhng cu di y v in vo ch trng.

1. The woman is ________ the street with a broom.

2. The woman is ________ the girls arm.

3. He is ________ a phone conservation at the desk.

4. The speaker is ________ into the microphone.

5. He is ________ on the paper with a pen.

6. She is ________ the liquid into the cup.

7. The man is ________ at the computer.

8. A woman is ________ something under her arm

Vocabulary
street ng ph microphone micr
broom ci chi pour A into B /rt A vo B
examine kim tra liquid nc, cht lng
conduct a phone conversation ni chuyn carry mang, vc
in thoi

Bi tp 2: 003.mp3
Nghe cc cu miu t v bc hnh di y, nu c ni dung ph hp vi hnh, chn T; ngc li
chn F.

1. (A) [T/F]
(B) [T/F]
(C) [T/F]
(D) [T/F]
(E) [T/F]
(F) [T/F]

Vocabulary
at the water's edge ti mp nc put on mc (qun o)
carry mang, vc, khun wear c ~ trn ngui, mc, i, mang
tire lp, v xe cut down cht
by oneself t ai glove gng tay

Bi tp 3 004.mp3
ng k hc: 0962 60 8801 04 6260 3948 195
a ch: S 18 Trn i Ngha Q Hai B Trng H ni
NGOI NG 24H 19
WWW.NGOAINGU24H.VN 6

Nghe cc cu miu t v bc hnh di y, nu c ni dung ph hp vi hnh, chn T; ngc li


chn F.
(A) [ T / F ]
(B) [ T / F ]
(C) [ T / F ]
(D) [ T / F ]
(E) [ T / F ]
(F) [ T / F ]

Vocabulary
type nh my monitor mn hnh
keyboard bn phm conduct a phone conversation ni chuyn
reach for vi ly qua in thoi

ng k hc: 0962 60 8801 04 6260 3948 196


a ch: S 18 Trn i Ngha Q Hai B Trng H ni
NGOI NG 24H 19
WWW.NGOAINGU24H.VN 7

Bi tp 4: 006.mp3
Chn cu ng qua li miu t v cc bc hnh di y, lng nghe v in vo ch trng.

1. 2.

A B C D A B C D
(A) The man is ___________________ . (A) He is ________________________ .
(B) The man is ___________________ . (B) He is ________________________ .
(C) The man is ___________________ . (C) He is ________________________ .
(D) The man is ___________________ . (D) He is ________________________ .

3. 4.

A B C D A B C D
(A) The woman is_________________ . (A) She is _______________________ .
(B) The woman is_________________ . (B) She is _______________________ .
(C) The woman is_________________ . (C) She is _______________________ .
(D) The woman is_________________ . (D) She is _______________________ .

ng k hc: 0962 60 8801 04 6260 3948 197


a ch: S 18 Trn i Ngha Q Hai B Trng H ni
NGOI NG 24H 19
WWW.NGOAINGU24H.VN 8

II. DNG CU HI C NHIU NGI TRONG HNH.


Trng tm ca bi hc
Trng hp trong hnh c nhiu hn hai ngi, nu nhng ngi trong hnh khng c ng tc
chung th phi ch quan st xem c hnh ng no khc bit hay khng. Ch lm quen vi cc t
vng i km vi gii t nhm din t mi quan h v tr ca con ngi hoc s vt v hc cch gii
thch cc im ging v khc nhau ca nhng ngi trong hnh.
Phn tch hnh 007.mp3

Vocabulary
relax th gin have ones legs crossed bt cho chn
outdoors ngoi tri (indoors trong side by side cnh bn nhau
nh) be placed c b tr / t
in different directions theo cc hng khc along dc theo
nhau fence hng ro

Bi tp 5: 008.mp3
Nghe nhng cu di y v in vo ch trng.

1. They are ___________ a document.

2. They are ___________ the audience.

3. The musicians are ___________ instruments.

ng k hc: 0962 60 8801 04 6260 3948 198


a ch: S 18 Trn i Ngha Q Hai B Trng H ni
NGOI NG 24H 19
WWW.NGOAINGU24H.VN 9

4. They are ___________ in a row.

5. They are ___________ from each other.

6. They are ___________.

7. Children are ___________ the water fountain.

8. The travelers are ___________ their suitcases.

Vocabulary
look at nhn line up xp hng
document ti liu in a row thnh mt hng
audience khn gi meeting cuc hp
musician nhc s water fountain vi nc
instrument nhc c (= musical instrument) suitcase vali

Bi tp 6 009.mp3
Nghe cc cu miu t v bc hnh di y, nu c ni dung ph hp vi hnh, chn T; ngc li
chn F.

1. (A) [T/F]
(B) [T/F]
(C) [T/F]
(D) [T/F]
(E) [T/F]
(F) [T/F]

Vocabulary
walk i b enter i vo
ground mt t front gate cng trc
water ti nc

Bi tp 7 010.mp3
Nghe cc cu miu t v bc hnh di y, nu c ni dung ph hp vi hnh, chn T; ngc li
chn F.

ng k hc: 0962 60 8801 04 6260 3948 199


a ch: S 18 Trn i Ngha Q Hai B Trng H ni
NGOI NG 24H 20
WWW.NGOAINGU24H.VN 0

2. (A) [T/F]
(B) [T/F]
(C) [T/F]
(D) [T/F]
(E) [T/F]
(F) [T/F]

Vocabulary
gesture lm iu b
in two rows thnh hai hng
face nhn i din
brief gii thch
colleague ng nghip
gymnasium phng tp th dc
speaker ngi ni

ng k hc: 0962 60 8801 04 6260 3948 200


a ch: S 18 Trn i Ngha Q Hai B Trng H ni
NGOI NG 24H 20
WWW.NGOAINGU24H.VN 1

Bi tp 8 012.mp3
Chn cu ng qua li miu t v cc bc hnh di y, lng nghe v in vo ch trng.

1. 2.

A B C D A B C D
(A) People are____________________ . (A) The picture is _________________ .
(B) The band is ___________________ . (B) People are ____________________ .
(C) People are____________________ . (C) People are ____________________ .
(D) Some people are _______________ . (D) One of the men is ______________ .

3. 4.

A B C D A B C D
(A) One woman is ________________ . (A) All the people are ______________ .
(B) One of the women is ___________ . (B) People are ____________________ .
(C) The women are _______________ . (C) People are ____________________ .
(D) The women are ________________ (D) People are ____________________ .

ng k hc: 0962 60 8801 04 6260 3948 201


a ch: S 18 Trn i Ngha Q Hai B Trng H ni
NGOI NG 24H 20
WWW.NGOAINGU24H.VN 2

BI TP KIM TRA
Example 1: Choose the statement that best describes the
photograph. 001.mp3
(A) She is painting a picture.
(B) She is sitting on a bench
(C) She is crossing the street
(D) She is looking at a paiting on the wall

Example 2. Choose the statement that best describes the


photograph.
(A) People are looking at fish
(B) People are fishing
(C) People are diving
(D) People are washing their cars.

Bi tp 1: Listen and choose the statement that best


describes each photograph. 002.mp3
(B) She is holding a map
1. 2.

( (A) A man is taking a picture.


A) (B) A man is in an art gallery.
She
is reading a book.
3. phone is being used
(B) A woman is using her cell phone.
4.

(
A)
A
pub (A) A public phone is being used
lic (B) A woman is using her cell phone.

ng k hc: 0962 60 8801 04 6260 3948 202


a ch: S 18 Trn i Ngha Q Hai B Trng H ni
NGOI NG 24H 20
WWW.NGOAINGU24H.VN 3

5.

(A) The women are looking at each other,


(B) The women are seated side by side.

ng k hc: 0962 60 8801 04 6260 3948 203


a ch: S 18 Trn i Ngha Q Hai B Trng H ni
NGOI NG 24H 20
WWW.NGOAINGU24H.VN 4

Bi tp 2:
Listen and choose the statement that best describes each photograph. 003.mp3
1. 2.

( (A) Some people are walking on the path.


A) (B) They are riding their bicycles uphill.
The (C) Bicycles are parked along the path.
man is watering the plants 4.
(B) There are leaves on all of the trees.
(C) He is putting the hose away
3.

(
A)
The
(A y
) are taking the elevator down.
Veg (B) They are going down the escalator.
etab (C) They are walking down the steps.
les are being placed on the table. 6.
(B) The food is in her shopping cart.
(C) She is shopping for some produce. (
5.

(
A) A) They are moving in a line.
The (B) They are sitting on the ground.
ma (C) They are climbing over a wall.
n is getting out of the vehicle. 7.
(B) The man is standing beside the car.
(C) The
man is parking
his car.

ng k hc: 0962 60 8801 04 6260 3948 204


a ch: S 18 Trn i Ngha Q Hai B Trng H ni
NGOI NG 24H 20
WWW.NGOAINGU24H.VN 5

(A) A truck is moving down the street. 8


(B) The street is filled with cars.
(C) Some people are in the street.

(A) The front of the


house is being
painted.
(B) A man is
working on the
roof.
(C) The ladder goes up to the roof

Bi tp 3: Practice with Possible statements 004.mp3


Listen and choose the statement that best describes each photograph. Then, listen again and fill the
missing words in the gapped statements below
1. (A) She _____ _____ _____ the stair.
(B) She _____ _____ _____ on the handrail.
(C) She _____ _____ _____ the stairs.

2. (A) She ______ _______ a ______.


(B) She ______ _______ a ______ from the shelf.
(C) She ______ _______ a ______ at the library.

3. (A) A _____ _____ a _____.


(B) She _____ _____ a from the shelf.
(C) She _____ _____ a _____ at a library.

4. (A) The man _____ _____ a suitcase.


(B) The man _____ _____ _____ a suit.
(C) The man _____ _____ _____ the sofa.

ng k hc: 0962 60 8801 04 6260 3948 205


a ch: S 18 Trn i Ngha Q Hai B Trng H ni
NGOI NG 24H 20
WWW.NGOAINGU24H.VN 6

5. (A) They _____ just _____ the finish line.


(B) The bikers _____ _____ helmets.
(C) The bicycles _____ _____ on the front.

6. (A) The man is _____ _____ the lawn.


(B) The man is _____ some _____.
(C) The man is _____ _____ _____.

7. (A) A woman _____ _____ a broom.


(B) The area is _____ _____.
(C) A woman is _____ the _____.

8. (A) They are _____ _____ .


(B) They are _____ in a circle.
(C) She is _____ _____ .

10. (A) He is _____ some _____.


(B) He is _____ some _____.
(C) He is _____ some _____.

11. (A) She is _____ her _____.


(B) She is _____ at her _____.
(C) She is _____ some _____.

ng k hc: 0962 60 8801 04 6260 3948 206


a ch: S 18 Trn i Ngha Q Hai B Trng H ni
NGOI NG 24H 20
WWW.NGOAINGU24H.VN 7

12. (A) They are sitting _____ _____ each other.


(B) They are _____ by the _____.
(C) The flower pot _____ _____ the _____

ng k hc: 0962 60 8801 04 6260 3948 207


a ch: S 18 Trn i Ngha Q Hai B Trng H ni
NGOI NG 24H 20
WWW.NGOAINGU24H.VN 8

ng k hc: 0962 60 8801 04 6260 3948 208


a ch: S 18 Trn i Ngha Q Hai B Trng H ni
NGOI NG 24H 209
WWW.NGOAINGU24H.VN

Bi tp 4: Practice with TOEIC Actual Questions 005.mp3


Listen and choose the statement that best describes each photograph
1.

2.

3.

4.

5.

6.

ng k hc: 0962 60 8801 04 6260 3948 209


a ch: S 18 Trn i Ngha Q Hai B Trng H ni
NGOI NG 24H 210
WWW.NGOAINGU24H.VN

7.

8.

9.

10.

ng k hc: 0962 60 8801 04 6260 3948 210


a ch: S 18 Trn i Ngha Q Hai B Trng H ni
NGOI NG 24H 211
WWW.NGOAINGU24H.VN

NGY 12: DNG CU HI C HNH NH VT


HOC PHONG CNH THIN NHIN
I. DNG CU HI C HNH NH VT.
Trng tm ca bi hc
Trong hnh khng xut hin con ngi m ch xut hin vt nn cn nm c cc p n c li
gii thch v mi lin h v tr hay trng thi ca cc s vt xung quanh. Th b ng be + p.p (qu
kh phn t) l cu trc cu vi hnh thc c bn ch ng b tc ng bi hnh ng. S vt
khng t n thc hin ng tc nn vi trng hp ca Part 1 th cc p n cho sn c ch ng l
vt v thng c trnh by vi hnh thc b ng.
Phn tch hnh 013.mp3

Vocabulary
stack sp xp thnh cm / ng load cht, ti ln
in lines theo hng item hng ha
identical ging nhau

Bi tp 1 014.mp3
Nghe nhng cu di y v in vo ch trng.

1. The kitchen is ___________________________.

2. The shoes ________________ on the shelf.

3. ___________________________ over both doors.

ng k hc: 0962 60 8801 04 6260 3948 211


a ch: S 18 Trn i Ngha Q Hai B Trng H ni
NGOI NG 24H 212
WWW.NGOAINGU24H.VN

4. The driveway is ___________________________.

5. A vehicle ____________ next to the building.

6. The boats have ___________________ at the dock.

7. Several cars __________________ at a stop sign.

8. The desk ___________________________.

Vocabulary
be cleaned c lau dn park u (xe)
display trng by dock bn tu
arch vm, mi vm line up xp hng
pave lt / ph stop sign bin bo dng

Bi tp 2 015.mp3
Nghe cc cu miu t v bc hnh di y, nu c ni dung ph hp vi hnh, chn T; ngc li
chn F.

1. (A) [T/F]
(B) [T/F]
(C) [T/F]
(D) [T/F]
(E) [T/F]
(F) [T/F]

Vocabulary
be lined up c xp theo hng be paved c lt
along dc theo pedestrian ngi i ng
fire hydrant vi rng sidewalk va h
curb mp ng quiet yn tnh

Bi tp 3 016.mp3
Nghe cc cu miu t v bc hnh di y, nu c ni dung ph hp vi hnh, chn T; ngc li
chn F.

ng k hc: 0962 60 8801 04 6260 3948 212


a ch: S 18 Trn i Ngha Q Hai B Trng H ni
NGOI NG 24H 213
WWW.NGOAINGU24H.VN

2. (A) [T/F]
(B) [T/F]
(C) [T/F]
(D) [T/F]
(E) [T/F]
(F) [T/F]

Vocabulary
shelf k purchase mua bn
be filled with c cht y candy ko
be stocked with c cht ~ product hng bottom di cng
ha empty trng
plastic bag ti nylon upper trn

Bi tp 4 018.mp3
Chn cu ng qua li miu t v cc bc hnh di y, lng nghe v in vo ch trng.

1. 2.

A B C D A B C D
(A) Food is ______________________ . (A) The window is ________________ .
(B) There is _____________________ . (B) The rug is ____________________ .
(C) The cake is ___________________ . (C) There is ______________________ .
(D) The dishes are ________________ . (D) The coffee cups have ___________ .

3. 4.

ng k hc: 0962 60 8801 04 6260 3948 213


a ch: S 18 Trn i Ngha Q Hai B Trng H ni
NGOI NG 24H 214
WWW.NGOAINGU24H.VN

A B C D A B C D
(A) People are____________________ . (A) Some people are ______________ .
(B) There is _____________________ . (B) The suit is ____________________ .
(C) Some people are _______________ . (C) The lights are _________________ .
(D) The cars are __________________ . (D) The clerk is ___________________ .

ng k hc: 0962 60 8801 04 6260 3948 214


a ch: S 18 Trn i Ngha Q Hai B Trng H ni
NGOI NG 24H 215
WWW.NGOAINGU24H.VN

II. Dng cu hi c hnh phong cnh thin nhin

Trng tm ca bi hc
L dng cu hi ly phong cnh thin nhin lm i tng trng tm ch khng phi con ngi. Cc
p n c th hin bng hnh thc b ng do ch ng din t s vt hoc ni chn. Cc bn hy
c gng nm vng cc cm t miu t thin nhin v phong cnh.
Phn tch hnh 019.mp3

Vocabulary
path ng mn on both sides hai bn ng
woods rng cy have thick leaves c tn I dy
fallen leaves l rng be wooded c nhiu cy
be scattered lc c, ri rc heavily nhiu, dy dc

Bi tp 5 020.mp3
Nghe nhng cu di y v in vo ch trng.

1. The path is ______________________ a forest.

2. Skyscrapers ______________________ the city.

3. There is a ______________________ along the side walk.

4. The mountain peak is ______________________ the snow.

5. The lake is ______________________.

6. There are ______________________ around the tall buildings.

ng k hc: 0962 60 8801 04 6260 3948 215


a ch: S 18 Trn i Ngha Q Hai B Trng H ni
NGOI NG 24H 216
WWW.NGOAINGU24H.VN

7. The boats are ______________________ in the water.

8. There is a ______________________ the lake.

Vocabulary
path ung mn peak nh, chp
passthrough i qua be covered with c ph ln
skyscraper nh chc tri calm yn tnh
tower cao vt hn float lnh bnh, ni
grassy ph y c bridge chic cu
sidewalk va h

Bi tp 6 021.mp3
Nghe cc cu miu t v bc hnh di y, nu c ni dung ph hp vi hnh, chn T; ngc li
chn F.

1. (A) [T/F]
(B) [T/F]
(C) [T/F]
(D) [T/F]
(E) [T/F]
(F) [T/F]

Vocabulary
shovel xc be cut down b cht, n
being used ang c s dng a pile mt ng
be covered with c bao ph bi lose the leaves rng l

Bi tp 7 022.mp3
Nghe cc cu miu t v bc hnh di y, nu c ni dung ph hp vi hnh, chn T; ngc li
chn F.

2. (A) [T/F]
(B) [T/F]
(C) [T/F]
(D) [T/F]
(E) [T/F]
(F) [T/F]

Vocabulary

ng k hc: 0962 60 8801 04 6260 3948 216


a ch: S 18 Trn i Ngha Q Hai B Trng H ni
NGOI NG 24H 217
WWW.NGOAINGU24H.VN

duck con vt
swim bi li
lake h
be docked cp bn
water's edge mp nc
object s vt, vt th
be reflected phn chiu
feed cho n
wave vy tay
deck boong tu

ng k hc: 0962 60 8801 04 6260 3948 217


a ch: S 18 Trn i Ngha Q Hai B Trng H ni
NGOI NG 24H 218
WWW.NGOAINGU24H.VN

Bi tp 8 024.mp3
Chn cu ng qua li miu t v cc bc hnh di y, lng nghe v in vo ch trng.

1. 2.

A B C D A B C D
(A) Some people are _______________ . (A) The trees _____________________ .
(B) The mountain is _______________ . (B) The buildings are ______________ .
(C) The trees are __________________ . (C) The buildings are ______________ .
(D) There are ____________________ . (D) There are _____________________ .

3. 4.

A B C D A B C D
(A) Most of the trees are ____________ . (A) There are _____________________ .
(B) The area is ___________________ . (B) The water is __________________ .
(C) The road _____________________ . (C) People are ____________________ .
(D) There are ____________________ . (D) People are ____________________ .

ng k hc: 0962 60 8801 04 6260 3948 218


a ch: S 18 Trn i Ngha Q Hai B Trng H ni
NGOI NG 24H 219
WWW.NGOAINGU24H.VN

BI TP KIM TRA
Example 1: Choose the statement that best describes the photograph. 006.mp3
(A) The store is crowded with shoppers.
(B) Shoes are on display in a store
(C) A woman is taking off her shoes
(D) Some shoppers are trying on shoes.

Example 2: Choose the statement that best describes the


photograph.
(A) The building are under construction.
(B) Construction vehicles have been parked near the tree.
(C) Cars are parked on both sides of the street.
(D) A person is getting in a car.

Bi tp 1: Listen and choose the statement that best

des
cribes each photograph.
007.mp3
1.

2.
(A) Books are arranged in rows.
(B) Books are stacked on the table.

3.

(A) A man is painting a wall.


(B) A ladder is leaning against a building.
4.
(
A)
Th
ere is an armchair by the window.
(B) There are vases on the table.
ng k hc: 0962 60 8801 04 6260 3948 219
a ch: S 18 Trn i Ngha Q Hai B Trng H ni
NGOI NG 24H 220
WWW.NGOAINGU24H.VN

(B) Papers are spread out across the table.


(A) Chairs have been placed around the table
6.

(A) There is a fence in front of the building


(B) There are columns in the front of building.

5.

(A) The

buildings are under construction.


(B) The window is widely open

Bi tp 2:
Listen and choose the statement that best describes each
photograph.
1. (
A)
So
me
peo
ple

ng k hc: 0962 60 8801 04 6260 3948 220


a ch: S 18 Trn i Ngha Q Hai B Trng H ni
NGOI NG 24H 221
WWW.NGOAINGU24H.VN

are sitting in the bench. 2.


(B) There is a bicycle on the path.
(C) Some buildings overlook the park.

(A) The ship has been tied to the dock.


(B) Some people are sitting in chairs.
(C) A man is swimming in the water.
3. 4.

(A) The train platform is empty.


(B) The train doors are open.
(C) There are passengers waiting to board the
train
(A) The
man is
looking
through 6.
a telescope.
(B) There is a microscope on the desk.
(C) The doctor is reading a patient's record.
5.

(A) The
dining
room is
full of people. (A) P
(B) A couple is waiting to take their seats. (B) A vehicle is parked in front of a plane.
(C) Several tables are by the window. (C) The plane is landing on the ground.

ng k hc: 0962 60 8801 04 6260 3948 221


a ch: S 18 Trn i Ngha Q Hai B Trng H ni
NGOI NG 24H 222
WWW.NGOAINGU24H.VN

7.
8.

(A)
So
me (A) There
box is a fence
es are placed on the floor. in front
(B) There is some fruit on display. of the
(C) The garden is full of fruit trees.

building
(B) The man is climbing up the ladders.
(C) Some ladders are leaing against the
building.

ng k hc: 0962 60 8801 04 6260 3948 222


a ch: S 18 Trn i Ngha Q Hai B Trng H ni
NGOI NG 24H 223
WWW.NGOAINGU24H.VN

Bi tp 3: 009.mp3
Listen and choose the statement that best describes each photograph. Then, listen again and fill
the missing words in the gapped statements below
1. (A) The woman is _____ some bread.
(B) Different kinds of bread _____ _____ _____ for sale.
(C) the woman is _____ a shopping cart.

2. (A) The ______ door is _____.


(B) There are _____ on _____ of the stairs.
(C) A _____ is _____ down the stairs.

3. (A) Some people are _____ their bikes.


(B) The bikes are _____ next to a _____.
(C) Some of the bikes _____ baskets.

4. (A) One woman is _____ a cup of _____.


(B) One man is _____ in a notebook.
(C) There are some _____ on the _____.

5. (A) Some _____ is on the _____.


(B) The tables are all _____.
(C) A waiter is _____ the _____.

6. (A) The woman is _____ her suitcase.


(B) The suitcase is ____ the ____.
(C) The porter is _____ the bag

7. (A) The _____ are being _____.


(B) The wagon is full of _____.
ng k hc: 0962 60 8801 04 6260 3948 223
a ch: S 18 Trn i Ngha Q Hai B Trng H ni
NGOI NG 24H 224
WWW.NGOAINGU24H.VN

(C) There are several types of _____.

8. (A) There are some _____ on the _____.


(B) The river is _____ with _____.
(C) A ship is _____ under the _____.

9. (A) There are some _____ on the _____.


(B) The river is _____ with _____.
(C) A ship is _____ under the _____.

10. (A) There are no _____ in the _____.


(B) People are wheeling carts out of the ____.
(C) The statue is _____ _____ the building.

11. (A) There are monitors all _____ the _____.


(B) All of the people are _____ the _____ channel.
(C) The men are _____ monitors.

12. (A) Many ____ are ____ in the ground.


(B) The wheelbarrow ____ _____ on the path.
(C) The gardener is _____ _____.

Bi tp 4: Listen and choose the statement that best describes


each photograph. 010.mp3
1.

2.

ng k hc: 0962 60 8801 04 6260 3948 224


a ch: S 18 Trn i Ngha Q Hai B Trng H ni
NGOI NG 24H 225
WWW.NGOAINGU24H.VN

3.

4.

5.

6.

7.

ng k hc: 0962 60 8801 04 6260 3948 225


a ch: S 18 Trn i Ngha Q Hai B Trng H ni
NGOI NG 24H 226
WWW.NGOAINGU24H.VN

8.

9.

10.

ng k hc: 0962 60 8801 04 6260 3948 226


a ch: S 18 Trn i Ngha Q Hai B Trng H ni
NGOI NG 24H 227
WWW.NGOAINGU24H.VN

BI 13: KIM TRA PART 1

TEST 01:
Listening Comprehension
In the Listening test, you will be asked to demonstrate how well you understand spoken English. The
entire Listening test will last approximately 45 minutes. There are four parts, and directions are given
for each part. You must mark your answers on the separate answer sheet. Do not write your answers
in your test book.

Part 1
Directions: For each question in this part, you will hear four statements about a picture in your test
book. When you hear the statements, you must select the one statement that best describes what you
see in the picture. Then, find the number of the question on your answer sheet and mark your answer.
The statements will not be printed in your test book and will be spoken only one time.

Look at the example item below.

Now listen to the four statements.


Sample Answer
(A) (B) (D)
Statement (C), Theyre standing near the table, is the best description of the picture, so you should
select answer (C) and mark it on your answer sheet.

ng k hc: 0962 60 8801 04 6260 3948 227


a ch: S 18 Trn i Ngha Q Hai B Trng H ni
NGOI NG 24H 228
Test01.wmv
WWW.NGOAINGU24H.VN

1.

2.

ng k hc: 0962 60 8801 04 6260 3948 228


a ch: S 18 Trn i Ngha Q Hai B Trng H ni
NGOI NG 24H 229
WWW.NGOAINGU24H.VN

3.

4.

ng k hc: 0962 60 8801 04 6260 3948 229


a ch: S 18 Trn i Ngha Q Hai B Trng H ni
NGOI NG 24H 230
WWW.NGOAINGU24H.VN

5.

6.

ng k hc: 0962 60 8801 04 6260 3948 230


a ch: S 18 Trn i Ngha Q Hai B Trng H ni
NGOI NG 24H 231
WWW.NGOAINGU24H.VN

7.

8.

ng k hc: 0962 60 8801 04 6260 3948 231


a ch: S 18 Trn i Ngha Q Hai B Trng H ni
NGOI NG 24H 232
WWW.NGOAINGU24H.VN

9.

10.

ng k hc: 0962 60 8801 04 6260 3948 232


a ch: S 18 Trn i Ngha Q Hai B Trng H ni
Test02.wmv NGOI NG 24H 233
WWW.NGOAINGU24H.VN

TEST 02:

2.

ng k hc: 0962 60 8801 04 6260 3948 233


a ch: S 18 Trn i Ngha Q Hai B Trng H ni
NGOI NG 24H 234
WWW.NGOAINGU24H.VN

3.

4.

ng k hc: 0962 60 8801 04 6260 3948 234


a ch: S 18 Trn i Ngha Q Hai B Trng H ni
NGOI NG 24H 235
WWW.NGOAINGU24H.VN

5.

6.

ng k hc: 0962 60 8801 04 6260 3948 235


a ch: S 18 Trn i Ngha Q Hai B Trng H ni
NGOI NG 24H 236
WWW.NGOAINGU24H.VN

7.

8.

ng k hc: 0962 60 8801 04 6260 3948 236


a ch: S 18 Trn i Ngha Q Hai B Trng H ni
NGOI NG 24H 237
WWW.NGOAINGU24H.VN

9.

10.

ng k hc: 0962 60 8801 04 6260 3948 237


a ch: S 18 Trn i Ngha Q Hai B Trng H ni
NGOI NG 24H 238
Test03.wmv WWW.NGOAINGU24H.VN

TEST 03:
1.

2.

ng k hc: 0962 60 8801 04 6260 3948 238


a ch: S 18 Trn i Ngha Q Hai B Trng H ni
NGOI NG 24H 239
WWW.NGOAINGU24H.VN

3.

4.

ng k hc: 0962 60 8801 04 6260 3948 239


a ch: S 18 Trn i Ngha Q Hai B Trng H ni
NGOI NG 24H 240
WWW.NGOAINGU24H.VN

5.

6.

ng k hc: 0962 60 8801 04 6260 3948 240


a ch: S 18 Trn i Ngha Q Hai B Trng H ni
NGOI NG 24H 241
WWW.NGOAINGU24H.VN

7.

8.

ng k hc: 0962 60 8801 04 6260 3948 241


a ch: S 18 Trn i Ngha Q Hai B Trng H ni
NGOI NG 24H 242
WWW.NGOAINGU24H.VN

9.

10.

ng k hc: 0962 60 8801 04 6260 3948 242


a ch: S 18 Trn i Ngha Q Hai B Trng H ni
NGOI NG 24H 243
WWW.NGOAINGU24H.VN

NGY 14: CU HI WHO/ WHERE/ WHEN/ WHICH

T IM CAO TRONG PART 2:

1. Nm vng cu hi c bn
Part 2 yu cu th sinh chn cu tr li thch hp nht cho cu hi va c nghe. D bn nghe
c ht c 3 la chn nhng khng nghe c cu hi th cng v ch. Vic nghe nhiu cu hi
gn ging nhau c th lm bn d nhm ln. Gio trnh s cung cp cc bi tp gip bn luyn tp
phn ny. Hy nghe cc cu hi thng gp sau y v in vo ch trng bng cc t nghi vn
(question word).

Nghe v in vo ch trng cc t nghi vn. 038.mp3


1. ___________ are you going to Seattle this afternoon?
2. ___________ are you going to Seattle this afternoon?
3. ___________ are you going to Seattle for your trip?
4. ___________ are you going on your vacation this summer?
5. ___________ time are you going to leave for the workshop?
6. ___________ about going to a movie tonight?
7. ___________ are you going to take, a bus or the subway?

Gi :
* be going to + danh t ch ni chn c ngha l i n ni no. be going to + ng t nguyn
mu ngha l d nh lm g.
V d:
He's going to New York. (Anh y ang i ti New York.)
He's going to meet the clients. (Anh y d dnh gp khch hng.)

Xc nh cu tr li ng.
1. How are you going to Seattle this afternoon?
Dng How hi cch i n Seattle (bng phng tin g)
2. Why are you going to Seattle this afternoon?
Dng Why hi l do i n Seattle
3. When are you going to Seattle for your trip?
Dng When hi thi im i n Seattle
4. Where are you going on your vacation this summer?
Dng Where hi ni ngh h
5. What time are you going to leave for the workshop?
Dng What kt hp vi time hi thi gian i n cuc hi tho
6. How about going to a movie tonight?
Dng How kt hp vi about ngh i xem phim
7. Which are you going to take, a bus or the subway?

ng k hc: 0962 60 8801 04 6260 3948 243


a ch: S 18 Trn i Ngha Q Hai B Trng H ni
NGOI NG 24H 244
WWW.NGOAINGU24H.VN

Dng Which kt hp vi a bus or the subway chn la mt trong hai

Vocabulary
trip (n) chuyn i
vacation (n) (chuyn i) ngh mt
leave for ri (ni no ) n ~ workshop (n) hi tho
take a bus bt xe but

2. Nm phng php loi tr p n sai


Trong s 30 cu hi ca Part 2 th khong 10 cu hi c p n ng l cu tr li gin tip.
Ngha l nu cu hi l Where th trong 10 trng hp, p n ng khng phi l cu tr li ch
ni chn.
Q: Where did you get that camera?
A: Oh, Its a gift from my friend, John.
R rng, cu tr li cho cu hi trn khng phi l mt ni chn nh ta thng mong i. Do ,
trc ht cc bn phi hc cch loi tr cc cu tr li hin nhin sai, sau bn s d dng chn
c p n ng hn.

Nghe 2 p n cho mi cu hi di y, nh X vo p n sai, v O vo p n ng.


039.mp3

1. How much sugar do you want for your coffee?


(A) __________ (B) __________

2. Which do you prefer, a window seat or an aisle seat?


(A) __________ (B) __________
Gi :
* Nu nghe m khng phn on c u l p n sai th khng th s dng phng php loi
tr.
* Thng thng trong cc p n ca Part 2, p n no xut hin nhng t nghe trong cu
hi ri th thng l p n sai. Do , khi nghe nu bn ch nh long thong cc m m
khng hiu ngha th khng th chn ng c.

Xc nh cu tr li ng.
1.(A) O (B) X
How much sugar do you want for your coffee? Hi v lng ng cn b vo c
ph
(A) I prefer black. Tr li l thch c ph en (tc l
khng cn ng)
B) Yes, were out of sugar. y l cu hi c t nghi vn nn
khng c tr li bng Yes
2.(A) O (B) X
Which do you prefer, a window seat or an aisle seat? Hi v ch bn thch ngi, gn ca s
hay gn li i
(A) Either is fine with me. Ch no cng c (khng chn 1
ch nh mong i)
(B) The cold food is in aisle 3. Thc n ngui dy th 3

ng k hc: 0962 60 8801 04 6260 3948 244


a ch: S 18 Trn i Ngha Q Hai B Trng H ni
NGOI NG 24H 245
WWW.NGOAINGU24H.VN

Vocabulary
prefer (v) thch hn
be out of: ht
aisle (n) li i
either ci ny hoc ci kia (ci no cng c)
cold food thc n ngui

I. Cu hi Who/ Where
Trng tm ca bi hc
Dng cu hi bt u bng Who /Where xut hin trung bnh 2 ln trong thi mi thng. Tuy th
sinh ch nghe cu hi nhng cng c th d dng tr li c. Who l cu hi v ngi cn Where l
cu hi v ni chn. Tuy nhin khng phi lc no cng nh vy nn khi lm bi, th sinh nn cn
thn pht hin v trnh c cc by c trong thi.

Phn tch cu hi v cc p n la chn 025.mp3


Cu hi Who
Who called you this morning?
One of my clients from Hong Kong.
Who will announce the report tomorrow?
The president will do it
L cu hi v con ngi; thng qua t called
hi ngi gi vo bui sng l ai.
Client c ngha l khch hng.
L cu hi v con ngi, thng qua will announce hi v nhn vt s pht biu l ai.
Ni n mt chc danh c th.

Cu hi Where
Where did you go for your vacation?
I went to Florida this year.
Where will the meeting be held?
In the conference room.
L cu hi v ni chn, thng qua go for/ vacation hi v ni ngh mt.
cp n mt a im c th.
L cu hi v ni chn, thng qua will/ meeting /be held hi a im s t chc hi ngh.
cp n mt a im c th.

Vocabulary
call gi in president ch tch
client khch hng be held c t chc
announce thng bo conference hi ngh
report bi bo co, bi tng thut

Bi tp 1 026.mp3
Nghe cc cu hi v chn ra cu tr li ng.
1. [ A / B ]
2. [ A / B ]
3. [ A / B ]
ng k hc: 0962 60 8801 04 6260 3948 245
a ch: S 18 Trn i Ngha Q Hai B Trng H ni
NGOI NG 24H 246
WWW.NGOAINGU24H.VN

Vocabulary
design thit k be held c t chc
downtown khu trung tm thnh ph sell well bn chy
local (thuc) a phng revised c sa li
architect kin trc s annual hng nm, thng nin
sales meeting hi ngh v bn hng be canceled hy b

Bi tp 2 027.mp3
Nghe cc cu hi v chn ra cu tr li ng.
4. [ A / B ]
5. [ A / B ]
6. [ A / B ]
Vocabulary
be in charge of ph trch cash register my tnh tin
office supplies vn phng phm entrance cng vo
stapler ci dp ghim next to k
pay for tr, thanh ton vice president ph ch tch
suit b com-l present the award trao gii thng

Bi tp 3 028.mp3
Nghe cc cu hi v chn ra cu tr li ng.
7. [ A / B ]
8. [ A / B ]
9. [ A / B ]
Vocabulary
report bn bo co, thng bo leave the company ri khi cng ty, ngh vic
missing mt, tht lc necklace si dy chuyn
be submitted c np gift qu tng
retirement party tic chia tay ngh hu expensive t

Bi tp 4 029.mp3
Nghe nhng cu di y v in vo ch trng.
1. __________________ this report?
2. __________________ did you __________________ this camera?
3. __________________ the meeting?
4. __________________ can I take the __________________ near here?
5. __________________ are you planning to __________________?
6. __________________ me with this proposal?
7. __________________ did you __________________ the news?
8. __________________ the sales department?
9. __________________ public phone?
ng k hc: 0962 60 8801 04 6260 3948 246
a ch: S 18 Trn i Ngha Q Hai B Trng H ni
NGOI NG 24H 247
WWW.NGOAINGU24H.VN

10. __________________ be working on this project?


Vocabulary
report bn bo co proposal d kin, ngh
camera my quay phim, chp nh in charge of ph trch
chair lm ch ta, ch tr

Luyn nghe nhng cu cn thit 030.mp3

Luyn nghe nhng cu hi Who/Where thng gp

Cu hi Who 1. Who is going to be working on the Thomson project?


2. Who does this bag belong to?
3. Who took over when your supervisor was away?
4. Who has the copy of the revised schedule?
5. Who is responsible for the repairs to the copier?

Cu hi Where 6. Where is the nearest train station?


7. Where can I pay for this shirt?
8. Where did you buy that briefcase?
9. Where can we get an ink cartridge for the printer?
10. Where can I find the accounting office?

Bi tp 5 031.mp3

Lng nghe v chn cu tr li ng cho tng cu hi, sau khi nghe li ln 2, hy in vo ch trng.

1. A B C 4. A B C
________________the expense report? __________________________belong to?
(A) It costs more than ______________ .
(A) It is mine.
(B) The trees are __________________ .
(B) No, I didnt ___________________ .
(C) In your ______________________ .
(C) Its not_______________________ .

2. A B C 5. A B C
________________ take her to the ____________________ pay phone?
airport? (A) I think you should ______________ .
(A) The plane leaves_______________ .
(B) Yes, you can __________________ .
(B) Robert can do it.
(C) Theres one ___________________ .
(C) _____________________.

ng k hc: 0962 60 8801 04 6260 3948 247


a ch: S 18 Trn i Ngha Q Hai B Trng H ni
NGOI NG 24H 248
WWW.NGOAINGU24H.VN

3. A B C 6. A B C
________________ a good place to ____________________ about my missing
eat? credit card?
(A) ______________and steak. (A) Tell the man __________ back there.
(B) Try the restaurant ______________ . (B) No, Ill pay _______________.
(C) I ___________________ in the morning. (C) I__________ to mechanic yesterday.

II. Cu hi When / Which


Trng tm ca bi hc
Dng cu hi bt u bng When / Which xut hin trung bnh hn 2-3 cu trong thi mi thng.
Tuy th sinh ch c nghe cu hi v cc p n nhng vic tr li ng cc cu hi l tng i d
dng. When l cu hi v thi gian, cn Which l cu hi la chn. Tuy nhin, th sinh nn cn thn
vi nhng by tr li gin tip v chng khng cp n thi im hoc s la chn mt cch r
rng.

Phn tch cu hi v cc p n la chn 032.mp3

Cu hi When
When will the meeting be held?
After lunch.
When did you last talk to Mr. Kim?
Yesterday at the meeting.
1. L dng cu hi v thi im, thng qua will/ be held hi v mt thi im tng lai.
Khng cp n thi gian c th, ch cho bit l bui chiu qua cm t after lunch.

2. L dng cu hi v thi im, thng qua did/ last talk c th bit c l thi im qu
kh.
cp n thi im c th qu kh.

Cu hi Which
Which hotel should I reserve?
The one close to downtown.
Which way is faster to the station?
The highway is faster.
3. L dng cu hi la chn, thng qua hotel/ reserve hi v vic s ng k khch sn no.
chn the one.

4. L dng cu hi la chn ung no (which way). Thng qua t station c th on l hi v


cch i n nh ga.
C th ni n highway.

Vocabulary
meeting hi ngh last cui cng, gn y
be held c t chc reserve ng k trc

ng k hc: 0962 60 8801 04 6260 3948 248


a ch: S 18 Trn i Ngha Q Hai B Trng H ni
NGOI NG 24H 249
WWW.NGOAINGU24H.VN

faster nhanh hn highway quc l

Bi tp 6 033.mp3
Nghe cc cu hi v chn ra cu tr li ng.
1. [ A / B ]
2. [ A / B ]
3. [ A / B ]

Vocabulary
leave for ri khi ~ n ~ workshop nh marketing job cng vic tip th
xng, hi tho far away xa deadline hn cht
at the latest tr nht market survey kho st th trng
position chc v marketing department phng tip th
apply for np n as I know of nh ti bit

Bi tp 7 034.mp3
Nghe cc cu hi v chn ra cu tr li ng.
4. [ A / B ]
5. [ A / B ]
6. [ A / B ]
Vocabulary
airport sn bay either ci ny hoc ci kia
employee training hun luyn nhn vin arrive n
prefer thch hn for a while mi lc
window seat gh ngi cnh ca s actually thc s
aisle seat gh ngi cnh li i

Bi tp 8 035.mp3
Nghe cc cu hi v chn ra cu tr li ng.
7. [ A / B ]
8. [ A / B ]
9. [ A / B ]

Vocabulary
proposal d kin, ngh wallpaper giy dn tng
due n hn, o hn patterned c hoa vn
be ready sn sng brighter sng hn
approve chp nhn

Bi tp 9 036.mp3
Nghe nhng cu di y v in vo ch trng.

1. ___________________ supposed to start?


2. ___________________ turns off the camcorder?
ng k hc: 0962 60 8801 04 6260 3948 249
a ch: S 18 Trn i Ngha Q Hai B Trng H ni
NGOI NG 24H 250
WWW.NGOAINGU24H.VN

3. ___________________ are you going to ___________________?


4. ___________________ should I ___________________ for the staff meeting?
5. ___________________ today?
6. ___________________, the red one or the blue one?
7. ___________________ you start your ___________________?
8. ___________________ goes to London?
9. ___________________ get to know each other?
10. ___________________ is yours?
1. When is the show 2. Which button 3. When / finish the project 4. When / reserve 5. When are you
leaving 6. Which do you prefer 7. When will / new job 8. Which train 9. When did you 10. Which

Luyn nghe nhng cu cn thit 037.mp3

Luyn nghe nhng cu hi When/Which thng gp

Cu hi When 1. When do you usually get to the office in the morning?


2. When is the seminar scheduled to begin?
3. When does the warranty on the camcorder expire?
4. When do you expect the repairman to arrive?
5. When are you taking your vacation this year?

Cu hi Which 6. Which do you prefer, the black one or the white one?
7. Which hotel should I reserve for the visitors?
8. Which place would you like to go for lunch today?
9. Which number should I press to be connected with the front desk?
10. Which way is quicker to get to your home?

Bi tp 10 038.mp3
Lng nghe v chn cu tr li ng cho tng cu hi, sau khi nghe li ln 2, hy in vo ch trng.

1. A B C 4. A B C
_______ is Sophia ________at the station? ___________________________________
to the train station?
(A) We need one more _____________ . (A) _______________________than a car.
(B) She didnt tell me. (B) Take the route 5.
(C) The train _____________________ . (C) _______, taking train is _____________.

2. A B C 5. A B C
ng k hc: 0962 60 8801 04 6260 3948 250
a ch: S 18 Trn i Ngha Q Hai B Trng H ni
NGOI NG 24H 251
WWW.NGOAINGU24H.VN

________________ would you like, the ____________________ the international


white ones or the blue ones? conference___________?
(A) Can I ________________________? (A) ___________this time.
(B) Those are ____________________ . (B) ____________________________ .
(C) There is ________________________ (C) It will __________________ at least.
across the street.

3. A B C 6. A B C
________________________________ ____________________ are you looking for
of my medical check-up?
(A) ________________________. (A) Ive been seeking ______________ .
(B) The results are_________________ . (B) A convertible ____________________.
(C) Id like to ________________________. (C) I ____________ behind the building.

BI TP KIM TRA
Questions with Who
Example. Listen and choose the best response to the question. 011.mp3
Who will replace Mr. Freeman when he leave?
(A) Mr. Washington will.
(B) Yes, its a beautiful area.
(C) Im not sure where he lives.

Question structures
Structure 1: Who is/are/was/were + noun? 012.mp3
Q: Who is the woman?
A: She is my new secretary.
Possible responses
Her name is Jessica.
She is the manager of the Maintenance Department.
I have no idea./ I am not sure.
Listen and choose the correct response to each of the questions below.
1. Who is the new receptionist?
(A) I have no idea. (B) Its on my desk.
2. Who is the person in charge of payroll?
(A) That is Mrs. Smith. (B) Once a year.
3. Who is the woman sitting next to Mr. Thomas?
(A) She can do everything.
(B) She is the manager of the Maintenance Department.

Structure 2: Who + verb (present tense/past tense) + object? 013.mp3


ng k hc: 0962 60 8801 04 6260 3948 251
a ch: S 18 Trn i Ngha Q Hai B Trng H ni
NGOI NG 24H 252
WWW.NGOAINGU24H.VN

Q: Who has this month's sales report?


A: I have it right here.
Possible responses:
Michael is taking care of it.
Its on my desk
My manager should know it.
I gave it to your secretary.
Listen and choose the correct response to each of the questions below.
1. Who left this file in the meeting room?
(A) You're right. (B) I did.
2. Who handles the complaints about shipping costs?
(A) Michael takes care of them. (B) We only take cash.
3. Who called you early this morning?
(A) It was Mr. Ford. (B) In the kitchen.

Structure 3: Who will + bare infinitive/be + V-ing? 014.mp3


Who is going to + bare infinitive?
Who is supposed to + bare infinitive?
Q: Who will replace Ms. Clinton after she retires?
A: Mr. Chang, our marketing manager.
Possible responses.
Mr. Washington will.
Someone from the Washington office will.
They havent found anyone yet.

Listen and choose the correct response to each of the questions below.
1. Who will Inform the employees of the schedule changes?
(A) Mr. Washington will. (B) At the hospital.
2. Who is going to work on the design project?
(A) Thats fine. (B) We have not decided yet,
3. Who is supposed to pick up this document?
(A) I think I can. (B) Yes, usually.

Practice - Questions with Who (1)


Listen to the questions and responses below and choose the correct response to each question.
Then, listen again and fill the missing words in the gapped questions and responses. 015.mp3
Practice A Practice B
1. Who is the woman? 1. Who _____ the _____?
(A) She is _____ new _____. (A) The reception is _____ _____.
(B) She is _____ with her clients. (B) Mr. Robinson and _____ _____ did.
2. Who organized the reception? 2. Who is supposed to _____ the _____?

ng k hc: 0962 60 8801 04 6260 3948 252


a ch: S 18 Trn i Ngha Q Hai B Trng H ni
NGOI NG 24H 253
WWW.NGOAINGU24H.VN

(A) That would be the _____ _____. (A) Please _____ it _____ me.
(B) The _____ is complicated. (B) My office is _____ the _____ _____.
3. Who is supposed to pay the bill? 3. Who _____ the _____?
(A) The _____ _____ is _____. (A) She is the _____ _____.
(B) The Maintenance Department is _____ (B) She is _____ _____ her _____.
_____ _____ it. 4. Who _____ _____ the annual _____?
4. Who will organize the annual meeting? (A) _____ a _____.
(A) I _____ no _____. (B) The Planning Department is _____ _____ it.
(B) _____ a _____. 5. Who _____ _____ _____ _____ the copy
5. Who has a copy of the contract? machine?
(A) Im _____ _____. . (A) A technician _____ _____ it tomorrow.
(B) I _____ _____ it to your (B) _____ the _____ across the street.
6. Who is going to repair the copy machine? 6. Who _____ a _____ of the contract?
(A) _____ the coffee shop _____the street. (A) I'm _____ now.
(B) _____ Steven _____ the maintenance office. (B) Mr. Smith _____it.

Structure 4: Who is + V-ing + object? 016. mp3


Q: Who is giving the sales presentation?
A: I think Mr. Gomez is.
Possible responses.
I said I would.
I have asked Robert to do it
I think Thompson is scheduled to.
One of my colleagues will.
Listen and choose the correct response to each of the questions below.
1. Who is giving the presentation at todays meeting?
(A) On the radio. (B) I said I would.
2. Who is looking into the problems with the computer?
(A) I asked Robert to do it. (B) About five months.
3. Who is driving you to the airport?
(A) One of my colleagues will. (B) Thats true.

Structure 5: Who is in charge of + noun phrase/gerund phrase? 017.mp3


Who is responsible for + noun phrase/gerund phrase?
Q: Who is in charge of hiring new employees?
A: That's personnel manager's job.
Possible responses:
That would be Mr. Taylor.
Mr. Thompson, the personnel director.
Thats my job.
Listen and choose the correct response to each of the questions below

ng k hc: 0962 60 8801 04 6260 3948 253


a ch: S 18 Trn i Ngha Q Hai B Trng H ni
NGOI NG 24H 254
WWW.NGOAINGU24H.VN

1. Who is in charge of making hiring decisions?


(A) Its his report. (B) Mr. Thompson, the personnel director.
2. Who is responsible for setting up the tables for the meeting?
(A) At a local hotel. (B) That would be Mr. Taylor.
3. Who is responsible for purchasing office supplies?
(A) Thats my job. (B) Thats a good idea.

Structure 6: Who should/can + subject + bare infinitives? 018.mp3


Who did/do/does + subject + bare infinitives?
Q: Who should I submit this report to?
[= To whom should I submit this report?]
A: Ask the secretary.
Possible responses:
The secretary will let you know.
You can give it to Mr. Johnson in the Planning Department.
I am not sure.
Listen and choose the correct response to each of the questions below.
1. Who should I ask for the annual sales report?
(A) Im not sure. (B) At about 9 a.m.
2. Who did you give the document to?
(A) To Mr. Kim in the Sales Department (B) I dont know him at all.
3. Who does this laptop belong to?
(A) Yes, its very interesting. (B) Mark in the R&D Department, I think.

Practice Questions with Who (2) 019.mp3


Listen to the questions and responses below and the correct response to each question.
Then, listen again and fill the missing words in the gapped questions and responses.
Practice A Practice B
1. Who is driving you to the train station? 1. Who is in _____ of the _____ Department?
(A) It _____ _____ 10 minutes. (A) _____ _____ be Ms. Elliot.
(B) I _____ a taxi. (B) I ____ so.
2. Who should I contact to confirm the 2. Who does this handbag ____ _____?
reservation? It's Janes.
(A) _____ the receptionist. (A) Its _____ Janes.
(B) I _____ a taxi. (B) Its very _____.
3. Who does this handbag belong to? 3. Who should I _____ _____ confirm the
(A) It would _____ _____. reservation?
(B) I am not _____. (A) I reserved a _____ _____.
4. Who is in charge of the Planning (B) _____ this _____.
Department? 4. Who is _____ _____ the _____ tonight?
(A) Mr. Kent is _____ that. (A) The _____ _____ in the Marketing
(B) I _____ so. Department.
ng k hc: 0962 60 8801 04 6260 3948 254
a ch: S 18 Trn i Ngha Q Hai B Trng H ni
NGOI NG 24H 255
WWW.NGOAINGU24H.VN

5. Who is responsible for hiring salespeople? (B) The _____ _____ at 10.
(A) You _____ _____ _____ any higher. 5. Who is _____ you to the _____ _____?
(B) Mr. Corner usually _____ _____ _____ that. (A) It will _____ _____ _____.
6. Who is coming to the party tonight? (B) Johnson will _____ with _____ .
(A) Some _____ _____ from my college. 6. Who is ____ for _____ salespeople?
(B) It is a _____ party. (A) You _____ _____ be there.
(B) _____ would be the personnel _____.

Practice the possible questions. 020.mp3


Listen and choose the correct response to each of the questions.
1. (A) (B)
2. (A) (B)
3. (A) (B)
4. (A) (B)
5. (A) (B)
6. (A) (B)
7. (A) (B)
8. (A) (B)
9. (A) (B)
10. (A) (B)

Practice with TOEIC Actual Questions. 021.mp3


Listen and choose the correct response to each of the questions.
1. (A) (B) (C)
2. (A) (B) (C)
3. (A) (B) (C)
4. (A) (B) (C)
5. (A) (B) (C)
6. (A) (B) (C)
7. (A) (B) (C)
8. (A) (B) (C)
9. (A) (B) (C)
10. (A) (B) (C)
11. (A) (B) (C)
12. (A) (B) (C)
13. (A) (B) (C)
14. (A) (B) (C)
15. (A) (B) (C)

Questions with Where


Example: Listen and choose the best response to the question. 022.mp3
Where is your new office building?
(A) At the end of King Street.
ng k hc: 0962 60 8801 04 6260 3948 255
a ch: S 18 Trn i Ngha Q Hai B Trng H ni
NGOI NG 24H 256
WWW.NGOAINGU24H.VN

(B) Yes, thats my job.


(C) I worked for five years.

Question Structures:
Structure 1: Where is/are + noun/noun phrase? 023.mp3
Where is the nearest/closest + noun/noun phrase?
Q: Where is your company's head office?
A: It's in New York.
Possible responses:
In Singapore.
You can find it on the main page of our website.
The head office was recently moved to Seoul.
Listen and choose the correct response to each of the questions below.
1. Where is our new branch office?
(A) In Singapore. (B) For several hours.
2. Where is the nearest bookstore?
(A) Thanks for letting me know. (B) On the next corner.

Structure 2: Where is/are + subject + past participle? 024.mp3


Q: Where is your company based?
A: It is based in Tokyo.
Possible responses:
We have our main office in Seoul.
We are based in Manhattan.
In San Francisco.
Listen and choose the correct response to each of the questions below.
1. Where is your new office located?
(A) We have our main office in Seoul. (B) Ill do it tomorrow.
2. Where is your head office located?
(A) We are based In Manhattan. (B) Early Saturday evening.
3.Where was the sales conference held last month?
(A) With my friend. (B) In San Francisco.

Structure 3: Where can I + bare infinitive + object? 025.mp3


Q: Where can I get a copy of this newsletter?
A: I don't think there are any left.
Possible responses:
You can get one at the reception hall,
I will get one for you.
Why dont you go to the PR office?
Listen and choose the correct response to each of the questions below.
ng k hc: 0962 60 8801 04 6260 3948 256
a ch: S 18 Trn i Ngha Q Hai B Trng H ni
NGOI NG 24H 257
WWW.NGOAINGU24H.VN

1. Where can I get a pamphlet?


(A) I will get one for you. (B) I just moved here yesterday.
2. Where can I buy an umbrella?
(A) Its ten dollars. (B) Try the shop next to the bank.
3. Where can I find the business magazines?
(A) They are in the magazine section. (B) Its possible, I suppose.

Practice - Questions with Where (1) 026.mp3


Listen to the questions and responses below and choose the correct response to each question.
Then, listen again and fill the missing words in the gapped questions and responses.
Practice A Practice B
1. Where is the copy machine? 1. Where are the documents _____?
(A) On the _____ _____. (A) The store is _____ _____ _____
(B) I need _____ _____. (B) _____ Sharon.
2. Where are the documents stored? 2. Where is your _____ _____ _____?
(A) The store is _____ on _____. (A) _____, _____ _____ in L.A.
(B) _____ the drawer of my _____. (B) _____ to the convention _____.
3. Where can I submit my application? 3. Where is the _____ _____?
(A) You can ______ it _____. (A) _____ _____ Michaels office.
(B) I _____ it. (B) I need ____ copies.
4. Where is the closest supermarket around 4. Where can I ____ my ____?
here? (A) Please go to the _____ _____.
(A) I will _____ some _____. (B) He _____ it.
(B) Next to the _____ _____. 5. Where is the _____ _____ _____?
5. Where is the guest list for Friday's reception? (A) I will _____ _____ cheese.
(A) Mr. Smith _____ knows. (B) Just _____ the _____.
(B) It will be _____ _____ the banquet hall. 6. Where is the _____ _____ for Fridays
6. Where is your new office located? _____?
(A) Yes, his office is _____ New York. (A) The _____ _____ has it.
(B) In the _____ _____. (B) It will _____ _____ at the Hilton Hotel.

Structure 4: Where should I + bare infinitive + object? 027.mp3


Q: Where should I put this plant?
A: On Mr. Johnson's desk.
Possible responses:
In the back room.
On the shelf.
Over there in the corner.

Listen and choose the correct response to each of the questions below.
1. Where should I put these catalogs?
(A) At nine tomorrow. (B) On the shelf.
2. Where should I store these boxes of copy paper?
ng k hc: 0962 60 8801 04 6260 3948 257
a ch: S 18 Trn i Ngha Q Hai B Trng H ni
NGOI NG 24H 258
WWW.NGOAINGU24H.VN

(A) Thanks a lot. (B) In the back room.


3. Where should I park the car?
(A) Over there. (B) The park is wonderful.

Structure 5: Where did/do/does + subject + verb? 028.mp3


Q: Where did you work before you got this job?
A: At a small design company.
Possible responses:
I worked at an export firm.
I helped with my fathers business.
I studied at a university.
Listen and choose the correct response to each of the questions below.
1. Where did Kate go on her holiday?
(A) With her family. (B) To Singapore.
2, Where do you keep the office supplies?
(A) In the cabinet. (B) At noon.
3, Where did Mr. Gibson leave the report?
(A) He left it on his desk. (B) He will leave for Sydney.

Structure 6: Where is/are + subject + going to + bare infinitive/be + past participle?


Where is/are + subject + V-ing? 029.mp3
Where will + subject + bare infinitive/be + V-ing/be + past participle?
Q: Where is the next convention going to be held?
A: At the Plaza Hotel.
Possible responses:
It will be in Las Vegas.
It has not been decided yet.
Listen and choose the correct response to each of the questions below.
1. Where is the seminar going to be held next year?
(A) Yes, it will. (B) It will be in Las Vegas.
2. Where will they move the office?
(A) It has not been decided yet. (B) That is surprising.
3. Where is your department having the welcome party?
(A) At the Plaza Hotel. (B) At about 5 o'clock.

Practice - Questions with Where (2) 030.mp3


Listen to the questions and responses below and choose the correct response to each question.
Then, listen again and fill the missing words in the gapped questions and responses.
Practice A Practice B
1, Where did you hear the news? 1. Where will the _____ _____ be _____?
(A) A _____ of mine _____ me. (A) To show _____ _____ _____.

ng k hc: 0962 60 8801 04 6260 3948 258


a ch: S 18 Trn i Ngha Q Hai B Trng H ni
NGOI NG 24H 259
WWW.NGOAINGU24H.VN

(B) He will be here _____ _____ _____. (B) At the _____ _____.
2. Where should I put my suitcase? 2. Where are they going to _____ a new factory?
(A) The handbag suits _____ _____. (A) Yes, they _____ in the ____.
(B) In the closet in the _____. (B) Several options are being _____.
3. Where are you traveling next summer? 3. Where did you ____ the _____?
(A) I haven't _____ _____. (A) Everyone _____ it.
(B) _____ _____ the Bahamas! (B) He will be _____ ____.
4. Where will the trade show be held? 4. Where should I _____ my _____?
(A) _____ _____ some new clothing lines. (A) The _____ _____ you well.
(B) _____ Chicago. (B) You can _____ _____ _____.
5. Where are they going to build a new factory? 5.Where did Samantha _____ the _____ _____?
(A) On the outskirts _____ _____. (A) She left _____ _____ ago.
(B) Is _____ the _____? (B) I have no _____.
6. Where did Samantha leave the application 6. Where are you _____ next summer?
form? (A) Hawaii.
(A) She left an _____ _____. (B) ______ ______ Hong Kong
(B) In the _____.

Practice the possible questions. 031.mp3


Listen and choose the correct response to each of the questions.
1. (A) (B)
2. (A) (B)
3. (A) (B)
4. (A) (B)
5. (A) (B)
6. (A) (B)
7. (A) (B)
8. (A) (B)
9. (A) (B)
10. (A) (B)

Practice with TOEIC Actual Questions. 032.mp3


Listen and choose the correct response to each of the questions.
1. (A) (B) (C)
2. (A) (B) (C)
3. (A) (B) (C)
4. (A) (B) (C)
5. (A) (B) (C)
6. (A) (B) (C)
7. (A) (B) (C)
8. (A) (B) (C)
9. (A) (B) (C)
10. (A) (B) (C)
ng k hc: 0962 60 8801 04 6260 3948 259
a ch: S 18 Trn i Ngha Q Hai B Trng H ni
NGOI NG 24H 260
WWW.NGOAINGU24H.VN

11. (A) (B) (C)


12. (A) (B) (C)
13. (A) (B) (C)
14. (A) (B) (C)
15. (A) (B) (C)

Questions with When 033.mp3


Listen and choose the best response to the question.
When is the workshop supposed to end?
(A) At 3 o clock
(B) It does not work.
(C) At the end of the corridor.

Question structures:
Structure 1: When is + Subject + .? 034.mp3
Q: When is the train to London?
A: In thirty minutes.
Possible responses:
At 5 oclock sharp.
Not until 7 o'clock tomorrow morning.
Please ask the conductor.
Why dont you go to the information center?
Listen and choose the correct response to each of the questions below.
1. When is the dinner appointment with Mr. Tanaka?
(A) At 5 oclock.
(B) By telephone.
2. When is the next available flight to Hawaii?
(A) I want to take the train.
(B) Not until 7 o'clock tomorrow morning.

Structure 2: When is/are + subject + due? 035.mp3


When is the deadline for + noun/noun phrase?
Q: When is the project due?
A: Next Monday.
Possible responses:
By the end of this month.
We have a few more days.
You should finish it before five oclock today.
Listen and choose the correct response to each of the questions below.
1. When are the assignments due?
(A) In the mailboxes. (B) You should finish it before five oclock today.

ng k hc: 0962 60 8801 04 6260 3948 260


a ch: S 18 Trn i Ngha Q Hai B Trng H ni
NGOI NG 24H 261
WWW.NGOAINGU24H.VN

2. When is this phone bill due?


(A) You can use my phone. (B) By the end of this month.
3. When is the deadline for registration?
(A) We have a few more days. (B) We have the plans.

Structure 3: When did + subject + bare infinitive? 036.mp3


Q: When did you join the Marketing Department?
A: About three years ago.
Possible responses:
I believe it was about two years ago.
Last month.
Listen and choose the correct response to each of the questions below.
1. When did you make the decision?
(A) We decided last week. (B) Next month.
2. When did you arrive from Tokyo?
(A) Three days ago. (B) From Platform 1.

Structure 4: When do /does + subject + bare infinitive? 037.mp3


Q: When does the train leave for Boston?
A: In ten minutes.
Possible responses:
At 2 o'clock.
It is being delayed.

Listen and choose the correct response to each ot the questions below.
1. When do you need the report by?
(A) Next Friday. (B) In the office.
When does the concert begin?
2. (A) In New York. (B) At 2 o'clock.

Practice - Questions with When (1)


Listen to the questions and responses below and choose the correct response to each question.
Then, listen again and fill the missing words in the gapped questions and responses. 038.mp3
Practice A Practice B
1. When is the next bus to Manhattan? 1. When did you _____ _____ this morning?
(A) Its _____ _____. (A) I have not _____ yet.
(B) ______ _____. (B) Very _____.
2. When is the ____ for the gas bill payment?
2. When is your appointment with the dentist?
(A) In _____.
(A) This _____. (B) Next _____.
(B) In the _____ _____ town. 3. When do you ____ into your new _____?
3. When does your passport expire? (A) No ____ _____ October 10.
(A) _____ _____ from now. (B) Its on the ______ _______.
4. When _____ your _____ with the dentist?
ng k hc: 0962 60 8801 04 6260 3948 261
a ch: S 18 Trn i Ngha Q Hai B Trng H ni
NGOI NG 24H 262
WWW.NGOAINGU24H.VN

(B) First of all, you should _____the _____. (A) At the _____.
4. When did you wake up this morning? (B) I have to be ____ by _____.
(A) _____ _____ oclock. 5. When _____ your passport _____?
(A) Next _____.
(B) I have not _____ it yet.
(B) I lost my ____.
5. When do you move into your new office? 6. When is the ____ ____ to Manhattan?
(A) _____ two ____. (A) Not until ____ _____.
(B) Its _____ the _____ floor. (B) Its my _____.
6. When is the deadline for the gas bill
payment?
(A) By _____ _____.
(B) On _____ 15.

Structure 5: When will + subject + bare infinitive/be + V-ing? 039.mp3


When will + subject + be + past participle?
When is/are + subject + going to + bare infinitive/be + past participle?
When is/are + subject + V-ing /being + past participle?
Q: When will the meeting take place?
A: On Monday afternoon.
Possible responses:
At 10 o'clock.
The day after tomorrow.
It is going to be held after lunch.
The committee has not made a decision yet.
Listen and choose the correct response to each of the questions below.
1. When are you moving into the new building?
(A) On the second floor. (B) Next week.
2. When are you going to release our new products?
(A) Just 10 miles from here. (B) As soon as possible.
3. When will the order be delivered?
(A) In two weeks. (B) Not at all.

Structure 6: When is /are + subject + supposed to + bare infinitive? 040.mp3


When is/are + subject + scheduled to + bare infinitive?
Q: When is the workshop supposed to end?
A: At three o'clock.
Possible responses:
On Friday.
Next Tuesday.
Before noon, I hope.
At five o'clock, I think.
Listen and choose the correct response to each of the questions below.

ng k hc: 0962 60 8801 04 6260 3948 262


a ch: S 18 Trn i Ngha Q Hai B Trng H ni
NGOI NG 24H 263
WWW.NGOAINGU24H.VN

1. When is he supposed to start work?


(A) He was busy. (B) Next Tuesday.
2. When are we supposed to hear from them?
(A) At five o'clock, I think. (B) No problem
3. When are the renovations scheduled to begin?
(A) Four days a week. (B) On Friday

Structure 7: When can/should I + bare infinitive? 041.mp3


When do we have to + bare infinitive?
Q: When can I expect to get the results of the test?
A: In a couple of days.
Possible responses:
It will not take very long.
You will get the results by next week.
'Can you be here tomorrow?
Listen and choose the correct response to each of the questions below.
1. When should I pick up my car?
(A) Can you be here tomorrow? (B) At the repair store.
2. When can I get my money back?
(A) It will not take very long. (B) Yes, thats fine.
3. When do we have to submit the application forms?
(A) To the manager of the Personnel Department.
(B) By the end of the month.

Practice - Questions with When (2) 042.mp3


Listen to the questions and responses below and choose the correct response to each question. Then,
listen again and till the missing words in the gapped questions and responses
Practice A Practice B
1. When is the concert supposed to begin? 1. When are you _____ your _____?
(A) It is written on the _____. (A) At ______ tomorrow.
(B) At the _____ _____. (B) I will _____ the _____.
2. When are you taking your driving test? 2. When are you ____ _____ _____ the _____?
(A) I failed _____ _____. (A) Early _____ _____.
(B) _____ _____ from now. (B) _____ weeks _____.
3. When will your new book be published? 3. When can I ____ my ____ to _____?
(A) _____ next month, I ______. (A) Tomorrow ____, I think.
(B) That sounds _____. (B) I ____ a new ____.
4. When are you going to start the project? 4. When is the construction ____ to be _____?
(A) _____ _____, I will. (A) Of course ____.
(B) _____ _____ this work. (B) At the ____ _____ next month.
5. When is the construction scheduled to be 5. When is the concert _____ ____ _____?
finished? (A) At the _____.
ng k hc: 0962 60 8801 04 6260 3948 263
a ch: S 18 Trn i Ngha Q Hai B Trng H ni
NGOI NG 24H 264
WWW.NGOAINGU24H.VN

(A) I'm _____ _____ it. (B) In ___.


(B) Not for _____ _____ weeks. 6. When will your new book ____?
6. When can I expect my order to arrive? (A) ____ next week.
(A) I _____ a new printer. (B) At the ____ of the _____
(B) It will take _____ _____ days.

Practice the possible questions. 043.mp3


Listen and choose the correct response to each of the questions.
1. (A) (B)
2. (A) (B)
3. (A) (B)
4. (A) (B)
5. (A) (B)
6. (A) (B)
7. (A) (B)
8. (A) (B)
9. (A) (B)
10. (A) (B)

Practice with TOEIC Actual Questions. 044.mp3


Listen and choose the correct response to each of the questions.
1. (A) (B) (C)
2. (A) (B) (C)
3. (A) (B) (C)
4. (A) (B) (C)
5. (A) (B) (C)
6. (A) (B) (C)
7. (A) (B) (C)
8. (A) (B) (C)
9. (A) (B) (C)
10. (A) (B) (C)
11. (A) (B) (C)
12. (A) (B) (C)
13. (A) (B) (C)
14. (A) (B) (C)
15. (A) (B) (C)

ng k hc: 0962 60 8801 04 6260 3948 264


a ch: S 18 Trn i Ngha Q Hai B Trng H ni
NGOI NG 24H 265
WWW.NGOAINGU24H.VN

NGY 15: CU HI WHY/ HOW/ WHAT

Trng tm ca bi hc
Ln lut hc k nhng cu hi d b t by. Khi gp dng cu hi bt u bng Why/ How/ What
cn phi ch nghe k cc t theo sau t nghi vn nm bt c ngha ca cu hi. Do cch
tt nht l hc chi tit tng loi cu hi.
Phn tch cu hi v cc p n la chn 039.mp3

Cu hi Why
Why were you late this morning?
There was a road construction.
Why dont you take a few days off?
No, I have too many things to do.
1. L cu hi v l o, thng qua t late hi l do ti sao i tr.
cp c th l do i tr l v c mt con ng ang thi cng.

2. L cu trc Why dont you...?, thng qua cm t days off ngi hi a ra ngh xin
ngh vi ngy.
t chi ngh bng t No v trnh by l do many things to do.

Cu hi How
How do you get to work?
I drive by myself.
How many workers did you hire?
Less than 10 people.
How long will it take to complete this project?
Two more months from now.
3. L cu hi v cch thc, thng qua t get to work hi ngi nghe i lm bng cch no.
trnh by cch thc c th l i lm bng xe hi qua t drive.

4. L cu trc How many...? hi v s lng, thng qua workers /hire hi v s nhn vin
c thu.
Ni v s nhn vin c th l 10 ngi.

5. L cu trc How long...? hi v khong thi gian, thng qua t complete hi v thi gian
hon thnh d n.
Ni n thi gian c th l mt hn 2 thng.

Cu hi What
What should I do to turn on the copier?
Press the green button.
What time does the flight depart?
At 3 P.M.
ng k hc: 0962 60 8801 04 6260 3948 265
a ch: S 18 Trn i Ngha Q Hai B Trng H ni
NGOI NG 24H 266
WWW.NGOAINGU24H.VN

What do you think of the new computer?


Its much faster than the old one.
6. L dng cu hi v cch thc m my photocopy.
Cu tr li miu t ng tc c th nhn nt xanh.
7. Cu trc What time hi v thi gian, thng qua t depart hi thi gian my bay ct cnh.
cp c th thi gian ct cnh.
8. Cu trc What do you think of...? hi v kin, thng qua t computer hi nhn xt ca
ngi nghe v my vi tnh mi.
So snh vi my vi tnh trc y th hin kin ca mnh.

Vocabulary
road construction vic sa cha, tu b ng complete hon thnh
s copier my photocopy
take a few days off ngh vi ngy press nhn
get to work i lm depart khi hnh, ct cnh
hire thu, mn

Ghi ch
Cn phn bit chc nng ca cc t nghi vn Why/How/What trong cu hi c th d dng tm ra
cu tr li nh sau:
Why hi l do p n thng l cu tr li trnh by l do.
Why nu li ngh p n thng l cu tr li ng hay t chi li ngh.
How hi v cch thc p n thng l li trnh by hng dn c th.
How hi v mc How many cn cu tr li v s lng,
How often cn cu tr li ch s thng xuyn,
How long cn cu tr li ch khong thi gian.
What hi v cch thc p n thng l cu tr li trnh by ni dung hng dn hay tng hng
mc c th.
What hi kin p n thng l cu tr li trnh by kin c nhn.

Cn ghi nh rng dng cu nghi vn bt u bng Why/How thng khng tr li bng Yes/No.
Tuy nhn trong cc trng hp Why dont you ...?, How about...? mang ngh c th c tr
li bng Yes/ No.

Bi tp 1: 040.mp3
Nghe cc cu hi v chn ra cu tr li ng.
1. [ A / B ]
2. [ A / B ]
3. [ A / B ]

Vocabulary
page trang (giy) avenue i l
drive to work li xe i lm since k t

Bi tp 2 041.mp3
Nghe cc cu hi v chn ra cu tr li ng.
4. [ A / B ]

ng k hc: 0962 60 8801 04 6260 3948 266


a ch: S 18 Trn i Ngha Q Hai B Trng H ni
NGOI NG 24H 267
WWW.NGOAINGU24H.VN

5. [ A / B ]
6. [ A / B ]
Vocabulary
join tham gia open m (ca)
often thng key cha kha
go on business trips i cng tc security guard nhn vin bo v
mainly ch yu holiday ngy l

Bi tp 3 042.mp3
Nghe cc cu hi v chn ra cu tr li ng.
7. [ A / B ]
8. [ A / B ]
9. [ A / B ]

Vocabulary
department phng, ban open m ca
head trng phng be late for work i lm tr
by taxi bng taxi be delayed b tr hon
late tr due to do (= because of, owing to)

Bi tp 4 043.mp3
Nghe nhng cu di y v in vo ch trng.
1. __________________ to have for lunch?
2. __________________ have you worked here?
3. __________________ join us for coffee?
4. __________________ about the damaged shipment?
5. __________________ learn about the bid?
6. __________________ do you need?
7. __________________ the new manager?
8. __________________ the copier working?
9. __________________ is the presentation?
10. __________________ send out the memo right now?

Luyn nghe nhng cu cn thit 044.mp3

Luyn nghe nhng cu hi Why / How / What thng gp

Cu hi Why 1. Why is the construction being delayed?


2. Why dont you come to the beach with us?
3. Why is the road closed today?

ng k hc: 0962 60 8801 04 6260 3948 267


a ch: S 18 Trn i Ngha Q Hai B Trng H ni
NGOI NG 24H 268
WWW.NGOAINGU24H.VN

4. How did you leam about this position at Samsung?


5. How big is that apartment?
Cu hi How 6. How much does it cost to repair the equipment?
7. How many workers do you have in your factory?

Cu hi What 8. What time can we have a break?


9. What date is the deadline for the proposal?
10. What type of accommodation would you like?

Bi tp 5 045.mp3
Lng nghe v chn cu tr li ng cho tng cu hi, sau khi nghe li ln 2, hy in vo ch trng.

1. A B C 4. A B C
________________________________ ________________________ get to the
from here? ______________ around here?
(A) No, ____________________ today. (A) 20 dollars,please.
(B) _______________________ away. (B) Ill send it by ___________________.
(C) I______________________ by bus. (C) Just cross the street and its
_______________________.

2. A B C 5. A B C
________________________________ ___________________ closed today?
to get more office supplies? (A) There will be a parade ___________ .
(A) We are ________________________.
(B) It is ______________________my
(B) You should _______________ first.
neighborhood.
(C) Yes, my office has a _____________. (C) No, Ill ______________ not the bus.

3. A B C 6. A B C
________________________________ ___________________ have you been
at the lobby? to Sydney?
(A) ________________________. (A) ___________________ altogether.
(B) We should ___________________ . (B) ________________ to get there.
(C) Because ____________________. (C) This is my ____________________ .

ng k hc: 0962 60 8801 04 6260 3948 268


a ch: S 18 Trn i Ngha Q Hai B Trng H ni
NGOI NG 24H 269
WWW.NGOAINGU24H.VN

BI TP KIM TRA
Questions with Why
Example. Listen and choose the best response to the question. 045.mp3
Why is the bank closed today?
(A) It is just two blocks away.
(B) It is a holiday.
(C) Right across the street.

Question Structures:
Structure 1: Why is/are/was/were + subject + adjective/adverb/adverbial phrase? 046.mp3
Why is/are/was/were + subject + past participle?
Why isn't/aren't/wasn't/weren't + subject + past participle?
Q: Why is the library closed?
A: It's a national holiday.
Its operating hours ended.
It is being repaired.
There was a fire last night.
Listen and choose the correct response to each of the questions below.
1. Why was your flight delayed?
(A) Because of the bad weather. (B) It was too late.
2. Why are you here?
(A) Here you are. (B) For the job interview.

Structure 2: Why is/are/isn't/arent + subject + V-ing? 047.mp3


Q: Why are so many people standing on the street?
A: There is a street concert tonight.
Possible responses
There is a parade on the street.
A festival is taking place.
They are watching a street performance.
Some street musicians are having a concert.
Listen and choose the correct response to each of the questions below.
1. Why is he working so late tonight?
(A) He should finish the report. (B) He is always late.
2. Why isnt the elevator working?
(A) Its downstairs. (B) It Is out of order.
3. Why are you walking so fast?
(A) I will work faster. (B) I have to catch the last train.

Structure 3: Why do/does + subject + need/have/want to + bare infinitive. 048.mp3


Q: Why do you need to reschedule your appointment?

ng k hc: 0962 60 8801 04 6260 3948 269


a ch: S 18 Trn i Ngha Q Hai B Trng H ni
NGOI NG 24H 270
WWW.NGOAINGU24H.VN

A: Something came up unexpectedly.


Possible responses:
My client asked me to do that.
One of my team members had an accident.
My flight has been canceled because of the bad weather.
Listen and choose the correct response to each of the questions below.
1. Why do you want to cancel the appointment?
(A) My schedule has been changed. (B) She was appointed to the position.
2. Why do they have to depart so early in the morning?
(A) They get up early. (B) To avoid traffic jams.
3. Why do we need to send them another invoice?
(A) Its on my desk. (B) They lost the first one.

Practice - Questions with Why (1) 049.mp3


Listen to the questions and responses below and choose the correct response to each question.
Then, listen again and fill the missing words in the gapped questions and responses.
Practice A Practice B
1. Why does Rachel want to move to Boston? 1. Why is the ______ _____ so _____ today?
(A) She got a _____ _____ there. (A) You can _____ this _____ _____.
(B) Because she _____ _____ transportation. (B) Most people _____ _____.
2. Why are you in such a hurry? 2. Why is the _____ _____ its _____?
(A) I will _____ _____ _____ me. (A) My office is _____ the _____ floor.
(B) To _____ the first train. (B) _____ a new building.
3. Why do I have to submit this form again? 3. Why does Rachel _____ _____ _____ to
(A) We never received the _____ _____. Boston?
(B) To the _____ Department. (A) She wants to _____ _____ her family.
4. Why was the meeting cancelled? (B) Because she _____ traveling.
(A) The meeting _____ at 10:30. 4. Why are you _____ _____ a _____?
(B) Our manager had a _____ _____. (A) I don't want to be _____ _____.
5. Why is the parking lot so empty today? (B) I will _____ _____ you.
(A) You _____ _____ _____ your car here. 5. Why do I have to _____ this _____ _____?
(B) Today is a _____. (A) Some information is _____.
6. Why is the company moving its office? (B) To the _____ _____.
(A) My office is on the _____ _____. 6. Why was the _____ _____?
(B) The current office building is _____ _____. (A) The meeting _____ at noon.
(B) The president's flight has been _____.

Structure 4: Why dont you/we + bare infinitive. 050.mp3


Q: Why dont you call customer service?
A: That is probably a good idea.
Possible responses:
Thats a good idea.
ng k hc: 0962 60 8801 04 6260 3948 270
a ch: S 18 Trn i Ngha Q Hai B Trng H ni
NGOI NG 24H 271
WWW.NGOAINGU24H.VN

I think I should.
Actually, I just did that.
Listen and choose the correct response to each of the questions below.
1. Why dont you take some time off?
(A) I think I should. (B) The plane will take off in a minute.
2. Why dont you ask Paul to come to your party?
(A) No, he is not busy. (B) Actually, I just did.
3. Why dont we share a taxi to the airport?
(A) Thats a good idea. (B) Because I called a taxi.

Structure 5: Why did/didnt + subject + bare infinitive? 051.mp3


Q: Why did Michael leave early?
A: To avoid heavy traffic.
Possible responses:
He had an appointment.
He had to take the first train.
He received a phone call from his client.
Listen and choose the correct response to each of the questions below.
1. Why did you decide to install a new system?
(A) It is much better. (B) I think I will.
2. Why didnt you get to work on time?
(A) It does not work. (B) My train was delayed.
3. Why did Michael quit his job?
(A) He works as a doctor. (B) He got another job.
Structure 6: Why have/has/havent/hasnt + subject + past participle? 052.mp3
Q: Why have you been away so long?
A: The meeting lasted all day.
Possible responses:
We got stuck in heavy traffic.
I have been in the warehouse checking on the inventory.
I have been at the airport picking up Mr. Masao from Tokyo.
Listen and choose the correct response to each of the questions below.
1. Why has Taylor requested a leave of absence?
(A) He will leave soon. (B) His mother is in the hospital.
2. Why have you stopped producing the product?
(A) We found a detect. (B) Yes, they are good products.
3. Why hasn't Rachel come to the office yet today?
(A) She is on vacation. (B) She works very hard

Practice - Questions with Why (2) 053.mp3


Listen to the questions and responses below and choose the correct response to each question.
ng k hc: 0962 60 8801 04 6260 3948 271
a ch: S 18 Trn i Ngha Q Hai B Trng H ni
NGOI NG 24H 272
WWW.NGOAINGU24H.VN

Then, listen again and fill the missing words in the gapped questions and responses.
Practice A Practice B
1. Why dont you get some rest? 1. Why dont you _____ us for _____ tonight?
(A) Let me _____ this _____. (A) Thanks, but I have _____ _____.
(B) I will _____ a _____. (B) _____ is ready.
2. Why didn't John get the promotion? 2. Why did Mr. Thompson _____ the _____?
(A) To promote a _____ _____. (A) He _____ a _____ for you.
(B) He _____ qualified. (B) He started ______ ______ business.
3. Why did Mr. Thompson leave the company? 3. Why did Linda _____ the letter?
(A) He _____ his office this _____. (A) To _____ somebody about the _____ in the
(B) It was due to his ______ _____. schedule.
4. Why did Linda send the letter? (B) At the ______.
(A) She _____ us to her _____. 4. Why did you stay up _____ _____?
(B) At the _____. (A) I stayed _____ for two _____.
5. Why dont you join us for dinner tonight? (B) I had to _____ my _____.
(A) I'd _____ to. 5. Why dont you _____ _____ _____?
(B) _____ is waiting for you. (A) I will _____ the _____.
6. Why did you stay up so late? (B) The deadline is just _____ _____ _____.
(A) I had _____ _____ to do. 6. _____ _____ John _____ the promotion?
(B) I _____ _____ _____ very long. (A) _____ advertise a new _____.
(B) I _____ _____ _____.

Practice the possible questions. 054.mp3


Listen and choose the correct response to each of the questions.
1. (A) (B)
2. (A) (B)
3. (A) (B)
4. (A) (B)
5. (A) (B)
6. (A) (B)
7. (A) (B)
8. (A) (B)
9. (A) (B)
10. (A) (B)
Practice with TOEIC Actual Questions. 055.mp3
Listen and choose the correct response to each of the questions.
1. (A) (B) (C)
2. (A) (B) (C)
3. (A) (B) (C)
4. (A) (B) (C)
5. (A) (B) (C)
6. (A) (B) (C)
7. (A) (B) (C)
ng k hc: 0962 60 8801 04 6260 3948 272
a ch: S 18 Trn i Ngha Q Hai B Trng H ni
NGOI NG 24H 273
WWW.NGOAINGU24H.VN

8. (A) (B) (C)


9. (A) (B) (C)
10. (A) (B) (C)
11. (A) (B) (C)
12. (A) (B) (C)
13. (A) (B) (C)
14. (A) (B) (C)
15. (A) (B) (C)

Questions with What


Example:
Listen and choose the best response to the question 056.mp3
What time did Mr. Jackson leave the office?
(A) An hour ago.
(B) He doesnt live there anymore.
(C) Yes, he did.

Question Structures:
Structure 1: What is/are/was/were + noun/noun phrase 057.mp3
Q: What is the purpose of your visit to Tokyo?
A: Im here for a business meeting.
Possible responses:
I'm here for a conference.
Im going to see my parent
Listen and choose the correct response to each of the questions below.
1. What is your plan for summer vacation?
(A) I'm going to see my parents. (B) By airplane.
2. What is the price of this item?
(A) Later this week. (B) Its 10 dollars.

Structure 2: What time is + noun /noun phrase? 058.mp3


What time is/are/was/were + subject + V-ing/past participle?
What time does/do/did + subject + bare infinitive?
What time will/can/should + subject + bare infinitive?
Q: What time are you leaving tomorrow?
A: At seven o'clock in the morning.
Possible responses:
I will set off after ten o'clock.
We will start at eight o'clock sharp in the morning.
After finishing my sales report.
What will the traffic be like tomorrow morning? (The second speaker is not sure about the
answer.)
ng k hc: 0962 60 8801 04 6260 3948 273
a ch: S 18 Trn i Ngha Q Hai B Trng H ni
NGOI NG 24H 274
WWW.NGOAINGU24H.VN

It depends on the weather conditions. (The second speaker is not sure about the answer.)
Listen and choose the correct response to each of the questions below.
1. What time is the flight scheduled to depart?
(A) It depends on the weather conditions. (B) It takes about two hours.
2. What time can you take a break?
(A) Thats a good idea. (B) After finishing my sales report.
3. What time will you start the meeting?
(A) At ten oclock. (B) In the conference room.

Structure 3: What the best/fastest/quickest/shortest way + to infinitive? 059.mp3


Q: What is the best way to get to the airport?
A: Take the subway.
Take a taxi.
I think a taxi is the fastest way to get there.
There is a shuttle bus service.
Listen and choose the correct response to each of the questions below.
1. What is the fastest way to get to the convention center from here?
(A) Take a taxi. (B) At three o'clock.
2. What is the shortest way to get to the hospital?
(A) Two hundred meters. (B) Through the park.
3. What is the best way to reach your manager?
(A) Here is his phone number. (B) He is a sales manager.

Practice - Questions with What (1) 060.mp3


Listen to the questions and responses below and choose the correct response to each question.
Then, listen again and fill the missing words in the gapped questions and responses.
Practice A Practice B
1. What is the problem with my computer? 1. What is _____ _____ to contact you?
(A) It isnt plugged _____. (A) We should _____ _____ _____.
(B) Sure, you can ____ _____. (B) _____my office _____.
2. What time are you leaving the office today? 2. _____ _____ will the _____ _____?
(A) It takes about _____ _____. (A) It runs __________ a day.
(B) _____ _____ this report. (B) _____ _____ a.m.
3. What is Mr. Jacksons position at the 3. What is the _____ ____ my _____?
company? (A) I think its infected _____ a ____.
(A) He is a _____ _____. (B) _____ ____.
(B) In the _____ _____. 4. _____ _____ are you _____ the _____ today?
4. What time will the train arrive? (A) That _____ like a _____ _____.
(A) In _____ _____. (B) In _____ _____.
(B) It runs _____ _____ _____. 5. What is the _____ _____ of the construction?
5. What is the best way to contact you? (A) It will not exceed _____ _____ _____.

ng k hc: 0962 60 8801 04 6260 3948 274


a ch: S 18 Trn i Ngha Q Hai B Trng H ni
NGOI NG 24H 275
WWW.NGOAINGU24H.VN

(A) I _____ _____ _____ my mobile number. (B) It will take _____ _____ three _____.
(B) Its a _____ contract. 6. What is Mr. Jacksons _____ at the _____?
6. What is the total cost of the construction? (A) He is _____ _____ _____ the _____
(A) It will _____ at least _____ _____. Department.
(B) It is estimated at _____ _____. (B) The _____ _____ me.

Structure 4: What is the weather (going to be) like + date/place? 061.mp3


What is the weather forecast for + date/place?
Q: What is the weather going to be like tomorrow?
A: I heard it's going to rain.
Possible responses:
It is going to be cloudy and windy.
You will probably need your umbrella.
The forecast for tomorrow calls for clear skies.
Listen and choose the correct response to each of the questions below.
1. What is the weather like in New York?
(A) It is warm. (B) For a month.
2. What is the weather forecast for the weekend?
(A) From 3 to 7 o'clock. (B) It is going to be cloudy and windy.
3. What is the weather forecast for tomorrow?
(A) I suppose you would be right. (B) The forecast for tomorrow calls for clear skies.

Structure 5: What do/did you think of/about + noun phrase/gerund phrase? 062.mp3
Q: What do you think about our marketing strategy?
A: It seems like a great plan.
Possible responses:
It looks like a good Idea.
It sounds good in theory.
Thats an excellent idea.
I am not sure.
I doubt if it is possible in reality.
Listen and choose the correct response to each of the questions below.
1. What do you think about signing the contract?
(A) I am not sure. (B) Thanks a lot.
2. What do you think of the companys plan to expand?
(A) I doubt if it is possible in reality. (B) I dont think t can take a vacation.
3. What is your opinion of the new product?
(A) Anytime you want. (B) It looks like a good idea.

Structure 6: What do/does/did + subject + bare infinitive? 063mp3


What will/would/should + subject + bare infinitive?

ng k hc: 0962 60 8801 04 6260 3948 275


a ch: S 18 Trn i Ngha Q Hai B Trng H ni
NGOI NG 24H 276
WWW.NGOAINGU24H.VN

What is/are + subject + V-ing?


Q: What do you need for your trip?
A: A credit card.
A small backpack.
A credit card is all I need.
I need some camping equipment.
Nothing in particular. It will be a short visit to see my friend.
Listen and choose the correct response to each of the questions below.
1. What does Mr. Thompson intend to do after he retires?
(A) He came in yesterday. (B) He plans to start his own business.
2. What should I do with this report?
(A) I finished my report. (B) Please review it.

Structure 7: What kind/type/sort of + noun + auxiliary verb/be + subject + verb? 064.mp3


What + noun + auxiliary verb + subject + verb?
Q: What kind of job are you looking for?
A: Im interested in web design.
Im seeking a challenging position.
I have experience in marketing.
Listen and choose the correct response to each of the questions below.
1. What color would you like to paint your house?
(A) Yellow or green. B) Its a new house.
2. What kind of car are you interested in?
(A) l will take care of it. (B) I have a pickup truck in mind.
3. What sort of food would you like to have?
(A) Italian food. (B) Lets have lunch.

Practice - Questions with What (2) 065.mp3


Listen to the questions and responses below and choose the correct response to each question.
Then, listen again and fill the missing words in the gapped questions and responses.
Practice A Practice B
1. What do you think of the new plan? 1. What is the weather forecast _____ ____?
(A) I dont have any plans _____ _____. (A) I will _____ a _____.
(B) It _____ _____ a good one. (B) I _____ it will be _____.
2. What did they discuss at the meeting? 2. What do you _____ of the _____ _____?
(A) A _____ _____. (A) I have a _____ _____.
(B) They will _____ it _____. (B) It looks like we'll _____ _____ _____.
3. What is the weather forecast for Thursday? 3. What did they _____ _____ the _____?
(A) I dont _____ whether _____ will _____. (A) _____ raises.
(B) It's _____ _____ _____ pretty _____ then. (B) They have _____ _____ it.
4. What kind of book are you reading? 4. What do you think of _____ a _____in

ng k hc: 0962 60 8801 04 6260 3948 276


a ch: S 18 Trn i Ngha Q Hai B Trng H ni
NGOI NG 24H 277
WWW.NGOAINGU24H.VN

(A) I am _____ a comic book. Manila?


(B) I am _____ the _____ team. (A) What is the advantage of that ____?
5. What should I do with these books? (B) We will _____ the _____ at 7.
(A) _____ _____ to Mr. Jones. 5. What kind of ____ are you _____?
(B) Yes. he is a _____ _____. (A) I ____ _____ the library.
6. What do you think of opening a branch in (B) It is about _____.
Manila? 6. What should I _____ _____ these ____?
(A) I _____ _____ it is a good idea. (A) Yes, he is a _____ _____.
(B) I think we should _____ the _____. (B) Ill _____ _____ _____ them.

Practice the possible questions. 066.mp3


Listen and choose the correct response to each of the questions.
1. (A) (B)
2. (A) (B)
3. (A) (B)
4. (A) (B)
5. (A) (B)
6. (A) (B)
7. (A) (B)
8. (A) (B)
9. (A) (B)
10. (A) (B)

Practice with TOEIC Actual Questions. 067.mp3


Listen and choose the correct response to each of the questions.
1. (A) (B) (C)
2. (A) (B) (C)
3. (A) (B) (C)
4. (A) (B) (C)
5. (A) (B) (C)
6. (A) (B) (C)
7. (A) (B) (C)
8. (A) (B) (C)
9. (A) (B) (C)
10. (A) (B) (C)
11. (A) (B) (C)
12. (A) (B) (C)
13. (A) (B) (C)
14. (A) (B) (C)
15. (A) (B) (C)

ng k hc: 0962 60 8801 04 6260 3948 277


a ch: S 18 Trn i Ngha Q Hai B Trng H ni
NGOI NG 24H 278
WWW.NGOAINGU24H.VN

NGY 16: CU HI YES/ NO

Trng tm ca bi hc
Bi ny hc v nhng cu hi Yes/No, bt u bng cc ng t Do/Be/Have v tr li bng Yes/No.
Khi p n bt u bng Yes/No, cn phi xc nh xem cc ni dung sau Yes/No: (1) Tuy tr li l
Yes nhng c mang ni dung ph nh khng. (2) Ni dung sau Yes c lin quan n cu hi hay
khng, cn phi nghe k cc t cng nh ch n th ca ng t. Hc trng tm cc cu hi c
t ra theo hng nm bt chnh ca cu hi.

Phn tch cu hi v cc p n la chn 046.mp3

Did you call a taxi for Mr. Nunez?


Yes, it should be here shortly.
9. Hi xem call a taxi cha.
Qua t Yes c th bit gi taxi ri.

Have you made copies of the booklet?


I already distributed them to the
attendees.
10. Hi xem make copies cha.
D t Yes b lc b nhng qua cm t already distributed them c th bit bn sao c
thc hin.

Is your business going well?


Not really. Im having financial problems.
11. Hi cng vic kinh doanh c tt hay khng.
Qua cm t not really c th bit l No, lm r l do chnh l gp vn v ti chnh.

Vocabulary
call a taxi gi taxi distribute phn pht
shortly khng lu, sm attendee ngi tham d
make copies sao chp ti liu financial (thuc) ti chnh
booklet sch mng

Bi tp 1 047.mp3
Nghe cc cu hi v chn ra cu tr li ng.
1. [ A / B ]
2. [ A / B ]
3. [ A / B ]

Vocabulary

ng k hc: 0962 60 8801 04 6260 3948 278


a ch: S 18 Trn i Ngha Q Hai B Trng H ni
NGOI NG 24H 279
WWW.NGOAINGU24H.VN

sunny tri nng contractor nh thu


picnic i d ngoi offer li ngh
place the order t hng sign k kt
ask for yu cu contract hp ng
along the wall dc theo bc tng decline t chi
completely hon ton

Bi tp 2 048.mp3
Nghe cc cu hi v chn ra cu tr li ng.
4. [ A / B ]
5. [ A / B ]
6. [ A / B ]

Vocabulary
on time ng gi in a minute chc na
fortunately may mn thay enjoy vui thch, tn hng
meet the deadline kp hn trip chuyn i
estimate c tnh learn hc
arrival vic ti ni

Bi tp 3 049.mp3
Nghe cc cu hi v chn ra cu tr li ng.
7. [ A / B ]
8. [ A / B ]
9. [ A / B ]

Vocabulary
shipment chuyn hng bring mang theo
be scheduled to do uc sp xp / ln k newly mi
hoch lm g release pht hnh
change thay i funny gy ci
receipt giy bin nhn weekend cui tun
seem c v

Bi tp 4 050.mp3
Nghe nhng cu di y v in vo ch trng.
1. ________________________ to process the applications?
2. ________________________ will replace Carols work?
3. ________________________ Tim recently?
4. ________________________ while I was out?
5. ________________________ to work today?
6. ________________________ her name, K-A-T-Y?
7. ________________________ the invitations yesterday?
ng k hc: 0962 60 8801 04 6260 3948 279
a ch: S 18 Trn i Ngha Q Hai B Trng H ni
NGOI NG 24H 280
WWW.NGOAINGU24H.VN

8. ________________________ to the headquarters?


9. ________________________ bottled water here?
10. ________________________ each other before?

Luyn nghe nhng cu cn thit 051.mp3


Luyn nghe nhng cu hi Do / Be / Have thng gp

Cu hi Do 1. Do you know where the cold food is?


2. Did the president approve the proposal?
3. Does she work with Jackson cooperation?
4. Did Mr. White go on his business trip?
Cu hi Be
5. Are you still seeking employees for the job openings?
6. Is the director available for tonights charity event?
Cu hi Have
7. Are there any steaks left?

8. Have you ever been to the national museum?


9. Has she transferred to Busan branch?
10. Have we met before?

Bi tp 5 052.mp3
Lng nghe v chn cu tr li ng cho tng cu hi, sau khi nghe li ln 2, hy in vo ch trng.

1. A B C 4. A B C
________________________ in the ___________ to buy office furniture?
kitchen? (A) _______________________,please.
(A) Yes, he is a ___________________ .
(B) _____________ and lamps.
(B) Why dont you
(C) There is a shop ________________
____________________?
across the street.
(C) No, they are on ________________ .

2. A B C 5. A B C
____________________about the ___________________ of the revised
Johns ____________________? plan?
(A) He has been working here for (A) There is one ___________________ .
____________. (B) ____________ needs to be changed.

ng k hc: 0962 60 8801 04 6260 3948 280


a ch: S 18 Trn i Ngha Q Hai B Trng H ni
NGOI NG 24H 281
WWW.NGOAINGU24H.VN

(B) I havent heard yet. (C) He advised me not to buy ________ .


(C) Yes, he ____________________.

3. A B C 6. A B C
Are you going to ________________ Have you seen my ___________________?
tonight? (A) It _______________ by plane.
(A) I have _______________ today.
(B) Its ________________.
(B) No, she is the new____________.
(C) No, he ______________ last week.
(C) Peter will __________________.

BI TP THC HNH
Questions with Be/Do/Have
Example 1: Listen and choose the best response to the question 068.mp3
Are you going to attend the seminar next week?
(A) I am still looking for a |ob.
(B) That s very kind of you.
(C) Yes, we all have to be there.
Example 2: Listen and choose the best response to the question
Do you have my business card?
(A) No, could you give it to me?
(B) I'll send a postcard.
(C) He may be busy.
Example 3: Listen and choose the best response to the question.
Have you seen any movies lately?
(A) No, t haven't seen her.
(B) Ive been too busy.
(C) She was late for the party.

Question Structures:
Structure 1: Are you + V-ing/going to + bare infinitive? 069.mp3
rd
Is + subject (3 person singular) + V-ing/going to + bare infinitive?
Q: Are you coming to the party tonight?
A: Yes, Ill be there.
Possible responses:
Sure, I will not be late.
No, I have other plans.
I will be there on time.
I am looking forward to it.
Listen and choose the correct response to each of the questions below.
1. Are you going to show up at the charity event?

ng k hc: 0962 60 8801 04 6260 3948 281


a ch: S 18 Trn i Ngha Q Hai B Trng H ni
NGOI NG 24H 282
WWW.NGOAINGU24H.VN

(A) I want to help you. (B) No, I have other plans.


2. Are you going to attend the conference?
(A) I will be there on time. (B) No, thanks.
3. Are you going to visit the Alps this winter?
(A) I am looking forward to it. (B) We had a great time.

Structure 2: Is/Are/Was/Were there + noun + prepositional phrase? 070.mp3


Q: Are there any tickets available for today's performance?
A: Only a few are left.
Possible responses:
Yes, there are some left.
Sorry. They are all sold out.
Let me check.
How many tickets do you want?
Listen and choose the correct response to each of the questions below.
1. Is there a grocery store in the neighborhood?
(A) Yes, I'll call you later. (B) Yes, right across the street.
2. Are there any more apples left in the refrigerator?
(A) I will cook with apples. (B) Yes, there are some left.

Structure 3: Is/Are/Was/Were there + noun + prepositional phrase? 071.mp3


Q: Are gas and electricity included in the rent?
A: Yes, and water, too.
Possible responses:
Yes, they are.
All utility bills are included.
No, utility bills are not included.
Utility bills are charged separately.
Listen and choose the correct response to each of the questions below.
1. Are you free later this afternoon?
(A) No, Im busy. (B) Three times a day.
2. Is Ms. Lopez still on vacation?
(A) Yes, she is working now. (B) Yes, but she will be here on Friday.

Structure 4: Do/Does/Did + subject + bare infinitive (+ object)? 072.mp3


Q: Do you have an extra copy of the schedule?
A: Yes, it's on my desk.
Possible responses:
I will bring it to you.
Let me check.
Here are some extra copies on the table.

ng k hc: 0962 60 8801 04 6260 3948 282


a ch: S 18 Trn i Ngha Q Hai B Trng H ni
NGOI NG 24H 283
WWW.NGOAINGU24H.VN

Sorry. That is the last one.


Listen and choose the correct response to each of the questions below.
1. Does this gym have indoor tennis courts?
(A) Yes, on the first floor. (B) Yes, once a week.
2. Did you move into the new house?
(A) That sounds good. (B) Yes, I did.
3. Did the plane arrive at the airport on time?
(A) No, I took a taxi. (B) No, it was delayed.

Practice - Questions with Be /Do /Have (1 ) 073.mp3


Listen to the questions and responses below and choose the correct response to each question. Then,
listen again and fill the missing words in the gapped questions and responses.
Practice A Practice B
1. Are you leaving work early today? 1. Is there a _____ _____ around here?
(A) _____, I will. (A) It _____ ten minutes _____.
(B) Yes, I will _____ on it. (B) The _____ ____ is five miles away _____.
2. Do you have Jessica's phone number? 2. Is it okay to _____ in _____ of the building?
(A) Let me _____ my _____. (A) Yes, there is a _____.
(B) Yes. she _____ me this morning. (B) No, it is a no- _____ _____.
3. Is there a gas station around here? 3. Are you _____ _____ early today?
(A) It _____ five minutes ______. (A) Yes, I will _____ _____ to ______.
(B) There is one _____ the _____. (B) Yes, at _____ o'clock.
4. Is it okay to park in front of the building? 4. Are you satisfied _____ the _____ you
(A) I couldn't _____ a _____ _____. received?
(B) I dont _____ _____. (A) It was _____ _____.
5. Are you satisfied with the service you (B) No, it is not _____ _____.
received? 5. Does your new laser _____ _____?
(A) Are you _____? (A) Yes, it _____ _____.
(B) Yes, _____ _____. (B) I think _____ is _____ now.
6. Does your new laser printer work? 6. Do you _____ Jessicas phone _____?
(A) Yes, it is _____ than the old one. (A) Yes, I _____.
(B) I think he is _____ _____. (B) No, she _____ me.

Structure 5: Do you know + question word + subject + verb? 074.mp3


Q: Do you know where the nearest bus stop is?
A: Just across the street.
Possible responses:
Over there.
Around the corner.
At the corner of the street.
Next to the taxi stand.
There is one in front of the post office.
ng k hc: 0962 60 8801 04 6260 3948 283
a ch: S 18 Trn i Ngha Q Hai B Trng H ni
NGOI NG 24H 284
WWW.NGOAINGU24H.VN

Listen and choose the correct response to each of the questions below.
1. Do you know where I can buy a newspaper?
(A) Over there. (B) At noon.
2. Do you know when the report is due?
(A) To New York. (B) By Friday.
3. Do you know who is in charge of this project?
(A) Its tonight. (B) That would be Mr. James.
Structure 6: Do you mind if I/we + verb? 075.mp3
Do you mind + V-ing?
Q: Do you mind if I park my car here for a moment?
A: Sorry. It is a restricted area.
Possible responses:
That's fine with me.
Not at all.
By all means.
This place is reserved for emergency vehicles only.
Listen and choose the correct response to each of the questions below.
1. Do you mind if I turn on the radio?
(A) I like that TV program. (B) Actually, I do mind.
2. Do you mind waiting for a while?
(A) For thirty minutes. (B) Thats fine with me,
3. Do you mind if I close the window?
(A) I opened the door. (B) Not at all.
Structure 7: Have you + past participle? 076.mp3
Has/Have + subject (3rd person) + (been) past participle?
Q: Have you received the sample?
A: No, it hasn't arrived yet.
Possible responses:
Yes, I received it just a while ago.
It arrived ten minutes ago.
I am still waiting for it.
Listen and choose the correct response to each of the questions below.
1. Has the package been delivered?
(A) Twice a day. (B) Yes, it has just arrived.
2. Have you finished reading the files I gave you?
(A) Not yet. (B) No, its mine.
3. Has the software been installed on your computer?
(A) No, it hasnt. (B) Yes, I repaired my computer.

Practice Question with Be/Do/Have (2) 077.mp3


Listen to the questions and responses below and choose the correct response to each question.

ng k hc: 0962 60 8801 04 6260 3948 284


a ch: S 18 Trn i Ngha Q Hai B Trng H ni
NGOI NG 24H 285
WWW.NGOAINGU24H.VN

Then, listen again and fill the missing words in the gapped questions and responses.
Practice A. Practice B
1. Do you mind if I leave early today? 1. Has your car _____ _____ yet?
(A) Not ____ _____. (A) Yes, Ill ____ a ____.
(B) ______ ______. (B) Yes, I'll go to the ____ ____ to ____ it
2. Has your car been repaired yet? _____ this afternoon.
(A) Yes, I'll ____ a _____. 2. Do you know ______ the report is _____?
(B) It was _____ today. (A) Yes, I received it _____ _____.
3. Do you know who has the copy of the revised (B) No, I havent _____.
schedule? 3. Have you _____ my wallet?
(A) There is _____ on my _____. (A) It's ____ your ____.
(B) Sorry. I cant _____ my ______. (B) No, I dont have any _____.
4. Have you seen my wallet? 4. Do you _____ if I _____ early today?
(A) No, I havent. (A) Dont be _____.
(B) _____ very _____. (B) Id rather you _____.
5. Do you know when the report is due? 5. Has the _____ for the meeting been yet?
(A) _____ _____ morning. (A) I ____ it.
(B) Yes, I _____ it ______. (B) I am still ____ ____ it.
6. Has the report for the meeting been 6. Do you know _____ has the ____ of the
completed yet? _____ schedule?
(A) Yes, they _____ together. (A) It's _____ Ms. Sanchez.
(B) No, not _____. (B) Its ____ for _____.

Practice the possible questions. 078.mp3


Listen and choose the correct response to each of the questions.
1. (A) (B)
2. (A) (B)
3. (A) (B)
4. (A) (B)
5. (A) (B)
6. (A) (B)
7. (A) (B)
8. (A) (B)
9. (A) (B)
10. (A) (B)

Practice with TOEIC Actual Questions. 079.mp3


Listen and choose the correct response to each of the questions.
1. (A) (B) (C)
2. (A) (B) (C)
3. (A) (B) (C)
4. (A) (B) (C)
5. (A) (B) (C)
ng k hc: 0962 60 8801 04 6260 3948 285
a ch: S 18 Trn i Ngha Q Hai B Trng H ni
NGOI NG 24H 286
WWW.NGOAINGU24H.VN

6. (A) (B) (C)


7. (A) (B) (C)
8. (A) (B) (C)
9. (A) (B) (C)
10. (A) (B) (C)
11. (A) (B) (C)
12. (A) (B) (C)
13. (A) (B) (C)
14. (A) (B) (C)
15. (A) (B) (C)

Cu hi vi Can/Could/ Will/ Would/ Should/ May


Example 1: Listen and choose the best response to the question. 080.mp3
Could you take me to the railway station?
(A) No problem.
(B) I missed the train.
(C) There is a stationery store down the street.
Example 2: Listen and choose the best response to the question.
Would you like to taste a free sample of our new chocolate?
(A) No, its free of charge.
(B) This camera Is very simple to use.
(C) Sure, it looks delicious.
Example 3: Listen and choose the best response to the question.
Should I wear a warm coat today?
(A) Where did you put it?
(B) Yes, its too cold today.
(C) The coat looks great on you.

Question Structure:
Structure 1: Can/Could you + bare infinitive + object? 081.mp3
Q: Can you help me move this sofa?
A: Sure, if its not too heavy.
Possible responses:
Sure, just give me a minute.
I'd be glad to.
Where should we put it?
Listen and choose the correct response to each of the questions below.
1. Can you meet me this afternoon?
(A) At the cafeteria. (B) I'd love to, but I'm busy.
2. Could you take a look at my report now?
(A) Sure, just give me a minute. (B) I didn't give it to you.
3. Could you please answer the phone?
ng k hc: 0962 60 8801 04 6260 3948 286
a ch: S 18 Trn i Ngha Q Hai B Trng H ni
NGOI NG 24H 287
WWW.NGOAINGU24H.VN

(A) I'll call you later. (B) Id be giad to.

Structure 2: Can/Could you tell me + question word + subject + verb? 082.mp3


Q: Can you tell me where the closest cash machine is?
A: Go to the post office across the street.
Possible responses:
There is one at the next comer.
There is one in front of the tall building.
I am sorry. I dont know.
Listen and choose the correct response to each of the questions below.
1. Can you tell me what you are doing this evening?
(A) I will stay at home. (B) I can do it today.
2. Can you tell me how I can get to the museum?
(A) Take bus number 11. (B) Before noon.
3. Could you tell me who the new manager is?
(A) Im a member of this club. (B) I am sorry. I don't know.

Structure 3: Can/Could l/we + bare infinitive? 083.mp3


May I + bare infinitive?
Q: Can I borrow your calculator?
A: No problem.
Possible responses:
Yes, you can.
Of course, you can.
Here you are./Here it is.
Be my guest.

Listen and choose the correct response to each of the questions below.
1. May I ask you for a small favor?
(A) Yes, it Is very small. (B) Yes, of course.
2. Could we take a break after the next presentation?
(A) Of course, we can. (B) I'd like to know.
3. Can I see your ID card, please?
(A) Here it is. (B) I will pay in cash.

Practice - Questions with Can/Could/Will/Would/Should/May (1 ) 084.mp3


Listen to the questions and responses below and choose the correct response to each question.
Then, listen again and fill the missing words in the gapped questions and responses.
Practice A: Practice B:
1. Can you tell me how many copies of this 1. May I _____ _____ it with a _____ card?
report you need? (A) We don't _____ _____.
(A) _____ will be _____. (B) _____ _____, you can.

ng k hc: 0962 60 8801 04 6260 3948 287


a ch: S 18 Trn i Ngha Q Hai B Trng H ni
NGOI NG 24H 288
WWW.NGOAINGU24H.VN

(B) ____ noon. 2. Could you ____ me _____ the conference


2. Can I look at the new catalog? room is?
(A) _____, it is on the _____. (A) Its on the _____ floor.
(B) I ____ it on the _____. (B) The ____ was cancelled.
3. Could you please turn down the television? 3. Can you ____ me a _____ to the _____?
(A) Sure _____ ______ ______. (A) I _____ it at the ____ desk.
(B) _____, I turned down his _____. (B) Sorry. I didn't ____ my car.
4. May I pay for it with a credit card? 4. Can I ____ ____ the new catalog?
(A) We dont _____ any ____. (A) I ____ it on TV.
(B) _____, you ____. (B) Im ____. There is ____ ____.
5. Can you give me a ride to the airport? 5. Can you tell me ____ ____ copies of this
(A) Sorry. I don't _____ _____. report you need?
(B) _____ be _____ to. (A) I am ____ ____.
6. Could you tell me where the conference room (B) Before ____.
is? 6. _____ you please _____ _____ the television?
(A) The seminar was ____. (A) I ____ turn down _____ _____.
(B) Yes, ____ straight and _____ _____. (B) ____. I didnt ____ you were here.

Structure 4: Would you like/care to + bare infinitive? 085.mp3


Would you like/prefer + noun phrase?
Q: Would you like to take a look at this report?
A: Is there anything wrong?
Possible responses:
Sure, I will.
Would you pass it to me?
Wait for a second.
I should finish this report first.

Listen and choose the correct response to each of the questions below.
1. Would you like to apply for the job?
(A) Sure, I will. (B) My job is interesting.
2. Would you like some milk in your coffee?
(A) I prefer coffee. (B) Yes, please.
Structure 5: Would you like me/us to + bare infinitive? 086.mp3
Q: Would you like me to review the report with you?
A: That would be great.
Possible responses:
Yes, please.
If you wouldnt mind.
Id appreciate that.
Thanks. That would be very helpful.

Listen and choose the correct response to each of the questions below.

ng k hc: 0962 60 8801 04 6260 3948 288


a ch: S 18 Trn i Ngha Q Hai B Trng H ni
NGOI NG 24H 289
WWW.NGOAINGU24H.VN

1. Would you like me to send you a sample of our new product?


(A) Thanks. That would be very helpful. (B) I sent you an e-mail.
2. Would you like me to take you to the airport?
(A) Next to the airport. (B) Id appreciate that.
3.Would you like me to arrange the meeting?
(A) The meeting was boring. (B) If you wouldn't mind.
Structure 6: Would you + bare infinitive? 087.mp3
Will you + bare infinitive/be + V-ing?
Q: Would you tell Michael I will call him tomorrow morning?
A: No problem. What time are you calling?
Possible responses:
Yes. I will tell him.
Okay. I will let him know.
Sure, I will leave a memo on his desk.
He will not be here tomorrow.

Listen and choose the correct response to each of the questions below.
1. Would you fax this to the sales manager?
(A) Sure. (B) It will be fixed soon.
2. Will you be taking your vacation in July?
(A) Yes, I will call you back. (B) No, in August.
3. Would you please press the button for the 9th floor?
(A) Certainly. (B) It's on the 7th floor.
Structure 7: Should I/we + bare infinitive + object? 088.mp3
Q: Should we finish the report today?
A: We have a few more days until the deadline.
Possible responses:
Yes, we have to hurry,
It must be submitted before noon,
Yes, it should not be delayed anymore.

Listen and choose the correct response to each of the questions below.
1. Should we accept their proposal?
(A) Yes, we should. (B) At the convention center.
2. Should I call a doctor for you?
(A) He is a doctor. (B) I would appreciate it.
3. Should I come to see you again next week?
(A) Yes, we need further discussion. (B) No. Ive never seen him before.

Practice - Questions with Can/Could/ Will/ Would/Should/May (2) 089.mp3


Listen to the questions and responses below and choose the correct response to each question.
Then, listen again and fill the missing words in the gapped questions and responses.
Practice A: Practice B:

ng k hc: 0962 60 8801 04 6260 3948 289


a ch: S 18 Trn i Ngha Q Hai B Trng H ni
NGOI NG 24H 290
WWW.NGOAINGU24H.VN

1. Would you please help me move the table? 1. Will you _____ me _____ after work?
(A) Yes, I am _____ to. (A) _____ are you going to be _____?
(B) _____, _____. I am ____. (B) I don't _____ _____ it works.
2. Would you like to try on a smaller size? 2. Should I _____ the package by _____ _____?
(A) I will _____ my _____. (A) I received it _____.
(B) I think I _____. (B) Is there a _____ _____?
3. Will you pick me up after work? 3. ____ you please _____ me _____ the table?
(A) I'd like to know _____ it _____. (A) I'm _____ to _____ that.
(B) I think I _____. (B) ______. Do you _____ _____ _____ it now?
4. Would you like me to come on Tuesday? 4. Would you like _____ _____ of tea?
(A) I will _____ it to you. (A) Yes, ________.
(B) Is that okay _____ _____? (B) I _____ it on the ____.
5. Should I send the package by express mail? 5. Would you like to _____ ______ a _____
(A) I _____ it this _____. size?
(B) Yes, it _____ ______ there by tomorrow (A) I always _____ ______ _____.
morning. (B) Yes, I'd _____ to.
6. Would you like another cup of tea? 6. _____ you ______ _____ to come on
(A) No, ______ _______. ______?
(B) I _____ it on the _____. (A) I will _____ an ______.
(B) Yes, I will _____ _____ you.

Practice the possible questions. 090.mp3


Listen and choose the correct response to each of the questions.
1. (A) (B)
2. (A) (B)
3. (A) (B)
4. (A) (B)
5. (A) (B)
6. (A) (B)
7. (A) (B)
8. (A) (B)
9. (A) (B)
10. (A) (B)

Practice with TOEIC Actual Questions. 091.mp3


Listen and choose the correct response to each of the questions.
1. (A) (B) (C)
2. (A) (B) (C)
3. (A) (B) (C)
4. (A) (B) (C)
5. (A) (B) (C)
6. (A) (B) (C)
7. (A) (B) (C)
ng k hc: 0962 60 8801 04 6260 3948 290
a ch: S 18 Trn i Ngha Q Hai B Trng H ni
NGOI NG 24H 291
WWW.NGOAINGU24H.VN

8. (A) (B) (C)


9. (A) (B) (C)
10. (A) (B) (C)
11. (A) (B) (C)
12. (A) (B) (C)
13. (A) (B) (C)
14. (A) (B) (C)
15. (A) (B) (C)
Embedded Questions, Alternative Questions, and Statements
1. Embedded Questions.
Exercise
Sample 092.mp3
Q: Do you know who has the key to the warehouse?
(A) To the warehouse.
(B) Yes, Amy usually has it.
(C) Just get the key for me.
Step 1: Listen and choose the correct response to each question.
1. (A) (B) (C)
2. (A) (B) (C)
Step 2: Listen and fill in the missing words.
1. Could you let me ______________________complained about the service?
(A) Approximately
(B) The service was poor.
(C) The customer complained bitterly.
2. Can you _________________________ you usually do in your free time?
(A) Yes, I have some time.
(B) I ____________________.
(C) Usually after work.
Step 3: Check the filled-in words.
1. Could you let me know how many people complained about the service?
(A) Approximately 50 or more.
(B) The service was poor.
(C) The customer complained bitterly.
2. Can you tell me what you usually do in your free time?
(A) Yes, I have some time.
(B) I _play online games .
(C) Usually after work.
2. Alternative questions.
Exercise 093.mp3
Q: Should we ship the incomplete orders or wait another day?
(A) Yes, for two days.
(B) I have to leave soon.
(C) Let's just send what we have on hand.
ng k hc: 0962 60 8801 04 6260 3948 291
a ch: S 18 Trn i Ngha Q Hai B Trng H ni
NGOI NG 24H 292
WWW.NGOAINGU24H.VN

Step 1: Listen and choose the correct response to each question.


1. (A) (B) (C)
2. (A) (B) (C)
Step 2: Listen and fill in the missing words.
1. Do you prefer __________________ or in a group?
(A) Im not feeling well.
(B) ___________________ with me.
(C) No, I have a job.
2. Would you rather prefer ______________________ or analyzing the materials?
(A) I dont trust the news report.
(B) In the filing cabinet.
(C) _________________________ matter to me.
Step 3: Check the filled-in words.
1. Do you prefer working alone or in a group?
(A) Im not feeling well.
(B) Either is fine with me.
(C) No, I have a job.
2. Would you rather prefer entering the data or analyzing the materials?
(A) I dont trust the news report.
(B) In the filing cabinet.
(C) It doesnt matter to me.
3. Statements
Exercise
Sample 094.mp3
Q: This package seems to have been damaged during delivery.
(A) Yes, ten packages.
(B) Overseas delivery is not available.
(C) You'd better take it back to the dealer.
Step 1: Listen and choose the correct response to each question.
1. (A) (B) (C)
2. (A) (B) (C)
Step 2: Listen and fill in the missing words.
1. You will ____________________ in 10 working days.
(A) I received your email.
(B) __________________________ right after I get it.
(C) No, its too expensive.
2. Lets _______________________ for a cup of coffee.
(A) Thats _________________.
(B) Near the Forbes coffeehouse.
(C) You can borrow mine.
Step 3: Check the filled-in words.
1. You will receive the bill in 10 working days.
(A) I received your email.
ng k hc: 0962 60 8801 04 6260 3948 292
a ch: S 18 Trn i Ngha Q Hai B Trng H ni
NGOI NG 24H 293
WWW.NGOAINGU24H.VN

(B) I pay for it right after I get it.


(C) No, its too expensive.
2. Lets take a break for a cup of coffee.
(A) Thats a good idea.
(B) Near the Forbes coffeehouse.
(C) You can borrow mine.

Practice test 095.mp3


1. Mark your answer.
(A) (B) (C)
2. Mark your answer.
(A) (B) (C)
3. Mark your answer.
(A) (B) (C)
4. Mark your answer.
(A) (B) (C)
5. Mark your answer.
(A) (B) (C)
6. Mark your answer.
(A) (B) (C)
7. Mark your answer.
(A) (B) (C)
8. Mark your answer.
(A) (B) (C)
9. Mark your answer.
(A) (B) (C)
10. Mark your answer.
(A) (B) (C)

Practice: 096.mp3
1. Q: _________________________ to attend Mr. Coles retirement party?
A: Yes, he is my supervisor.
2. Q: Do you know _________________________ to get the job done?
A: About 2 months.
3. Q: That was a _________________________.
A: Yes, i enjoyed it a lot.
4. Q: Can you tell me _________________________ a subway station?
A: Yes, go this way and turn right at the first intersection.
5. Q: I need a better way _________________________.
A: I know a website for carpooling.
6. Q: _________________________ we can meet the deadline.
A: Im glad to hear that.
7. Q: _________________________ we have to order more office supplies?
ng k hc: 0962 60 8801 04 6260 3948 293
a ch: S 18 Trn i Ngha Q Hai B Trng H ni
NGOI NG 24H 294
WWW.NGOAINGU24H.VN

A: No, we still have enough.


8. Q: Would you like to _________________________ today or tomorrow?
A: It doesnt matter to me.
9. Q: Mr. Peterson _________________________ twice this morning.
A: Did he leave any message?
10. Q: Do _________________________ Mr. Ben asked for a transfer?
A: She wants to work in a new environment.

Practice: Identifying an Opinion 097.mp3


Direction: Listen to the questions, which are followed by three responses. They will not be written
out for you. Choose the best response to each question.
1. (A) (B) (C)
2. (A) (B) (C)
3. (A) (B) (C)
4. (A) (B) (C)
5. (A) (B) (C)
6. (A) (B) (C)
7. (A) (B) (C)
8. (A) (B) (C)
9. (A) (B) (C)
10. (A) (B) (C)

Practice: Identifying a choice


Practice: Identifying a choice 098.mp3
Direction: Listen to the questions, which are followed by three responses. They will not be written
out for you. Choose the best response to each question.
1. (A) (B) (C)
2. (A) (B) (C)
3. (A) (B) (C)
4. (A) (B) (C)
5. (A) (B) (C)
6. (A) (B) (C)
7. (A) (B) (C)
8. (A) (B) (C)
9. (A) (B) (C)
10. (A) (B) (C)

ng k hc: 0962 60 8801 04 6260 3948 294


a ch: S 18 Trn i Ngha Q Hai B Trng H ni
NGOI NG 24H 295
WWW.NGOAINGU24H.VN

NGY 18:
PART 2: KIM TRA TIN
PART 3: GII THIU CHUNG
PART 7: TH TN V TH IN T (E-MAIL)

A PART 2: KIM TRA TIN 099.mp3


1. (A) (B) (C)
2. (A) (B) (C)
3. (A) (B) (C)
4. (A) (B) (C)
5. (A) (B) (C)
6. (A) (B) (C)
7. (A) (B) (C)
8. (A) (B) (C)
9. (A) (B) (C)
10. (A) (B) (C)
11. (A) (B) (C)
12. (A) (B) (C)
13. (A) (B) (C)
14. (A) (B) (C)
15. (A) (B) (C)
16. (A) (B) (C)
17. (A) (B) (C)
18. (A) (B) (C)
19. (A) (B) (C)
20. (A) (B) (C)
21. (A) (B) (C)
22. (A) (B) (C)
23. (A) (B) (C)
24. (A) (B) (C)
25. (A) (B) (C)
26. (A) (B) (C)
27. (A) (B) (C)
28. (A) (B) (C)
29. (A) (B) (C)
30. (A) (B) (C)

ng k hc: 0962 60 8801 04 6260 3948 295


a ch: S 18 Trn i Ngha Q Hai B Trng H ni
NGOI NG 24H 296
WWW.NGOAINGU24H.VN

B- PART 3: GII THIU CHUNG


Part 3:
Dng 1: Bi i thoi trong cng ty
Trng tm ca bi hc
Trong cc bi i thoi ca Part 3, dng cu hi v ch c tn sut xut hin cao nht l cc bi
i thoi lin quan ti vn phng, c tnh khong 40% - 50% mi thi TOEIC hng thng. Hy
xem bi i thoi thng gp di y.
Phn tch bi i thoi 060.mp3

Questions 1 through 3 refer to the following conversation.


W: Rodger, the new receptionist, (1) Ms. Maria Cortez, is waiting for you in the lobby.
Nhn vin tip tn mi ang i ngi n ng tin snh.
M: (2) Could you take her over to the conference room? I'll meet you both there in about 10
minutes.
Ngi n ng nh a c Maria Cortez n phng hi ngh.
W: Okay, I'll do that. Is she starting her job today?
Hi xem liu c Mara Cortez c bt u cng vic t hm nay cha.
M: Yes, but (3) shell be trained by Lisa for a week to understand how things are done here first.
Khng bt u cng vic ngay m phi hc nghip v trong mt tun.

Q1 : Who is waiting for the man?


A: A new employee
Nghe ni dung chnh v cn phi suy lun v vn trc tin. Trong cu hi c t nghi vn Who
nn phi on xem ngi ang i ngi n ng l ai.

Q2: What does the man ask the secretary to do?


A: Take the visitor to the conference room
C th ni dung ngi n ng nh th k lm g? s c nhc n. Trong bi i thoi, khi c
cc cu trc nh Could you ? / Please th hy ch cc ni dung xut hin ngay sau .

Q3: What does the man say about Lisa?


A: She will help the new employee with the work.
C th bit ngi ph n c nhc n l Lisa. Khi xut hin tn Lisa cn ch lng nghe
on c cu tr li ca Q3.

Vocabulary
receptionist nhn vin tip tn conference hi ngh
lobby hnh lang, tin snh start the job bt u cng vic
take ~ to a ~ vo

Bi tp 1: 061.mp3
Nghe bi i thoi nhiu ln ri in vo ch trng.

M: Hi, Sally. Are you attending (1) ________________________ Friday? Its at 11 oclock.
Dont be late.
ng k hc: 0962 60 8801 04 6260 3948 296
a ch: S 18 Trn i Ngha Q Hai B Trng H ni
NGOI NG 24H 297
WWW.NGOAINGU24H.VN

W: Friday? You mean the day after tomorrow? I thought (2) ______________________ because
the president is (3) ______________________.
M: Well, he decided to have the meeting since many staff members are (4)
__________________________ next week. So he will be back tonight.
W: Okay, Im (5) __________________________ from London branch at 11 oclock. So Ill be
at the meeting about 11:30.

Vocabulary
attend tham d go on vacation i ngh
quarterly meeting cuc hp hng qu expect mong i
be pushed back c di li branch vn phng chi nhnh (= branch
business trip chuyn i cng tc office)
staff nhn vin

Ghi ch:
(1) Ch cu hi c cc p n c din t trong 1-2 t.
Nhng ngi mi bt u hc s gp kh khn khi phi c cu hi v cc cu tr li c trnh by
trn giy trc, sau mi nghe bi i thoi. Nhng i vi trng hp ca hnh thc cu tr li
c din t trong 1-2 t th tt nht nn xem trc cu tr li ri mi nghe bi i thoi. Trong
trng hp p n la chn c th hin di dng cu th cc bn cn luyn tp nghe ton b bi
i thoi tm ra c p n. Qua tham kho, nhng cu hi c p n c din t bng 1-2 t
a s u c kh thp do cc bn hy tp trung tr li nhng cu nh vy.

Ch 1 062.mp3

BC 1: Trc tin lng nghe bi i thoi.

BC 2: Kim tra p n.
1. The first speaker is probably [an office manager / a receptionist].
2. The man wants to [get a job / hire someone].

BC 3: Xc nh ni dung.
W: (1) Welcome to Global Tech Inc. How can I help you?
M: Im here to meet with Sophia White for (2) a job interview.
W: Oh I see. Wait a moment, please. (3) Ill call up to Ms. Whites office. Could you please let
me have your name?
M: Sure. My name is Craig Powell. I have a 1 oclock appointment.

Vocabulary
office manager trng phng job interview phng vn cng vic
receptionist nhn vin l tn call up to gi in
hire tuyn dng appointment cuc hn

BC 4: Phn tch ni dung.


1. Ngi ni u tin c th l [trng phng/ tip tn].

ng k hc: 0962 60 8801 04 6260 3948 297


a ch: S 18 Trn i Ngha Q Hai B Trng H ni
NGOI NG 24H 298
WWW.NGOAINGU24H.VN

ang h ngh nghip ca ngi ni th nht, do vy phi lng nghe k t vng suy ra ngh
nghip. C gi tip n khch v da vo t Inc. ta c th on c y l trng phng hay tip
tn.
2. Ngi n ng mun [tm vc/ tuyn ngi].
Nghe k li ni ca ngi n ng suy ra iu ng ta cn l g. Mc ch chi tit s c lm
r bng ni dung sau cu ni Im here to meet with ...

Ch 2 063.mp3

BC 1: Trc tin lng nghe bi i thoi.

BC 2: Kim tra p n.
3. [Jonathan/ Lisa] will be transferred to Taipei office.
4. They will prepare a [birthday party / farewell party].

BC 3: Xc nh ni dung.
M1: I heard that (3) Jonathan is transferring to Taipei office in a month.
M2: Really? I havent heard about it yet. Who did you hear it from?
M1: Lisa told me that she had overheard the manager saying something about it.
M2: Well. I think we should have (4) a farewell party for him.

Vocabulary
be transferred to c chuyn sang sang in a month trong mt thng
prepare chun b overhear tnh c nghe c
farewell party tic chia tay manager trng phng

BC 4: Phn tch ni dung.


3. [Jonathan/ Lisa] s c chuyn n vn phng i Bc.
Nhiu trng hp c th on ra cu tr li t cu thoi ca ngi th nht trong bi i thoi. Do
vy trong phn 3, ch lng nghe khng b l cu thoi u tin. Trong bi i thoi c
cp trc tip n vic Jonathan chuyn i, cn Lisa l ngi nghe c tin .
4. H s chun b [tic sinh nht / tic chia tay].
Ton bi i thoi ni v vic chuyn cng tc ca Jonathan v trong ni dung chnh c ni n
ba tic chia tay nn c th tm cu tr li mt cch d dng.

ng k hc: 0962 60 8801 04 6260 3948 298


a ch: S 18 Trn i Ngha Q Hai B Trng H ni
NGOI NG 24H 299
WWW.NGOAINGU24H.VN

Bi tp 2:
Cu hi kim tra 065.mp3
1. What does the woman want? 6. What does the woman offer to do?
(A) A supervisor's phone number (A) Revise a schedule
(B) Where supplies are located (B) Write a note
(C) Promotion approval (C) Review a document
(D) The location of the warehouse (D) Extend a deadline

2. Where is the department head? 7. Who is the woman most likely talking to?
(A) At home (A) Her supplier
(B) In her office (B) Her co-worker
(C) In the supply room (C) Her neighbor
(D) In the print shop (D) Her customer

3. What does the man suggest the woman do? 8. What does the man say about the software
(A) Make a purchase order installation?
(B) Speak with Emiko (A) There is a technical problem with it.
(C) Buy a new phone (B) It has to be completed by tomorrow.
(D) Get a receipt (C) It will be discussed at a conference.
(D) He wants it done on his computer.
4. What are the speakers discussing?
(A) A printing error 9. What does the woman suggest the man do?
(B) A staff meeting (A) Meet Richard
(C) A new program (B) Get permission
(D) A travel itinerary (C) Talk to her supervisor
(D) Apply for a job
5. What does the woman ask the man to do?
(A) Cancel a flight
(B) Attend a seminar
(C) Proofread the seminar schedule
(D) Call a publisher

BI TP KIM TRA
Bi tp 1: Nghe bi i thoi, sau tr li cu hi
100.mp3
1. Where does the man probably work?
(A) At the post office
(B) In the payroll department
2. What will the man probably do next?
(A) Send the womans paycheck again.
(B) Call the woman's new apartment
3. What are the speakers talking about?
(A) A stalling shortage.
(B) Law wages.
4. What can be inferred about the speakers?
ng k hc: 0962 60 8801 04 6260 3948 299
a ch: S 18 Trn i Ngha Q Hai B Trng H ni
NGOI NG 24H 300
WWW.NGOAINGU24H.VN

(A) They want to postpone the deadline.


(B) They have to work late for a while.
5. Where is Mr. Wilson now?
(A) In Seoul.
(B) In New York.
6. When will the meeting most likely take place?
(A) Wednesday.
(B) Friday.
7. How many times has the woman redone the design?
(A) 5 times.
(B) 6 times.
8. Why is IPD Magazine significant to them?
(A) It is the biggest account.
(B) It is the important supplier.
9. What does the man want?
(A) To buy new computers.
(B) To replace his cell phone.
10. What does the man finally suggest?
(A) To go on a vacation.
(B) To buy from a different company.

Bi tp 2: Nng cao kh nng c bn | Luyn tp paraphrasing


Lm quen vi cm gic thi tht 101.mp3
Nghe k bi i thoi v tr li cu hi.
1. What is being discussed?
(A) A shareholder conference
(B) A staff meeting
(C) A training program
(D) Money management
2. What did the man say about the information he received?
(A) It raised many questions.
(B) It was practical.
(C) It was detailed.
(D) It was predictable.
3. What does the man want to do?
(A) Increase productivity
(B) Reduce costs
(C) Improve quality
(D) Start a new project
4. What are the speakers discussing?
(A) A repair work

ng k hc: 0962 60 8801 04 6260 3948 300


a ch: S 18 Trn i Ngha Q Hai B Trng H ni
NGOI NG 24H 301
WWW.NGOAINGU24H.VN

(B) A floor arrangement


(C) A new building
(D) A special notice
5. What does the woman ask the man about?
(A) The number of workers
(B) The weekend schedule
(C) The basement lighting
(D) The meeting time
6. What does the man ask the woman to do?
(A) Send a memo
(B) Call the contractor
(C) Close the facility
(D) Inform maintenance staff
7. Why is the man asking about his phone?
(A) He wants to make a call, but is unable to.
(B) He is trying unsuccessfully to answer a call.
(C) It doesnt appeal to be working properly
(D) He is inquiring about a part replacement.
8. Where is this conversation likely taking place?
(A) In the expensive restaurant.
(B) In the rental vehicle
(C) In the office
(D) In the meeting room
9. What does the man finally decide to do?
(A) Order the necessary supplies
(B) Talk to the person that is holding on his line
(C) Ask his co-worker to take a message
(D) Take the rest of the afternoon off

C- PART 7: TH TN V TH IN T (LETTERS AND EMAILS)

CH 1: Trc tin hy nm vng cu trc ca bi c

1. Th tn (Letter)

Ms. Imai 1934 Main St. 1


Binghamton 3Z492B 2
December 10 3

Mr. Park 4
Reader's Book Club 5
1756 East St. 6
New Camden 6C721L
ng k hc: 0962 60 8801 04 6260 3948 301
a ch: S 18 Trn i Ngha Q Hai B Trng H ni
NGOI NG 24H 302
WWW.NGOAINGU24H.VN

Dear Mr. Park, 7

Im writing to express my dissatisfaction with your service. 8


The delivery of the books is usually late. I want the books to be delivered on time.

Sincerely, 9
Masako Imai 10
Masako Imai
1. Tn ngi gi
2. Tn cng ty v a ch
3. Ngy gi
4. Tn ngi nhn
5. Tn cng ty
6. a ch
7. Li cho (vit theo thi quen)
8. Mc ch v l do vit th (thng c trnh by u bi vit)
9. Li cho (vit theo thi quen)
10. Chc v hoc tn ngi gi
Bi tp 1: Cu hi Quiz
1. Ngi nhn l ai? ___________________________
2. Ngi gi l ai? ___________________________
3. L do gi th l g? ___________________________

Vocabulary
express (v) by t late (adj, adv) tr
dissatisfaction (n) s khng hi lng on time ng gi
usually (adv) thng thng

2. Th tn (Letter)

To : All managers 1
From : Linda Palmer, Human Resources Directo 2
tn chc v
Subject : Upcoming employee seminar 3
Date: July 10 4
I'm writing to set up a meeting to discuss the details of the employee seminar. 5
I think that this Friday will be perfect to have the meeting.
Please forward your opinions as soon as possible.

ng k hc: 0962 60 8801 04 6260 3948 302


a ch: S 18 Trn i Ngha Q Hai B Trng H ni
NGOI NG 24H 303
WWW.NGOAINGU24H.VN

1. Ngi nhn
2. Ngi gi
3. Ch
4. Ngy gi
5. Trnh by mc ch vit th
(thng trnh by u bi vit)

Bi tp 2: Cu hi Quiz
1. Ngui nhn l ai? ___________________________
2. Ngi gi l ai? ___________________________
3. L do gi th l g? ___________________________

Vocabulary
upcoming (adj) sp ti
set up (v) t chc
discuss (v) tho lun
detail (n) chi tit
perfect (adj) rt tt, tuyt vi
forward (v) gi
opinion (n) kin
as soon as possible cng sm cng tt

ng k hc: 0962 60 8801 04 6260 3948 303


a ch: S 18 Trn i Ngha Q Hai B Trng H ni
NGOI NG 24H 304
WWW.NGOAINGU24H.VN

CH 2: Nm vng cc dng cu hi thng gp

1. Nhng cu hi v ngi gi
- Who sent this e-mail?
- Who wrote this letter?

Gi tm t kha:
Th tin: Tm thng tin (tn, chc v, a ch) v ngi gi phn u v phn cui ca bi c.
E-mail: Tm phn cui ca bc th hoc mc ngi gi (From: ...).

2. Nhng cu hi v ngi nhn


- Who is this letter intended for?
- To whom is the e-mail intended?

Gi tm t kha:
Th tn: Tm thng tin ca ngi nhn bn dui thng tin ca ngui gi.
E-mail: Tm trong mc (To: ...) trnh by v ngi nhn.

3. Nhng cu hi v mc ch vit th
- What is the purpose of this e-mail?
- Why was this letter written?
- Why was this letter sent to Ms. Kim?
- What is the main purpose of this letter?

Gi tm t kha:
Cc cu hi v mc ch vit th a s c cu tr li nm phn u ca bi c.
c bit l mc ch vit th hay e-mail u th hin r rng bng cm t Im writing to do ~.

Bi tp 3:
Questions 1 ~ 3 refer to the following letter.
Ms. Sara Emma, President
Value Construction Company
1547 Remington St.
Austin, TX.
Dear Ms. Emma,
On behalf of the entire staff of our company, I would like to express our thanks for your p successfully
completing the reconstruction of our headquarters building. You have finished the building on July 7 as
you promised. Thanks!
Sincerely yours,
Jimmy Carnell
Jimmy Carnell
CEO, Pacific Holdings Co.

1. What is the purpose of this letter?


(A) To inform staff change
(B) To express gratitude

ng k hc: 0962 60 8801 04 6260 3948 304


a ch: S 18 Trn i Ngha Q Hai B Trng H ni
NGOI NG 24H 305
WWW.NGOAINGU24H.VN

(C) To inform schedule change


(D) To express dissatisfaction
2. When was the construction finished?
(A) Last week
(B) August 20
(C) Last year
(D) July 7
3. To whom is this letter intended?
(A) The CEO of Pacific Holdings Co.
(B) Employees at Pacific Holdings Co.
(C) The President of Value Construction Company
(D) Jimmy Carnell
Questions 4 ~ 6 refer to the following email.

To: All employees


From: James Morgan, Facilities Manager
Re: Company Parking Lot
Date: June 18
This is a reminder of the upcoming maintenance work on the company parking lot. The parking lot will be
closed for the work. The maintenance work will begin tomorrow morning at 9:00. The work will be
completed by 5:00 P.M.
Thanks for your cooperation.

4. What is the main purpose of the e-mail?


(A) To inform maintenance work
(B) To ask suggestions
(C) To respond to a letter
(D) To introduce a new company policy
5. When will the work be finished?
(A) Today
(B) June 18
(C) Tomorrow afternoon
(D) Tomorrow morning
6. Who sent the e-mail?
(A) The CEO
(B) Maintenance office
(C) Employees
(D) Facilities Manager

Questions 7 ~ 11 refer to the following job advertisement and e-mail.

CUSTOMER SERVICE REPRESENTATIVES NEEDED


We are looking for full-time customer service representatives to work at our Customer Service Center in
our headquarters.
Job Descriptions:
Primary duties are to answer customer questions.
And also provides advice on problems regarding orders.

ng k hc: 0962 60 8801 04 6260 3948 305


a ch: S 18 Trn i Ngha Q Hai B Trng H ni
NGOI NG 24H 306
WWW.NGOAINGU24H.VN

Qualifications:
Excellent communication skills
Ability to use personal computers
Applicants must be able to work any shift, including weekends.
To apply, send a rsum and cover letter to: recruits@ABCmart.com

To: Recruits
From: Denis Ram
Subject: Customer Service Representative
Date: September 20
Im writing to apply for the advertised position. Im really interested in working as a customer service
representative. Although I have no previous experience in the field, I will do my best.
I enclosed my resume and cover letter.

7. Which position is being advertised?


(A) Sales representative
(B) Customer service representative
(C) Accountant
(D) Computer technician
8. What is NOT stated as requirements for the position?
(A) College degree
(B) Communication skills
(C) Ability to work weekends
(D) Computer skills
9. What is the main duty of the position?
(A) To respond to questions
(B) To support sales personnel
(C) To assist managers
(D) To prepare for a meeting
10. Why Denis Ram wrote the letter?
(A) To request a day off
(B) To apply for a position
(C) To explain his career
(D) To answer questions
11. The word enclosed in line 4 is closest in meaning to
(A) closed
(B) terminated
(C) detailed
(D) included

BI TP KIM TRA
Bi tp 1: Read the following letter and answer the questions
Max Office Supplies
433 Western Lane
New York, NY 10987
Dear Ms. Johnson:

ng k hc: 0962 60 8801 04 6260 3948 306


a ch: S 18 Trn i Ngha Q Hai B Trng H ni
NGOI NG 24H 307
WWW.NGOAINGU24H.VN

It was a pleasure meeting you at the interview. I enjoyed our conversation so much, especially hearing
about the needs of the Marketing Department.
I would appreciate it if you could send me a letter concerning your hiring decision. Thank you again for
your time, and I am looking forward to hearing from you soon.
Sincerely,
Jane Omaley
especially appreciate concerning
1. By whom was the letter sent?
(A) A job applicant
(B) An interviewer
2. What is Ms. Johnson asked to do?
(A) Arrange a meeting
(B) Send a letter about a decision

ng k hc: 0962 60 8801 04 6260 3948 307


a ch: S 18 Trn i Ngha Q Hai B Trng H ni
NGOI NG 24H 308
WWW.NGOAINGU24H.VN

Bi tp 2: Practice paraphrasing the following sentences by choosing the correct option.


01 We assure you that your order will arrive within seven days.
= Delivery is to take place within a specific time.
(A) likely (B) guaranteed
02 I send a copy of your flight itinerary with my letter.
= please find the information on your travel plans.
(A) Enclosed (B) Revised
03 I have attached the technical specifications for the AC20 to this e-mail.
= A list of the product's_ _ is included with the e-mail.
(A) costs (B) details
04 Thank you for finding the data that I had asked you about.
= I appreciate your taking the time to the requested information.
(A) locate (B) fill out
05 Schedule your appointment a week before you want to have your car checked.
(A) Appointments for car maintenance should be made a week in advance.
(B) You should check your car service appointment one week ahead or time.
06 Please give me information about the conference facilities at your hotel.
(A) I would like some details about the meeting rooms at your hotel.
(B) I want to have a meeting about the services available at your hotel.

07 Improperly disposing of motor oil can contaminate drinking water.


(A) Motor oil must be disposed of in specific areas.
(B) Disposing motor oil inappropriately can pollute drinking water.

08 Mr. Sigh Is in charge of organizing the reception for the overseas investors.
(A) Mr. Sigh takes responsibility for planning a welcome party,
(B) Mr. Sigh is the receptionist for the overseas branch.

Question 9 refers to the following letter.


Dear Ms, Jackson,
Our law firm has recently decided to move to a more spacious office due to an increase. In the
number of employees In the last year. Additional furniture, office supplies, arii equipment are needed
for our new location. As we were very satisfied with the qual goods and services you have provided
in the past, we look forward to working with you to furnish our new office. I would appreciate your
giving us a cost estimate for the iter mentioned in the enclosed document.
09 What is enclosed in the letter?
(A) A product catalog (B) A list (C) A quote
Question 10 refers to the following e-mail.
FROM: Christopher Knight Personnel Manager TO: All staff
This e-mail is to notify staff that all data contained in the personnel information files is confidential.
It should only be shown to authorized executives. Reviewing or sharing any information you find
ng k hc: 0962 60 8801 04 6260 3948 308
a ch: S 18 Trn i Ngha Q Hai B Trng H ni
NGOI NG 24H 309
WWW.NGOAINGU24H.VN

with a third party is strictly forbidden and could result in termination of employment If you are given
access to a file by mistake, please report it to me immediately.
10. What must one do if one receives a confidential file accidentally?
(A) Contact the technician
(B) Send it to the supervisor
(C) Inform Mr. Knight
Questions 11-12 refer to the following letter.
Jones
Viking Wireless Lexington New York.
Dear Mr. Jones,
For the past several months, my cell phone bill has been much higher than usual. So, I contacted the
customer service department to find out why. It turns out that there are many *?
phone calls that I did not make on bill. However, the customer service insists that these calls were
made from my cell phone.
But many of the calls were made when I was out of the country Enclosed is a copy of my plane ticket
to prove that I was not in the country. I have also attached a copy of my recent bills and highlighted
the calls that I did not make.
Please reimburse me for the incorrect charges and send a statement that reflects the changes. Thank
you for your attention to this matter.
Susan Hay
11 What is the purpose of this letter?
(A) To ask about a phone service plan
(B) To praise the customer service
(C) To exchange a defective phone
(D) To complain about a billing issue
12 What does Ms. Hay request the company do?
(A) Refund the extra charges
(B) Change her service plan
(C) Contact the travel agency
(D) Update her subscription

Bi tp 3:
Questions 1 -2 refer to the following e-mail.
TO: editor_NewYork@newyrork.net
FROM: Brian Simpson (simpson@googler.com)
SUBJECT: The article regarding PK Co.
To the editor,
I am writing about the article entitled PK Co. Negative Sales. My name is Brian Simpson, and I am
the public relations director at PK Co. After my coworkers and I read your article, we found some
incorrect information.
First of all, you wrote that we are planning to lay off 30% of our staff members this coming August.
However, that has not been decided yet. Also you wrote that our sales decreased 10% in the first
quarter of the year. This is not true.

ng k hc: 0962 60 8801 04 6260 3948 309


a ch: S 18 Trn i Ngha Q Hai B Trng H ni
NGOI NG 24H 310
WWW.NGOAINGU24H.VN

We would like you to publish a correction to the article immediately, and we need an official apology
to our company.
Thank you,
Brian Simpson
1. Why was this e-mail written?
(A) To ask an editor for some advice
(B) To request some correction be made
(C) To apologize to the editor for the artice
(D) To introduce some new staff members
2. Who is Brian Simpson?
(A) The executive official at PK Co
(B) The CEO of PK Co.
(C) The writer of an article.
(D) A publisher.

ng k hc: 0962 60 8801 04 6260 3948 310


a ch: S 18 Trn i Ngha Q Hai B Trng H ni
NGOI NG 24H 311
NGY 19: WWW.NGOAINGU24H.VN
PART 3: CC DNG BI
PART 7: CH QUNG CO

A- PART 3:

Dng 2: Bi i thoi khi mua sm


Trng tm ca bi hc
Trong cc bi i thoi ca Part 3, ni dung lun xut hin trong thi mi thng l bi i thoi
lin quan n mua sm. Hy xem xt cc bi i thoi thng gp v cc t vng c lin quan n
mua sm.
Phn tch bi i thoi 066.mp3
Questions 1 through 3 refer to the following conversation.
M: (1) I'd like to buy some shirts.
Ngi n ng mun mua o s mi.
W: Sure, what style are you most interested in?
Nhn vin bn hng hi ngi n ng thch kiu no.
M: (2) I like dark colors. Do you have any?
ng y tr li l thch kiu s mi mu ti.
W: I can show you some of those over here, (3) please follow me.
Nhn vin bn hng ni rng o s mi mu ti hng kia v ngh dn ngi n ng i
n .
Q1 : Where are the speakers most likely?
A : In a department store
Trng hp cu hi hi v a im, c khi cu tr li xut hin ngay trong cu thoi u tin.
Nhng cng nn ch cc t kha khc quyt nh p n ng. do nn c cu hi nh
hng thong tin cn nghe. V d nh nghe c cc t shirts/ style/ color th c th bit l ca
hng bch ha.

Q2 : What is the man concerned about?


A : Color
Khi nghe c nhn vin hi ng thch kiu s mi no? th ngi n ng tr li Ti thch mu ti
c th bit c p n ca Q2 l mu sc.

Q3 : What does the woman ask the man to do?


A : Come with her
Cm t come with her y c dung theo ngha ging vi follow me.

Vocabulary
would like to do mun ~ be interested in thch, quan tm
dark color mu ti follow i theo
over here hng kia department store ca hng bch ha

Bi tp 1: 067.mp3
Nghe bi i thoi nhiu ln ri in vo ch trng.

ng k hc: 0962 60 8801 04 6260 3948 311


a ch: S 18 Trn i Ngha Q Hai B Trng H ni
NGOI NG 24H 312
WWW.NGOAINGU24H.VN

M: Excuse me. I bought this book yesterday and (1) ______________________________ at the
end of the book. (2) ______________________________________?
W: You certainly can. Do you have (3) _____________________________________________?
I need to see (4) _______________________________________.
M: No, Im afraid not. I dont have the receipt (5)
____________________________________________.
W
M: Oh, Im terribly sorry for the inconvenience but, you can only get a full refund if you have
the receipt.
Vocabulary
miss mt, tht lc purchase vic mua hng
get a refund c hon li tin terribly rt, thc s
original gc inconvenience s bt tin
receipt giy bin nhn full refund s tin hon tr y
proof bng chng

Ghi ch:
(2) Cc t / cm t ng ngha
Cc p n ca cu hi trong Part 3 c chia thnh 2 loi:
1. Ly t vng trong bi i thoi a vo p n.
2. Chn t c cch din t khc nhng c cng ngha vi cc t c trong on i thoi. V d
nh trng hp trong bi i thoi nghe l please follow me v trong p n c cm t tng t
l come with her. Vic ny c gi l paraphrasing.
Nu va nghe bi i thoi va nm c cc yu t ng ngha vi nhau th s c kh nng t
c im s cao.

Ch 1 068.mp3

BC 1: Trc tin lng nghe bi i thoi.

BC 2: Kim tra p n.
1. The newspaper is [on the printer / in the employee lounge].
2. Sophia suggests the man [go to the electronics store / access the Internet].

BC 3: Xc nh ni dung.
M: Sophia, have you seen the yesterdays newspaper? I placed it on the printer before I left the
office yesterday. I want to look at an ad in the paper again.
W: (1) I saw it on the table in the employee lounge this morning. What is the ad about?
M: I need a fridge. The new electronics store advertised 30% off in every item.
W: Well, (2) I think you should search on the Internet first. I can show you a website where you can
get a much better deal.

Vocabulary
newspaper t bo employee lounge phng ngh dnh cho nhn
place t, vin
printer my in fridge t lnh
leave ri khi electronics store ca hng in t
ad mu qung co advertise qung co
ng k hc: 0962 60 8801 04 6260 3948 312
a ch: S 18 Trn i Ngha Q Hai B Trng H ni
NGOI NG 24H 313
WWW.NGOAINGU24H.VN

offer tng search tm kim


item mn hng deal s tho thun

BC 4: Phn tch ni dung.


1. T bo [trn my in / trong phng ngh ca nhn vin].
Ch nghe thng tin v v tr ca t bo. Ngi n ng nhc n t printer nhng l by. Ch
n li ca c gi c
nhc n phng ngh dnh cho nhn vin.
2. Sophia ngh ngi n ng [i n ca hng in t / tm kim trn mng].
Cn ch nghe k cu ni ca Sophia v ngi n ng nhc n ca hng in t, tuy nhin
c gi ni l you should search on the Internet. p n thay th t search bng t khc c
ngha tng t l access.

Ch 2 069.mp3

BC 1: Trc tin lng nghe bi i thoi.

BC 2: Kim tra p n.
3. The first speaker is looking for [some seafood / some meat].
4. The second speaker will [help the first speaker locate an item / pay for the cashier].

BC 3: Xc nh ni dung.
W1: Excuse me. (3) Which aisle is frozen shrimp in?
W2: l believe that its in aisle 6 or 7, the frozen food section.
W1: Well, I just looked in the frozen food section but I didnt see it.
W2: Oh, it must be in the seafood section then. That would be the first aisle by the meat corner. I
have to buy some chicken. So, (4) Ill show you if you follow me.

Vocabulary
aisle li i, gian seafood hi sn
frozen ng lnh meat corner gian hng tht
shrimp tm follow i sau, theo sau
section ngn

BC 4: Phn tch ni dung.


3. Ngi ni th nht ang tm [cc mn hi sn / tht].
Ngi ni th nht hi Which aisle is frozen shrimp in? nn c th on ngi ang tm cc
mn hi sn. Thng qua vic ngi ni th hai cp n chicken c th bit meat l th ngi
th hai ang tm.
4. Ngi ni th hai s [gip ngi th nht tm hng ha / tnh tin].
Trong cu thoi ca ngi ni th hai c cu tr li. Cm t Ill show you ca ngi ni th
hai c din t li thnh help ~ locate (gip tm v tr).

ng k hc: 0962 60 8801 04 6260 3948 313


a ch: S 18 Trn i Ngha Q Hai B Trng H ni
NGOI NG 24H 314
WWW.NGOAINGU24H.VN

Bi tp 2: 071.mp3

1. Who most likely is the woman? 6. Why does the man say he is not interested?
(A) A newspaper reporter (A) He likes a different model.
(B) A delivery person (B) He wants a bigger car.
(C) A cashier (C) He is concerned about gas.
(D) A restaurant waitress (D) He cannot afford the price.

2. What did the man learn about? 7. What is the man selling?
(A) Where he can find a bed (A) Newspapers
(B) When a sale will begin (B) A motorcycle
(C) Whether he can use coupons (C) Accident insurance
(D) What products are available (D) Gas

3. What is inferred about the mans next visit? 8. What does the woman want to do?
(A) He will pay for tax. (A) Check the size
(B) He will pay by check. (B) Visit the man
(C) He will use a credit card. (C) Avoid an accident
(D) He will buy grocery items. (D) Change the color

4. Who most likely is the woman? 9. What does the man plan to do?
(A) An automobile dealer (A) Work in his yard
(B) A truck manufacturer (B) Go to his office
(C) A car mechanic (C) Prepare his schedule
(D) A van driver (D) Have dinner

5. What does the woman offer to do?


(A) Give a discount
(B) Show various vehicles
(C) Take a test drive
(D) Accept a purchase

ng k hc: 0962 60 8801 04 6260 3948 314


a ch: S 18 Trn i Ngha Q Hai B Trng H ni
NGOI NG 24H 315
WWW.NGOAINGU24H.VN

Dng 3: Bi i thoi trong nh hng

Trng tm ca bi hc
Dng bi i thoi c lin quan ti mn n hay nh hng hu nh u xut hin trong thi TOEIC
mi thng. Hy ch luyn tp cc mu i thoi thng gp v nhng t vng c lin quan n
cuc hn n ung cng nh vic trnh by kin v nh hng v mn n.

Phn tch bi i thoi 072.mp3

Questions 1 through 3 refer to the following conversation.


M: I heard that (1) a new Italian restaurant has opened recently on 14th street? Have you been
there?
Hi l i n nh hng mi khai trng cha .
W: I havent eaten there yet, but Kathy said that the pizzas there are very good.
Ngi ph n tr li l vn cha n nh hng .
M: Thats great! (2) I hope they arent expensive though. I think Ill go there this evening.
Care to join me?
ngh cng i n vo ti nay.
W: Id love to, but (3) Ive promised my daughter that I'd go shopping with her tonight.
Ni rng d mun nhng khng th i cng v c cuc hn vi con gi.
Q1 : What are the speakers discussing?
A : A restaurant
Hi ch ca cuc i thoi. Ni chung khong 80% ch c th hin trong cu thoi u
tin. Ni v nh hng mi, hy xem xt cu: that a new Italian I restaurant's opened recently v hi
ngi ph n i th cha.
Q2: What is the man concerned about?
A: How much a price is
Cu tr li nm trong cu ni ca ngi n ng. Ngi ph n c nghe ni rng pizza rt ngon
v ngi n ng ni l hi vng gi khng mc I hope they aren't expensive though.
Q3: What does the woman plan to do in the evening?
A: Go out with her daughter
Ngi n ng ngh: Care to join me? xem ngi ph n c cng i c khng. Ngi
ph n t chi li mi i n nh hng bng cu: I'd love to, but... v nhc n k hoch i mua sm
cng con gi (I'd go shopping with her).

Vocabulary
restaurant nh hng promise ha
open m, khai trng go shopping i mua sm
recently gn y price gi c
expensive t tin plan to do d nh lm g
join tham gia go out i ra ngoi

Bi tp 3 073.mp3
Nghe bi i thoi nhiu ln ri in vo ch trng.
M: Good evening. (1) _______________________________ choosing what to eat tonight?
W: Thanks. This is our first visit here. (2) ________________________________ something for
us?
ng k hc: 0962 60 8801 04 6260 3948 315
a ch: S 18 Trn i Ngha Q Hai B Trng H ni
NGOI NG 24H 316
WWW.NGOAINGU24H.VN

M: Well. Today, we have a chefs special salad and Sophias gourmet sirloin (3)
__________________ and of course we have a delicious Kim-chi cream soup too.
W: Well, I think we need more time to (4) __________________________________. Could you
bring us (5)______________________________ while we think about that?

Vocabulary
choose chn la sirloin tht thn b
recommend gii thiu delicious thm ngon
chef bp trng soup sp
special c bit get ready sn sng
gourmet ngon, cht lng cao ice water nc lc (lnh)

Ghi ch:
(3) Phi nghe n ht bi xem xt chn cu tr li ph hp.
Trng hp cu hi v thng tin hay k hoch ca ngi ni, khng nn ch nghe ni dung lin quan
n ngi ni ri quyt nh cu tr li. V d trong trng hp ny ngi n ng ni l I think I'll
go to the pizza place this evening. Care to join me?. Nu ngi ph n p li Id love to th hnh
ng ca c y s l cng i n ca hng pizza. Nhng nu ngi ph n ni rng Id love to, but
Ive promised my daughter that Id go shopping with her tonight. th hnh ng ca c y khng
phi l i n ca hng pizza m l i mua sm cng con gi. Nu ch nghe mt phn cuc i thoi
m b st phn ni dung quan trng khc ng vai tr quyt nh n ni dung ca cuc i thoi th
th sinh rt c th s ri vo by v chn cu tr li sai.

Ch 1 074.mp3
BC 1: Trc tin lng nghe bi i thoi.

BC 2: Kim tra p n.
1. The speakers are mainly talking about [health problems / a place to eat].
2. The man will probably have [fish / meat] for his lunch today.

BC 3: Xc nh ni dung.
M: Jessica, (1) have you been to that new Indian restaurant across the street?
W: Sure, I have. They serve excellent dishes. I love the food there. You should try fish dish if you
plan to go.
M: Sounds great. Actually my doctor suggested that I have fish instead of meat.
W: (2) Why dont you go there for lunch special today? Im sure youll like it.

Vocabulary
mainly chnh, ch yu fish dish mn c
health sc khe suggest khuyn
place a im instead of thay cho
Indian (thuc) n lunch special mn n c bit dnh cho ba
across the street bn kia ng tra
serve phc v

BC 4: Phn tch ni dung.


1. Hai ngi ang ni chuyn v [vn d sc khe / a im n ung].

ng k hc: 0962 60 8801 04 6260 3948 316


a ch: S 18 Trn i Ngha Q Hai B Trng H ni
NGOI NG 24H 317
WWW.NGOAINGU24H.VN

ang hi v ch i thoi. Ch ca cuc i thoi c trnh by trong cu thoi u tin.


Ngi n ng m u cu chuyn v nh hng n . Vic ni v bc s gia cuc i thoi
nhm nh lc hng th sinh.
2. Ngi n ng c th s dng [c / tht] cho tra nay.
Ngi n ng bt u cp n bc s v ni rng mnh cn phi n c v ngi ph n ngh
dng mt ba tra c bit, do c th suy ra ngi n ng s n c trong ba tra.

Ch 2 075.mp3

BC 1: Trc tin lng nghe bi i thoi.

BC 2: Kim tra p n.
3. The first speaker is most likely [a waiter / a customer],
4. The first speaker wants to [order food / get the check].

BC 3: Xc nh ni dung.
W: Excuse me. (3) Do you know where the waiter is? (4) Id like to order another glass of wine.
M: The same wine, maam? I'll see if I can find him for you.
W: I've been waiting for more than 15 minutes. And (4) I'd like to order some bread, too.
M: Ill have him come to this table as soon as I find him.

Vocabulary
waiter nhn vin phc v another khc
customer khch hng wine ru
order gi mn have him come gi anh y n
get the check ly ha n as soon as ngay khi

BC 4: Phn tch ni dung.


3. Ngi ni th nht c th l [nhn vin phc v bn / khch].
ang hi ngh nghip ca ngi ni th nht. ng nhm ln vi ngh nghip ca ngi kia.
Ngi ni th nht hi Do you know where the waiter is?" nn c th on c ngi th
nht l khch ti nh hng.
4. Ngi ni th nht mun [gi mn n / ly ha n].
Ngi ni th nht mun bnh m v ru nn c th on l vn cha kt thc ba n. Cu ni
nh i ly ha n l khng c. Lu trong nh hng, ha n tnh tin c gi l check.

Bi tp 4: 077.mp3

1. Who most likely is the man? 6. Why did the woman change her order?
(A) A cashier (A) It was no longer on the menu.
(B) A waiter (B) She is allergic to clams.
(C) A chef (C) The restaurant is out of clams.
(D) A customer (D) That was too expensive.

2. Why doesnt the woman want appetizers? 7. What are the speakers discussing?
(A) She wants to save time. (A) Favorite restaurants in town
(B) She doesnt see anything she likes. (B) Important business mergers
(C) She wants to read the menu further. (C) Client meetings
(D) She wants to pay now. (D) Where the woman eats
ng k hc: 0962 60 8801 04 6260 3948 317
a ch: S 18 Trn i Ngha Q Hai B Trng H ni
NGOI NG 24H 318
WWW.NGOAINGU24H.VN

3. What does the woman want to know? 8. How does the man sound?
(A) Where she can get a table (A) Surprised
(B) When the restaurant closes (B) Excited
(C) Whether a drink is included (C) Confused
(D) Who willserve the food (D) Depressed

4. What was not mentioned in the menu? 9. What does the woman plan to do tomorrow?
(A) Lobster (A) Eat at home
(B) King crab (B) Meet clients
(C) Steak (C) Sign a contract
(D) Shrimp (D) Start a vacation

5. What was the womans final order?


(A) Lobster and clams
(B) Crab and shrimp
(C) Lobster and shrimp
(D) Pasta and clams

IV. Dng 4: Bi i thoi trong ngn hng

Trng tm ca bi hc
Dng i thoi ny lin quan ti nghip v ngn hng v hu nh lun xut hin trong cc k thi
TOEIC hng thng. Hy nm vng hnh thc cng nh dng t vng c lin quan n nghip v
ngn hng.

Phn tch bi i thoi 078.mp3


Questions 1 through 3 refer to the following conversation.

W: (1) Bens computer terminal doesnt work properly. (2) It looks like a problem with the new
software.
Ni rng phn mm my tnh ca Ben trc trc nn gp kh khn khi lm vic.
M: So will that counter be closed down for the day?
Hi nu nh vy c phi ng ca quy giao dch hm nay khng.
W: Yes. I think the other tellers will be asked to do his work.
Ni rng v khng th sp xp cng vic nn nhng nhn vin khc phi lm cng vic ca
anh y.
M: It might be a busy day today. Just let me know (3) if you need any help with any of the
transactions.
Ni rng nu cn gip th hy cho anh y bit.

Q1 : Who most likely are the speakers?


A : Bank clerks
Qua cc t counter / transaction / tellers c th on c y l cuc i thoi ca cc nhn vin
ngn hng.

Q2 : What problem does Ben have?


A : His computer isnt functioning.

ng k hc: 0962 60 8801 04 6260 3948 318


a ch: S 18 Trn i Ngha Q Hai B Trng H ni
NGOI NG 24H 319
WWW.NGOAINGU24H.VN

Trong cu u tin Bens computer terminal doesnt work properly nu r l my vi tnh ca


Ben hon ton khng hot ng c.

Q3 : What does the man tell the woman?


A : He will be able to help out with some of the work.
Qua cu Just let me know if you need any help c th bit ngi n ng ng gip mt s vic.

Vocabulary
computer terminal my vi tnh trm transaction v giao dch
software phn mm bank clerk nhn vin ngn hng
counter quy giao dch function hot ng
teller nhn vin thu ngn help out gip tm thi

Bi tp 5 079.mp3
Nghe bi i thoi nhiu ln ri in vo ch trng.
W: Excuse me. Can you tell me how the (1)________________________________________ work?
M: You can do all your (2)_________________________________________ over the telephone, 24
hours a day.
W: Thats great. How do I (3)__________________?
M: Just call the bank, key in your (4)____________________ and
(5)_______________________________
available.

Vocabulary
telephone banking (dch v) ngn hng thng qua in thoi
day-to-day hng ngy access truy cp
account ti khon
key in nhp (d liu)
PIN number m s nhn din c nhn (= Personal Identification Number)
option s la chn

Ghi ch:
(4) Cn kt hp nhiu chi tit a ra la chn chnh xc.
Trong Part 3, thnh thong c nhng trng hp p n ca cu hi th nht c trnh by sau p
n ca cu hi th hai. tr li cu hi th nht, hy ch i p n khi nghe ht on i thoi.
Trong nhiu trng hp, cu hi th nht hi v ngh nghip ca ngi ni hoc a im cuc i
thoi ang din ra. Trong trng hp nh vy nn xem qua cc cu tr li cho sn v luyn tp nm
bt ton b ni dung tng hp cc chi tit nghe uc trong cc on i thoi. Cn luyn tp thi
quen ghi nh ni dung tt c cc cu hi trc khi nghe ni dung bi i thoi c th tng hp cc
chi tit v tr li chnh xc cc cu hi c t ra.

Ch 1 080.mp3
BC 1: Trc tin lng nghe bi i thoi.

BC 2: Kim tra p n.
1. The woman's problem is that [her homepage is crashed / she cannot enter a webpage].
2. The woman is looking for [how to revise the information / how to repair her homepage].

ng k hc: 0962 60 8801 04 6260 3948 319


a ch: S 18 Trn i Ngha Q Hai B Trng H ni
NGOI NG 24H 320
WWW.NGOAINGU24H.VN

BC 3: Xc nh ni dung.
W: David, Im looking for some advice on (1) the Montreal Bank site as to (2) how to change my
personal information, but I cant enter.
M: Did you log in? Go to the top of the webpage, then click the space that says log in.
W: Okay, I have done that. Now, (3) how can I change my personal information?
M: Go to the part of the page that says Personal Information. If you have any problems, then let
me know. Ill help you fix the data.

Vocabulary
crash trc trc advice khuyn
enter vo personal information thng tin c nhn
revise chnh sa fix chnh sa
repair chnh sa data d liu

BC 4: Phn tch ni dung.


1. Vn ca ngi ph n l [trang ch ca c y b trc trc / c y khng vo c web].
ang hi v vn ca ngi ph n nn bn cn hng s ch vo li ni ca c ta. Ngi
ph n c nhc ti the Montreal Bank site v ni tip cant enter nn vn y l khng th
vo c trang web.
2. Ngi ph n ang tm kim [cch chnh sa thng tin / cch chnh sa trang ch ca c
y].
C th bit vic ngi ph n mun thay i thng tin c nhn ca mnh thng qua cu how can I
change my Personal information?. Cu ni v trang ch ca c y cha tng c nhc ti.

Ch 2 081.mp3
BC 1: Trc tin lng nghe bi i thoi.

BC 2: Kim tra p n.
3. The man wants to [find an ATM location / deposit some money at the bank].
4. The nearest ATM is [one block away / two blocks away].

BC 3: Xc nh ni dung.
M: Hello. (3) I'm calling to ask for information on ATM location. Could you please direct me to the
nearest ATM?
W: No problem, Sir. Where are you calling from?
M: I'm calling from Harvardway at 10th street, Boston.
W: Okay. The nearest ATM is just (4) one block away on Harvardway and 8th street. Oh, sorry. (4)
Two blocks, not one block.

Vocabulary
ATM my rt tin t ng (= Automated ask for yu cu
Teller Machine) location a im
deposit gi tin vo ngn hng direct hng n, ch ng
block dy nh nearest gn nht
ng k hc: 0962 60 8801 04 6260 3948 320
a ch: S 18 Trn i Ngha Q Hai B Trng H ni
NGOI NG 24H 321
WWW.NGOAINGU24H.VN

BC 4: Phn tch ni dung.


3. Ngi n ng mun [tm v tr ATM / gi tin vo ngn hng].
p n c sn trong cu ni ca ngi n ng. Cu u tin ca bi i thoi Im calling to ask
for information on ATM location lm r l do v sao ngi n ng gi in thoi.
4. ATM gn nht cch y khong [mt dy nh / hai dy nh].
L cu hi v khong cch, c lin quan n con s. Khi nhc n v tr ca my ATM hy ch
lng nghe cc con s v cui on i thoi c gi cho bit v tr ca ATM. Hy ch
tp trung nghe n ht khng chn sai p n.

Bi tp 6: 083.mp3

1. Where is this conversation taking place? 5. What does the customer want to discuss?
(A) At a bank (A) A credit card
(B) At a travel agency (B) A tax return
(C) At a supermarket (C) An investment plan
(D) At an airport (D) A personal loan

2. What does the woman suggest the man 6. When will the man see the customer?
do? (A) As soon as he comes into the office
(A) Travel only to safer places (B) After he finishes a document
(B) Not carry cash (C) Before leaving for the day
(C) Go abroad (D) When the office opens
(D) Fill out a form to get a refund
7. What does the woman need to know?
3. What should the man do if he loses the (A) Account numbers
checks? (B) Mailing addresses
(A) Go to the bank directly (C) Branch phone numbers
(B) Report to the police (D) Package tracking numbers
(C) Call the helpline
(D) Go to the travel agency 8. What problem does the woman mention
about the Washington branch?
4. Who is the man most probably? (A) It is not yet open for business.
(A) An insurance agent (B) She knows no one there.
(B) A counselor (C) Its computers are down.
(C) A banker (D) It doesnt have information.
(D) A stockbroker
9. Who is Mr. Palmer?
(A) A supplier
(B) A client
(C) An executive
(D) A mailman

ng k hc: 0962 60 8801 04 6260 3948 321


a ch: S 18 Trn i Ngha Q Hai B Trng H ni
NGOI NG 24H 322
WWW.NGOAINGU24H.VN

BI TP KIM TRA
Bi tp 1: 103.mp3 104.mp3
1. What was the original plan?
(A) Going to a movie
(B) Going to a dinner
(C) Going to meet the womans cousin
(D) Going to the woman's house
2. Why have the plans changed?
(A) She is hungry
(B) She forgot about a meeting
(C) The womans cousin is in town.
(D) There are no seats at the movies.
3. Who is the man going to see the movie with?
(A) Mary
(B) Robert
(C) By himself
(D) Mary's family

Bi tp 2: Nghe bi i thoi, sau tr li cu hi 105.mp3


1. Why are the speakers going to the city center?
(A) To attend a show
(B) To have dinner
2. How will the speakers got to the city center?
(A) By bus
(B) By ferry
3. Who is Mr. Park talking to?
(A) A magazine salesman
(B) His boss
4. Why does Jean call Mr. Park?
(A) To inform him of the subscription
(B) To inform him of the method of payment
5. When will the woman visit the dinosaur exhibit?
(A) Saturday
(B) Wednesday
6. What did the man say about the exhibit?
(A) The kids love it.
(B) He went there three times.
7. How long is the man going to stay in Edinburgh?
(A) Four days
(B)Five days
8. What does the woman ask the man to do?
(A) Pay with a credit card
(B) Sign his name

ng k hc: 0962 60 8801 04 6260 3948 322


a ch: S 18 Trn i Ngha Q Hai B Trng H ni
NGOI NG 24H 323
WWW.NGOAINGU24H.VN

9. When is the woman available?


(A) At 4 o'clock
(B) At 4:10
10. What does the man say?
(A) The meeting wont take very long
(B) He has time at 4:40.

Bi tp 3: Nng cao kh nng c bn | Luyn tp paraphrasing


Lm quen vi cm gic thi tht 106.mp3
Nghe k i thoi v tr li cu hi
1. Where did Sally go?
(A) Alaska
(B) Junes house
(C) A birthday party
(D) Khalils house
2. What did Sally NOT do on her vacation?
(A) Fishing
(B) Hiking
(C) Going downtown
(D) Skiing
3. What can be assumed about Sally's vacation?
(A) She went there during the winter.
(B) She went there during the summer.
(C) She stayed at her friends house.
(D) She spent her birthday there.
4. Who are the speakers?
(A) Coworkers
(B) Best friends
(C) Sisters
(D) A businessman and a client
5. What is happening to Tyler?
(A) He is leaving to another company.
(B) He is presiding a meeting.
(C) He is going to Australia.
(D) He is going to have a party at his house.
6. What does Julie say she will do?
(A) Come back to the office by Friday
(B) Go to a restaurant on Wednesday
(C) Go to Michigan on Friday
(D) Stop by Tyler's office
7. How many people are expected to attend the reception?
(A) 19
(B) 30
ng k hc: 0962 60 8801 04 6260 3948 323
a ch: S 18 Trn i Ngha Q Hai B Trng H ni
NGOI NG 24H 324
WWW.NGOAINGU24H.VN

(C) 35
(D) 40
8. What does the woman say about the reception?
(A) There isnt enough room for 30 people.
(B) More people are coming than she thought.
(C) Fewer people are coming than she thought.
(D) She is looking forward to it.
9. What can be assumed about last years reception?
(A) The reception wasnt as big as this years.
(B) They had a lot of people last year.
(C) The reception last year was bigger.
(D) The reception last year lasted longer.

Bi i thoi lin quan n mua sm / nh hng


Hc cc mu cu hi thng gp 107.mp3
What is the problem with the clothes?
When did the man buy the pants?
Where most likely are the speakers?
Why is the woman concerned about furniture?
What kind of business most likely is Hanovers?
Where does the woman work?
What does the man say about the dinning area?
What does the customer ask the man to do?

Bi tp 4:
Lm quen vi ti thng gp 108.mp3 109.mp3
1. Who is Kevin?
(A) Wendys friend
(B) Wendy's brother
(C) Wendys boss
(D) Sejoons girlfriend
2. What does Kevin say about Sejoon?
(A) He gets cranky often.
(B) His birthday is on Christmas.
(C) He has children.
(D) He is a nice person.
3. When will Kevin probably go shopping?
(A) Friday
(B) Anytime this week
(C) Anytime next week
(D) Never

Bi tp 5: Nghe bi i thoi, sau tr li cu hi 110.mp3


ng k hc: 0962 60 8801 04 6260 3948 324
a ch: S 18 Trn i Ngha Q Hai B Trng H ni
NGOI NG 24H 325
WWW.NGOAINGU24H.VN

1. What is the conversation about?


(A) A now restaurant
(B) Office work
2. What did the man do last night?
(A) He went to the new restaurant.
(B) He reserved tables at the restaurant.
3. Where is this conversation taking place?
(A) The doctor's office
(B) A leather bag store
4. Why is the bag so cheap?
(A) It is on a big sale.
(B) There is a scratch on the bag.
5. Where most likely is this conversation held?
(A) In a movie theater ticket booth
(B) In a convenience store
6. What are the benefits of paying in cash?
(A) You can get a discount next time you come in.
(B) You can get a coupon.
7. What is wrong with the computer?
(A) The fan has a problem.
(B) It is making strange noises.
8. Under what circumstances can they fix the computer for free?
(A) If the warranty is expired
(B) If it wasnt abused
9. Why has the woman called the store?
(A) To order some products
(B) To check a delivery date
10. What does the man say about the delivery date?
(A) It will be delivered on April 2.
(B) It is scheduled for April 4.

Bi tp 6: Lm quen vi cm gic thi tht 111.mp3


Nghe k bi i thoi v tr li cu hi.
1. Where is this conversation taking place?
(A) In a restaurant
(B) In a shoe store
(C) At a birthday party
(D) At a friends house
2. What is the mans problem?
(A) The shoes have a loose bottom.
(B) The shoes are not what he was looking for.
(C) His friend got him wrong shoes.
(D) He doesnt know how to cook.
ng k hc: 0962 60 8801 04 6260 3948 325
a ch: S 18 Trn i Ngha Q Hai B Trng H ni
NGOI NG 24H 326
WWW.NGOAINGU24H.VN

3. What must you have to get an exchange?


(A) Proof of purchase
(B) The item you have bought
(C) The bag in which the item was in
(D) Identification
4. What is about to take place?
(A) A concert
(B) A big company meeting
(C) An announcement of a new caterer
(D) An opening of a new restaurant
5. Why will the dinner and presentation be switched?
(A) Not enough food is prepared.
(B) The caterer misunderstood the schedule.
(C) The presentation isnt ready.
(D) Not enough people showed up.
6. What will the caterer do?
(A) Make as much food as he can
(B) Make more food for the same price
(C) Make food free of charge
(D) Take his time
7. Where is this conversation taking place?
(A) In a restaurant
(B) In a library
(C) In a classroom
(D) In an office
8. What does the girl ask the man to do?
(A) Call her if he finds the book
(B) Pay for her book
(C) Find her book
(D) Check her out a book
9. Why does the girl need her book?
(A) She needs to study.
(B) She needs it for a project.
(C) She needs to return it.
(D) She has to pay for it.

Bi i thoi lin quan n ngn hng / bu in / giao thng


Hc cc mu cu hi thng gp: 112.mp3
Who is the man speaking to?
Why does the woman want insurance?
What does the man ask about the travelers check?
What will the woman do with her pension?
Where are the speakers?
ng k hc: 0962 60 8801 04 6260 3948 326
a ch: S 18 Trn i Ngha Q Hai B Trng H ni
NGOI NG 24H 327
WWW.NGOAINGU24H.VN

What will they deliver over by the red building?


Why does the man recommend a different location?
What does the man need to do to get a loan?
Bi tp 7: Lm quen vi ti thng gp 113.mp3 114.mp3
1. Who most likely is the man?
(A) An engine driver
(B) A mailman
(C) An Internet web designer
(D) A postal worker
2. What method of shipping did the man recommend?
(A) Train mail
(B) Airmail
(C) Surface mail
(D) E-mail
3. When will the package probably arrive?
(A) Thursday
(B) Friday
(C) Sunday
(D) Next week
Bi tp 8: Nghe bi i thoi, sau tr li cu hi 115.mp3
1. Who are the speakers?
(A) A delivery man and an office employee
(B) A delivery man and his co-workor
2. What does the woman offer to do?
(A) Take the package for Mr. Thompson
(B) Place orders
3. What does the man from the hotel ask?
(A) How he can get to the office
(B) Which train to ride
4. What does the man offer the man in the hotel?
(A) To pick him up from the hotel himself
(B) To send someone to pick him up
5. Where is the conversation taking place?
(A) In a car
(B) On a bus
6. What does the woman suggest that the man do to get to his destination?
(A) Get off at the next stop
(B) Stay on the bus
7. Where does this conversation take place?
(A) In a post office
(B) In a library
8. Why doesnt the man need express mail?
(A) He doesn't have enough money.
ng k hc: 0962 60 8801 04 6260 3948 327
a ch: S 18 Trn i Ngha Q Hai B Trng H ni
NGOI NG 24H 328
WWW.NGOAINGU24H.VN

(B) He isnt in a hurry.


9. Who is the man talking to?
(A) A telemarketer
(B) A bank teller
10. What is the woman asked by the man?
(A) His balance
(B) His account number

Bi tp 9: 116.mp3
Nghe k bi i thoi v tr li cu hi.
1. Where is the man working at?
(A) A telephone company
(B) A shipping company
(C) A heating gas company
(D) A bank
2. What does the man say about the woman's bill?
(A) The price has increased.
(B) The account number has been changed.
(C) The company made a mistake.
(D) The woman might have used a lot of gas.
3. What information is the woman request-ed to give?
(A) Her address
(B) Her account number
(C) Her phone number
(D) Her credit card number
4. Where does this conversation most likely take place?
(A) In front of a persons house
(B) Inside a person's living room
(C) In a restaurant
(D) In a car
5. What is the problem?
(A) Two houses share the same address.
(B) The man brought a wrong order.
(C) The man was late.
(D) The man has no change.
6. What is said about the Johnsons?
(A) They order food often.
(B) They love pizza.
(C) They have a lot of money.
(D) They have a large family.
7. What is this conversation about?
(A) Mr. Colemans package
(B) Wrong address for packages
ng k hc: 0962 60 8801 04 6260 3948 328
a ch: S 18 Trn i Ngha Q Hai B Trng H ni
NGOI NG 24H 329
WWW.NGOAINGU24H.VN

(C) Cathy's credit card limit


(D) Packages in the security office
8. Who is NOT receiving a package?
(A) Mr. Meyers
(B) Cathy
(C) Mr. Johnson
(D) Mr. Lee
9. What does Cathy say she will do?
(A) Pick up the packages herself
(B) Give all the packages to Mr. Lee
(C) Thank Mr. Meyers
(D) Talk to Mr. Lee

B- PART 7: Ch qung co
CH 1: Trc tin hy nm vng cu trc ca bi c
1. Qung co sn phm

Save up to 50% more on your phone bill 1


1. C th nm bt thng tin v sn phm c qung co hay mc ch ca vic qung co.
Unlimited local & long distance calls
Keep your current phone number just $19.99/month. 2
2. Gii thiu c nt c trng ca sn phm c qung co.
This offer applies only to those who use services for at least 6 months. 3
3. Thng tin cn ch hay ni dung v khuyn mi
* Trng hp nh gim gi hay phiu mua hng c quy nh thi gian s dng.

Bi tp 1: Cu hi Quiz
1. Qung co sn phm no? a) Th in thoi b) Dch v in thoi
2. c im ca sn phm? a) Phi thay s mi b) C th gi nguyn s hin ti
3. Ai c th tham gia dch v ny? a) Tt c mi ngi b) Ngi s dng ti thiu 6 thng.

Vocabulary
save (v) tit kim current (adj) hin ti
up to n ~ bill (n) ha n offer (n) qu tng, khuyn mi (v) cung cp
unlimited (adj) khng b gii hn/ hn ch apply to p dng
local (adj) a phng use (v) s dng
long distance call cuc gi ng di at least ti thiu
keep (v) gi

ng k hc: 0962 60 8801 04 6260 3948 329


a ch: S 18 Trn i Ngha Q Hai B Trng H ni
NGOI NG 24H 330
WWW.NGOAINGU24H.VN

2. Qung co tuyn dng

Job openings 1

Human Tech Co. is looking for a supervisor for our Technical Support Department. The department
offers a variety of after-sales advice on hardware to our clients. 2

Job Description: 3
The supervisor will train new and existing employees.
Qualifications: 4
College degree
Two years experience in a supervisory position
Knowledge of computer programs
Contacts: 5
Send a rsum, two letters of reference and a cover letter to Samuel Lima, Human Resources
Manager 6
Human Tech Corporations
1789 Industry Ave.
Los Angeles, CA

1. Cho bit y l qung co tuyn dng


2. a ra thng tin ngn gn v cng vic hoc thng tin gii thiu v cng ty.
* p n cho cu hi v thng tin ca cng ty a s c th tm thy phn u bi c.
3. Gii thiu cng vic/ M t cng vic
4. Yu cu v trnh chuyn mn (=requirements)/bng cp
* Gii thch trnh chuyn mn cng ty yu cu. Tm p n cho cu hi v trnh chuyn
mn phn ny.
5. Thng tin lin h
6. Yu cu v h s
* Thng tin lin h v hng dn v h s xin vic thung xut hin phn cui bi c.

Bi tp 2: Cu hi Quiz

1. Cn tuyn v tr no? a) Trng phng ti chnh b) Nhn vin qun l


b phn t vn k thut

2. Nu c tuyn th s lm cng vic g? a) T vn k thut b) o to nhn vin

3. Yu cu chuyn mn ra sao? a) Bng cp c lin quan b) Kinh nghim.

Vocabulary
look for (v) tm after-sales (adj) hu mi
supervisor (n) nhn vin qun l advice (n) li khuyn
a variety of a dng job description m t cng vic

ng k hc: 0962 60 8801 04 6260 3948 330


a ch: S 18 Trn i Ngha Q Hai B Trng H ni
NGOI NG 24H 331
WWW.NGOAINGU24H.VN

train (v) o to
existing (adj) hin ti
qualifications trnh chuyn mn
degree (n) bng cp
supervisory (adj) gim st
knowledge (n) kin thc
contact (n) lin h
rsum (n) s yu l lch
letters of reference (n) th gii thiu
cover letter (n) th gii thiu (c gi km
vi ti liu khc)

ng k hc: 0962 60 8801 04 6260 3948 331


a ch: S 18 Trn i Ngha Q Hai B Trng H ni
NGOI NG 24H 332
WWW.NGOAINGU24H.VN

CH 2: Nm vng cc dng cu hi thng gp

1. Cu hi v i tng c qung co
- What is being advertised?
- What is this advertisement for?

Gi tm t kha:
a s cu tr li cho cu hi ny u c th c tm thy phn mc (title), l nt c trng ca
qung co. Trng hp khng c mc, i tng qung co c nu r phn u.

2. Cu hi v chng trnh gim gi


- What is being offered?

Gi tm t kha:
Ch cc t gim gi (sale /off) hay t khuyn mi c bit (special offer).

3. Cu hi v v tr cn tuyn hay thng tin cng ty


- Where is the main office of the company?
- In what department is the advertised position?

Gi tm t kha:
C th tm p n trong phn gii thiu cng ty (Company profile) thng c trnh by phn u
qung co.

4. Qung co tuyn dng : Cu hi v yu cu trnh chuyn mn


- What is required for the position?
- What is a requirement for the position?

Gi tm t kha:
C th tm p n trong phn Qualifications hoc Requirements ngha l trnh chuyn mn.

Bi tp 3:
Questions 1 ~ 3 refer to the following advertisement.

Computer Desk Sale


Shop desks at Staples!
Free delivery on orders over $50
To celebrate our 10th anniversary, weve just reduced our prices on a wide selection of furniture items.
Visit our website today and check out the savings now.

1. What is being advertised?


(A) Furniture
(B) Clothes
(C) A delivery service
(D) Educational programs
2. What will happen if customers order items more than $50?
(A) The company will open a new website.

ng k hc: 0962 60 8801 04 6260 3948 332


a ch: S 18 Trn i Ngha Q Hai B Trng H ni
NGOI NG 24H 333
WWW.NGOAINGU24H.VN

(B) The company will deliver items for free.


(C) The company will reduce prices.
(D) The company will give customers a gift certificate.
3. How long has the company been in business?
(A) One year
(B) Five years
(C) Ten years
(D) Twenty years

Questions 4 ~ 6 refer to the following job advertisement.

The Pacific Investment Co. is currently looking for an experienced financial assistant in our Hong Kong
office.

Job Description:
The financial assistant will prepare daily reports for the senior financial advisor.

Qualifications:
College degree in accounting or related field
Knowledge of accounting software
One-year experience in accounting

To apply for the position, please send a rsum and cover letter to Human Resources by May 15.

4. Which position is being advertised?


(A) Sales manager
(B) Financial assistant
(C) Accounting manager
(D) Financial advisor
5. What is stated as a requirement?
(A) Drivers license
(B) Previous experience
(C) A letter of recommendation
(D) Sales and marketing skills
6. Which branch is the advertised position for?
(A) Headquarters
(B) London
(C) Hong Kong
(D) New York

ng k hc: 0962 60 8801 04 6260 3948 333


a ch: S 18 Trn i Ngha Q Hai B Trng H ni
NGOI NG 24H 334
WWW.NGOAINGU24H.VN

Questions 7 ~ 11 refer to the following job advertisement and e-mail.

Weekly Online Survey


Your opinions are valuable to Bestbuy.com.
Click below to answer a few questions about our products and services.
Free delivery coupons will be given to all participating customers.
CLICK here!
Yes, Ill take the survey.

To: Bestbuy.com
From: Judy Carol
Subject: Free delivery coupon
Date: January 30
Im writing to express my disappointment at your service.
Your company promised to provide free delivery coupons if I completed the survey. But I didnt get any
coupons even though I completed it 10 days ago.
I want your company to apologize to me for this error and I would like the coupons delivered as soon as
possible.

7. What is this advertisement for?


(A) Delivery services
(B) Discount coupons
(C) Customer survey
(D) New products
8. What will happen if customers complete the survey?
(A) A gift certificate will be given.
(B) Free delivery coupons will be given.
(C) Discount coupons will be given.
(D) Product samples will be given.
9. What is the purpose of the e-mail?
(A) To appreciate excellent services
(B) To announce an event
(C) To complain about poor service
(D) To suggest an idea
10. When did Judy Carol complete the survey?
(A) A month ago
(B) January 30
(C) A week ago
(D) January 20
11. What did Judy Carol ask the company to do?
(A) Have a meeting with her
(B) Send her the coupons
(C) Give a full refund
(D) Exchange items

BI TP KIM TRA
Bi tp 1: Read the job advertisement below and answer the questions.
Marketing Position at the Sharon Manufacturing Co.
The Sharon Manufacturing Co. is the nations leading producer of home appliances. Currently, we are
looking for a marketer.

ng k hc: 0962 60 8801 04 6260 3948 334


a ch: S 18 Trn i Ngha Q Hai B Trng H ni
NGOI NG 24H 335
WWW.NGOAINGU24H.VN

Requirements:
Bachelors degree
Minimum of three years of experience in marketing
Good interpersonal skills
If you are interested in the job position, please send your resume and cover letter to
applicances@sharon.com by March 1.
home appliances interpersonal skill
1. What is NOT required for the position?
(A) A university degree
(B) Fluency in three languages
2. What should an applicant do to apply for the job?
(A) E-mail a resume and cover letter
(B) Fax a resume and cover letter

ng k hc: 0962 60 8801 04 6260 3948 335


a ch: S 18 Trn i Ngha Q Hai B Trng H ni
NGOI NG 24H 336
WWW.NGOAINGU24H.VN

Bi tp 2: Practice paraphrasing the following sentences by choosing the correct option.


01 The Internet is an inexpensive means of advertising,
= A website is a way to promote your business.
(A) creative (B) low-cost
02 All of our monitors are guaranteed to last more than ten years.
= Our displays are well made and extremely
(A) durable (B) relevant
03 To make up for the drop in profits, the company fired 30 workers last quarter.
= The company employees last quarter.
(A) hired (B) dismissed
04 We are seeking those who have acquired broad experience in the service industry.
= Those who have experience in the hospitality industry will be welcomed.
(A) expensive (B) extensive
05 We offer the most competitive prices in the automobile industry
(A) We provide high-quality and low-price auto transport service.
(B) Compared to most car companies, our prices are quite low.
06 The product will be sold across the country on October 15
(A) The product will be available nationally in the middle of October.
(B) By the second week of October, the product will be sold out.
07 Researchers will primarily work in the office, but some fieldwork will be required
(A) Researchers have to work outside of the office at times.
(B) Researchers are required to do fieldwork after completing their office work.
08 S-Mart stores will now be open 24 hours a day in order to batter serve their customers
(A) To improve service. S-Mart will be open around the clock.
(B) Poor service has led S-Mart to extend its hours.

Question 9 refers to the following advertisement.


Smith Falls Apartments have two spacious bedrooms, a sunny living room, bathroom with both a tufa
and shower, and a fully-equipped kitchen. Our amenities include a swimming pool, a fitness center,
and a Laundromat. There is plenty of parking nearly. The maintenance costs of the complex are
included in the rent. In order to become a tenant, a $2.000 security deposit and a recommendation
from a previous landlord are required.
09 What is being advertised?
(A) A parking area
(B) An apartment complex
(C) A business space

Question 10 refers to the following advertisement.


Alfredos, western Canada's favorite Italian restaurant chain, is looking for regional man- IS agers to
help with its expansion into two new provinces. All regional managers will be based at our corporate
headquarters in Calgary, but will be asked to travel extensively throughout the country. Qualified
applicants must be proficient in both English and J French and understand the market differences
among the various regions of Canada. A minimum of three years of experience in the hospitality
industry is preferred.
10 What is staled as a requirement for the job?
(A) A sense of hospitality
(B) Proof of residence in Calgary
(C) Fluency in two languages

ng k hc: 0962 60 8801 04 6260 3948 336


a ch: S 18 Trn i Ngha Q Hai B Trng H ni
NGOI NG 24H 337
WWW.NGOAINGU24H.VN

Questions 11-12 refer to the following job advertisement.


Quality Control Engineer

The Coleman Co. is searching for a quality control engineer who will provide support and technical
advice regarding all of our manufactured goods. The quality control engineer will be expected to
support the operational division to find solutions for engineering and quality concerns. The quality
control engineer will also be expected to communicate with customers on technical issues.
We require that all candidates applying for this position have a bachelors degree in mechanical
engineering as well as a minimum of five years' experience In the field. Additionally, a successful
candidate will have strong communication skills. Candidates who are able to speak several different
languages will be preferred. A rewarding salary will be offered and will depend upon the candidate's
experience.
11 What is one of the stated job responsibilities?
(A) Giving instructions to the technical staff
(B) Supervising the operational division
(C) Maintaining the factory's facilities
(D) Providing technical information to clients
12 What is NOT a requirement for the job?
(A) Proficiency in a foreign language
(B) A university degree
(C) Related experience
(D) Excellent interpersonal skills

Bi tp 3:
Questions 1-2 refer to the following job advertisement.
Job Openings at Golden Office Supplies
Golden Office Supplies is looking for some experienced managers. All the managers will start to
work in Chicago and will have opportunities to work at other branches in the future. The qualified
candidates will be able to:
manage contracts with suppliers
develop marketing plans
conduct customer surveys

Download the application from our website and fill out all the information. Please send the form with
two letters of recommendation by October 10 to the following address:
Golden Office Supplies
17 Broadway
Chicago, IL 60602
1 What position is being advertised?
(A) Teller
(B) Secretary
(C) Supplier
(D) Manager

2 What is NOT true about the advertisement?


(A) candidate can download an application from the website.
(B) The application should be sent by October 10.

ng k hc: 0962 60 8801 04 6260 3948 337


a ch: S 18 Trn i Ngha Q Hai B Trng H ni
NGOI NG 24H 338
WWW.NGOAINGU24H.VN

(C) The application should be sent by e-mail.


(D) Successful candidates will work in Chicago.

ng k hc: 0962 60 8801 04 6260 3948 338


a ch: S 18 Trn i Ngha Q Hai B Trng H ni
NGOI NG 24H 339
WWW.NGOAINGU24H.VN

NGY 20:
PART 4: GII THIU CHUNG
PART 7: THNG BO
A- PART 4
DNG BI C NI DUNG HNG DN V TIN NHN GHI M QUA IN THOI.
Trong bi ni ca Part 4, ni dung c t ra a s l cc ch nh tin nhn ghi m qua in thoi
v cc ni dung truyn t, thng bo hay hng dn. C t 6 n 9 cu hi c t ra mi thng
v vic phn tch cc cu tr li trc khi nghe ng mt vai tr quan trng. Hn na trong nhiu
trng hp cc bi ni c s dng cc cm t ng ngha vi nhau nn luyn tp paraphrasing l
vic rt cn thit.

Cu hi kim tra nng lc ngn ng 089.mp3 084.mp3

Questions 1 - 3 refer to the following announcement.


Attention, all passengers! (1) The ferry to South Island will be delayed due to bad weather
conditions. (2)The safety of our passengers is the prime concern of all of us here at Victor Feny
Services. So we decided to reschedule all the boarding and departure times. (3) The next weather
update is expected at 10:00 A.M. Please contact our staff at the ferry office if you have any
questions.

Q1 : What has caused the delay?


A : Unpleasant weather
Nu nm c ni dung cu hi s bit bi ni cp n vic tr hon. T cm t due to bad
weather conditions c th bit tu b hon do thi tit xu

Q2 : What is Victor Ferry Services prime concern?


A : Safety of passengers
Nh vo chi tit trong cu hi m c th tm ra p n khi c nhc n mi quan tm ca Victor
Ferry Services. Trong cu The safety of our passengers is the prime concern nhn mnh mi quan
tm hng u ca hng l s an ton ca hnh khch

Q3 : What is expected to happen at 10 oclock?


A : There will be some more information about the weather.
Hi v s vic s xy ra trong thi gian xc nh nn phi nghe k t ch thi im. Qua cu The
next weather update is expected at 10:00 A.M. c th bit c mt s thng tin v thi tit.

Vocabulary
ferry ph concern mi quan tm
be delayed b tr hon reschedule sp xp li
due to do boarding ln tu
bad weather thi tit xu (= unpleasant departure khi hnh
weather) update s cp nht
safety an ton contact lin lc
prime chnh staff nhn vin
ng k hc: 0962 60 8801 04 6260 3948 339
a ch: S 18 Trn i Ngha Q Hai B Trng H ni
NGOI NG 24H 340
WWW.NGOAINGU24H.VN

Bi tp 1: 085.mp3
Nghe bi ni nhiu ln ri in vo ch trng.
Youve reached International Food Bank in Korea,
(1)__________________________________________
Our hours are 9:00 A.M. to 5:00 P.M. (2)____________________________________________ We
are not open on (3)______________________________________________. If you want to get
information about donation, (4)___________________________. If you want to speak to one of our
representatives, please call back during regular hours or
(5)_______________________________________after the tone. Thank you for calling and have a
nice day.

Vocabulary
reach lin lc call back gi in li
international quc t during regular hours trong gi lm vic
national holiday quc l record ghi m
get information ly thng tin message tin nhn
donation vt tng, cng tone ting bp
representative ngi i din

Luyn cch trnh by 1 086.mp3

BC 1: Trc tin lng nghe bi ni.

BC 2: Kim tra p n.
1. People wanting to join the summer event should [visit the homepage / call at a different number].
2. The new store will open in [summer / autumn].

BC 3: Xc nh ni dung.
Thank you for calling the Seoul Department store in Hong Kong. (1) If you are calling about joining
Summer Festival Event, please call back at 02-544-3528 for our Public Relations office. (2) This fall,
the Seoul Department store will open a new store in Taiwan. If you are interested in our Taiwan
store, visit our website www.seoulstore.com.

Vocabulary
department store ca hng bch ha open khai trng
lines are busy cc ng dy in thoi ang store ca hng
bn be interested in quan tm
handle x l visit gh thm
join the event tham gia s kin website trang web
public relations quan h cng chng

BC 4: Phn tch ni dung.


1. Nhng ngi mun tham gia vo s kin ma h nn [vo trang ch / gi vo s khc].
Phi lng nghe nhng ni dung ch dn lin quan n nhng s kin din ra vo ma h. Lng
nghe cc cu trc cu thng gp nh Please /Could you ... / Why don't you .... Chi tit quan
trng l please call back at 02-544-3528 v trong p n c thay bng a different number.
2. Ca hng mi s khai trng vo [ma h / ma thu].
ng k hc: 0962 60 8801 04 6260 3948 340
a ch: S 18 Trn i Ngha Q Hai B Trng H ni
NGOI NG 24H 341
WWW.NGOAINGU24H.VN

Lng nghe thi im v vic khai trng mt a im mi ch khng phi ni hin ti.
Summer l thi im t chc event ch khng phi l thi im khai trng ca hng mi.
Luyn cch trnh by 2 087.mp3

BC 1: Trc tin lng nghe bi ni.

BC 2: Kim tra p n.
3. This announcement is being made [at the theater / at the sports facility].
4. In section A and B, people are allowed to use [fireworks / whistles].

BC 3: Xc nh ni dung.
Attention please! Beer sales are not allowed inside (3) the stadium. And you may not bring canned or
bottled beer into (3) the stadium, either. Our uniformed guards will inspect your package at the
entrance. (4) Whistles and other noisemakers may only be used by persons seated in section A and B.

Vocabulary
beer sales bun bn bia inspect kim tra
be allowed to do c cho php (lm g) package gi hng
inside trong stadium sn vn ng entrance cng vo
canned beer bia lon whistle ci
bottled beer bia chai noisemakers nhng th pht ra m thanh
uniformed mc ng phc seat ngi
guard nhn vin bo v

BC 4: Phn tch ni dung.


3. Thng bo ny ang c pht [ti nh ht / ti khu th thao].
Thng tin v a im thng bo c pht c cung cp phn u nn cn nghe k 2 cu
u tin. Trong 2 cu ny c nhc n vic cm mang bia vo sn vn ng.
4. khu A v B, mi ngi c php dng [pho hoa / ci].
xc nh a im l khu A v B nn kh nng bit c p n t phn ny l rt cao. Trong
phn cui ca bi th hin r l ci v nhng th pht ra m thanh (whistles and
noisemakers) ch c s dng 2 khu A v B.

ng k hc: 0962 60 8801 04 6260 3948 341


a ch: S 18 Trn i Ngha Q Hai B Trng H ni
NGOI NG 24H 342
WWW.NGOAINGU24H.VN

Bi tp 2: 089.mp3

1. What is the purpose of the message? 6. Why is John calling Cindy?


(A) To sell a product (A) He wants to know if the fax machines
(B) To ask for payment are working.
(C) To announce a service (B) The department wants to try out a new
(D) To request a visit product.
(C) He wants some important information
2. When does the store close? from her.
(A) 8:00 P.M. (D) He wants her to make a decision about
(B) 9:00 P.M. supplies.
(C) 10:00 P.M.
(D) 11:00 P.M. 7. Who is the speaker most probably
addressing?
3. What does the speaker ask the listener to use (A) Salespersons
to place an order? (B) Tourists
(A) Directions (C) Students
(B) Opening date (D) Office employees
(C) Purchase requirements
(D) Promotional code 8. What is being offered?
(A) A new insurance plan
4. What problem is mentioned? (B) A retirement benefit
(A) Approval was denied for a request. (C) A new service
(B) A product is not available. (D) A performance incentive
(C) The new supplier is not reliable.
(D) The paper has low quality. 9. When will the offer come into effect?
(A) Today
5. What is said about the new dealer? (B) Tomorrow
(A) His prices are lower. (C) Next week
(B) He delivers orders within 10 days. (D) Next month
(C) His specialty is fax machines.
(D) He contacted the office a short while
ago.

BI TP KIM TRA
Thng bo / Hng dn / Qung co
1. Thng bo 118.mp3
Thank you all for coming to this meeting at such a short notice. (1) I am happy to announce that
Piczell Co. has just given us our next assignment, which is a bigger contract than anything we have
attempted till now. (2) Please give me your initial estimates of the time and staff required for the job,
at the latest by tomorrow evening. (3) We will meet on Wednesday at 10.00 a.m. to finalize dates
and recruitment details, before I meet our clients again. Oh, one more thing. Please join me for a
lunch soon after this meeting to celebrate our success.

(1) Hi v ch ca c bi
Q: What is the topic of the talk?
A: To announce a new contract
ng k hc: 0962 60 8801 04 6260 3948 342
a ch: S 18 Trn i Ngha Q Hai B Trng H ni
NGOI NG 24H 343
WWW.NGOAINGU24H.VN

Dng p n c ni dung trc tip


(2) Hi v cng vic ngi nghe cn phi lm
Q: What are the listeners asked to do by tomorrow
evening?
A: Submit a report with draft estimate
Dng p n paraphrasing
(3) Hi v k hoch sp ti
Q: What will happen on Wednesday?
A: An important meeting will take place.
Dng p n paraphrasing

2. Qung co 119.mp3
Attention, shoppers. While you are stocking up on food for your next barbecue, (1) why not stop by
the cosmetics department and pick up some No-Burn Sun Block? Today it is only seven ninety-nine
for a jumbo 14-ounce tube. No-Bum Sun Block has a sun-protection factor up to forty and is oil (2)
and dye-free so it wont irritate your skin. We have several varieties including waterproof, sweat-
proof, and baby-gentle. (3) So, stop by the cosmetics department and protect your skin from the
summer sun. Thank you for shopping and have a great day!
(1) Hi v ni dung qung co
Q: What is the woman advertising?
A: Skin care products
Dng p n paraphrasing
(2) Hi v gi c, chng loi, c im ca sn phm
Q: What is true about all of the products?
A: They contain no dyes.
Dng p n c ni dung trc tip
(3) Hi v ni dung yu cu hoc cch thc mua bn
Q: What are the listeners being asked to do?
A: Visit the cosmetics section in the store
Dng p n paraphrasing

Bi tp 1: Nghe bi ni, sau tr li cu hi 120.mp3


1. What is being advertised?
(A) TV
(B) A restaurant
2. When is Grandma Jessie's closed?
(A) Saturday
(B) Sunday
3. What will the passengers receive?
(A) A ticket
(B) A free meal
4. What should passengers do?
(A) Go to the ticket counter
(B) Go to the restaurant
5. To whom is this talk directed?
(A) Food company employees
(B) Food mart shoppers
ng k hc: 0962 60 8801 04 6260 3948 343
a ch: S 18 Trn i Ngha Q Hai B Trng H ni
NGOI NG 24H 344
WWW.NGOAINGU24H.VN

6. What is mentioned about Jumbo Hamburger?


(A) It has 147 employees.
(B) It has a market share of 8%.
7. What will the passengers be traveling on?
(A) A plane
(B) A train
8. Who will collect the boarding passes?
(A) The conductor
(B) The ticket agent
9. When does the sale end?
(A) 5th
(B) 15th
10. What is the largest discount being offered during the sale?
(A) 15%
(B) 70%

Bi tp 2: 121.mp3
Nghe k bi ni v tr li cu hi.

1. What is the advertisement about?


(A) A sailboat
(B) A restaurant
(C) A food market
(D) A beach house in Vietnam
2. What is mentioned about Saigon House?
(A) The prices are economical.
(B) It is well known for its service.
(C) The interior is unique.
(D) It is located on the beach.
3. What can be seen in one of the dining rooms?
(A) A sailboat
(B) A map of Vietnam
(C) A couch
(D) A TV set
4. Where will the company picnic be held if it doesn't rain?
(A) Nancy Hadley's house
(B) Greentree Park
(C) San Francisco
(D) Elm Restaurant
5. Who will be providing the food?
(A) Nancy Hadley
(B) The cafeteria staff
(C) A caterer
(D) Greentree Park staff
6. Who will be attending the picnic?
(A) Elm Restaurant employees
(B) Greentree Park patrols
ng k hc: 0962 60 8801 04 6260 3948 344
a ch: S 18 Trn i Ngha Q Hai B Trng H ni
NGOI NG 24H 345
WWW.NGOAINGU24H.VN

(C) Company employees


(D) Community basketball players
7. Why is the boat departing late?
(A) Bad weather
(B) Technical problems
(C) Not all passengers are boarded.
(D) The docks are under construction.
8. What time can the passengers board after the delay?
(A) 4:30
(B) 6:00
(C) 6:30
(D) 8:00
9. What does the announcement ask the passengers to do?
(A) Hold on to their tickets
(B) Return to the boarding area before 6
(C) Wait until 4:30 to ride the boat
(D) Have some snacks while waiting

ng k hc: 0962 60 8801 04 6260 3948 345


a ch: S 18 Trn i Ngha Q Hai B Trng H ni
NGOI NG 24H 346
WWW.NGOAINGU24H.VN

B PART 7:
CH 1: Trc tin hy nm vng cu trc ca bi c

1. Thng bo

NOTICE 1
We invite you to sign up to receive information by e-mail regarding special offers, new services,
and important information from Macy Department Stores. 2

Yes. Id like to receive e-mail updates from Macy Department Stores. 3


(You may stop receiving e-mail at any time by clearing this check box.)
For more information, please visit our website.

1. Thng bo
* Trnh by ni dung thng bo bng mc.
2. Mc ch thng bo
* Thng trnh by mc ch thng bo hoc s kin phn u bi c.
3. Trnh by ni dung mt cch chi tit nh v gii hn, iu khon b sung hay gii thch v ni
dung chi tit ca thng bo.
Bi tp 1: Cu hi Quiz
1. Mc ch ca th thng bo l gi?
a) Mi nhn thng tin bng e-mail b) Thng bo v t gim gi c bit
2. Nu mun nhn thng tin th khch hng phi lm g?
a) Phi tr tin b) Kim tra hp th
3. Thng bo ny dnh cho i tng no?
a) Sinh vin i hc b) Khch hng ca ca hng bch ha

Vocabulary
invite (v) mi update (n) thng tin cp nht
sign up (v) ng k stop (v) dng, ngng
regarding v, v vic at any time vo bt c lc no
special offer khuyn mi c bit clear (v) xa

2. Th bo (Memo hoc Memorandum)


Th bo l hnh thc vn bn c s dng vi mc ch cng vic nh thng bo, hng dn trong
ni b cng ty.

Memo
To : All sales personnel 1
From : Marco Silva, sales manager 2
Subject: Sales promotions 3
Date: June 7 4
Im thinking about having a promotional event for our new digital cameras.
ng k hc: 0962 60 8801 04 6260 3948 346
a ch: S 18 Trn i Ngha Q Hai B Trng H ni
NGOI NG 24H 347
WWW.NGOAINGU24H.VN

If you have any ideas or suggestions for the event, please contact me.
The promotional event will take place on July 30.
Marco Silva

1. Ngi nhn
2. Ngi gi
3. Ch
4. Ngy gi
* Mc ch ca dng vn bn ny c trnh by phn u.
* Trnh by thng tin b sung hoc ni dung chi tit.
* Thng tin v ngi gi xut hin li phn cui.

Bi tp 2: Cu hi Quiz
4. Ngi nhn l ai?
a) Trng phng kinh doanh b) Nhn vin phng kinh doanh
5. Mc ch ca th bo l g?
a) Thng bo thi vic b) Yu cu cho kin
6. S kin g s xy ra vo ngy 30 thng 7?
a) T chc s kin qung co sn phm b) T chc tic v hu
7. Nu mun ngh th phi lm th no?
a) Tham gia s kin b) Lin lc vi Marco Silva

Vocabulary
personnel (n) nhn vin contact (v) lin h
promotion (n) s khuyn mi / qung co take place (v) din ra
think about ngh v ~ suggestion (n) xut

CH 2: Nm vng cc dng cu hi thng gp

1. Cu hi v mc ch thng bo
- What is the purpose of the notice?

Gi tm t kha:
Mc ch ca bn thng bo thng xut hin phn u hoc phn mc.

2. Cu hi v ch v i tng thng bo
- Who issued the notice?
- To whom is this notice intended?

Gi tm t kha:
Mun tr li cu hi v ch hay i tng ca th thng bo th phi c k bi c nm
bt
ng k hc: 0962 60 8801 04 6260 3948 347
a ch: S 18 Trn i Ngha Q Hai B Trng H ni
NGOI NG 24H 348
WWW.NGOAINGU24H.VN

(1) ai l ngi ra thng bo, (2) i tng ca thng bo l ai, (3) ni dung thng bo, (4) l do ra
thng bo.

3. Cu hi v mc ch ca th bo
- What is this memo about?
- What is the purpose of this memo?

Gi tm t kha:
Mc ch ca th bo c th c tm thy phn u ca bi vit hoc mc trnh by ch
(Subject:, Re:).

4. Cu hi v thng tin ngi gi & ngi nhn th bo


- For whom is this memo intended?
- Who wrote this memo?

Gi tm t kha:
Tm thng tin ngi nhn trong mc ngi nhn (To:); thng tin ngi gi trong mc ngi gi
(From:) hoc phn cui ca bi c.

Bi tp 3: Questions 1 ~ 3 refer to the following notice.

NOTICE
Library Loan Periods
New Books -14 Days
High Demand Books (no renewals) -14 Days
Other Books - 21 Days
Audio Books - 21 Days
Music CDs - 21 Days
Magazines (no renewals) - 7 Days
Videos (VHS & DVD) - 7 Days
Feature Films / Fiction - 3 Days
Non-Fiction -14 Days

1. Which of the following is not renewable?


(A) New Books
(B) Audio Books
(C) Magazines
(D) Music CDs
2. Where can this notice most likely be found?
(A) At a train station
(B) At a library
(C) At an office
(D) At a bank
ng k hc: 0962 60 8801 04 6260 3948 348
a ch: S 18 Trn i Ngha Q Hai B Trng H ni
NGOI NG 24H 349
WWW.NGOAINGU24H.VN

3. Which item has the shortest loan period?


(A) Videos
(B) Non-Fiction
(C) Other Books
(D) Feature Films / Fiction

Questions 4 ~ 6 refer to the following memo.

MEMO
TO: All managers
FROM: Andy McCoy, Director of Human Resources
RE: Andrea Cathy
DATE: February 22
I am very pleased to introduce our new marketing director, Andrea Cathy.
She has been working in the marketing department for more than 5 years.
She will be in charge of developing our new marketing strategies.
Andy McCoy

4. To whom is this memo intended?


(A) Andy McCoy
(B) Andrea Cathy
(C) Employees
(D) Managers
5. What is the main purpose of the memo?
(A) To apologize for inconvenience
(B) To set up a meeting
(C) To inform of staff change
(D) To ask for assistance
6. Who is Andrea Cathy?
(A) A manager
(B) The new marketing director
(C) The new CEO
(D) The Director of Human Resources

Questions 7 ~ 11 refer to the following memo and e-mail

To: All Employees


From Kevin Redford
Date: April 25
Subject: Staff Meeting
There will be a monthly staff meeting next Monday at 2 P.M. in room 101.
The meeting will last about an hour. All employees should attend the meeting.
Agenda: (1) Dress Code
(2) Vacation Policy
(3) Expense Reduction

To: Kevin Redford


ng k hc: 0962 60 8801 04 6260 3948 349
a ch: S 18 Trn i Ngha Q Hai B Trng H ni
NGOI NG 24H 350
WWW.NGOAINGU24H.VN

From: Jason Kay


Subject: Staff Meeting
Date: April 26
I am afraid that I have to miss the monthly meeting because I have a previous appointment with JD
Drew Co. at noon.
I will ask one of my associates about the meeting later.
7. What is the purpose of the memo?
(A) To inform of a meeting
(B) To apply for a position
(C) To request suggestions
(D) To distribute meeting materials
8. What is not included in the agenda?
(A) Dress Code
(B) Maintenance Work
(C) Vacation Policy
(D) Expense Reduction
9. Who should attend the staff meeting?
(A) Managers
(B) New employees
(C) Directors
(D) All employees
10. What will Jason Kay do next Monday?
(A) He will attend the staff meeting.
(B) He will take a day off.
(C) He will go on a vacation.
(D) He will meet with a client.
11. When is the staff meeting scheduled to end?
(A) At noon
(B) At 2 P.M.
(C) At 3 P.M.
(D) At 4 P.M.

BI TP KIM TRA
Bi tp 1: Read the following memo and answer the questions.
To: all the members
From: Jessica Shaydon, Personnel Department
Re: The promotion of Joe Endley
I am pleased to announce that Joe Endley, a manager in the Sales Department, has been promoted to
vice president of the company. Mr. Endley has been working for our company for the last 18 years.
He won the Employee of the Year award in 2009 for increasing our sales. We are very excited to
see his leadership as a new vice president.
1. What is the purpose of this memo?
a. To inform employees of the new vice president
b. To notify employees of an annual meeting
2. How long has Mr. Endley been working for the company?
ng k hc: 0962 60 8801 04 6260 3948 350
a ch: S 18 Trn i Ngha Q Hai B Trng H ni
NGOI NG 24H 351
WWW.NGOAINGU24H.VN

a. 18 years
b. 29 years

Bi tp 2: Practice paraphrasing the following sentences by choosing the correct option.


01 As requested, the swimming pool will remain open tor three extra hours.
= The request to the pool hours of operation has been approved.
(A) extend (B) extract
02 Pre-registration is required to secure a place in the lecture course.
= You need to sign up to ensure a seat in the lecture.
(A) in a hurry (B) in advance
03 If you have a question, please ask a salesperson.
= All should be directed to one of the sales representatives.
(A) inquiries (B) concerns
04 Led by Mr. Collins, the seminar on safety procedures at the factory will be held this Friday.
= Mr. Collins will the seminar on the proper use of machinery.
(A) conduct (B) train

05 A ban on this movie is in effect from February 14 until further notice.


(A) It is illegal to watch the movie until February 14.
(B) A ban of indefinite duration has been placed on this film.

06 A schedule of the weeks events is available to conference attendees on the website.


(A) A conference program can be found on the website.
(B) Attendees may sign up for events on the conference website.

07 Due to a server update, there will be an interruption to the online services.


(A) Due to a server error, the online system requires updating.
(B) A service disruption is expected while our system is being updated.

08 A $50 non-refundable deposit is required to make a reservation at our hotel.


(A) Customers will not get their deposit back if they cancel their reservations.
(B) Customers must pay a penalty of $50 when they cancel their reservations.

Question 9 refers to the following announcement.


The 23rd Annual Batteford Science Fair will take place at Riedel University from May 1 to 7. The fair
draws over 3000 students, parents, and industry leaders from Riedel County each year and is in need
of volunteers. Volunteers will organize the awards ceremony. In addition, we need a team to take
responsibility for selling snacks and refreshments. If you are interested in volunteering, contact Bob
Marshall, Battleford Science Fair coordinator, for futher information.
09 What is the purpose of this announcement?
(A) To recruit volunteers for the science fair
(B) To provide information on an awards ceremony

ng k hc: 0962 60 8801 04 6260 3948 351


a ch: S 18 Trn i Ngha Q Hai B Trng H ni
NGOI NG 24H 352
WWW.NGOAINGU24H.VN

(C) To draw students to the science fair

Question 10 refers to the following announcement.


Tickets for the performance can be exchanged no later than 48 hours before the time stated on the
tickets. All evening performances will begin at 8 p.m., and weekend matinees start at 2:30 p.m.
Audience members are asked to arrive early, as latecomers will not be allowed to enter the concert
hall. Please be aware that there is limited parking near the Art Center. Street parking is also available
for a small fee.
10 What is mentioned in the announcement?
(A) Tickets can be purchased online.
(B) There are no afternoon performances on weekends.
(C) They may have difficulty finding a place to park.
Questions 11-12 refer to the following notice.
Belhaven is pleased to provide on-site classes from Harvey University to our staff. Until now, our
company has supported staff who take courses on the university campus. However, management
believes that it will be more beneficial to bring the classes to the company. This will save time by
reducing unnecessary commuting. Half of the enrollment fee will be paid by the company, and a
university credit will be given upon completion of each course.
Class Time Classroom

Introduction to Accounting Mon, Fri 6:00 p.m ~ Room 202


8:00 p.m

Introduction to Marketing Tue, Thur 6:00 p.m. ~ Room 202


9:00 p.m.

Bussiness Administration Mon, Wed 6:00 p.m. ~ Room 203


8:00 p.m.

If you are Interested in taking a class, please download a registration form from the HR Department's
website. Further instructions are displayed on the bulletin board.

11. What is not a stated feature of the companys on-site classes?


(A) They will award university credits
(B) They will save staff time by minimizing travel time
(C) They will be available at no charge.
(D) They are run by a university.

12. What should employees do if they are interested in attending a class?


(A) Call the HR Department
(B) Ask for their supervisors approval

ng k hc: 0962 60 8801 04 6260 3948 352


a ch: S 18 Trn i Ngha Q Hai B Trng H ni
NGOI NG 24H 353
WWW.NGOAINGU24H.VN

(c) contact the university


(D) Obtain a document.
Bi tp 3:
Questions 1 -2 refer to the following memorandum.
To: All staff
From: Jessica Bennett, Maintenance Department Subject: Building safety inspection
The building safety inspection was scheduled for Monday, September 27. However, the company
decided to change the date. The safety inspection will take place on Thursday, September 30.
During the building safety inspection, a special team from the headquarters will give you all the
information about what you have to do. The
team will arrive on Wednesday to make safety
preparations. I would appreciate it if you would inspection
cooperate with the inspection. be
1 When will the inspection take place? scheduled
a. September '27 for take
b. September 30 place
headquarte
c. September 20 .
rs
d. September 13 preparation
2 What are the employees asked to do during the appreciate
cooperate
inspection?
participate
a. Leave the office building in
b. Participate in the preparations
c. Do what the special team asks
d. Inspect some information

Bi tp 4: Reading Comprehension
Simple tips for exciting vacations:
Dont Miss Your Ride Always check the bus schedule and plan to board the bus at least 15 minutes
before departure. Bus schedules are available at ail bus stops and with all bus drivers.
Be Prepared
Prepare your bus ticket, motley, and identification before boarding the bus.
Safety First
Stay seated until the bus stops. When getting off the bus, tightly hold the handrails.
Show Manners
Talk in a soft voice and use earphones when listening to music. Be sure not to block the aisle.
Thank you for choosing to ride with Triple Transportation.
1 What is the purpose of the notice?
a. To make suggestions to passengers
b. To threaten passengers with warnings
c. To inform bus drivers of the rules
d. To inform passengers of a bus fare increase
ng k hc: 0962 60 8801 04 6260 3948 353
a ch: S 18 Trn i Ngha Q Hai B Trng H ni
NGOI NG 24H 354
WWW.NGOAINGU24H.VN

2 According to the notice, what can the passengers obtain from the driver?
a. Snacks and drinks
b. Books for reading
c. Disposable cameras
d. Bus schedules

ng k hc: 0962 60 8801 04 6260 3948 354


a ch: S 18 Trn i Ngha Q Hai B Trng H ni
NGOI NG 24H 355
WWW.NGOAINGU24H.VN

ng k hc: 0962 60 8801 04 6260 3948 355


a ch: S 18 Trn i Ngha Q Hai B Trng H ni
NGOI NG 24H 356
WWW.NGOAINGU24H.VN
NGY 21:
PART 4: CC DNG BI
PART 7: N TP

A- PART 4
DNG BI NI GII THIU V NGI V GII THIU CHUYN THM
QUAN
Trong Part 4, dng bi ni c ni dung gii thiu ngi v chuyn tham quan thng xuyn xut
hin trong thi hng thng. Cch trin khai ni dung thng c n nh v c ni dung c
nh, do vy hy c gng hc ng trng tm cng nh ni dung, hnh thc ca bi ni.

Phn tch cu hi v cc p n la chn 090.mp3


Questions 1 - 3 refer to the following announcement.

(1) On your right is one of the oldest castles in Europe. It has been rebuilt several times (2) because
of its unique location. Historically, it has been a major center of Hungary. It is almost 600 years old.
Our next stop will be a fort built in the 16th century, about 3 miles from here. (3) But right now, we
will go to lunch.

Q1 : Where are the listeners most likely?


A: At a historical site
ang hi v ni mi ngi ang tham quan. Ni dung c th hin trong cu u one of the
oldest castles in Europe v c nhc li trong cu th ba Historically, it has been a major center.
Qua ta bit c a im l mt khu di tch.

Q2 : Why was the site rebuilt?


A: It is in a special area.
c trc cu hi v nh hng tp trung lng nghe cc thng tin cn thit. y l cu hi v l
do ta lu i c xy li. p n nm cu th hai It has been rebuilt several times because of its
unique location. Trong cm t unique location c thay th bng a special area.

Q3: What is scheduled to happen next?


A: The listeners will eat.
ang hi v lch trnh tip theo. Trong nhiu trng hp, a im tham quan k tip c trnh
by phn kt thc bi ni. Nh y trnh by trong cu cui l But right now, we will go to
lunch.

Vocabulary
castle lu i historically v mt lch s
be rebuilt c xy dng li major chnh, quan trng
several mt vi center trung tm
unique c nht, c o fort pho i

ng k hc: 0962 60 8801 04 6260 3948 356


a ch: S 18 Trn i Ngha Q Hai B Trng H ni
NGOI NG 24H 357
WWW.NGOAINGU24H.VN

lunch ba n tra

Bi tp 1: 091.mp3
Nghe bi ni nhiu ln ri in vo ch trng.

Im pleased to introduce our (1)_____________________________________, Mr. Yamamoto. His


speech today is about recent advances in skin protection technologies in
(2)______________________________Specifically, he will talk about new products containing
newly-developed compounds by his laboratory. Mr. Yamamoto will
(3)______________________________ following his talk. So please
(4)_____________________________ of any questions while he speaks. After this speech, our
assistant will (5)_____________________________________ to help others hear you well. Thank
you.

Vocabulary
be pleased to do hn hnh contain gm c
introduce gii thiu newly mi
guest speaker din gi khch mi compound hp cht
speech bi din vn laboratory phng th nghim
recent gn y following sau
advances nhng tin b take notes ghi ch
protection s bo v welcome hoan nghnh
technology cng ngh assistant tr l
cosmetics industry ngnh m phm microphone micr
specifically mt cch c th

Luyn cch trnh by 1 092.mp3

BC 1: Trc tin lng nghe bi ni.

BC 2: Kim tra p n.
1. Sophia Lopez is [receiving the award / presenting the award].
2. She has been working for the company for [15 years /10 years].

BC 3: Xc nh ni dung.
(1) Its my pleasure to introduce the award recipient, Sophia Lopez for this years Employee of the
Year. Many of you know, (2) Ms. Sophia has been working for our company for more than fifteen
years. After three year working in the office, she started working as one of the sales representatives.
Remarkably, she has been the top seller in her department for more than ten years. Lets give a nice
round of applause for Sophia Lopez.

Vocabulary
receive nhn recipient ngi nhn
present trao tng sales representative nhn vin bn hang
pleasure nim vinh hnh remarkably mt cch ng ch
introduce gii thiu top seller ngi bn hng xut sc nht
award gii thng department phng
ng k hc: 0962 60 8801 04 6260 3948 357
a ch: S 18 Trn i Ngha Q Hai B Trng H ni
NGOI NG 24H 358
WWW.NGOAINGU24H.VN

a round of applause mt trng v tay

BC 4: Phn tch ni dung.


1. Sophia Lopez [ang nhn gii thng / ang trao tng gii thng].
Ni dung cu u tin l introduce the award recipient, Sophia Lopez. Ngi ni ang gii thiu
ngi ot gii l Sophia Lopez.
2. Sophia lm vc cho cng ty c [15 nm / 10 nm].
ang hi v s nm lm vic. Nhng con s c trnh by trong p n cho sn nn cn
ch nghe k. 10 nm l phn ni v thnh tch ca n nhn vin kinh doanh. Cn p n ng
c th hin trong cu Ms. Sophia has been working for our company for more than fifteen years.

Luyn cch trnh by 2 093.mp3

BC 1: Trc tin lng nghe bi ni.

BC 2: Kim tra p n.
3. The current owner of the property is [Philip Edward / the government].
4. The tourists will have lunch [in a restaurant / in a hotel].

BC 3: Xc nh ni dung.
Hello everyone! My name is Peter Choi and Ill be leading the tour today. The castle in front of you
was originally built 300 years ago by Philip Edward. In the 1970s, the government decided to buy
this property from the owner to preserve this historical site. (3) So this castle was purchased by the
government and turned into this history museum. After visiting this castle, (4) we will have lunch in
a local restaurant.

Vocabulary
current hin ti
owner ch s hu
property ti sn
tourist du khch
lead the tour hng dn chuyn i
castle lu i
originally ban u
preserve bo tn
historical site di tch lch s
purchase mua
government chnh ph
turn into tr thnh
history museum vin bo tng lch s
local a phng

ng k hc: 0962 60 8801 04 6260 3948 358


a ch: S 18 Trn i Ngha Q Hai B Trng H ni
NGOI NG 24H 359
WWW.NGOAINGU24H.VN

BC 4: Phn tch ni dung.


3. Hin ti ch s hu ta nh l [Philip Edward / chnh ph].
Ni dung cu hi l v ch s hu ta nh. ng nhm vi originally built 300 years ago by
Philip Edward. Philip Edward vn l ch ta nh nhng qua cu So this castle was purchased by
the government and turned into this history museum, c th bit chnh ph chnh l ch s hu.
4. Cc du khch s n tra [ti nh hng / ti khch sn].
a im n tra c ni n l nh hng qua cu we will have lunch in a local restaurant.

Bi tp 2: 095.mp3

1. Where is the tour taking place? 5. What will Anjali Sardesai do today?
(A) A botanical garden (A) Present an award
(B) A theme park (B) Introduce another speaker
(C) A manufacturing facility (C) Read from her book
(D) A science museum (D) Talk about her youth

2. What will the senior technical managers do? 6. Why is the live audience mentioned?
(A) Host the lunch (A) They are personal friends of the
(B) Make a presentation on the history of novelist.
the company (B) They will ask questions.
(C) Interview the visitors (C) They are all famous critics.
(D) Inform the production process (D) They are waiting to congratulate her.

3. When will the question and answer session 7. Who is Amrita Rai?
be held? (A) A consultant
(A) During lunch (B) A company director
(B) At the end of the tour (C) A top researcher
(C) During the break (D) A project assistant
(D) Before the tour begins
8. What problem is mentioned?
4. What is the main program of todays show? (A) A position has to be filled.
(A) Tips on writing novels (B) A director has become ill.
(B) Where to look for inspiration (C) A research requirement has to be met.
(C) Common problems faced while writing (D) A project was rejected.
(D) An interview with a celebrity
9. What will most likely happen next?
(A) A program will be detailed.
(B) The speaker will announce an award.
(C) A field visit will be planned.
(D) Names will be suggested.

BI TP KIM TRA
1. Tin tc / Pht thanh 122.mp3
International motor giant Corda Corporation has decided (1) to build a new production facility in
South Valley. (2) The company has chosen South Valley for its high-class transportation facilities and
its closeness to the port. Spokespersons addressing the media said that the superior transportation
ng k hc: 0962 60 8801 04 6260 3948 359
a ch: S 18 Trn i Ngha Q Hai B Trng H ni
NGOI NG 24H 360
WWW.NGOAINGU24H.VN

system would help materials reach the factory easily and ensure that goods are shipped out without
any delay. Cordas Chief Executive Officer (3) Steve Davis said that the company intended to hire
staff for the new facility locally.
(1) Hi v ch chnh ca bn tin (ni dung tng qut)
Q : What is the main subject of this news report?
A : Choice of a location for a new plant
Dng p n paraphrasing
(2) Hi thng tin v South Valley (chi tit)
Q : According to this report, what is special about South Valley?
A : It has an excellent transportation system.
Dng p n paraphrasing
(3) Hi v thng bo ng Davis a ra (lin quan n ngi ni hoc nhn vt c nhc n)
Q : What announcement does Mr. Davis make?
A : He will hire local people.
Dng p n paraphrasing
2. Du lch 123.mp3
Ladies and gentlemen, welcome to Bell Town. (1) Todays bus tour will take us to all the important
landmarks in the town, like the Centenary Hall building, the Bell Town Botanical Garden and the new
Hobart Aquarium. This tour begins and ends at Carson Street and will take about 3 hours. (2) At the
aquarium, a marine biologist will speak about some of the sea life you see there. (3) Please feel free to
take pictures and ask questions during the tour.
(1) Hi v loi hnh du lch (ni dung tng qut)
Q : What type of tour is being discussed?
A : A bus tour
Dng p n c ni dung trc tip
(2) Hi v lch trnh ca chuyn i (chi tit)
Q : At which stop will there be a guest speaker?
A : Hobart Aquarium
Dng p n paraphrasing
(3) Hi v nhng vic du khch c khuyn nn lm (chi tit)
Q : What are visitors invited to do during the trip?
A : Take pictures

ng k hc: 0962 60 8801 04 6260 3948 360


a ch: S 18 Trn i Ngha Q Hai B Trng H ni
NGOI NG 24H 361
WWW.NGOAINGU24H.VN

Dng p n c ni dung trc tip


Bi tp 1: Nghe bi ni, sau tr li cu hi 124.mp3
1. Who is Gina?
(A) A disk jockey
(B) An English teacher
2. What is Sarahs request?
(A) Seoul DJ Hot
(B) Pretty Day by Eugene
3. What transportation is used for this tour?
(A) Foot
(B) Bus
4. What can the visitors get for free on the tour?
(A) Cold drinks
(B) Breakfast and coffee
5. What does the Fisher Company make?
(A) Food
(B) Cosmetics
6. What does Fisher products have a reputation in?
(A) Products that are free of toxic chemicals
(B) The cheapest cosmetic products
7. Who is the speaker?
(A) A museum tour guide
(B) A museum guard
8. What will the tourists do next?
(A) They will have a look around in the video arts section.
(B) They will learn about an artists life.
9. Why do the developers keep their prices low?
(A) They are a non-profit organization.
(B) Their programs are used more widely.
10. What have the developers done to the software?
(A) Simplified
ng k hc: 0962 60 8801 04 6260 3948 361
a ch: S 18 Trn i Ngha Q Hai B Trng H ni
NGOI NG 24H 362
WWW.NGOAINGU24H.VN

(B) Make it complicated

Bi tp 2: Lm quen vi cm gic thi tht 125.mp3


Nghe k bi ni v tr li cu hi.
1. What will the government do with the park?
(A) Protect the park
(B) Make it a national arboretum
(C) Make the park larger
(D) Make the park smaller
2. What is NOT a reason the park was chosen?
(A) Its quiet setting
(B) Its size
(C) Its unique color
(D) Its natural beauty
3. What do some residents say about the decision?
(A) It is a very good choice.
(B) It will increase traffic.
(C) They are unhappy with the decision.
(D) They want some money.
4. When will the construction begin?
(A) June 1st
(B) June 4th
(C) July 1st
(D) August 5th
5. Where is the construction taking place?
(A) On Route 2 between Main St. and Diamond St.
(B) On Route 4 between Harper Lane and Main St.
(C) On Route 2 between Harper Lane and Main St.
(D) On Route 3 between Harper Lane and Arctic Blvd.
6. How long will the construction last?
(A) 2 days
(B) 2 weeks
(C) 2 months
(D) 3 months
7. What is the good news for the travelers?
(A) There is no good news.
(B) Ticket prices will be lower this month than last month.
(C) There will be extra seats.
(D) Weather is great
8. What is the bad news for the travelers?
(A) Some flights will be less comfortable.
(B) Ticket prices are higher.
(C) There is no bad news.
(D) There will be extra seats
9. What can be assumed about the airlines?
(A) They have the best airplanes.
(B) They are overbooked.
(C) It is a vacation season.

ng k hc: 0962 60 8801 04 6260 3948 362


a ch: S 18 Trn i Ngha Q Hai B Trng H ni
NGOI NG 24H 363
WWW.NGOAINGU24H.VN

(D) They are not doing well in business.

3. Tin nhn ghi m 126.mp3


Hello, Jane Brighton. This is (1) Noah Leroy from the marketing department. My client, Luke, went
over to the research department about two weeks ago to get the brochure and layout for the new
plumbing system he was getting from us and you told him they will be delivered to me within a week.
(2) lts been two weeks now and I still haven't received anything and Luke is very upset. Please tell
me why it is taking so long. (3) Call me back as soon as possible. Thanks.

(1) Hi v thng tin ngi gi hoc ngi nhn in thoi


Q : Who is calling?
A : Mr. Noah Leroy
Dng p n c ni dung trc tip
(2) Hi v l do thc hin cuc gi
Q : Why is Noah calling Ms. Brighton?
A : To request a call back
Dng p n paraphrasing
(3) Hi v ni dung ca tin nhn
Q : How late are the brochure and layout?
A : Two weeks.
Dng p n paraphrasing
4. Gii thiu ngi 127.mp3
Ladies and gentlemen, it's my pleasure to introduce our next award recipient, Jen Lopez, who has
been named this years Employee of the Year by the Community Health Association. Many of you
know Ms. Lopez as the Director of Patient Services at Central Hospital, (1) a position shes held for 5
years. (2) Under her guidance, national surveys have ranked Central Hospital among the most patient-
friendly medical centers in the country for three years running. (3) Lets give a nice round of applause
for Jen Lopez.

(1) Hi chi tit v ngi c gii thiu


Q : How long has Jen Lopez been the Director of Patient Services?
A : For 5 years
Dng p n c ni dung trc tip
(2) Hi v thnh tch ca ngi c gii thu
Q: Why is Jen Lopez being awarded?
A: She tried hard to help the Central Hospital to be the best.
Dng p n paraphrasing
(3) Hi v ni dung yu cu hoc lch trnh tip theo
Q : What is the speaker asking people to do?
A : Give her a warm welcome
Dng p n c ni dung trc tip

ng k hc: 0962 60 8801 04 6260 3948 363


a ch: S 18 Trn i Ngha Q Hai B Trng H ni
NGOI NG 24H 364
WWW.NGOAINGU24H.VN

Bi tp 3: 128.mp3
1. Who is David West?
(A) A dentist
(B) An energy researcher
2. How is the speaker acquainted with David West?
(A) He went to one of David Wests previous talks.
(B) He has done research with him.
3. What does Ronald say about the projectors?
(A) The V-10 is too big.
(B) He needs a V-12.
4. What is Ronald going to need?
(A) A V-10 projector
(B) 100 more copies of the brochure
5. Why is there a party for Frank Thomas?
(A) Because hes 25 years old
(B) Because it's his retirement
6. For what reason is he being appreciated?
(A) Hes been a devoted employee for 25 years.
(B) Because he's smooth
7. Who is Michael?
(A) JCs boss
(B) A co-worker of Mary
8. What happens tomorrow?
(A) Presentation
(B) A movie premiere
9. Who probably is Martha?
(A) The owner of Taco Center
(B) An employee of a tortilla factory
10. What does Martha say about the new white com tortillas?
(A) It tastes the same as yellow com tortilla.
(B) They have gotten very good response from the customers.

Bi tp 4: 129.mp3
ng k hc: 0962 60 8801 04 6260 3948 364
a ch: S 18 Trn i Ngha Q Hai B Trng H ni
NGOI NG 24H 365
WWW.NGOAINGU24H.VN

Nghe k bi ni v tr li cu hi.
1. Who is Mark Hunt?
(A) A fire fighter
(B) A captain of the community service program
(C) The owner of the theatre
(D) A loan shark
2. What does Mark Hunt say about his classes?
(A) They last 2 hours.
(B) They last 3 hours.
(C) They start at 5:30 p.m.
(D) They end at 9:00 p.m.
3. Who does Mark teach?
(A) People who want to become a fire fighter
(B) People without jobs
(C) Other fire fighters
(D) Children
4. When will the store be open again?
(A) Monday
(B) Tuesday
(C) Friday
(D) Sunday
5. What does the store probably NOT sell?
(A) TVs
(B) DVD players
(C) Leather coats
(D) Radios
6. Which item is discounted?
(A) Speakers
(B) Home theatres
(C) CDs
(D) CD players
7. Why is the check being written?
(A) Payment for the business
(B) For the broken window
ng k hc: 0962 60 8801 04 6260 3948 365
a ch: S 18 Trn i Ngha Q Hai B Trng H ni
NGOI NG 24H 366
WWW.NGOAINGU24H.VN

(C) Money owed for the luncheon


(D) Anna Lees pay check
8. When will the check be mailed?
(A) This afternoon
(B) February 20th
(C) Lunch time
(D) Next time
9. What is Holiday Caterings job?
(A) Preparing corporate luncheon
(B) Replacing windows
(C) Fixing windows
(D) Making checks

ng k hc: 0962 60 8801 04 6260 3948 366


a ch: S 18 Trn i Ngha Q Hai B Trng H ni
NGOI NG 24H 367
WWW.NGOAINGU24H.VN

B- PART 7
n tp Part 7 (tit 1)
Directions: In this part you will read a selection of texts, such as magazine and newspaper articles,
letters, and advertisments. Each text is followed by several questions. Select the best answer for each
question and mark the letter (A), (B), (C) or (D) on your answer sheet.
Questions 153 154 refer to the following fax.
Attn: Mike Bradshaw
Dear Mike,
Thank you for your interest in the sales position at Topfield Electronics. We enjoyed speaking with
you in the interview last Thursday. The executives have reviewed your resume and were very
impressed. We have decided to hire you for the position. As you will notice, I have sent the contract
along with this fax. Please read it thoroughly, sign it, and fax it back to our office by Friday at 6:00
p.m. If possible, we would like you to begin working for us as early as May 23rd, approximately two
weeks from now. If you have any questions about the contract or the position, please call me at 895-
368-6457. Congratulations. We look forward to working with you.
Regards,
Samuel Stone
153. What does Samuel Stone include in the fax?
(A) A detailed work schedule
(B) A job application
(C) A work agreement
(D) A letter from the executives
154. What should Mike Bradshaw do by Friday at 6:00 p.m.?
(A) Interview with the employee
(B) Reply to the fax
(C) Schedule an appointment.
(D) Call Samuel Stone

Questions 155-156 refer to the following email.


From: Helen Martin
To: All advertising associates
Subject: Employee Lunch
Hello everyone,
I just wanted to remind you all that the annual employee luncheon will be held this Friday, March
16th. It will be held at 1 p.m. at the Emerald Country Club. Lunch will be provided, but we would like
each associate to bring one dessert. There is a sign-up sheet in the break room next to the water cooler.
Also, this year we will be celebrating the achievements of Bob Hopey, the Director of Advertising,
who will be retiring at the end of March. Mr. Hopey has worked for Best Advertising for thirty years,
and was responsible for the successful merger with McKinley Advertising in 2004. He will be missed
greatly, and we are asking each associate to contribute $25 for a goodbye gift. The gift will be handed
to him at the luncheon. Please give the money to Barbara Whales in Human Resources by Thursday,
March 15th.
ng k hc: 0962 60 8801 04 6260 3948 367
a ch: S 18 Trn i Ngha Q Hai B Trng H ni
NGOI NG 24H 368
WWW.NGOAINGU24H.VN

Thanks,
Helen Martin
155 What information is discussed in the email?
a. A possible merger with another company
b. A plan to hire a new director
c. A schedule for an upcoming event
d. A request to reschedule a meeting

156 What will Bob Hopey most likely receive on March 16th?
a. A promotion
b. A retirement gift
c. Retirement benefits
d. A campaign proposal

ng k hc: 0962 60 8801 04 6260 3948 368


a ch: S 18 Trn i Ngha Q Hai B Trng H ni
NGOI NG 24H 369
WWW.NGOAINGU24H.VN

Questions 157-160 refer to the following letter.


Dear Readers,
This July, Horse and Rider will celebrate its 27th anniversary. Since its first issue was published in
January of 1980, Horse and Rider has continued to bring its readers tasteful but entertaining articles
covering everything about horses. To celebrate our anniversary, we are offering a special promotion to
all our readers: a free three-month subscription. Plus, you will receive our special anniversary issue,
which will be published at the end of July. This special issue will feature an article about Marjorie
Clemmons, the Senior Editor and founder of Horse and Rider,
To be eligible for the special offer, just fill out the request form that is enclosed with this letter, and
mail it to us by June 15th. As always, we welcome your questions and comments, so if you would like
to be included in our From the Readers section, please visit our website www.horseandrider.com and
submit your review electronically. We thank you for your continued support, which has helped us
become the most popular horse magazine in the country.
Sincerely,
Katherine Fields Public Relations Advisor
157 Why did Katherine Fields write this 158 According to the letter, how can readers get
letter? their comments published?
a. To introduce a special offer a. By writing a letter to the editor
b. To remind readers to renew their b. By submitting a review online
subscription c. By contacting the PR department
c. To ask customers about their d. By calling Marjorie Clemmons
subscription preference
159 Which of the following is mentioned about A
d. To invite clients to the ceremony
Horse and Ridert
158 Who will be featured in the anniversary a. It is the only magazine of its kind.
issue? b. It is over thirty years old.
a. A famous rider c. It is very successful.
b. A new employee d. It is changing owners.
c. The winner of the race
d. The Senior Editor

Questions 161-163 refer to the following article.


The Health Factory will introduce a new line of vitamins, a company spokesman said yesterday. The
vitamins are intended for athletic men and women, who are looking for a vitamin that will replace vital
nutrients lost during strenuous exercise. Unlike regular vitamins, the Health Factory Active Essentials 2020
have more iron, calcium, and potassium. The new vitamins have been tested on professional athletes, such as
cyclist Luke Wilson, and are proven to increase athletic performance by up to 20 percent. Of course, health
products like the Active Essentials vitamins are not cheap. One months supply of the new vitamins costs
about $200. To promote the new product, however, the Health Factory is offering a special 30 percent
discount to anyone who purchases the new vitamins with their Health Factory card. For more information on
the new product or to apply for a Health Factory card, call the companys customer service hotline at 1-800-
326-HEALTH.
161. What can be inferred about the new vitamins?

ng k hc: 0962 60 8801 04 6260 3948 369


a ch: S 18 Trn i Ngha Q Hai B Trng H ni
NGOI NG 24H 370
WWW.NGOAINGU24H.VN

a. They are intended for athletes.


b. They are the most popular.
c. They are easy to swallow.
d. They are not sold in stores.
162. What is NOT mentioned about the new vitamins?
a. They increase performance.
b. They were tested on athletes.
c. They are cheaper than other brands.
d. They have more iron and calcium.
163. According to the article, how can people receive a discount?
a. Place an order online
b. Apply for a membership card
c. Enter a contest
d. Spend a certain amount

Questions 164 166 refer to the following letter.


12 Blue Bird Drive
Newark, NJ
064587
12 May 2007
Paul Thorntree Spotless Cleaner's 145 Market Drive Newark, NJ
Dear Mr. Thorntree,
I recently had three jackets dry cleaned at Spotless Cleaners. Unfortunately, when I got home and looked at
one of the jackets. I noticed there was a large rip on one of the sleeves. I immediately returned the item to
your building on Market Drive, but the sales clerk told me chat the company was not responsible for any
damages incurred during cleaning, i have been doing business with Spotless Cleaner's for three years and
have always received excellent service, but I am quite distressed about this recent incident. I would like the
company to pay for the repair of this item, and to give me a full refund on the cost of dry cleaning all three
items. Please contact me as soon as possible to discuss this matter. 1 can be reached Monday to Friday after
6 p.m., and Saturday and Sunday any time. My home number is 632-365-1456. You can also reach me at my
office at 653-362-9876.
Regards,
Linda Applebee
164. What is the purpose of the letter?
a. To complain about service
b. To apply for a membership
c. To discuss a late payment
d. To order a clothing item

165. What does Mrs. Applebee say about Spotless Cleaners?

ng k hc: 0962 60 8801 04 6260 3948 370


a ch: S 18 Trn i Ngha Q Hai B Trng H ni
NGOI NG 24H 371
WWW.NGOAINGU24H.VN

a. It has a bad reputation.


b. It does not repair clothing.
c. It usually provides good service.
d. It is the only dry cleaners in town

166. What does Mrs. Applebee request from the company?


a. A receipt
b. A refund
c. An apology
d. A coupon

ng k hc: 0962 60 8801 04 6260 3948 371


a ch: S 18 Trn i Ngha Q Hai B Trng H ni
NGOI NG 24H 372
WWW.NGOAINGU24H.VN

Questions 167-168 refer to the following article.


The Smallville Department of Transportation (SDT) announced that it will begin construction on a new bus
lane downtown. The construction is scheduled to begin at the end of July, a city official said. The new bus
lane will provide direct service to the main financial district, and will replace bus lines 3 and 4. Studies have
been conducted on traffic patterns and it is expected that the new service will reduce commuting time by 25
percent. Crane Construction has been contracted to build the new lane. Unfortunately, Main St. will be
closed from July 22nd to August 30th, while the construction is taking place. The SDT has asked all
commuters who drive to work to take 122 Avenue instead of Main Street.

167. What does the SDT say about the new 168. What is NOT mentioned in the article?
service? a. Commuters should take another
a. It will decrease travel time. route.
b. It will cost less money. b. Bus lines 3 and 4 will be closed.
c. It will take a year to complete. c. More studies are planned for July.
d. It will reduce pollution. d. The SDT hired Crane Construction

ng k hc: 0962 60 8801 04 6260 3948 372


a ch: S 18 Trn i Ngha Q Hai B Trng H ni
NGOI NG 24H 373
WWW.NGOAINGU24H.VN

Questions 169 172 refer to the following advertisement.


JP Store is proud to announce its new catalogue service, which will begin this September. Each
month all customers with a JP Gold Card will receive a free catalogue. What's so great about the JP
Catalogue? You don't have to go to our store to buy your clothes. You can simply call the customer
service hotline and place your order from the comfort of your own home. Plus, each month's
catalogue will include special discounts on all our clothing. If you have any questions about this
special promotion, contact us at 1-800-2365-9864. If you are not a cardholder and would like to
become one, then download an application from our website, www.jpclothes.com. All customers who
apply online before August 15th will receive an additional 20 percent off their first order.
169. What kind of product does JP Store sell?
a. Home furnishings
b. Kitchenware
c. Clothing
d. Office supplies
170. What will JP Gold Card holders receive?
a. Complimentary catalogues
b. A discount on shipping charges
c. Special coupons for the store
d. An online account

171. How often will cardholders receive the JP Catalogue?


a. Once a week
b. Every two weeks
c. Once a month
d. Once a year

172. How can interested customers apply for a card?


a. Get an application from the store
b. Call customer service
c. Email the company
d. Submit an online application

ng k hc: 0962 60 8801 04 6260 3948 373


a ch: S 18 Trn i Ngha Q Hai B Trng H ni
NGOI NG 24H 374
WWW.NGOAINGU24H.VN

Questions 173-175 refer to the following job advertisement.


Charity Hospital is looking for an organized, motivated individual to fill the position of Director of
Dining Services, starting August 15th.
Responsibilities of the position include planning weekly meals for patients, managing a staff of thirty
food service workers, and placing food supplies orders every' month. You may also be asked to
organize food for special events, such as hospital benefits.
The successful applicant will have at least five years experience working as a manager in some
business field, and a university degree. Specific experience in the hospitality industry is preferred, but
not required.
Interested individuals should pick up an application from the Food Service Department at Charity
Hospital, which is located on the first floor of the hospital. Please fill out the application and submit
it, along with 2 reference letters and a cover letter, by July 24. 2007. Applications will be available
starting June 21, 2007,
173. What kind of position is being advertised?
a. Medical assistant
b. Receptionist
c. Hospital director
d. Manager of food services
174. Which qualification is required of the successful candidate?
a. Five years experience
b. Foreign language skill
c. Cooking certificate
d. Knowledge of the hospitality sector

175. By what date must an applicant submit an application?


a. June 21st
b. July 24th
c. August 15th
d. August 24th

ng k hc: 0962 60 8801 04 6260 3948 374


a ch: S 18 Trn i Ngha Q Hai B Trng H ni
NGOI NG 24H 375
WWW.NGOAINGU24H.VN

Questions 176-180 refer to the following article.


McDowell Publishing plans to merge with Harvey Media this January, a spokesman for McDowell
Publishing said. The new company will be called McDowell and Harvey Media. The decision comes
after both companies experienced their third straight year of losses. Both companies have struggled to
compete with the world's largest publishing company, Watson Media Inc., which accounts for about
75 percent of the market. Since Watson Media Inc. was founded by Charles Duvet Jr. in 1999, the
company has successfully forced ten other small publishing companies to declare bankruptcy.
According to a spokesman from Harvey Media, after the merger is completed, the new company will
launch a 3 million dollar marketing campaign. Jackson Polkins, the famous children's writer, has
already announced he will be leaving Watson Media Inc. to pursue a contract with the new company.
176. The word losses in line 5 is closest in meaning to
a. Deficits
b. Fortunes
c. Properties
d. resources

177. According to the article, why are the two companies making the change?
a. To induce voluntary retirement
b. To hire more employees
c. To reduce operation costs
d. To remain competitive

178. According to the article, what happened in 1999?


a. Watson Media Inc. went bankrupt.
b. Charles Duvet Jr. started a company.
c. Harvey Media introduced a new campaign.
d. Jackson Polkins published a book.
179. What can be inferred from the newspaper article?
a. The new company will lay off many workers.
b. McDowell Publishing will invest considerable money.
c. Watson Media Inc. is a huge company.
d. Harvey Media will change locations.

180. What is Jackson Polkins's occupation?


a. Company spokesman
b. Financial advisor
c. Childrens writer
d. Publishing assistant

ng k hc: 0962 60 8801 04 6260 3948 375


a ch: S 18 Trn i Ngha Q Hai B Trng H ni
NGOI NG 24H 376
WWW.NGOAINGU24H.VN

Questions 181 -185 refer to the following weather forecast and email.
Three-Day Forecast
Friday March 22: Residents should carry their umbrellas because there is an 85 percent chance of
heavy rain. Morning temperatures will be in the low 70's and are expected to drop to around 65 in the
mid-afternoon.
Saturday March 23: If youre planning any outdoor activities, then this is the day to schedule them.
Thanks to a warm westerly wind, temperatures are forecasted to be in the high 80s, about five
degrees warmer than we might usually expect at this time of the year. Remember to wear sunscreen
and drink lots of water.
Sunday March 24: Make sure you enjoy the sunshine on Saturday, because on Sunday temperatures
will drop to the low 60s. There will be some sun in the morning, but the afternoon will be mostly
cloudy.
From: Belinda Pushkin To: All sales staff Subject: Company Picnic
Hello, 1 just wanted to let everyone know that the Tollins company picnic has been cancelled because
there is a chance of rain on the day we originally chose. As you know, we wanted to have the picnic
outdoors so that employees could participate in a variety of team-building exercises.
We asked the Humble Country Club if we could hold the lunch in their Club House instead, but
unfortunately they told us that Plymouth Autos will be holding a special banquet to celebrate their
25th anniversary there.
Attached to this email is an Employee Preference Form. I am asking all employees to list their
preferred date for the rescheduled company picnic. 1 sincerely hope that we can accommodate
everyones schedule. Please fill out the form and email it to me by Thursday, March 21st, at 3 p.m.
Regards,
Belinda Pushkin Human Resources
181. According to the weather forecast, what is unusual about Saturdays weather?
a. Low humidity
b. Strong winds
c. High temperatures
d. No rain

182. When was the picnic originally scheduled?


a. March 21st
b. March 22nd
c. March 23rd
d. March 24th

183. Why can't the event be held in the Club House?


a. It is closed for renovations.
b. It must be reserved in advance.
c. It cant accommodate all employees.
d. It is being used by another group.

184. What can be inferred about Plymouth Autos from the email?

ng k hc: 0962 60 8801 04 6260 3948 376


a ch: S 18 Trn i Ngha Q Hai B Trng H ni
NGOI NG 24H 377
WWW.NGOAINGU24H.VN

a. It will host an event on March 21st.


b. It is celebrating an employees achievements.
c. It has been in business more than twenty years.
d. It is located next to Humble Country Club.

185. What did Belinda Pushkin add to her email?


a. The email addresses of all employees
b. The Employee Preference Form
c. The list of famous restaurants
d. The menu for the luncheon

From: Mary Helmsford To: Linda Watercrest Subject: Conference


Dear Linda,
As you requested, I am writing to provide you with some more information about the conference at
the end of the month. It will be held at the Gillmor Conference Center in San Francisco, California,
on Saturday, May 14th.
The conference is a great way for New Media Inc. employees to meet other people in the
entertainment industry. The conference schedule is as follows:
8:00-10:00 A.M. Understanding the Customer" presented by market analyst, Richard Preyer
10:15-12:00 A.M. Making an Effective Presentation" presented by CEO of Anderson Entertainment,
Brian Thompson
12:00-1:00 P M Free lunch
1:30-3:30 P.M. Increasing Productivity" presented by Director of Sales at Phillip Mortin Media, Bob
Ross
3:45-5:45 P.M. Future Trends in the Entertainment Industry presented by Phillip Trent, author of
The 10 Most Successful Media Companies"
5:45-6:00 P.M. Closing remarks by Samuel Wright, conference coordinator
Please note that New Media Inc. will pay for the registration fee, accommodation, and all meals.
Employees are expected to pay for transportation to and from the conference. I hope you find this
information helpful.
Regards,
Mary Helms ford Director of New Media Inc.

From: Linda Wacercrest To: All employees Subject: Conference


Hi, everyone! I am just writing to inform you that there has been a slight schedule change for
the conference on Saturday, You may have already heard it on the news, but Richard Preyer
suffered a heart attack on Thursday night and is recovering in Mercy Hospital. Unfortunately,
he will not be able to give his presentation as planned, I have been informed by the
conference coordinator that Michel Tupper, the Director of Customer Relations at Fleet
Media, will be presenting instead. He will talk about his 20 years of experience dealing with
customer complaints under the title of How to Satisfy the Customer. Also, if you have any

ng k hc: 0962 60 8801 04 6260 3948 377


a ch: S 18 Trn i Ngha Q Hai B Trng H ni
NGOI NG 24H 378
WWW.NGOAINGU24H.VN

special meal requests for the conference, please contact Mary Helmsford at 895-326-6547 by
May 4th.
186. Why did Mary Helmsford write the email?
a. To provide information about an event
b. To change the date of a meeting
c. To discuss a new company policy .
d. To introduce a new work schedule

187. What does Mary Helmsford say about the conference?


a. It is held in San Francisco every year.
b. It is organized by Anderson Entertainment.
c. It is intended for new employees in the industry.
d. It is an excellent place to develop business relationships.

188. What are employees expected to pay for?


a. Registration
b. Transportation
c. Accommodation
d. Food and drinks

189. What is TRUE about Michel Tupper?


a. He is the CEO of New Media Inc.
b. He has written a book.
c. He organized the conference
d. He is a company executive.

190. Which seminar was cancelled at the conference?


a. Understanding the Customer
b. Making an Effective Presentation
c. Increasing Productivity
d. Future Trends in the Entertainment Industry

Questions 191 195 refer to the following e-mails.


From: Beth Volt <bvolt@runtcorp.com>
To: Chris Gately <cgately@spectron.com>
Date: November 22
Subject: Details for Conference on December 2 Hi, Chris!
First, I'd like to express my excitement that youve agreed to speak at this years technology
conference. As you know, Runt Corporation has been organizing this event for the last ten
years to promote innovation in the computer industry.
In my previous email, I told you that the conference would start at 8 a.m., but it has been
changed to 9 a.m. because a few of the companies who will be providing exhibits wanted a
little more time to set up their product displays. Now, when you get to the conference center,

378
ng k hc: 0962 60 8801 04 6260 3948
a ch: S 18 Trn i Ngha Q Hai B Trng H ni
NGOI NG 24H 379
WWW.NGOAINGU24H.VN

you will have to pass through security. Please hand the security guard the four-digit code that
I gave you, and he will issue you a temporary visitors ID badge. You will need to wear this
badge at all times while you are in the building.
If you need us to provide any equipment, such as a laptop computer, or overhead projector,
please contact the event organizer, Martin Walsh. His email address is
mwalsh@runtcorp.com. You can also reach him by phone at 1-520-236-1478.
Please feel free to contact me with any further questions that you might have. 1 look forward
to meeting you on the day of the conference.
Beth Volt, Public Relations Director
From: Chris Gately <cgately@spectron.com> To: Beth Volt <bvolt@runtcorp.com>
Date: November 23
Subject: Update on December 2 Conference
Dear Mrs. Volt,
Thank you so much for your kind email.I am also very excited to speak at your conference.
My companys innovative business strategy has greatly increased our profit margins and I am
confident that, if implement properly, it will help any computer company improvents. Since
the subject I plan to cover is rather technical, I have asked one of my colleagues, Bill Front, to
give a brief PowerPoint presentation to familiarize the audience with a few new technology
terms. Mr. Front is a software designer who has been working for Spectron Inc. for five years.
His innovative product designs have contributed greatly to Spectrons success in the last three
years.
With regard to equipment, I will need a couple of things for the presentation. I will leave a list
of required items with the event organizer when I visit his office on Wednesday, November
25. Thanks for all of your help.
Regards,
Chris Gately
191. Why did Beth Volt write the email?
a. To outline details of the conference
b. To ask an employee to attend a meeting
c. To inquire about a new software program
d. To postpone a public relations event

192. What is Mr. Gately required to do before entering Runt Corporation?


a. E-mail Beth Volt
b. Acquire a security badge
c. Contact the software designer
d. Meet with a Spectron Inc. executive

193. When will Ms. Volt and Mr. Gately meet?


a. On November 22nd
b. On November 23rd

379
ng k hc: 0962 60 8801 04 6260 3948
a ch: S 18 Trn i Ngha Q Hai B Trng H ni
NGOI NG 24H 380
WWW.NGOAINGU24H.VN

c. On November 25th
d. On December 2nd

194. Who is Bill Front?


a. Public Relations Director
b. Security guard
c. Software designer
d. Event organizer

195. What does Mr. Gately plan to do on Wednesday?


a. Pick up his registration form
b. Design the brochure for the event
c. Leave a document with Mr. Walsh
d. Meet with Mr. Front about the Conference.

Questions 196-200 refer to the following two letters.


12 April 2007
Employment Officer
Porter Investments
67 Waltham Ave.
Boston. MA
Dear Mr. Wright,
I was extremely interested in your recent advertisement for an Assistant Accountant, which
appeared in the March issue of Financial Times. I am confident that my educational
background and work experience make me the ideal candidate for this position.
I graduated last year with high honors from Georgetown University, where I double majored
in accounting and economics. My thesis. The Economics of Investing. received the highest
grade in my class. While at University, I was also president of the Finance Club, and the
Treasurer of the Student Council. In my junior year. I was responsible for organizing a
fundraising event that collected 50 thousand dollars for the new gymnasium on campus.
Last summer. I was accepted for an internship position with Harvey Investments, a famous
New York investment firm. During my two months at Harvey Investments, 1 managed over
20 corporate accounts, and was responsible for approximately 1 million dollars in
investments.
I have enclosed a detailed resume, along with two letters of recommendation, and my
University transcript. I hope you will find my personality and work experience a good match
for this position. Please feel free to contact me at 896-326-5478, if you have any questions.
Sincerely,
KLcfoard/Bcrlt
12 Frosty Rd.
Boston, MA
Dear Mr. Bolt,

380
ng k hc: 0962 60 8801 04 6260 3948
a ch: S 18 Trn i Ngha Q Hai B Trng H ni
NGOI NG 24H 381
WWW.NGOAINGU24H.VN

I am sorry to inform you that the position you applied for has been filled. I want you to know,
however, that I thought your resume and cover letter were extremely interesting. I think you
would be an excellent addition to our team here at Porter Investments. If you would like to
pursue other employment opportunities with our company, please contact Martin Plaster, the
Director of Finances. He might be able to tell you about some other positions that will be
available in the future. He can be reached at 895-654-2315. Thank you for your interest in our
company.
Sincerely, Paul Wright

196. What did Richard Bolt achieve while at Georgetown University?


a. He used to be an assistant of the University Treasurer.
b. He was president of the Debate Club.
c. He wrote a thesis that received an award.
d. He raised money for new sports facilities.

197. What does the letter suggest about Harvey Investments?


a. It is based in Boston.
b. It pays very well.
c. It is well known.
d. It is an international firm

198. What news does Mr. Wright give Richard Bolt?


a. The company does not accept personal interviews.
b. The Assistant Accountant position is no longer available.
c. The information in his transcript was incorrect.
d. The Employment Officer has not received the resume.

199. Why might Mr. Bolt contact Martin Plaster?


a. To inquire about a job
b. To schedule an interview
c. To submit a resume
d. To review a contract

200. The word extremely in line 2 of the second letter is closest in meaning to
a. Intelligently
b. Easily
c. Highly
d. Relatively

381
ng k hc: 0962 60 8801 04 6260 3948
a ch: S 18 Trn i Ngha Q Hai B Trng H ni
NGOI NG 24H 382
WWW.NGOAINGU24H.VN

NGY 22:
PART 3+4: KIM TRA

A- PART 3
Review Test 117.mp3
Nghe v chn p n ng nht
1. What did the woman do recently?
(A) Present a statement
(B) Make a career change
(C) Design a program
(D) Develop a strategy
2. What will happen in a month?
(A) A new team will be formed.
(B) A policy will be announced.
(C) A machine will be installed.
(D) An assignment will be completed.
3. What field do the speakers work in?
(A) Computers
(B) Accounting
(C) Security
(D) Education
4. What are the speakers mainly discussing?
(A) The location of an office
(B) The expenses for renting a truck
(C) The companys billing program
(D) A delay in shipment
5. What will happen at 5:00?
(A) Delivery will be made.
(B) The driver will call for directions.
(C) A discount offer will expire.
(D) An order will be finalized.
6. Who is waiting for the items?
(A) Manufacturers
(B) Investors
(C) Customers
(D) Suppliers
7. Why is the man calling?

382
ng k hc: 0962 60 8801 04 6260 3948
a ch: S 18 Trn i Ngha Q Hai B Trng H ni
NGOI NG 24H 383
WWW.NGOAINGU24H.VN

(A) To ask for instructions


(B) To offer an apology
(C) To respond to a complaint
(D) To market a new product
8. What problem does the woman mention?
(A) A program is not working.
(B) A form is not available.
(C) A technician made an error.
(D) A machine isnt working.
9. What does the man say he will do?
(A) Send a replacement
(B) Forward her message
(C) Consult his coworkers
(D) Come over himself
10. What are the speakers discussing?
(A) A project deadline
(B) An important report
(C) A new schedule
(D) Todays weather
11. What will happen in the afternoon?
(A) More news will be available.
(B) Rain will stop.
(C) The man will put away his umbrella.
(D) A plan for the evening will be made.
12. What does the woman suggest the man do?
(A) Carry a jacket
(B) Call the airport
(C) Cancel a ticket
(D) Turn on the lights
13. What are the speakers mainly discussing?
(A) A resignation
(B) A tour schedule
(C) A staff members absence
(D) A new hiring policy
14. What is the man concerned about?
(A) Covering a work load
(B) Meeting a deadline
(C) Filing a report
(D) Correcting a mistake
15. What will happen after 2 weeks?

383
ng k hc: 0962 60 8801 04 6260 3948
a ch: S 18 Trn i Ngha Q Hai B Trng H ni
NGOI NG 24H 384
WWW.NGOAINGU24H.VN

(A) Alexander and Yumiko will get married.


(B) Martha will return.
(C) A profit statement will be released.
(D) A holiday will begin.
16. What are the speakers manly discussing?
(A) What to do after lunch
(B) How to travel in Asia
(C) Which work schedule to follow
(D) Where to go for a meal
17. What concern does the woman have?
(A) Price
(B) Quality
(C) Time
(D) Amount
18. What benefit of Ayudho does the man mention?
(A) Its larger.
(B) Its cheaper.
(C) Its closer.
(D) Its newer.
19. What is the man calling about?
(A) A delivery schedule
(B) A presentation
(C) A staff meeting
(D) A delayed investment
20. What does the woman tell the man?
(A) Shes busy right now.
(B) Shell study the options.
(C) She already shipped the items.
(D) She will return his call.
21. When will the man and the woman meet?
(A) Tuesday
(B) Wednesday
(C) Thursday
(D) Friday
22. Who most likely are the speakers?
(A) Experts on a food show
(B) Participants at a cooking contest
(C) Customers at a restaurant
(D) Coworkers at a party
23. What is said about the vegetable pie?

384
ng k hc: 0962 60 8801 04 6260 3948
a ch: S 18 Trn i Ngha Q Hai B Trng H ni
NGOI NG 24H 385
WWW.NGOAINGU24H.VN

(A) Francois used a friends recipe.


(B) Lauren helped make it.
(C) It will be served with salad.
(D) Mini will bring it soon.
24. What will the man probably do next?
(A) Arrange the table
(B) Check on the dessert
(C) Invite guests to eat
(D) Call her grandmother
25. What does the woman want to do?
(A) Go to St. Louis
(B) Make a call
(C) Pick up an item
(D) Mail a package
26. What does the man ask for?
(A) An address
(B) A receipt
(C) A stamp
(D) A form
27. What will probably happen next?
(A) The woman will make a payment.
(B) The man will write down details.
(C) A complaint will be filed.
(D) A delivery will be confirmed.
28. What is the man calling about?
(A) A room location
(B) A ticket cancellation
(C) A compensation claim
(D) A flight reservation
29. What does the woman tell the man?
(A) He should leave a message.
(B) A flight will arrive soon.
(C) Information is not currently available.
(D) She will forward his call.
30. What does the man suggest the woman do?
(A) Return a call
(B) Send him a form
(C) Repeat a number
(D) Speak to a supervisor

385
ng k hc: 0962 60 8801 04 6260 3948
a ch: S 18 Trn i Ngha Q Hai B Trng H ni
NGOI NG 24H 386
WWW.NGOAINGU24H.VN

B- PART 4:
Review Test 130.mp3
Nghe k bi ni v tr li cu hi.
1. What does the company offer?
(A) Help with online registration
(B) Directory assistance
(C) Financial advice
(D) Security service
2. What must callers do to speak to staff?
(A) Follow instructions
(B) Make an appointment
(C) Visit a downtown office
(D) Call during working hours
3. What number should callers press to leave a message?
(A) 3
(B) 5
(C) 6
(D) 8
4. Who most likely is Sandra Ross?
(A) A popular author
(B) A professional cook
(C) An office manager
(D) A store owner
5. What is implied about the dessert?
(A) It is a new item.
(B) The flavor is unusual.
(C) Many people like it
(D) It takes a long time to make.
6. What can listeners receive?
(A) Samples of the dishes
(B) Special offers on kitchenware
(C) An invitation to the next show
(D) A free booklet of recipes

386
ng k hc: 0962 60 8801 04 6260 3948
a ch: S 18 Trn i Ngha Q Hai B Trng H ni
NGOI NG 24H 387
WWW.NGOAINGU24H.VN

7. Who most likely are the listeners?


(A) Visitors at an exhibit
(B) Guests at a restaurant
(C) New workers at an orientation
(D) Participants at a conference
8. What must listeners do to buy food at the cafeteria?
(A) Pay cash
(B) Fill out a form
(C) Purchase coupons
(D) Contact staff
9. What does the speaker invite listeners to do?
(A) Help out at the counter
(B) Explore the facility
(C) Give suggestions for the menu
(D) Ask questions
10. Who most likely is the speaker?
(A) A real estate agent
(B) A gardener
(C) A tour guide
(D) An investor
11. How old are the carved doors mentioned in the talk?
(A) 30 years
(B) 75 years
(C) 100 years
(D) 150 years
12. What will happen next?
(A) A different speaker will talk.
(B) The group will explore an area.
(C) The speaker will make an announcement.
(D) A tram will arrive to pick up the people.
13. How does the speaker try to convince listeners?
(A) By having users endorse products
(B) By emphasizing high quality

387
ng k hc: 0962 60 8801 04 6260 3948
a ch: S 18 Trn i Ngha Q Hai B Trng H ni
NGOI NG 24H 388
WWW.NGOAINGU24H.VN

(C) By promising free gifts


(D) By offering customized service
14. What does Kitchen Mate claim about its products?
(A) They are made of recycled material.
(B) They come in standard sizes.
(C) They are available in a range of designs.
(D) They can be exchanged any time.
15. What must buyers do to receive a discount?
(A) Call a number
(B) Visit a company outlet
(C) Respond to a questionnaire
(D) Make an online purchase
16. What is being announced?
(A) A list of programs
(B) Details of a contest
(C) A weather warning
(D) Events in the city
17. What is expected to happen soon?
(A) A storm will move inland.
(B) A beach resort will open.
(C) New shows will be broadcast.
(D) The station will give more interviews.
18. What are listeners asked to do?
(A) Remain in their houses
(B) Call the weather department
(C) Pass on a message to friends
(D) Visit the police station
19. Why is Marsha calling John?
(A) To place an order
(B) To confirm a schedule
(C) To pass on information
(D) To request quick service
20. What does the caller want to do today?

388
ng k hc: 0962 60 8801 04 6260 3948
a ch: S 18 Trn i Ngha Q Hai B Trng H ni
NGOI NG 24H 389
WWW.NGOAINGU24H.VN

(A) Meet a client


(B) Replace old equipment
(C) Make a delivery
(D) Contact her assistant
21. What will John probably do?
(A) Call back
(B) Direct a maintenance crew
(C) Wait for instructions
(D) Leave the office immediately
22. Who most likely are the listeners?
(A) Security guards at a company
(B) Meeting participants
(C) Workers preparing for an event
(D) Visitors at an exhibition
23. What problem is mentioned by the speaker?
(A) He needs more assistants.
(B) There is little time to complete a job.
(C) New furniture has not arrived.
(D) Decorations are missing.
24. What will happen at 7 p.m.?
(A) A show will begin.
(B) The listeners will finish a task.
(C) Guests will arrive.
(D) Food will be served.
25. Where most likely does the speaker work?
(A) At a food store
(B) At a restaurant
(C) At a factory
(D) At a delivery service
26. What is said about the main dishes?
(A) They are available all week.
(B) They have been newly introduced.
(C) They will be served with sauce.

389
ng k hc: 0962 60 8801 04 6260 3948
a ch: S 18 Trn i Ngha Q Hai B Trng H ni
NGOI NG 24H 390
WWW.NGOAINGU24H.VN

(D) They are in great demand.


27. What will the speaker do next?
(A) Take an order
(B) Serve a free beverage
(C) Accept a payment
(D) Speak with the chef
28. What is being announced?
(A) A record sales figure
(B) An increase in production
(C) An award given to the company
(D) A new quality policy
29. Who is the speaker?
(A) A Customer Relations manager
(B) A consultant
(C) A Sales manager
(D) A company vice-president
30. What are listeners invited to do?
(A) Wait for an announcement
(B) Join a celebration
(C) Suggest improvements
(D) Design a campaign

390
ng k hc: 0962 60 8801 04 6260 3948
a ch: S 18 Trn i Ngha Q Hai B Trng H ni
NGOI NG 24H 391
WWW.NGOAINGU24H.VN

NGY 23:
PART 7: N TP

PART 7
Questions 153-154 refer to the following email
From: Hazel Irwin <hirwin@csgmedia.com>
To: Lindsey Monotail <lmonotail@csgmedia.com> Subject: Speech Request Date: May 23
Hi Lindsey,
As you know, Robert Hulking will be awarded the CSG Excellence in Sales Prize at a special
ceremony at the Persian Hotel this Saturday. Mr. Hulking recently informed me that you were
his supervisor when he first came to CSG five years ago, and that you and your business
advice have greatly contributed to his recent success. To honor both your work as Senior
Sales Supervisor and Mr. Hulkings recent achievement, I would like to ask you to present the
award this Saturday. As part of the presentation ceremony, you will be asked to give a brief
speech about Mr. Hulking. It would be a good idea to include some personal anecdotes
Please contact Sara Benhunt, my secretary, before 2 p.m. today and let her know it attend the
event. Her extension is 3125.1 hope you will join us.
Sincerely,
Hazel Irwin
Director of Public Relations

153. Who is Robert Hulking?


a. The new sales associate
b. A supervisor in the sales department
c. An employee who will receive an award
d. The Public Relations Director
154. What should Ms. Monotail do before 2 p.m.?
a. Give a speech
b. Reply to the invitation
c. Attend the ceremony
d. Supervise a meeting

391
ng k hc: 0962 60 8801 04 6260 3948
a ch: S 18 Trn i Ngha Q Hai B Trng H ni
NGOI NG 24H 3
WWW.NGOAINGU24H.VN

Questions 155 156 refer to the following advertisement.


Do you enjoy working outdoors? Do you thrive in a physically demanding
work environment? Then Gateway Contractors has the perfect job for you!
We're looking for young, energetic individuals to work on Gateway's latest
construction project: the Maplewood Gymnasium. Successful candidates will
be responsible for unloading supply trucks and distributing supplies to
various key points around the site. No previous construction experience is
necessary, but applicants must be able to work well with other people.
Applications are available at www.gatewaycontractors.com. To schedule an
interview, contact Bruno Morton, the Construction Site Supervisor, at 245-
698-7895. Please, no calls after 9 p.m.
155. What kind of job is being advertised?
a. Construction worker
b. Shipping supervisor
c. Gym instructor
d. Truck driver

156. Why might the applicants contact Mr. Morton?


a. To ask for an application form
b. To make an appointment for an interview
c. To locate the main office
d. To inquire about the job detail

Questions 157-392 refer to the following news article


Deloit Industries to Open Another Factory
Deloii Industries, a leading producer of construction equipment, plans to
open another factory in 2007. The new factory will be built in Rayong.
Thailand, where a number of other large-scale manufacturing companies are
located.
The companys decision was criticized by many local officials, who claim
that Deloit Industries plans to exploit local laborers, Tn the last decade, many
companies have migrated to Thailand because the country's average wages
are significantly less than thuse in North America and Europe. By operating
in Thailand, companies are able to decrease their annual expenditures and
increase profitability.
Unfortunately, these profits do not always benefit the local communities.
According to a company spokesman, however, Deloit Industries will
implement a couple of strategies that will benefit the local laborers and their
families. Among these positive plans are providing health insurance for
employees and their families, and giving academic scholarships to
employees children.

392 04 6260 3948


ng k hc: 0962 60 8801
a ch: S 18 Trn i Ngha Q Hai B Trng H ni
NGOI NG 24H 3
WWW.NGOAINGU24H.VN

The company has implemented similar strategies for employees working at


its mines in South Africa, processing plants in the Philippines, and packaging
facilities in Brazil. In each place, the company has attempted to hire local
workers and integrate itself into the local community. In July, the company
hosted a charity event in Buenos Aires, Argentina, to raise money for local
schools.
157. According to the article, why has Deloit Industries decision been
criticized?
a. It will probably exploit the environment.
b. It does not encourage foreign investment.
c. It will take advantage of regional workers.
d. It makes employees work longer hours.

158. What does Deloit Industries plan to do at the facility in Rayong?


a. To provide workers better safety equipment
b. To give medical coverage to employees
c. To eliminate harmful waste products
d. To introduce a new mining technology

159. What is NOT mentioned as another one of Deloit Industries' businesses?


a. Mining facilities
b. Processing plants
c. Packaging factories
d. Automobile manufacturing

Questions 160 162 refer to the following advertisement.


Rollins University is looking for an energetic, personable individual to teach
a marketing class for students in the honors program during the fall semester.
The class, Advanced International Marketing 402, will be held on Tuesdays
and Thursdays from 4-6 p.m. This is a part-time position. Rollins University
is one of the top business schools in the world, and has campuses in Los
Angeles, Paris, Tokyo, and Geneva. This position will be at the university's
campus in Tokyo, Japan, so the successful applicant will be expected to
relocate if necessary. The successful candidate will have some teaching
experience (at least 1-2 years in an academic setting) and have worked
extensively in the marketing industry. Additionally, individuals without a
PhD need not apply. The incumbent will be required to work during regular
class hours, and spend two to three hours preparing for each class. A private
office will be provided. Since this is a short-term contract, compensation is
negotiable and will be determined based on the candidate's work experience
and professional history. For more information, or to request an application,
contact James Stewart at 1-541-265-6987.

393 04 6260 3948


ng k hc: 0962 60 8801
a ch: S 18 Trn i Ngha Q Hai B Trng H ni
NGOI NG 24H 3
WWW.NGOAINGU24H.VN

160. What does the advertisement suggest about the course mentioned?
a. It is usually only offered during the spring semester.
b. It is for students in their first year of study.
c. It is a part of the advanced program.
d. It is designed to give students practical experience.
161. Where will the successful candidate work?
a. Tokyo
b. Los Angeles
c. Geneva
d. Paris

162. What is NOT a requirement of the position?


a. To be prepared to live in a foreign country
b. To have worked in a field related to the subject
c. To have some form of teaching experience
d. To be able to work during the weekend

Questions 163-166 refer to the following article


TR Exhibition to be held
Tyler Remington, President of Tyler Remington Inc., announced that the
company would be hosting its fifth annual TR Programming Exhibition in
July. The event is held every year to give professional computer
programmers a chance to test new software developed by TR Inc. The event
is also a means for the company to consult with some of the world's most
talented programmers about technology trends.
The event attracts programmers from around the world, and most of the
individuals who attend are self-employed, or own their own web design and
consulting company. Since the exhibition began five years ago, it has become
known as the single most important programming event of the year. In the
past, it has been hosted in Seattle, Sydney, Berlin, and Moscow. This year's
event is scheduled to take place in Hyderabad, India, as it is rapidly
becoming the technology capital of the world.
To register, simply fill out the online form which can be downloaded from
the company's website at www.trindustries.com. Or, you can request a paper
application by calling 1-800-265-8795. A registration fee of $195 must also
be paid by June 25 with the application. Only credit card payments are
accepted.
163. What is the reason for having the exhibition?
a. To recruit employees
b. To test new computer programs
394 04 6260 3948
ng k hc: 0962 60 8801
a ch: S 18 Trn i Ngha Q Hai B Trng H ni
NGOI NG 24H 3
WWW.NGOAINGU24H.VN

c. To help small businesses prosper


d. To display popular electronics
164. Who will most likely attend the exhibition?
a. Business owners
b. Potential investors
c. College graduates
d. Professional gamers

165. Why will the exhibition be held in Hyderabad?


a. The company just opened a consulting firm in the area.
b. The programmers decided it was the most convenient location.
c. It is known to have many established software companies.
d. Its technology shows the most rapid growth in the world.
166. What is NOT a way to be eligible for the event?
a. Fill out an electronic application
b. Contact the company and request an application
c. Submit the form by the specified date.
d. Send a check to pay for the fee

Questions 167-395 refer to the following advertisement


Waldorf Books is excited to announce it will be offering a new print-on-
demand publishing service. The company has created a database of all the
major and minor book titles in the world and developed a program that can
access electronic versions of these books. If a customer is unable to find a
book in paperback or hardcover in the store, he can go to the Print-On-
Demand kiosk and with the push of a button print a copy of the book without
ever leaving the store! This new technology has a couple of advantages:
customers can choose from a more extensive selection of books, and the final
product is cheaper. The service is especially useful for individuals who are
most interested in less popular titles by relatively unknown authors. The
print-on-demand service will be available at all Waldorf Books stores
starting this September.
167. What service will Waldorf Books offer?
a. Express delivery service
b. Printable versions of books
c. Hardcover copies of bestsellers
d. Personal shopping assistants

168. According to the advertisement, who will benefit the most from the
service?
a. Small publishing companies looking to print more books
395 04 6260 3948
ng k hc: 0962 60 8801
a ch: S 18 Trn i Ngha Q Hai B Trng H ni
NGOI NG 24H 3
WWW.NGOAINGU24H.VN

b. Unpublished authors hoping to secure a contract


c. People who read books that arent very popular
d. Employees who are not familiar with the stores database

Questions 169 171 refer to the following letter.


32 Turtle Rd.
Wilmont, WI 20548
August 23
Dear Mrs. Malloy,
We recently discovered that your subscription to Gourmet Chef Monthly will
expire in September and you have not decided to renew it. To encourage you
to reconsider your decision, wed like to offer you a special, one-time-only
offer: if you choose to renew your subscription, we will give you a fifty
percent discount on each monthly issue from October to March. According to
our database, you were paying $6 per issue; with this discount you would
only have to pay $3.00. This offer is only valid if you apply to renew your
subscription before September 15th. Were offering to cut the cost of your
subscription in half. Can you really ask for a better deal?
To accept our offer, please call us at 1-800-254-9864. Our office is open
Monday to Friday from 9 a.m. to 5 p.m. Dont miss out on this wonderful
opportunity; call one of our helpful customer service agents today. We look
forward to your business.
Warmest Regards,
Ross Cather
Sales Director, Gourmet Chef Monthly
169. Why did Ross Gather write the letter?
(A) To request some personal information now?
(B) To encourage the customer to extend her service.
(C) To inquire about a bill that has not been paid.
(D) To introduce a new service at the restaurant
170. When will Mrs. Malloy's existing subscription expire?
(A) In March
(B) In August
(C) In September
(D) In October
171. How much is Mr. Hopkins paying per month
(A) $3.00
(B) $6.
(C) $9.00
(D) $15.00

396 04 6260 3948


ng k hc: 0962 60 8801
a ch: S 18 Trn i Ngha Q Hai B Trng H ni
NGOI NG 24H 3
WWW.NGOAINGU24H.VN

Questions 172-175 refer to the following announcement.


This month, Rockford Sports has experienced an unexpected increase in the
sale of running shoes. Executives feel that this was due in large part to the
new ad campaign released in February. The campaign was orchestrated by
Albert Sterling, the Director of Advertising. Mr. Sterling has dedicated
twenty years of his life to Rockford Sports; he started as a shoes salesman
and has been steadily working his way up the corporate ladder ever since. He
has held his current position for the last five years. Unfortunately, Mr.
Sterling will be retiring in March so the Hiring Committee has spent the past
year trying to find a suitable replacement. Just last week, a candidate was
chosen for the position; Roy Portsmouth. For the past ten years, Mr.
Portsmouth has been the Executive Advertising Assistant at Famous Feet's
corporate headquarters in Milan. Mr. Portsmouth is respected for his keen
marketing sense and strategic planning skills. In particular, he is famous for
introducing ad campaigns that helped increase Famous Feet's international
brand power, and boosted its total retail profits by 25 percent. Clearly, Mr.
Portsmouth will be a powerful addition to our company, and we hope all
employees make him feel welcome.
172. Where might this announcement be found?
a. A product catalogue
b. A local newspaper
c. A company newsletter
d. A shoe store

173. Who is Mr. Sterling?


a. A department store clerk
b. A retiring employee
c. The Executive Advertising Assistant
d. A member of the Hiring Committee

174. What is TRUE about Mr. Sterling?


a. He worked as a salesman for a decade.
b. He knows Mr. Portsmouth personally.
c. He took the lead of a new ad campaign.
d. He introduced a successful budget plan.
175. According to the announcement, what action is Mr. Portsmouth known
for?
a. Getting promoted very quickly
b. Increasing the companys profits
c. Designing a popular running shoe
d. Planning an important sporting event
397 04 6260 3948
ng k hc: 0962 60 8801
a ch: S 18 Trn i Ngha Q Hai B Trng H ni
NGOI NG 24H 3
WWW.NGOAINGU24H.VN

Questions 176-180 refer to the following Interview.


Jerry Walker has been the head coach for the Chicago Islanders for the last
eight years. On August 23, 2007, Mr. Walker will be inducted into the
prestigious Basketball Hall of Fame. Mr. Walker agreed to sit down with
Sports World to discuss his remarkable career.
Interviewer: So, what does it take to be a professional coach?
Mr. Walker: Actually, a lot of people ask me that, and I always tell them the
same thing: work, work, and more work. People are always surprised by my
answer.
Interviewer: Most people probably think that your job is pretty easy, am I
right?
Mr. Walker: They see me on TV and get the impression that my life is
glamorous. The reality of being a professional coach, however, is that you
have to be able to multi-task and perform under pressure. Im like a business
manager and athletic trainer, all in one package. I develop training programs
for my athletes, investigate other teams training habits, hire staff, fire staff,
schedule practice sessions, consult with injured players, and negotiate
contracts with new players.
Interviewer: That does sound like a lot of work for one man. What does your
wife think of your career?
Mr. Walker: Well, she wants me to teach in a university like she does rather
than work as a coach. She worries about my health and doesnt like the fact
that we cant spend much time together. I just love to see athletes excel, and
to know that I had something to do with their success. That's what I want the
most.
Interviewer: What would you change about your job, if you could?
Mr. Walker: I wish I had more time to coach amateur players, like high
school and college athletes. I think its important to develop the skills of up-
and-coming athletes, not just ones who sign a contract with us.
176. Why are many surprised about Jerry Walker's work?
a. It is not as easy as it appears to be.
b. it pays less than most people expect.
c. It doesnt require previous experience.
d. It benefits disadvantaged athletes.

177. What is NOT one of Mr. Walkers responsibilities?


a. Hiring new employees
b. Providing a training schedule
c. Recruiting new players
d. Attending athletic conferences

398 04 6260 3948


ng k hc: 0962 60 8801
a ch: S 18 Trn i Ngha Q Hai B Trng H ni
NGOI NG 24H 3
WWW.NGOAINGU24H.VN

178. What is Mr. Walkers wifes profession?


a. She is an athletic trainer.
b. She is an executive manager.
c. She is a professor.
d. She is an athletic recruiter.

179. What does Mr. Walker like most about his job?
a. He loves the competition.
b. He likes to help athletes succeed.
c. He enjoys traveling with his team.
d. He gets to be on television.

180. What would Mr. Walker like to be able to do?


a. Work with younger athletes
b. Spend more time with his team
c. Attend fewer media events
d. Reduce his managerial responsibilities

Questions 181 185 refer to the following two emails.


From: Howard Hilton To: Margaret Willow Subject: Defective Printer
Dear Mrs. Willow,
My company, Pip Imaging, purchased an Easy Jet 1435 printer from your
company four weeks ago. Three weeks ago, an employee reported that the
printer turned off every time she tried to print on both sides. A week after the
problem was reported, a repairman came to the office to fix the problem.
After spending an entire day taking apart the machine, he determined that the
printer was missing an important screw. It is critical that we have a
functioning printer in order to provide our customers with quality
photocopies. Therefore, it is important that we fix this problem as soon as
possible. We will be closing temporarily in two weeks from June 23rd to July
2nd, while we change locations, and we would like to resolve this problem
before then.
Sincerely,
Howard Hilton Technical Director
To: Howard Hilton From: Margaret Willow
Dear Mr. Hilton,
First, I would like to apologize for the problem you encountered with your
printer. This is the first defective product that we have ever been made aware
of, and we have already contacted the Director of Production, Glen Milton, to
investigate the current production practices for all Easy Jet 1435 printers. We
certainly want to resolve this problem as quickly as possible. We are happy
to pay for all repair costs, as well as all shipping fees. Please put the printer
in its original box and send it to the Product Repair Department. The address
399 04 6260 3948
ng k hc: 0962 60 8801
a ch: S 18 Trn i Ngha Q Hai B Trng H ni
NGOI NG 24H 4
WWW.NGOAINGU24H.VN

for this department is provided in the Owners Instruction Manual that came
with the printer. We will have a specialist take a look at the machine
immediately. You should have a working printer within three to four weeks.
Please feel free to contact me at any time, should you have questions or
comments.
Sincerely,
Margaret Willow
Director, Customer Relations, Easy Jet Office Supplies

400 04 6260 3948


ng k hc: 0962 60 8801
a ch: S 18 Trn i Ngha Q Hai B Trng H ni
NGOI NG 24H 401
WWW.NGOAINGU24H.VN

181. How long ago did Pip Imaging purchase the printer?
a. One week
b. Two weeks
c. Three weeks
d. One month

182. What kind of business is Pip Imaging most likely?


a. A copying company
b. A hardware store
c. A photography studio
d. A software supplier

183. According to the first email, when does the printer shut down?
a. Every time it finishes a printing task
b. If the machine is not cleaned regularly
c. When employees try to print on the front and back
d. After it has been used for a certain amount of time

184. What should Howard Hilton do before returning the product?


a. Check the receipt
b. Repackage it
c. Fill out a form
d. Call the Repair Department

185. Why might Howard Hilton not be satisfied with Margaret Willows offer?
a. He has to make an extra payment for parts.
b. It takes longer than he wants.
c. He does not want to pay for shipping fees.
d. The warranty expires before it can be fixed.

Questions 186 190 refer to the following advertisement and letter.


Job Openings at Felton Insurance
Position 1: Assistant Financial Officer
Requirements: A minimum of five years experience in a managerial position. Applicants will
have extensive knowledge of accounting systems, and significant experience developing
budgets and offering financial advice.
Position 2: Project Manager
Requirements: A minimum of 4 years experience organizing and executing projects. A strong
ability to plan new projects and lead people to complete them. Candidates will also be
required to relocate to our offices in Paris.
Position 3: Media Consultant

401
ng k hc: 0962 60 8801 04 6260 3948
a ch: S 18 Trn i Ngha Q Hai B Trng H ni
NGOI NG 24H 402
WWW.NGOAINGU24H.VN

Requirements: At least 3 years of experience in a related field. Applicants will have excellent
verbal and written communication skills. Experience writing press releases and holding press
conferences is preferred.
Position 4: Sales Copy Writer
Requirements: A Masters degree in either Business Communications or Journalism is a must.
Extensive marketing or advertising experience is also required.
To Apply: Send a cover letter, resume, and at least two letters of reference to the HR
Department at Felton Insurance:
Human Resources Department Felton Insurance 145 Liberty Rd. Seattle, WA 98101
Qualified candidates will be contacted by phone to schedule an interview. Please do
not contact the company directly.

To whom it may concern:


My name is Justin Thimble and Im writing in response to the Project Manager position you
advertised recently. I feel that my professional background and personality make me the ideal
candidate for the job. I have over six years of experience working with a small team to plan
and carry out new projects. Additionally, I can speak and write French fluently, and would be
very excited to live abroad. I would greatly appreciate the opportunity to work for such a
prestigious international company. The enclosed resume further details why I would make an
excellent choice for this position. Thank you for your time and consideration.
Sincerely,
Justin Thimble.
186. Which job description mentions education as a requirement for the position?
a. Project Manager
b. Media Consultant
c. Sales Copy Writer
d. Assistant Financial Officer

187. What are applicants asked NOT to do?


a. Call the company
b. Mail the cover letter
c. Provide references
d. Send the resume
188. What is the purpose of Mr. Thimbles letter?
a. To request more information about the company
b. To express interest in the position advertised
c. To change the date of the committee meeting
d. To inquire about the need for his marketing service

189. What is a requirement for the position that Mr. Thimble mentions in the letter?
a. Excellent written communication skills

402
ng k hc: 0962 60 8801 04 6260 3948
a ch: S 18 Trn i Ngha Q Hai B Trng H ni
NGOI NG 24H 403
WWW.NGOAINGU24H.VN

b. Extensive knowledge of accounting systems


c. Willingness to work in a foreign country
d. Ability to organize press conferences and seminars

190. What did Mr. Thimble include with his letter?


a. A project portfolio
b. A reference letter
c. A resume
d. A photograph

Questions 191 195 refer to the following two e-mail messages.


From: Gail Rogers <grogers@seriptors.com> To: John Harlow <jharlow@scriptors.com>
Date: June 21
Subject: Problems with the network Dear Mr. Harlow,
I dont know if youve received other complaints already this morning, but at the moment Im
being denied access to the companys network. Ive never experienced something like this
before. My co- worker, Jim Stevens, said he was having the same problem. When I enter my
employee ID and password, I receive this message: Employee ID invalid. I'm positive that Im
entering the correct ID. so 1 dont know why I cant access the network. I tried to contact the
Computer Technician, but he hasnt replied to my emails. I was hoping that you would be
able to help me, since youre the Technology Supervisor. I would really appreciate any
assistance you can provide at this time.
Thanks.
From: John Harlow <jharlow@striptors.com >
To: Gail Rogers cgrogers@scriptors.com >
Date: June 21
Subject: Re: Problems with the network Hi Gail,
I'm sorry that 1 did not reply sooner, but one of the computers on the fifth floor unexpectedly
shut down and I had to investigate the cause. Our Computer Technician, Thomas Benson, has
the flu and will not be in the office for the rest of the week. I have heard from many other
employees today, and they all mentioned the same error message. I think the problem might
be that we updated our computer security system, but I can't be certain until I do a little more
research. I'm not sure why, but we seem to be having a lot of problems with our computers
lately. This is the tenth complaint I've received this morning. 1 have scheduled a meeting with
the Administrative Supervisor, Philip Downs, at 3 p m. He should lx- able to tell me how to
fix the problem. Usually, the kind of error message you describe appears when there is
something wrong with the security settings. In the meantime, I recommend that you save all
of your current files on a hard disk because there is a possibility that we will have ro erase
your hard drive. If you have any questions before at ext. # 5498.
John Harlow Technology Supervisor
191. What is TRUE about Gail Rogers computer problem?
a. It is not the first time it has happened to her.
403
ng k hc: 0962 60 8801 04 6260 3948
a ch: S 18 Trn i Ngha Q Hai B Trng H ni
NGOI NG 24H 404
WWW.NGOAINGU24H.VN

b. Other employees have not experienced the problem.


c. It occurs when she saves a file.
d. The source of the problem is not known.

192. Who did Gail Rogers try to contact first?


a. John Harlow
b. Thomas Benson
c. Jim Stevens
d. Philip Downs

193. in the first e-mail, the word denied in line 2 is closest in meaning to
a. prevented
b. provoked
c. proven
d. promoted

194. How does John Harlow plan to resolve the problem?


a. By replacing some of the office computers
b. By repairing the electrical wires
c. By sending a computer technician
d. By speaking to a person from another department
195. What is Gail Rogers advised to do?
a. Turn off her computer immediately
b. Save her files to a disk
c. Contact her departments supervisor
d. Consult the instruction manual

Questions 196-200 refer to the following letter and e-mail.


HMG Computer Innovation Conference Lauren Hunt Watson and Rye Media 125 Dobson
Ave.
Atlanta, GA 54897
Dear Ms. Hunt,
l am pleased to inform you that the eighth annual Computer Innovation Conference will take
place from July 22-23 at the Henderson Hotel in Park County, Colorado. This year, we expect
to receive as many participants as in previous years: over 3,000 computer specialists from all
over the world, including some of the most highly respected individuals in the field. As
always, it is our sincerest hope that Watson and Rye Media will participate in the Publishers
Showcase for displaying your latest electronic publishing products, including journals,
software, and any other relevant materials.
The exquisite Boulder Room, which will be set up with tables, shelves, electronic equipment
and Internet access, will be available for you to display multi-media products. The adjoining

404
ng k hc: 0962 60 8801 04 6260 3948
a ch: S 18 Trn i Ngha Q Hai B Trng H ni
NGOI NG 24H 405
WWW.NGOAINGU24H.VN

Silver Springs Room will be reserved for showcasing printed media, like books and
magazines.
If you would like to reserve a space to set up a display, please fill out the registration form
enclosed with this letter. Please remember to indicate which room you would like to reserve,
and the times that you would like to reserve it for. Also, I should inform you that, due to
problems we had last year, all fees must be paid at the time of registration. You should submit
everything by June 15th. If you have any questions, please feel free to call me at 514- 698-
9874, or email me at torson@confercnce.hmg. com.
Sincerely,
Thomas Orson Conference Organizer From: Lauren Hunt <lhunt@watsonandrye.com>
To: Thomas Orson <torson@conference.hmg.com>
Date: Tuesday, June 8
Subject: Exhibits for HMG Conference
Dear Mr. Orson,
I was very excited to receive your invitation to attend the HMG Computer Innovation
Conference. Before I submit the application and fee, however, I would like to ask you an
important question about the rules for setting up an exhibit. Since our company handles both
electronic and print media products, we would prefer to display our product lines together. Is
there any possibility that we would be able to set up both of our exhibits in the Boulder
Room?
Thank you so much for your time. I look forward to hearing from you soon.
Lauren Hunt

196. Why did Mr. Orson write the letter to Ms. Hunt?
a. To introduce new hotel services
b. To provide details about a conference
c. To discuss technological advances
d. To inquire about schedule changes

197. In the letter, the word exquisite in paragraph 2, line 1 is closest in meaning to
a. Expensive
b. Exclusive
c. Exterior
d. elegant

198. How is the registration process this year different from previous years?
a. A maximum of three employees from each company can attend.
b. Applications can no longer be downloaded from the website.
c. A display design is required along with the application.
d. All documents must be submitted with the payment.

199. What does Ms. Hunt indicate in her e-mail to Mr. Orson?

405
ng k hc: 0962 60 8801 04 6260 3948
a ch: S 18 Trn i Ngha Q Hai B Trng H ni
NGOI NG 24H 406
WWW.NGOAINGU24H.VN

a. She does not plan to go to the conference this year.


b. She prefers to reserve a space in the Silver Springs Room.
c. She wants to set up both of her displays in the multi-media room.
d. She only plans to reserve enough space for her printed products.

200. What can be assumed about Ms. Hunt?


a. She is an employee of Watson and Rye Media.
b. She is helping to organize the conference.
c. She prefers print media to electronic media.
d. She will talk to her supervisor about the conference.

NGY 24:
LISTENING REVIEW

406
ng k hc: 0962 60 8801 04 6260 3948
a ch: S 18 Trn i Ngha Q Hai B Trng H ni
NGOI NG 24H 407
WWW.NGOAINGU24H.VN

Listening TEST
In the Listening test, you will be asked to demonstrate how well you understand spoken
English. The entire Listening test will last approximately 45 minutes. There are four parts.
and directions are given for each part. You must mark your answers on the separate answer
sheet. Do not write your answers in the test book.
Part 1
Directions: For each question in this part, you will hear four statements about a picture in
your test book. When you hear the statements, you must select the one statement that best
describes what you see in the picture. Then find the number of the question on your answer
sheet and mark your answer. The statements will not be printed in your test book and will be
spoken only one time.
Example: Sample Answer: A B C D

Statement (C). He is writing in a notebook." is the best description of the picture, so you
should select answer (C) and mark it on your answer sheet.

407
ng k hc: 0962 60 8801 04 6260 3948
a ch: S 18 Trn i Ngha Q Hai B Trng H ni
NGOI NG 24H 408
WWW.NGOAINGU24H.VN

1.

2.

408
ng k hc: 0962 60 8801 04 6260 3948
a ch: S 18 Trn i Ngha Q Hai B Trng H ni
NGOI NG 24H 409
WWW.NGOAINGU24H.VN

3.

409
ng k hc: 0962 60 8801 04 6260 3948
a ch: S 18 Trn i Ngha Q Hai B Trng H ni
NGOI NG 24H 410
WWW.NGOAINGU24H.VN

4.
5.

410
ng k hc: 0962 60 8801 04 6260 3948
a ch: S 18 Trn i Ngha Q Hai B Trng H ni
NGOI NG 24H 411
WWW.NGOAINGU24H.VN

7.

411
ng k hc: 0962 60 8801 04 6260 3948
a ch: S 18 Trn i Ngha Q Hai B Trng H ni
NGOI NG 24H 412
WWW.NGOAINGU24H.VN

8.

9.

412
ng k hc: 0962 60 8801 04 6260 3948
a ch: S 18 Trn i Ngha Q Hai B Trng H ni
NGOI NG 24H 413
WWW.NGOAINGU24H.VN

10.
Part 2:
Directions: You will hear a question or statement and three response spoken in English. They
will be spoken only one time and will not be printed in your test book. Select the best
response to the question or statement and mark the letter (A). (B), or (C) on your answer
sheet.
Example Sample Answer
You will hear: Where did you buy your tie? A B C
You will also hear: (A) Next time well do better.
(B) At the downtown shopping center.
(C) Well move to a new building soon.
The best response to the question "Where did you buy your tie?" is choice (B), "At the
downtown shopping center." so (B) is the correct answer. You should mark answer (B) on
your answer sheet.
11. Mark your answer on your answer sheet. 21 Mark your answer on your answer sheet.
12. Mark your answer on your answer sheet. 22. Mark your answer on your answer sheet.
13. Mark your answer on your answer sheet. 23. Mark your answer on your answer sheet.
14. Mark your answer on your answer sheet. 24. Mark your answer on your answer sheet.
15. Mark your answer on your answer sheet. 25. Mark your answer on your answer sheet.
16. Mark your answer on your answer sheet. 26. Mark your answer on your answer sheet.
17. Mark your answer on your answer sheet. 27. Mark your answer on your answer sheet.
18. Mark your answer on your answer sheet. 28. Mark your answer on your answer sheet.
19. Mark your answer on your answer sheet. 29. Mark your answer on your answer sheet.
20. Mark your answer on your answer sheet. 30. Mark your answer on your answer sheet.
413
ng k hc: 0962 60 8801 04 6260 3948
a ch: S 18 Trn i Ngha Q Hai B Trng H ni
NGOI NG 24H 414
WWW.NGOAINGU24H.VN

31. Mark your answer on your answer sheet.


32. Mark your answer on your answer sheet.
33. Mark your answer on your answer sheet.
34. Mark your answer on your answer sheet.
35. Mark your answer on your answer sheet.
36. Mark your answer on your answer sheet.
37. Mark your answer on your answer sheet.
38. Mark your answer on your answer sheet.
39. Mark your answer on your answer sheet.
40. Mark your answer on your answer sheet.

414
ng k hc: 0962 60 8801 04 6260 3948
a ch: S 18 Trn i Ngha Q Hai B Trng H ni
NGOI NG 24H 415
WWW.NGOAINGU24H.VN

Part 3:
Directions: You will hear some conversations between two people. You will be asked to answer
three questions about what the speakers say in each conversation. Select the best response to each
question and mark the letter (A). (B), (C), or (D) on your answer sheet. The conversations will be
spoken only one time and will not be printed in your test book.
41. Why has the man not purchased his ticket yet?
(A) He's waiting to schedule a meeting.
(B) When does the woman suggest the man go?
(C) The tickets are sold out.
(D) Some routes have been cancelled.
42. When does the woman suggest the man go?
(A) Sunday
(B) Monday
(C) Tuesday
(D) Wednesday
43. What is the man planning to do on Monday?
(A) Fly to Denver
(B) Attend a training program
(C) Go on a holiday
(D) Have a meeting
44. What are the speakers talking about?
(A) Making a presentation
(B) Going out socially
(C) Getting a discount at the cafeteria
(D) Preparing appetizers
45. What did the man do yesterday afternoon?
(A) Worked on a presentation
(B) Had a meeting with some clients
(C) Went out with some coworkers
(D) Attended a company luncheon
46. When will the speakers meet?
(A) 6:00
(B) 6:30
(C) 7:00
(D) 7:30
47. Where are the speakers?
(A) At a presentation
(B) In an office building
(C) At a financial institution
(D) In a post office
48. What does the woman suggest they do?
(A) Stay and wait in line
(B) Postpone the presentation
(C) Call the director
(D) Return again later
49. What is the woman concerned about?
ng k hc: 0962 60 8801 04 6260 3948 415
a ch: S 18 Trn i Ngha Q Hai B Trng H ni
NGOI NG 24H 416
WWW.NGOAINGU24H.VN

(A) If a report she asked for is ready


(B) Where the meeting will be held
(C) If the meeting room is too small
(D) What time the presentation will begin
50. What will most likely happen on Wednesday?
(A) The man will go home early.
(B) The man will be interviewed by Mr. Franklin.
(C) The man will read an accounting book.
(D) The man will change departments.
51. What does the woman suggest the man do?
(A) Go home early to prepare for a meeting.
(B) Visit the accounting department
(C) Buy Mr. Franklin breakfast
(D) Attend a training session
52. What will the woman probably do next?
(A) Interview an applicant
(B) Speak with Mr. Franklin
(C) Visit the accounting department
(D) Give the man a book for reference
53. What does the woman want to do?
(A) Go to Chicago
(B) Leave early
(C) Work late
(D) Arrive early tomorrow
54. What does the man say about Trevor?
(A) He can help with the report.
(B) He will contact the client in Chicago.
(C) He will arrive late tomorrow
(D) He can answer the phone.
55. What did the man do today?
(A) Finished the report
(B) Spoke with his client
(C) Contacted Trevor
(D) Visited Chicago
56. Where did Adam stay in Las Vegas?
(A) In a hotel
(B) His cousin's house
(C) In an express bus
(D) In a small inn
57. When did the woman go to Las Vegas?
(A) Last week
(B) Last month
(C) Six months ago
(D) Two years ago
58. How did Adam mostly spend his vacation?
(A) Fishing
ng k hc: 0962 60 8801 04 6260 3948 416
a ch: S 18 Trn i Ngha Q Hai B Trng H ni
NGOI NG 24H 417
WWW.NGOAINGU24H.VN

(B) Gambling
(C) Visiting his parents
(D) Relaxing at home
59. Why are drinks not included with lunch anymore?
(A) They lost one of their main suppliers
(B) There is a coffee shop nearby.
(C) Not many people drink coffee.
(D) The company is trying to save money.
60. What is the man's concern?
(A) Going to a nearby cafe
(B) Looking for a new job
(C) Paying for a drink
(D) Sending in a complaint
61. Why does the woman recommend the coffee shop?
(A) Her friend owns the shop.
(B) The company pays for the bill.
(C) The coffee tastes fresh
(D) They offer discount cards.
62. What are the speakers talking about?
(A) A shipment
(B) A document
(C) Home furnishings
(D) A colleague
63. Why is Grace Cameron unavailable?
(A) She isn't in the office today.
(B) She is on vacation.
(C) She is in a meeting.
(D) She is away at lunch
64. What does the woman offer to do?
(A) E-mail it to Grace
(B) Fax it to her boss
(C) Sign the form
(D) Make a copy
65. How did the man learn about the positions?
(A) His friend is in the HR department.
(B) He saw an ad in the paper.
(C) He was searching their website.
(D) He contacted the marketing division.
66. Why would the man like to work in marketing?
(A) He can make a lot of money.
(B) He has a friend in that division.
(C) It involves a lot of training.
(D) He has experience in that field.
67. What will the man probably do next?
(A) Visit another company
(B) Write out his resume
ng k hc: 0962 60 8801 04 6260 3948 417
a ch: S 18 Trn i Ngha Q Hai B Trng H ni
NGOI NG 24H 418
WWW.NGOAINGU24H.VN

(C) Wail for an interview


(D) Contact the marketing division
68. Who most likely is the man?
(A) A park supervisor
(B) A real estate agent
(C) A financial analyst
(D) An office receptionist
69. Why would the woman like to rush the purchase?
(A) She is interested in joining the team.
(B) Her office building is expensive.
(C) Her company is launching an advertising campaign soon.
(D) She would like to be closer to home.
70. What will determine the selling price?
(A) The number of people interested in the space
(B) Whether or not the seller is willing to sell
(C) If the building passes the inspection
(D) The location of the office tower
Part 4:
Directions: You will hear some short talks given by a single speaker. You will he asked to answer
three questions about what the speaker says in each short talk. Select the best response to each
question and mark the letter (A), (B). (C), or (D) on your answer sheet. The talks will be spoken only
one time and will not be printed in your test book.
71. Where is the speaker?
(A) In a medical clinic
(B) In an insurance office
(C) At school
(D) In a sports equipment store
72. What information should be included on the form?
(A) Height and weight
(B) Blood pressure reading
(C) Medications being taken
(D) Home address
73. What will happen after the form has been tilled out?
(A) The bill must be paid in full.
(B) Laboratory tests will be run.
(C) Blood pressure and temperature will be taken
(D) The doctor will prescribe new medication.
74. Where does the speaker probably work?
(A) Computer repair shop
(B) Internet cafe
(C) Software manufacturing plant
(D) Auto repair shop
75. When was the computer originally schedule to be delivered?
(A) Yesterday
(B) Two days ago
(C) Today
ng k hc: 0962 60 8801 04 6260 3948 418
a ch: S 18 Trn i Ngha Q Hai B Trng H ni
NGOI NG 24H 419
WWW.NGOAINGU24H.VN

(D) Last week


76. What time will the business open tomorrow
(A) At 6 a.m.
(B) At 7 a.m.
(C) At 8 am.
(D) At 9 am.
77. Where should the participants go first?
(A) To the front desk
(B) To the group lunch
(C) To the video screening
(D) To Wentworth Auditorium
78. Where is the guest speaker schedule posted?
(A) In the front foyer
(B) Outside the conference hall
(C) At the reception desk
(D) In auditorium
79. What will the participants do after lunch?
(A) There will be a surprise guest speaker.
(B) The opening speeches will be given.
(C) A short video will be screened.
(D) A financial paper will be presented.
80. When will the renovations start?
(A) In two weeks
(B) In November
(C) At the beginning of next month
(D) In March
81. What are the employees asked to do?
(A) Contact the design department
(B) Provide suggestions
(C) Clear out their desks
(D) Inform their colleagues
82. What does the speakers wish to talk about at the next meeting?
(A) When to begin the renovations
(B) How they will budget for the renovations
(C) What new departments to create
(D) Whose suggestions will be included
83. Who is listening to this announcement?
(A) Workers in a plant
(B) Visitors to a seminar
(C) Internet providers
(D) Software salesmen
84. What does the speaker say about the products?
(A) They were more expensive, a long time ago.
(B) They have helped millions of people.
(C) They used to be sold only in a small number of stores.
(D) They have always been available online.
ng k hc: 0962 60 8801 04 6260 3948 419
a ch: S 18 Trn i Ngha Q Hai B Trng H ni
NGOI NG 24H 420
WWW.NGOAINGU24H.VN

85. What is being advertised?


(A) New model of computer
(B) Computer training program
(C) Web pages
(D) New computer shop
86. When will the trade show finish?
(A) September 15th
(B) September 16th
(C) September 17th
(D) September 18th
87. What will be displayed in the show?
(A) Rare art
(B) Medical equipment
(C) Food and wine
(D) Office furniture
88. How will this event help Chicago?
(A) The air pollution in the city will decrease.
(B) Tourism to the city will increase
(C) Organizers are making a donation.
(D) Visitors will tell their friends to visit Chicago.
89. Who is Martino Lopez?
(A) A university student
(B) A business executive
(C) A columnist
(D) A radio talk show host
90. How might people know Martino Lopez?
(A) He has written nooks.
(B) He has a popular name
(C) He works for a famous company.
(D) He has appeared on the show before.
91. Where did Mr. Lopez teach?
(A) In England
(B) In the northwest
(C) In Columbia
(D) In New York
92. What is the purpose of this announcement?
(A) To welcome guests to this year's charity dinner
(B) To announce the winner of the raffle
(C) To thank guests for their contributions
(D) To raise awareness about cancer research
93. When will the promoters thank the guests?
(A) Before dinner
(B) Before the dance
(C) Just before dessert
(D) After dinner
94. How will this money most likely be spent?
ng k hc: 0962 60 8801 04 6260 3948 420
a ch: S 18 Trn i Ngha Q Hai B Trng H ni
NGOI NG 24H 421
WWW.NGOAINGU24H.VN

(A) To support cancer research projects


(B) To found a scholarship for students
(C) To build new office buildings
(D) To cover event's marketing costs
95. What will the city mayor announce?
(A) The number of new employees
(B) Building the city's new bridge
(C) The location of a new bridge
(D) Renovating old bridges
96. What benefits will the bridge bring?
(A) Add to the city skyline
(B) Reduce the number of Hampton residents
(C) Provide the mayor with exposure to the media
(D) Ease the traffic from east to west
97. When is construction for the bridge scheduled to start?
(A) This summer
(B) At the end of this year
(C) At the beginning of next year
(D) Next spring
98. Where most likely is this announcement taking place?
(A) At the customs office
(B) In an airport
(C) In a restaurant
(D) In a travel agency
99. What is the reason for the flight's delay?
(A) Bad weather conditions
(B) Power failure
(C) Mechanical problem
(D) Employee strike
100. What are passengers asked to do?
(A) Listen for further announcements
(B) Check into the nearest hotel
(C) Purchase drinks in the lounge
(D) Help the engineering team.

ng k hc: 0962 60 8801 04 6260 3948 421


a ch: S 18 Trn i Ngha Q Hai B Trng H ni
NGOI NG 24H 422
WWW.NGOAINGU24H.VN

NGY 25:
READING REVIEW

In the Reading test, you will read a variety of texts and answer different types of reading
comprehension questions. The entire Reading test will last 75 minutes. There are three parts, and
directions are given for each part. You are encouraged to answer as many questions as possible
within the time allowed.
You must mark your answers on the separate answer sheet. Do not write your answer in the test
book.

Part 5:
Direction: A word or phrase is missing in each of the sentences below. Four answer choices are
given below each sentence. Select the best answer to complete the sentence. Then mark the letter
(A), (B), (C) or (D) on your answer sheet.

101. Those individuals who want to reschedule their ---------- with a doctor should contact us two
days in advance.
(A) positions
(B) assignments
(C) subscriptions
(D) appointments
102. The report shows that builders reserve the right to control the volume of the construction project
on ---------- own.
(A) their
(B) theirs
(C) they
(D) them
103. Information on customers details can be stored on one diskette, as can the whole spreadsheet,
and the diskettes can be kept in a ---------- place.
(A) cautious
(B) distinguished
(C) secure
(D) strict
104. Superb ---------- has been paid to even the smallest detail, as the project was expected to cost
billions of dollars.
(A) attends
(B) attention
ng k hc: 0962 60 8801 04 6260 3948 422
a ch: S 18 Trn i Ngha Q Hai B Trng H ni
NGOI NG 24H 423
WWW.NGOAINGU24H.VN

(C) attended
(D) attendant
105. Mr. Hatcher is now curious about the general opinion on the matter of formatting a standard ----
------ deluxe system instead of formatting a premiere one.
(A) so
(B) nor
(C) or
(D) and
106. If you need to hire ---------- help for your growing business, our experts can assist with the
hiring process from start to finish.
(A) additions
(B) additionally
(C) additional
(D) addition
107. Another computer system failure is going to happen again unless appropriate steps are take to --
-------- it.
(A) prevent
(B) upgrade
(C) ignore
(D) improve
108. All ---------- to our fabulous national park will be given a complimentary lunch at the top of the
hill.
(A) visit
(B) visitations
(C) visitors
(D) visiting
109. Once youve decided to reduce ----------, there should be additional ways to halve the marketing
costs.
(A) values
(B) customs
(C) refunds
(D) expenses
110. Paulin Cornwall Inc. has made an ---------- discovery people can share in no time in the field of
information technology.
(A) amaze
(B) amazing
(C) amazement
(D) amazingly
111. The first draft of workshop proposals was due yesterday, --------- the director has granted us a
short extension to our surprise.
(A) beyond
(B) until
(C) that
(D) but
112. The company is going to carry out the proposal based on the information provided in your letter
of intent, so please submit ---------- application under this note.
ng k hc: 0962 60 8801 04 6260 3948 423
a ch: S 18 Trn i Ngha Q Hai B Trng H ni
NGOI NG 24H 424
WWW.NGOAINGU24H.VN

(A) you
(B) your
(C) yourself
(D) yours
113. After the introduction of collaboration tool program, the company, which was in temporary
financial trouble, was ---------- able to increase its market share.
(A) lastly
(B) at first
(C) meanwhile
(D) finally
114. Many people call fairy tales too ---------- and obvious, but thats why they are needed.
(A) predicting
(B) predicted
(C) predictable
(D) predictably
115. New recycling programs indicate that ---------- the landlord or the property owners may
schedule an appointment to get bulk pickup service in the same building.
(A) only
(B) easily
(C) simply
(D) merely
116. During the sessions, speakers will discuss the process of ----- a global service by opening doors
to economic opportunities.
(A) establish
(B) establishing
(C) establishes
(D) established
117. Please ---------- your survey results to the director by the date specified at the bottom of the
screen.
(A) advise
(B) urge
(C) comply
(D) submit
118. OFA representatives recently ---------- on-line tours and found those kinds of tours to promote
their products the most effective.
(A) conduct
(B) conducted
(C) to conduct
(D) will conduct
119. Please do not -------- to share your opinions in the comment space below so that people can fund
and consider possible alternatives.
(A) provide
(B) hesitate
(C) qualify
(D) compete

ng k hc: 0962 60 8801 04 6260 3948 424


a ch: S 18 Trn i Ngha Q Hai B Trng H ni
NGOI NG 24H 425
WWW.NGOAINGU24H.VN

120. Our GTA phone roaming service is suited for individuals who travel ------ anywhere in the
world for business.
(A) regular
(B) regularity
(C) regularly
(D) regulate
121. A banquet was held for researchers to be honored ---------- their exceptional achievements in
creative research activities.
(A) across
(B) over
(C) at
(D) for
122. We are proud to announce their upcoming ---------- in an event whose purpose is to educate
attendees on e-mail management tools.
(A) participation
(B) participated
(C) participating
(D) participate
123. Anyone who has questions concerning our community service can leave a message ---------- my
secretary or our answering service.
(A) off
(B) from
(C) by
(D) with
124. Requirements for the open position include basic computer skills and managerial experience in -
--------- sales.
(A) retailing
(B) retail
(C) retailer
(D) retailed
125. According to the medical report released today. cholesterol fighting medicines may ------ have
protective effects against numerous uncured diseases. (A) altogether
(B) alone
(C) also
(D) almost
126. Employees who participate in this training program can learn a great deal about themselves as
well as experience in ------ personal and profession growth. (A) every
(B) both
(C) either
(D) whether
127. According to studies released yesterday, a significant number of organizations experienced a
dramatic ---------- in electronic crime and data intrusions over the past year.
(A) increase
(B) impression
(C) access
(D) accent
ng k hc: 0962 60 8801 04 6260 3948 425
a ch: S 18 Trn i Ngha Q Hai B Trng H ni
NGOI NG 24H 426
WWW.NGOAINGU24H.VN

128. The president of Kasara Inc. ---------- announced that Lopez Davis, who has served the
company for more than 30 years, would retire at the end of next month.
(A) regretting
(B) regretted
(C) regretfully
(D) regretful
129. The new training program is intended to ---------- leadership to develop professional
competence through involvement in a range of professional development activities.
(A) remain
(B) grant
(C) serve
(D) provide
130. Due to recent changes, Nordon Express ---------- delivery of equipment effective the first day of
the coming year.
(A) to discontinue
(B) will discontinue
(C) discontinued
(D) have discontinue
131. Employees must get approval from their supervisors ---------- deleting data even if the data
seem unused.
(A) before
(B) until
(C) from
(D) during
132. In addition to ---------- a separate online site, new products will be available through traditional
retailers all over the world.
(A) launched
(B) launching
(C) launch
(D) launches
133. Tomorrow's one-to-one training session held in a boardroom is ---------- for supervisors and
new staff members.
(A) intended
(B) based
(C) agreed
(D) invited
134. You are advised to contact us immediately ---------- technical problems with your computer
system don't seem to be resolved.
(A) why
(B) if
(C) due to
(D) about
135. ---------- all the provisions of the merger are fully implemented. the two companies will expand
their partnerships to provide an even higher level of quality.
(A) Soon
(B) Then
ng k hc: 0962 60 8801 04 6260 3948 426
a ch: S 18 Trn i Ngha Q Hai B Trng H ni
NGOI NG 24H 427
WWW.NGOAINGU24H.VN

(C) Later
(D) Once
136. This letter is to solicit ---------- of candidates to be considered for the vacant positions in sales
that should be filled right away.
(A) nominate
(B) nominations
(C) nominating
(D) nominated
137. Our investment will lead to more improved public services, considerable cost reductions as well
as further ---------
(A) economics
(B) harvest
(C) productivity
(D) measures
138. The names of upper management to --------- you must report will be provided in order to get
budgets approved for projects.
(A) whom
(B) whoever
(C) what
(D) where
139. Successful candidates will receive a competitive salary and maximum benefits --------- to those
which they have received from their previous jobs.
(A) uniform
(B) even
(C) far
(D) equal
140. It is --------- through broadcast media such as television and radio that companies place an
advertisement to attract prospective customers.
(A) predominant
(B) predominantly
(C) predominating
(D) predominated

Questions 144-146 refer to the following article.


The threat of hospital-acquired infections is causing much more vigilance in health-care settings. In
addition, pharmaceutical companies are urged not only to develop therapies for these infections, -----
----- to strengthen their efforts at preventing them.
144 (A) and
(B) or
(C) but also
(D) nor
Obviously, in the health care environment better hand hygiene can significantly reduce infection
rates, ---------- can good practices for sterilizing processing equipment.
145. (A) but
(B) as
(C) not
ng k hc: 0962 60 8801 04 6260 3948 427
a ch: S 18 Trn i Ngha Q Hai B Trng H ni
NGOI NG 24H 428
WWW.NGOAINGU24H.VN

(D) if
Research announced at last year's annual conference shows that traditional cleaning has little ---------
- on removing bacterial contamination in ambulances and crews potentially at risk from microbial
infection, which can then be
146. (A) effected
(B) effecting
(C) effect
(D) effects
transported into hospitals.
Questions 147-149 refer to the following letter.
Dear Mr. Mackenzy,
I'm writing in response to your letter inquiring about "loose skin" resulting ---------- a diet.
147. (A) in
(B) from
(C) on
(D) upon
If you have a lot of weight to lose. you may be worried about "loose skin" on your chin. underarms
and thighs. Please aim to lose weight slowly in order for your body to adjust.
Whatever kind of diet you are on. your skin needs oil, ---------- it will become dry and less elastic.
148. (A) but
(B) yet
(C) so
(D) otherwise
It you add a dressing made with olive to your salads, it may increase your skin's radiance skin will
also become smoother and softer.
Please ---------- that water should be your primary beauty aid.
149. (A) remember
(B) remembering
(C) to remember
(D) be remembered
Not only does it fill you up without calories. but it also takes toxins out of your body system.
Sincerely yours,
Editor, Sara Williams.
Questions 150-152 refer to the following article.
In the past, customers had a tendency of shopping in places where they were certain that ----------
150. (A) whatever
(B) anywhere
(C) ever
(D) however
they wanted to buy was available, but things have changed.
Thanks to impressive advances in electronic commerce technology, consumers have become more
comfortable with using credit cards to make purchases on the Internet.
In addition, as it has become a preferred method of purchasing products and services, the popularity
of online shopping is expected to continue.
One of the greatest ---------- of shopping online over visiting the store in person
151. (A) incomes
ng k hc: 0962 60 8801 04 6260 3948 428
a ch: S 18 Trn i Ngha Q Hai B Trng H ni
NGOI NG 24H 429
WWW.NGOAINGU24H.VN

(B) interests
(C) goods
(D) advantages
is being able to seek out the lowest prices or the best deals available ---------- being provided with a
large selection, variety, convenience and rapid delivery.
152. (A) besides
(B) in addition
(C) without
(D) at

Part 7
Directions: In this part. you will read a selection of texts. such as magazine and newspaper articles,
letters, and advertisements. Each text is followed by several questions. Select the best answer for
each question and mark the letter (A). (B). (C). or (D) on your answer sheet.

Questions 153-154 refer to the following advertisement.


The exclusive Montgomery Caf, located on the first floor of the Gold Star Hotel, is the highest rated
cafe in Deville City. The cafe offers only the finest coffee, which is imported once a month from
Italy. You can choose from thirty different flavors of coffee, including French Vanilla, Mocha,
Raspberry, and Hazelnut. At Montgomery Cafe, we believe that making coffee is an art. All of our
new staff members are required to attend an intensive program where they learn how to prepare
gourmet coffee. In addition to coffee. the Montgomery Cafe offers the following services:
A daily selection of gourmet baked goods
Free Internet access (must spend a minimum of $5.00 at the cafe)
Fresh ground coffee to take home
A range of coffee machines and accessories
Coffee-making classes (applications available at the register)
153. According to the advertisement, what must the cafe's new employees do? (A) Watch an
introduction video
(B) Go to special classes
(C) Sample the product
(D) Purchase the uniform
154. What is NOT being offered in the cafe?
(A) Internet access
(B) Related equipment
(C) Membership discounts
(D) Useful courses
Questions 155-156 refer to the following letter.
July 15, 2007
Jason Holden
26 Humbolt Lane
LA Properties
Los Angeles, CA
035846
Dear Mr. Holden,

ng k hc: 0962 60 8801 04 6260 3948 429


a ch: S 18 Trn i Ngha Q Hai B Trng H ni
NGOI NG 24H 430
WWW.NGOAINGU24H.VN

The investment committee of TA Investors has reviewed your company's proposal and agreed with
your statement that I..A.'s 122nd Avenue has become more popular among tourists since the Flash
Mall was built on the street last year. The committee also agrees that the street needs a first-class
hotel to service the wealthy tourists who shop at the mall. Therefore, the committee decided to
accept your business proposal to open a new hotel on 122nd Avenue by January 2008, and is
prepared to invest a total of 1.5 million dollars in the project. Please call me as soon as possible, so
that we can schedule a meeting with the committee to discuss the details of this investment deal.
Sincerely,
Brian Wilson
TA Investors
155. What will most likely happen in January 2008?
(A) Flash Mall will open new stores.
(B) The city will change tourism policies.
(C) Brian Wilson will buy a property.
(D) LA properties will open a hotel.
156. What is Mr. Holden asked to do?
(A) Fax Mr. Wilson a detailed document
(B) Make a budget for a project
(C) Make an appointment by phone
(D) Examine the corrected proposal
Questions 157-159 refer to the following notice.
Volunteers Wanted!
CBG Steel is sponsoring the third annual Race for the Children. a ten kilometer run that raises
money for Mercy Children's Hospital. located near the company's main building. The race will be
held on Saturday. November 23. at 3:00 p.m. at the Headman Pavilion. The Planning Committee is
looking for volunteers (who are NOT running in the race) to work at the drinking stations. The job is
very easy. and only requires two to three hours of your time. You should arrive around 1:00 p.m. to
set up the drinking station for the race, which involves organizing cups, water. juice, and snacks for
the runners. Then. you will be responsible for handing out beverages and snacks to racers as they run
by. After the race finishes, there will be an awards ceremony and a brief speech by CBG president,
Walter Headman, followed by a small celebratory dinner for everyone. All volunteers will receive a
free meal pass. For anyone interested in volunteering. there will be a brief meeting on Thursday.
November 15, at 12:00 p.m. The meeting will be held in Room #203. Lunch will be provided.
157. Why is CBG Steel organizing the event?
(A) To collect money for a local facility
(B) To promote their new products
(C) To introduce a new staff member
(D) To support potential athletes
158. What will volunteers most likely do at 1 p.m.?
(A) Attend a brief meeting for a race
(B) Gather at Headman Pavilion
(C) Organize Room #203
(D) Meet a member of the Planning Committee
159. What is planned for after the president's speech?
(A) An awards ceremony
(B) A ten kilometer running race
ng k hc: 0962 60 8801 04 6260 3948 430
a ch: S 18 Trn i Ngha Q Hai B Trng H ni
NGOI NG 24H 431
WWW.NGOAINGU24H.VN

(C) A meal for all participants


(D) A meeting for volunteers
Questions 160-163 refer to the following articles.
Ready Air Introduces New In-flight Services
Ready Air has expanded its in-flight services for their new fleet of planes, which will start flying in
February 2008, a spokesman for the company said. Ready Air is the only airline in the world to offer
strictly economy class planes. The planes offer first class service for economy class passengers. the
spokesman said. Each seat is equipped with its own TV screen, and passengers can choose from 30
different movies in 20 different languages. At any time during the flight, passengers can enjoy real
espresso coffee from Magic Beans Inc., Seattle's most famous brand of coffee. On international
flights, every passenger will receive a complimentary travel kit. which includes toothbrush and
toothpaste. shampoo. mouthwash, comb. socks, and sleeping mask. Passengers who have a Ready
Air Frequent Flier card will also have unlimited access to Ready Air's in-flight library, which
includes an extensive selection of books, newspapers, and magazines. The CEO of Ready Air. Scott
Plow, said the fleet's new services "will change the way people in economy class fly."
160. Why is Ready Air different from other airlines?
(A) It has the highest number of planes.
(B) It offers the cheapest economy tickets.
(C) It doesn't have a first class section.
(D) It has the greatest variety of routes worldwide.
161. What can be inferred from the article?
(A) Frequent fliers get preferred seating.
(B) Ready Air is an award-winning company.
(C) Scott Plow is the founder of Ready Air.
(D) Magic Beans Inc. is based in Seattle.
162. Who is eligible to receive the travel kit?
(A) Passengers traveling to another country
(B) Passengers with a preferred customer card
(C) Passengers who have small children
(D) Passengers with connecting flights
163. Which of the following is available to Frequent Flier cardholders?
(A) Unlimited access to airport lounges
(B) Extensive access to reading materials
(C) Complimentary accommodation at airport hotels
(D) Double mileage on international flights
Questions 164-165 refer to the following advertisement.
Apartments for Rent
Come and live in the city's newest apartment complex, Dayville Suites. Located just two blocks from
the financial district, these apartments are perfect for businessmen and young professionals. The
building is twenty-five stories high and has 200 apartments. Anyone who lives in the apartments has
unrestricted access to the rooftop pool and the fitness center on the first floor. There is also a
restaurant on the first floor. and a supermarket in the basement that is open seven days a week. Three
different apartment styles are available_ The monthly rent for each style is as follows:
1 Bedroom $1,000
2 Bedrooms $2,000
3 Bedrooms $3.000
ng k hc: 0962 60 8801 04 6260 3948 431
a ch: S 18 Trn i Ngha Q Hai B Trng H ni
NGOI NG 24H 432
WWW.NGOAINGU24H.VN

NOTE: All apartments have air conditioning and floor heating. Anyone who provides the first six
months' rent up front will receive a ten percent discount.
164. What is TRUE about the facilities at Dayville Suites?
(A) The restaurant offers 24-hour-a-day service.
(B) The pool is located in the basement.
(C) The supermarket is closed on Sundays.
(D) The gym offers unlimited access to all tenants.
165. How can someone reduce the cost of their monthly rent?
(A) Make six consecutive payments on time
(B) Pay for the first six months in advance
(C) Choose an unfurnished apartment
(D) Use cash to pay for the rent
Questions 166-170 refer to the following article.
Local Company Builds Gym
Ask any health professional and they will tell you that the key to good physical and mental health is
to eat a balanced diet, get plenty of rest, and exercise regularly. Alarmingly, a recent study conducted
by the Institute of Balanced Living(IBL) revealed that the average person consumes some form of
fast food at least twice a week, sleeps less than 6 hours a night, and exercises less than once a week.
IBL said this is quite a dangerous situation for the nation's health.
In order to teach its employees to adopt healthy lifestyle habits. the popular software company.
MegaWare, has built a gym in its office building, and is hoping all employees will use the facilities
at least three times a week. To encourage the employees to take advantage of the gym, the company
is offering a $200 monthly bonus to every employee that uses the gym more than three times a week.
The company even developed a special program to keep track of the number rof times each
employee uses the gym. Last month every staff member was issued a special ID card, which the
employees must use to enter and exit the gym. Each time the card is used. the session is recorded in
the main database. The company also hopes to open an organic cafeteria in June 2008.
166. According to the article. what did the IBL conclude?
(A) People are spending more money on health products.
(B) People are not leading healthy lifestyles.
(C) Companies are starting to offer more training programs.
(D) Fitness trainers are increasing their Consultation fees.
167. What change recently occurred at MegaWare?
(A) A new health insurance plan was adopted.
(B) A longer lunch break was implemented.
(C) A fitness center was opened in the building.
(D) A staff psychologist was hired.
168. The word "encourage" in paragraph 2. line 4, is closest in meaning to
(A) inspire
(B) demonstrate
(C) organize
(D) prevent
169. Why did MegaWare give employees a special card?
(A) To substitute for a company card
(B) To reduce health insurance payments
(C) To improve their security system
ng k hc: 0962 60 8801 04 6260 3948 432
a ch: S 18 Trn i Ngha Q Hai B Trng H ni
NGOI NG 24H 433
WWW.NGOAINGU24H.VN

(D) To monitor how many times people used the gym


170. What does MegaWare wish to do in 2008?
(A) Offer more vacation time
(B) Provide better food
(C) Open a health clinic
(D) Build an outdoor track
Questions 171-173 refer to the following notice.
Attention all Club Members
The Highland Country Club regretfully informs all members that our pool will be closed from July
15 to August 2.5. While fixing a broken pipe, maintenance workers found a major problem with the
filtration system. Unfortunately, this problem is quite complex and requires the work of a plumbing
specialist. Because its plumbers are unexpectedly busy at the moment, QuickFix Plumbers, the only
company that offers the service we need, will not be able to help us for another two weeks. We are
terribly sorry for the inconvenience, and are willing to refund the cost of one month's club
membership to all individuals who paid for the 3-month Summer Membership.
Because we already offer a twenty percent discount to all members who sign up for the 1-year
membership, we will not be offering these individuals a refund. Anyone who has questions or
comments about the change should contact Melvin Woods at 869-654-9874.
171. What is the notice mainly about?
(A) The recent opening of a club building
(B) The temporary closing of a facility
(C) The changes to membership packages
(D) The schedule of a special competition
172. What does the notice mention about QuickFix Plumbers?
(A) They are unusually busy.
(B) They offer special discounts.
(C) They have the best service.
(D) They use a new technology.
173. What is TRUE about members with 1-year memberships?
(A) They will receive one month free.
(B) They can access the day spa.
(C) They can use a private pool.
(D) They won't get a refund.
Questions 174-177 refer to the following e-mail
To: Kathryn Hale khale@ftk.com
From: Stacy Mott <smott@ftk.com>
Subject: Budget proposal
Dear Ms. Hale,
After reviewing last years budget figures, I would like to propose a few changes to this year's
budget plan. If these changes are accepted, I predict that the company will reduce its operation costs
by approximately 40 percent. First, I noticed that the company allocated one fourth of its total budget
to purchasing office supplies. Of the office supplies that we purchase each year. paper is by far the
most expensive. I think that requiring all employees to print documents on both sides of the paper
will significantly cut down on costs. Second. the company currently employs 200 people; however,
only about 150 people are needed for the company to function property. Therefore, I propose the
company reduces its workforce by 50 people before the start of the next fiscal year. I realize that this
ng k hc: 0962 60 8801 04 6260 3948 433
a ch: S 18 Trn i Ngha Q Hai B Trng H ni
NGOI NG 24H 434
WWW.NGOAINGU24H.VN

second proposal may seem drastic, but according to my estimates, if the company does not
significantly cut costs by the end of December 2008, it will be forced to file for bankruptcy.
Sincerely,
Stacy Mott
Financial Advisor
174. What is the purpose of this e-mail?
(A) To ask for modifications to a financial plan
(B) To propose a schedule change
(C) To introduce an advertising campaign
(D) To request more office supplies
175. What does Stacy Mott point out about the company?
(A) It must hire more employees in order to grow.
(B) It spends too much money on office supplies.
(C) It failed to meet certain operational standards.
(D) It should invest more time in strategic planning.
176. How many people work for the company right now?
(A) 40
(B) 50
(C) 150
(D) 200
177. According to the e-mail, what might happen if the recommended changes are not made?
(A) Employees will lose certain benefits.
(B) They will forfeit their major contract.
(C) The company will have to close.
(D) The plan for expansion will be rejected.
Questions 178-180 refer to the following letter.
June 15, 2007
Charles Masters
15 Billows Rd.
Forestwood, CA
Dear Mr. Masters,
After conducting an extensive performance review of all the employees in the Advertising
Department here at Dunn and Dunn Inc., we have decided to promote you to Corporate Accounts
Manager. Since you started working for us five years ago, you have consistently demonstrated a
strong work ethic and the ability to produce quality work The executives feel strongly chat your
proposal to implement new advertising techniques helped the company to secure 20 new clients in
2004-2005. Additionally, in 2006 you voluntarily created an Office Policy Manual for new
employees, which demonstrates your dedication to the company and desire to make the office
environment as efficient and friendly as possible. Your responsibilities as Corporate Accounts
Manager will include delegating work to subordinates, researching potential advertising campaigns,
organizing meetings with clients, and determining the annual department budget. In return for your
extra work, you will receive a 25 percent increase in your current salary. seven additional vacation
days, and a $5,000 bonus.
Sincerely,
John Milestone
Director of Advertising
ng k hc: 0962 60 8801 04 6260 3948 434
a ch: S 18 Trn i Ngha Q Hai B Trng H ni
NGOI NG 24H 435
WWW.NGOAINGU24H.VN

178. What is the purpose of this letter?


(A) To introduce an advertising proposal
(B) To describe an investment opportunity
(C) To offer an employee a title change
(D) To respond to a client's request
179. How did Charles Masters improve the office in 2006?
(A) By recruiting the most clients
(B) By cutting down operational costs
(C) By proposing a modest annual budget
(D) By writing important guidelines
180. Which of the following is NOT the responsibility of the Corporate Accounts Manager?
(A) Determining the vacation schedule for employees
(B) Assigning work to employees in the department
(C) Creating a financial plan for the company
(D) Organizing meetings with customers
Questions 181-185 refer to the following advertisement and letter.
Al's Autos Super Saturday Sale
Al's Autos, the number one used car dealership in Florida, is having a mega -sale Saturday, August
25, 2007, from noon until 4 p.m. to celebrate our twenty years of operation. All two-door vehicles
with pink stickers are 10 percent off the ticketed price. All trucks with blue stickers are 15 percent
off the ticketed price. All vans are 20 percent off the ticketed price. Plus, if you pay in cash, we'll
reduce the price by another $300. As always, all purchases come with a 2-year Al's Autos Guarantee,
which covers the cost of labor and replacement of parts if your car experiences mechanical problems.
So if you're in the market for a used car, then come on down to Al's Autos this Saturday.
*Cars must be serviced by one of the following Al's Autos affiliates: Mike's Mufflers, Ace
Mechanics, or Carl's Car Parts.
l7 Washington Rd.
Tampa, FL
September 26, 2007
Allen Hanks
Al's Autos
56 Wheelbarrow Rd.
Tampa, FL
Dear Mr. Hanks,
Last month I purchased a used Capri 1025 from you. At first, the car seemed like a great deal,
especially with the $300 dollar discount I received. I bought the car because I have to travel a long
distance to work each day, and with gas being so expensive these days I needed a car that was more
fuel-efficient. You were right; the smaller car did save me about $30 in gas each week.
Unfortunately, after about two weeks, the car started making a clanking noise if I went faster than 5o
kilometers per hour. I took it to my mechanic. Smooth Rides. and had them evaluate the problem.
They determined that the entire engine would need to be replaced, due to a crack down the middle of
it. In total, the labor and engine parts will cost about $1,000. I assume that since I only purchased the
car a month ago, I am well within the 2-year limit to the guarantee and Al's Autos will cover the cost
of the repairs.
Please contact me at 852-321-6547 if you have any questions.
Sincerely,
ng k hc: 0962 60 8801 04 6260 3948 435
a ch: S 18 Trn i Ngha Q Hai B Trng H ni
NGOI NG 24H 436
WWW.NGOAINGU24H.VN

Floyd Baker
181. According to the advertisement, what will happen on Saturday, August 25?
(A) A retirement party for an employee
(B) A special discount on certain cars
(C) A big sale at a new car dealership
(D) An increase in the price of gas
182. How can a customer save more money?
(A) By making their purchase in cash
(B) By using their preferred customer card
(C) By applying for a drawing event
(D) By obtaining a coupon from the newspaper
183. What can be inferred about the Capri 1025 from the letter?
(A) It has two doors.
(B) It was twenty percent off.
(C) It is energy efficient.
(D) It is better for short distances.
86-190 refer to the following invoice and e-mail.
184. Why will Al's Autos probably reject Floyd Baker's request?
(A) He used a credit card to buy the car.
(B) He purchased the vehicle after the sale.
(C) He did not take his car to the right mechanic.
(D) He failed to sign and return the contract.
185. What does Floyd Baker's car need? (A) New tires (B) A bumper replacement (C) An engine
change (D) Car door painting
NanoMac Inc.
Finance Department Office
Office #402
26 Mountain Dr.
Sharonville, SC
Brooks Brothers Marketing
356 Brewster Rd.
Applewood, TN
Attn: Paul O'Reilly
INVOICE FOR:
1 desktop computer $590.75
1 laser jet printer $195.95
2 cartons glossy 8 x 11 inch copy paper $54.25
100 blue Shic pens $103.96
GST (Goods and Services Tax) $62.03
TOTAL PAYABLE: $1006.94
Customer Registration No. 878-96854-632
Invoice No. 856
Date of Invoice: July 15, 2007
To be paid within 30 days of invoice date. Should customers fail to pay within 30 days, a charge of
$50 will be added to the account each month that payment is overdue. Note: NanoMac Inc. is trying
to improve its customer service. Since your company has teen a loyal customer of ours, we invite
ng k hc: 0962 60 8801 04 6260 3948 436
a ch: S 18 Trn i Ngha Q Hai B Trng H ni
NGOI NG 24H 437
WWW.NGOAINGU24H.VN

you to fill out an online survey to assess our customer service. Fill out the survey at
www.nanomac.com and you can enter to win a 500 coupon.
If you have any questions about the invoice, please contact our department by e-mail or Atone.
To: Michael Shoe <mike@kmail.com>
From: Paul O'Reilly <paul@bbmarketing.com>
Subject: Surcharge Date: September 25. 2007
Hi Michael.
I recently received an invoice for the products we purchased from your company in July. I noticed
that we were charged an additional $100 dollars because our payment was two months overdue. I
would like to explain why we failed to make these payments. and hope that you will reverse the
charges immediately.
At the end of July, our department underwent restructuring and Brian Davies. our Finance Manager.
was asked to leave. Unfortunately, in the process of changing managers a few files were misplaced,
including the one containing all of our office supplies invoices. I hope you will accept my request so
that we continue doing business with your company in the future.
Sincerely.
Paul O'Reilly
186. What kind of service does NanoMac Technology provide?
(A) It develops computer software.
(B) It distributes office supplies.
(C) It repairs electronic devices.
(D) It provides financial advice.
187. Which of the following is TRUE about Brooks Brothers Marketing?
(A) It has offices in Sharonville.
(B) It is an electronics store.
(C) It works with NanoMac frequently.
(D) It is an established company.
188. What is the purpose of the email?
(A) To increase the volume of the orders
(B) To inquire about an upcoming sale
(C) To request an address change
(D) To respond to a service charge
189. What happened at Brooks Brothers Marketing in July?
(A) It merged with another marketing firm.
(B) The management structure was changed.
(C) A new marketing campaign was introduced.
(D) It relocated its headquarters to a different city.
190. What can be inferred about Michael Shoe?
(A) He works in the Finance Department at NamoMac.
(B) He usually delivers the orders himself.
(C) He was recently hired from Brooks Brother Marketing
(D) He was in charge of renovating in the bulding.
Questions 191-195 refer to the following ernalls
To: Fran Morris <fran@friendlyflowers.com>
From: Jerry Sprigs <jerrn@irvingcountryclub.com>
Subject: Delivery Request
ng k hc: 0962 60 8801 04 6260 3948 437
a ch: S 18 Trn i Ngha Q Hai B Trng H ni
NGOI NG 24H 438
WWW.NGOAINGU24H.VN

Dear Ms. Morris,


I am writing to inform you that the McGuiness Women's Foundation Luncheon. which was
scheduled to take place this Saturday at 3 p.m., has been postponed to Sunday at 2 p.m., due to
predictions of showers. Therefore, I would like to change the delivery time for the flower
arrangements we ordered. Would you be able to deliver the flowers either Saturday morning, or
between 2:00 p.m. and 4:00 p.m. Saturday afternoon? Also, if you choose to deliver the flowers on
Saturday afternoon, please do not take the flowers to the Club House: we will be hosting a charity
lunch for the Rotary Club at that time, and the area will be full of people. Instead. could you please
bring the flowers to the Maintenance building next to the botanical gardens? If you can bring the
flowers in the morning, just take them directly to the Club House as we originally discussed. Please
e-mail me immediately to confirm these changes.
Regards,
Jerry Sprigs
Manager, Irving Country Club
To: Jerry Sprigs <jerry@irvingcountryclub.com>
From: Fran Morris <fran@friendhiflowers.corn>
Subject: RE: Delivery Request
Dear Mr. Sprigs,
I received your request to change the date and time we deliver your flowers. Unfortunately, we are
completely booked in the afternoon on Saturday and will not be able to deliver the flowers at the
time you requested. We would be happy to deliver the flowers between 10:00 a m. and 12:00 p.m. on
Saturday. Also, I should inform you that Friendly Flowers charges 525 for any changes that are made
to orders on such short notice. I apologize for this inconvenience. but it helps us provide the most
reliable service possible. I will be out of the office this afternoon. so please call me on my cell phone
at 369-654-9876.
Sincerely,
Fran Morris
Owner, Friendly Flowers
191. Why does Jerry Sprigs want to change the delivery time?
(A) The planned event was delayed.
(B) The Club House is closed.
(C) The members requested it.
(D) The event starts earlier.
192. What is TRUE about the Irving Country Club?
(A) It is a popular place for weddings.
(B) It will host a fundraising event.
(C) It has international golf tournaments.
(D) It is famous for its botanical gardens.
193. What should Mr. Sprigs do to change an order?
(A) Provide at least 24 hours' notice
(B) Fill out an electronic form
(C) Submit a written request
(D) Pay a small fee
194. In the second e-mail. the word 'reliable' in line 6 is closest in meaning to
(A) arguable
(B) transferable
ng k hc: 0962 60 8801 04 6260 3948 438
a ch: S 18 Trn i Ngha Q Hai B Trng H ni
NGOI NG 24H 439
WWW.NGOAINGU24H.VN

(C) questionable
(D) dependable
195. How will Fran Morris most likely deliver the flowers?
(A) By meeting Jerry Springs at the office
(B) By going to the Rotary Club
(C) By dropping them off at the Club House
(D) By driving to the Maintenance building
Questions 196-200 refer to the following letters.
12 Buffalo Lane
Buffalo, New York
April 18
Bill Hipster
Epson Pharmaceuticals
157 Health Valley Dr.
Chicago. IL
Dear Mr. Hipster,
My name is Dr. Ivan Rigby, and I am a professor of Molecular Biology at Illinois State University
(ISU). I am writing on behalf of Melvin Tobin, who I believe would make an excellent candidate for
the Research Aide position your company is offering this summer.
Melvin is a junior at ISU. and for the last three years he has consistently received the highest grades
in his class. Last year. he submitted a research paper. "Mapping the Human Genome," which was
published in The Scientist, one of the most prestigious science journals in the country. Recently,
Melvin was awarded the Scientific Research Award for his outstanding work in the field of
molecular research.
Despite his busy academic schedule, Melvin manages to find time to volunteer at the local
elementary school (teaching science of course!). and write for the campus newspaper. ISU News. He
is friendly. compassionate, and hardworking, and he is destined fur great things. I feel strongly that
this young man would be a great addition to your company.
Sincerely,
Dr. Ivan Rigby
August 20
Dr. Ivan Rigby
Department of Molecular Biology
Dear Professor Rigby,
I would like to express my gratitude for the recommendation letter you wrote to Epson
Pharmaceuticals and I apologize for the lateness of this letter conveying my thanks.
I have learned more in my brief role as an Assistant Laboratory Technician than I have in all three
years at ISU. As you may have noticed, the company was unable to hire me for the position that you
recommended because it had already been filled, but I have really enjoyed working as an assistant
technician instead. I only have three more weeks before I return to ISU, but the company has already
asked that I join their team after I graduate!
They are offering me the position of Laboratory Technician and assigning me to work in the division
that researchers medication for patients recovering from heart surgery. In addition to an excellent
salary, they are also offering me free housing on the Epson Campus, which is located just three miles
from lake Michigan.

ng k hc: 0962 60 8801 04 6260 3948 439


a ch: S 18 Trn i Ngha Q Hai B Trng H ni
NGOI NG 24H 440
WWW.NGOAINGU24H.VN

Despite recent allegations that the company is producing drugs that do not meet federal standards, I
feel this is a great opportunity to work for the largest pharmaceutical company in the country. Thank
you once again for helping me get this wonderful opportunity. I look forward to attending your
classes next semester.
Sincerely,
Melvin Tobin
196. Why is Dr. Ivan Rigby sending the letter to Bill Hipster?
(A) To recommend a student for a position
(B) To suggest a student for a scholarship
(C) To propose an article for a journal
(D) To announce an awards ceremony
197. Which of the following was NOT mentioned about Melvin Tobin in the first letter?
(A) He received an award for his research work.
(B) He teaches children at a local school.
(C) He worked as a Research Aide before.
(D) He received the best grades in his courses.
198. What will Melvin receive if he accepts the full-time job offer?
(A) A research grant
(B) Free accommodation
(C) A pay rise
(D) Medical insurance
199. What happened to Epson Pharmaceuticals recently?
(A) It was accused of wrongdoing.
(B) It discovered a new drug.
(C) It appointed a new CEO.
(D) It increased the price of drugs.
200. What can be inferred about Melvin's position at Epson Pharmaceuticals?
(A) It did not teach him anything new.
(B) It offered very little compensation.
(C) It involved researching human genomes.
(D) It was not the original position he applied for.

ng k hc: 0962 60 8801 04 6260 3948 440


a ch: S 18 Trn i Ngha Q Hai B Trng H ni

You might also like